Quick viewing(Text Mode)

Aiims 21 Year Biology

Aiims 21 Year Biology

IIT JEE NEET eBooks: www.crackjee.xyz

1 The Living World

TYPE A : MULTIPLE CHOICE QUESTIONS 7. The system of classification based on evolutionary and genetic relationships among 1. Linnaeus system of classification is [1999] organisms, ignoring the morphological (a) Natural (b) Artificial similarities or differences, is called [2009] (c) Phylogenetic (d) Progressive (a) cladistics 2. The basic unit of classification is [2000] (b) phenetics (a) (b) species (c) classical systematics (c) variety (d) subspecies (d) new systematics 3. Binomial nomenclature was introduced by 8. Scientific name of Mango plant is Mangifera (a) Linnaeus [2000] indica (Linn.) Santapau. In the above name (b) Darwin Santapau refers to [2012] (c) Bentham and Hooker (a) Variety of Mango (d) Aristotle (b) A taxonomist who proposed the present 4. The usage of binomial names, for plant species nomenclature in honour of Linnaeus was accepted by all after the publication of the (c) A scientist who for the first time described work by [2001] Mango plant (a) Hooker (b) Linnaeus (d) A scientist who changed the name (c) Bentham (d) Darwin proposed by Linnaeus and proposed 5. Which one of the following is correctly matched present name regarding an Institute and its location? [2004] 9. The classification of Linnaeus was mainly based (a) National Institute of Virology - Pune on [2012] (b) National Institute of Communicable disease (a) Sepals (b) Carpels - Lucknow (c) Petals (d) Stamens (c) Central Drug Research Institute - Kasauli Institute 10. Which of the following is less general in (d) National Institute of Nutrition - Mumbai characters as compared to genus? [2013] 6. “Ordines Anomali” of Bentham and Hooker (a) Species (b) Division includes [2006] (c) Class (d) Family (a) seed plants showing abnormal forms of 11. Read the following statements growth and development. (i)Lower the taxon, more are the characteristics that the members within the (b) plants represented only in state. taxon share. (c) plants described in the literature but which (ii) Order is the assemblage of genera which Bentham and Hooker did not see in original. exhibit a few similar characters. (d) a few orders which could not be placed (iii) Cat and dog are included in the same family satisfactory in the classification. Felidae. IIT JEE NEET eBooks: www.crackjee.xyz EBD_7100

B-2 Topicwise AIIMS Solved Papers – BIOLOGY

(iv) Binomial Nomenclature was introduced by (a) If both Assertion and Reason are correct and Carolus Linnaeus. [2014] the Reason is a correct explanation of the Which of the following statements are NOT Assertion. correct? (b) If both Assertion and Reason are correct but (a) (i), (ii) and (iii) (b) (ii), (iii) and (iv) Reason is not a correct explanation of the (c) (i) and (iv) (d) (ii) and (iii) Assertion. 12. Choose the correct one [2015] (c) If the Assertion is correct but Reason is (i) Growth cannot be taken as a defining incorrect. (d) If both the Assertion and Reason are incorrect. property of living organism. (e) If the Assertion is incorrect but the Reason is (ii) Dead organism does not grow. correct. (iii) Reproduction cannot be an all inclusive 14. Assertion: Living organisms are regarded as defining characteristic of living organisms. closed systems. (iv) No non-living object is capable of Reason: Energy of living organisms can not be replicating itself. lost or gained from external environment. [2002] (v) Metabolism in a test tube is non-living. 15. Assertion: Systematics is the branch of biology (vi) Metabolism is a defining feature of all living that deals with classification of living organisms. organisms. Reason: The aim of classification is to group (a) (i) and (iii) (b) All except (v) the organisms. [2002] (c) All except (iii) (d) All of these Directions for (Q. 16) : Each of these questions 13. Match column I with column II and choose the contains an Assertion followed by Reason. Read them correct option. [2017] carefully and answer the question on the basis of Column-I Column-II following options. You have to select the one that A. Family I. tuberosum best describes the two statements. B. Kingdom II. Polymoniales C. Order III. Solanum (a) If both Assertion and Reason are correct and D. Species IV. Plantae Reason is the correct explanation of Assertion. E. Genus V. Solanaceae (b) If both Assertion and Reason are correct, but (a) A –IV; B –III; C –V; D –II; E –I Reason is not the correct explanation of (b) A –V; B –IV; C –II; D –I; E –III Assertion. (c) A –IV; B –V; C –II; D –I; E –III (c) If Assertion is correct but Reason is incorrect. (d) A –V; B –III; C –II; D –I; E –IV (d) If both the Assertion and Reason are incorrect. 16. Assertion : Taxon and category are different TYPE B : ASSERTION REASON QUESTIONS things. Directions for (Qs. 14-15) : These questions consist Reason : Category shows hierarchical of two statements, each printed as Assertion and classification. [2013, 14] Reason. While answering these questions, you are required to choose any one of the following five responses. IIT JEE NEET eBooks: www.crackjee.xyz

The Living World B-3

Type A : Multiple Choice Questions “cladogram” suggests that the lengths of the branches in the diagram are arbitrary, 1. (b) Carolus Linnaeus system of classification while in a “phylogeny,” the branch lengths is artificial. indicate the amount of character change. 2. (b) The basic unit of classification is species 8. (d) Collection of rules regarding scientific which consists of a natural population of nomenclature of plants is known as ICBN individuals having similar, morphological, or International Code of Botanical anatomical, biochemical, cytological and Nomenclature. It was first accepted in 1961. reproductive characters so that they can According to ICBN, if any scientist has interbreed freely and produce fertile proposed wrong name then his name offsprings. should be written in bracket and the 3. (a) Carolus Linnaeus gave the binomial system scientist who corrected the name should of nomenclature. be written after the bracket. 4. (b) Carolus Von Linnaeus in his book Genera 9. (d) Classification proposed by Linnaeus is Plantarum made use of the artificial system artificial. Linnaeus classified plant kingdom of classification. He distinguished between on the basis of only two characters the natural and artificial systems. He used (1) stamens (2) style the binomial nomenclature system and classified organisms into genus and 10. (a) Species is the lowest taxonomic category. species. Class is a category made of one or more 5. (a) National Institute of Virology is located in related orders possessing similar correlated Pune. characters. Family is composed of one to The National Institute of Virology (NIV) is many related genera. Division comprises a premier virus research laboratory in India. of several related classes. It is one of the major Institutes of the Indian 11. (d) Order being higher category is the Council of Medical Research (ICMR). It was assemblage of families which exhibit a few established at Pune, similar characteristics. 6. (d) Bentham and Hooker system was Dog (Canis familaris) and Cat (Felis published well before there were domesticus) belong to two different internationally accepted rules for botanical families—Cancideae and Felidae nomenclature. In this system, termination respectively. for families was not so as it is now. 12. (b) Metabolism takes place only inside the 7. (a) “Cladogram” emphasizes that the diagram body of a living organism. represents a hypothesis about the actual 13. (b) A - V; B - IV; C - II; D - I; E - III evolutionary relationships of a group. This history is deduced on the basis of certain Type B : Assertion Reason Questions shares like homologous characters. that are thought to indicate common ancestry. 14. (d) Living organisms are regarded as open While “phylogenies” represent true system as energy flow is the key function evolutionary history. To other biologists, of an ecosystem. IIT JEE NEET eBooks: www.crackjee.xyz EBD_7100

B-4 Topicwise AIIMS Solved Papers – BIOLOGY

15. (b) Systematics is the science of identification, represent any level of grouping of naming and classification of the organisms organisms based on certain common into groups. characteristics. There is some confusion 16. (a) A category is a rank or level in the in the use of taxon and category, for example hierarchial classification of organisms. Bryophyta is a taxon while division is a Taxon is a unit in classification which may category. IIT JEE NEET eBooks: www.crackjee.xyz

2 Biological Classification

TYPE A : MULTIPLE CHOICE QUESTIONS 11. Powdery mildews of crops are caused by [2001] (a) bacteria (b) ascomycetes 1. The symbiotic relationship of algae and fungus is found in [1997] (c) basidiomycetes (d) phycomycetes (a) lichen (b) mycorrhiza 12. Pullorum disease of poultry is caused by [2001] (c) pneumatophore (d) bacteriophage (a) Mycobacterium (b) Salmonella 2. The virus that infects bacteria is called [1997] (c) Clostridium (d) Haemophilus (a) cyanophage (b) mycophage 13. Viroids have [2003] (c) bacteriophage (d) none of these (a) single stranded RNA not enclosed by 3. Citrus canker is caused by [1997] protein coat. (b) single stranded DNA not enclosed by (a) bacterium (b) fungus protein coat. (c) alga (d) virus (c) double stranded DNA enclosed by protein 4. Which of the following is a cyanobacterium ? coat. (a) Nostoc (b) Chara [1997] (d) double stranded RNA enclosed by protein coat. (c) E. coli (d) Polysiphonia 14. Myxomycetes are [2006] 5. Virus has [1998] (a) saprobes or parasites, having mycelia, (a) DNA asexual reproduction by fragmentation. (b) RNA (b) slimy mass of multinucleate protoplasm, (c) both (a) and (b) having pseudopodia like structures for (d) either DNA or RNA engulfing food, reproduction through 6. Hormogonia are vegetative reproductive fragmentation or zoospores. structure of [1998] (c) prokaryotic organisms, cellular or acellular, saprobes or autotrophic, reproduce by (a) Spirogyra (b) Ulothrix binary fission. (c) Oscillatoria (d) Yeast (d) eukaryotic, single-celled or filamentous, 7. Which is a unicellular fungus ? [1998] saprobes or autotrophic, asexual (a) Yeast (b) E.coli reproduction by division of haploid (c) Nostoc (d) Albugo individuals, sexual reproduction by fusion 8. Bacterial cell wall is made up of [1999] of two cells or their nuclei. (a) xylan (b) chitin 15. Among rust, smut and mushroom all the three (c) cellulose (d) murein [2006] (a) are pathogens (b) are saprobes 9. Cell wall is absent in [2000] (c) bear ascocarps (d) bear basidiocarps (a) Amoeba (b) Chara 16. "Fairy rings" in lawns result from (c) yeast (d) E. coli outward,spreading circles of mycelia of 10. Which of the following is a fungus? [2000] mushrooms producing, at their periphery, fruiting bodies called [2009] (a) Nostoc (b) E. coli (a) ascocarps (b) basidiocarps (c) Yeast (d) Chara (c) sorocarps (d) pseudocarps IIT JEE NEET eBooks: www.crackjee.xyz EBD_7100

B-6 Topicwise AIIMS Solved Papers – BIOLOGY

17. Which one of the following statements is The correct combination is – correct? [2010] (a) A – II; B – I; C – IV; D – III (a) Prions are the smallest free-living cells. (b) A – II; B – IV; C – I; D – III (b) The cell wall of Mycoplasmas is made up (c) A – IV; B – I; C – II; D – III of amino sugars. (d) A – IV; B – III; C – II; D – I (c) Viroids consist of single-stranded RNA molecule. 22. Which of the following is correct about the slime (d) Rickettsiae lack cell wall. mould ? [2015] 18. Bacterial cell wall is composed of peptidoglycan, (i) Its thalloid body, Plasmodium has a complex of oligosaccharides and proteins. The pseudopodia for locomotion and engulfing oligosaccharide component consist of [2010] organic matter. (a) linear chain of alternating NAG and NAM (ii) During unfavourable conditions linked by D (1 – 4) linkage. Plasmodium differentiates and produces (b) linear chains of alternating NAG and NAM fruiting bodies, sporangium. linked by E (1 – 4) linkage. (iii) Spores posses no true cell wall. (c) linear chain of glucose linked by E(1 – 4) (iv) They are dispersed by air current. linkage. (v) Being extremely resistant, spores survive for (d) linear chain of glucose linked by D(1 – 4) many years. likage. (vi) Plasmodium can grow upto several feet. 19. Choose the correct names of the different bacteria Choose the answer from the following options according to their shapes. [2013] (a) (i),(ii), (iv), (v) and (vi) (b) (i),(ii) and (iii) (c) (i),(ii), (iii) and (vi) (d) (ii),(iii) and (vi) 23. (i) Green algae occur in fresh water, brackish (a) A – Cocci, B – Bacilli, C – Spirilla, D – Vibrio water, salt water. (b) A – Bacilli, B – Cocci, C – Spirilla, D – Vibrio (ii) Habitat of Brown algae-fresh water (rare), (c) A – Spirilla, B – Bacilli, C – Cocci, D – Vibrio brackish water, salt water (d) A – Spirilla, B – Vibrio, C – Cocci, D – Bacilli (iii) Some red algae are found in fresh water, 20. Which pair of the following belongs to mostly occur in salt water, some are in Basidiomycetes? [2013] brackish water. (a) Puffballs and Claviceps (iv) Most of the red algae are multicellular. (b) Peziza and Alternaria (v) Red alga may occur in both well lighted (c) Morchella and mushrooms regions close to water-surface and also at (d) Birds nest fungi and puffballs great depths in oceans where light 21. Match column-I with column-II and select the penetration is little. option . [2014] (vi) Cell wall of red algae consists of cellulose Column-I Column-II + agar. (Classes of fungi) (Common name) (vii) 2 – 8, equal and apical flagella in green A. Phycomycetes I. Sac fungi algae [2015] B. Ascomycetes II. Algal fungi (a) All are correct C. Basidiomycetes III. Fungi imperfecti (b) All are false D. Deuteromycetes IV. Club fungi (c) (i) and (vi) are correct (d) (ii), (iii) and (v) are correct IIT JEE NEET eBooks: www.crackjee.xyz

Biological Classification B-7

24. Fungi are filamentous with the exception of "X" 28. Assertion : Bacteria have three basic shapes, which is unicellular. Identify X. [2017] i.e., round , rod, spiral. (a) Yeast (b) Albugo Reason : Cocci and Bacilli may form clusters (c) Mucor (d) Lichen or chain of a definite length. [2000] 29. Assertion : Aflatoxins are produced by 25. Which of the following statements is not correct Aspergillus flavus. for viruses? [2017] Reason : These toxins are useful to mankind. (a) Viruses are obligate parasites. [2000] (b) Viruses can multiply only when they are 30. Assertion : Escherichia coli, Shigella sp. and inside the living cells. Salmonella sp. are all responsible for diarrhoeal (c) Viruses cannot pass through bacterial diseases. [2006] filters. Reason : Dehydration is common to all types of (d) Viruses are made up of protein and DNA or diarrhoeal diseases and adequate supply of RNA (never both DNA and RNA). fluids and electrolytes should be ensured. 26. Which of the following statements regarding 31. Assertion : Gram-negative bacteria do not retain cyanobacteria is incorrect? [2017] the stain when washed with alcohol. (a) It is also called blue green algae. Reason : The outer face of the outer membrane (b) They are chemosynthetic heterotrophs. of gram-negative bacteria contains (c) It forms blooms in polluted water bodies. lipopolysaccharides, a part of which is integrated into the membrane lipids. [2006] (d) It is unicellular, colonial or filamentous, marine or terrestrial bacteria. 32. Assertion : Neurospora is commonly called water mould. TYPE B : ASSERTION REASON QUESTIONS Reason : It belongs to basidomycetes fungi. Directions for (Qs. 27-32) : These questions consist [2007] of two statements, each printed as Assertion and Directions for (Qs. 33-34) : Each of these questions Reason. While answering these questions, you are contains an Assertion followed by Reason. Read them required to choose any one of the following five carefully and answer the question on the basis of following options. You have to select the one that responses. best describes the two statements. (a) If both Assertion and Reason are correct and (a) If both Assertion and Reason are correct and the Reason is a correct explanation of the Reason is the correct explanation of Assertion. Assertion. (b) If both Assertion and Reason are correct, but (b) If both Assertion and Reason are correct but Reason is not the correct explanation of Reason is not a correct explanation of the Assertion. Assertion. (c) If Assertion is correct but Reason is incorrect. (d) If both the Assertion and Reason are incorrect. (c) If the Assertion is correct but Reason is 33. Assertion : Chemosynthesis is an autotrophic incorrect. nutrition. (d) If both the Assertion and Reason are incorrect. Reason : Chemoautotrophs contain chlorophyll (e) If the Assertion is incorrect but the Reason is pigments. [2009] correct. 34. Assertion : TMV is a virus which causes mosaic 27. Assertion: Viruses are not considered organism. disease. Reason: Viruses are nucleoproteins and lack cell Reason : TMV has RNA as genetic material. organelle, etc. [1998] [2001, 2017] IIT JEE NEET eBooks: www.crackjee.xyz EBD_7100

B-8 Topicwise AIIMS Solved Papers – BIOLOGY

Type A : Multiple Choice Questions 8. (d) Bacterial cell wall consists of peptidoglycan/murein/mucopeptide which 1. (a) Lichens are the symbiotic association contains polymers of modified sugars between algae and fungi. Fungi provides (N-acetyl glucosamine and N- the raw material and algae manufacture the acetylmuramic acid) cross linked by short food. peptides. 2. (c) The virus which attacks bacteria is called 9. (a) Cell wall is a characteristic of bacteria, bacteriophage, e.g. T II phage, M13, I174 plants and fungi. etc. 10. (c) Heterotrophs were the first to be evolved and fungi in the plant kingdom are 3. (a) Citrus canker is caused by bacterium, heterotrophs. Yeast with its characteristic Xanthomonas citri. Bacterium is rod absence of chlorophyll and its vegetative shaped, motile, flagellate and gram propagation through budding and negative. saprophytic nature shows it is a fungus. 4. (a) Nostoc is a cyanobacterium. Cyanobacteria 11. (b) Powdery mildews are caused by fungi are gram (–) ve prokaryotes which perform belonging to the family Erysiphaceae oxygenic photosynthesis like plants. (class-Ascomycetes). Ascomycetes Cyanobacteria can be unicellular (e.g. spores are harmful and it makes the crop Spirulina), colonial. (e.g. Nostoc) or vulnerable to its toxin. The stem becomes filamentous (e.g. Oscillatoria) a powdery mass. 5. (d) Virus are obligate intracellular parasite 12. (b) Pullorum disease (Salmonellosis), a which can reproduce only by invading and contagious disease of young birds, is taking over other cells as they lack the caused by bacteria Salmonella pullorum. cellular machinery for self reproduction. It is usually transmitted by infected hens Virus has either DNA or RNA and never through their eggs. Its symptoms are loss both. of appetite and thirst, diarrhoea with white 6. (c) Hormogonia is a part of a filament of a faeces, drooping of wings etc. cyanobacterium that detaches and grows 13. (a) Viroids are the smallest known agents of by cell division into a new filament. infectious diseases comprising of small Cyanobacteria mostly multiply by asexual single stranded molecule of RNA. Viroids means like binary fission, small fragments lack capsid and have no proteins (hormogones), hormospores, akinetes, associated with them. endospores etc. Common cyanobacteria 14. (b) Myxomycetes are acellular plasmodial are-Nostoc, Anabaena, Oscillatoria etc. slime molds i.e. a group of 4 fungus which 7. (a) Yeast is the only member in kingdom fungi reproduces through fragmentation or which is unicellular. zoospore. Nostoc : Blue green algae 15. (d) Rust, smut and mushroom bear short lived E. coli : Bacterium reproductive bodies called basidiocarps. Albugo : Parasitic fungi IIT JEE NEET eBooks: www.crackjee.xyz

Biological Classification B-9

16. (b) Some basidiomycetes or club fungi (e.g., only replicate inside the cells of another Agaricus campestris) produce organism. Viruses are not considered as basidiocarps. These aerial structures also organisms as they have no independent called mushrooms. They occur in rings machinery. called 'fairy rings'. Basidiocarps of some 28. (e) Bacteria are microscopic organisms whose fungi are edible. single cells have neither a membrane 17. (c) Viroid is a small infective segment of enclosed nucleus nor other membrane nucleic acid, usually RNA. enclosed orgnelles like mitochondria and Prion is an infectious pathogenic agent chloroplast. Bacteria possess various devoid of nucleic acid and composed mainly forms and shapes like coccus (round), of proteins. bacillus (rod shaped), vibrio (comma Mycoplasmas lack a cell wall while shaped) and spirullum (spiral like cork Rickettsiae possess a cell wall. screw). 18. (b) The oligosaccharide component of bacterial 29. (c) Aflatoxin, produced by Aspergillus flavus, cell wall consists of linear chains of is a carcinogenic fungal toxin. The conidia alternating NAG and NAM linked by E not only causes allergies in human beings (1 – 4) linkage. but also causes death of birds. 19. (a) 20. (d) 30. (b) E. coli, Shigella sp. and Salmonella sp. 21. (a) A – II; B – I; C – IV; D – III causes diarrhoea. It is due to intake of 22. (a) contaminated water. 23. (a) All the given statements about algae are 31. (a) Gram negative bacteria contain correct. lipopolysaccharide in its membrane. 24. (a) Yeast being a unicellular fungus does not show filamentous nature. It is a microscopic 32. (d) Neurospora belongs to class ascomycetes fungus consisting of a single oval cell that in which ascospores are produced in sac reproduces by budding. like asci . Neurospora is commonly called 25. (c) Virus is a small infectious agent that as pink or red bread mould. It can be grown replicates only inside the living cells of easily in laboratory conditions so, used in other organisms. Viruses can infect all genetics experiments. types of life forms, from and plants 33. (b) Chemoautotrophs developed the ability to to microorganisms, including bacteria and synthesize organic molecules from archaea. Viruses can pass through bacterial inorganic raw materials. Chlorophyll proof filters as they are smaller than pigments are present in photoautotrophs. bacteria. The photo-autotrophs utilize the solar 26. (b) Cyanobacteria are photosynthetic energy absorbed by them in synthesizing (containing a blue photosynthetic and organic compounds. Certain chemo- chemosynthetic pigment) autotrophs. autotrophs include green sulphur bacteria. They are prokaryotic and represent the earliest known form of life on the Earth. 34. (b) In TMV, the chromosome consists of single stranded, linear RNA molecule coiled into Type B : Assertion Reason Questions a regular spiral. TMV causes mosaic 27. (a) Virus is a small infectious agent that can disease in tobacco and some other plants. IIT JEE NEET eBooks: www.crackjee.xyz EBD_7100

B-10 Topicwise AIIMS Solved Papers – BIOLOGY

Chapter 3 Plant Kingdom

TYPE A : MULTIPLE CHOICE QUESTIONS 9. In Dryopteris, the opening mechanism of sporangium is effectively operated by [1999] 1. Meiosis in Dryopteris takes place during [1997] (a) stalk (b) stomium (a) spore formation (c) annulus (d) peristome (b) gametic formation 10. 13 celled male gametophyte of Selaginella has (c) spore germination [2000] (d) zygote formation (a) 12 cells of antheridium + 1 prothallial cell 2. Sometimes a ladder like structure in Spirogyra (b) 10 cells of antheridium + 3 prothallial cell is formed due to [1998] (c) 9 cells of antheridium + 4 prothallial cell (a) lateral conjugation (d) 8 cells of antheridium + 6 prothallial cell (b) asexual reproduction 11. Which of the following is found in algal zone of Cycas coralloid roots? [2000] (c) scalariform conjugation (a) Blue green algae (b) Red algae (d) direct conjugation (c) Diatoms (d) Brown algae 3. In which group of the following would you place 12. Sex organs in Funaria develop [2001] the plants having vascular tissue and lacking (a) in protonema. seeds? [1998] (b) outside capsule. (a) Algae (b) Fungi (c) in the axil of leaf. (c) Bryophytes (d) Pteridophytes (d) at the tip of gametophore. 4. Largest ovule in plant kingdom are found in 13. In which portion of Cycas diploxylic vascular (a) Pinus (b) Cycas [1998] bundles are found ? [2001] (c) Thuja (d) Gnetum (a) Root (b) Stem 5. Plant material, which is used in culture medium (c) Leaflet (d) Rachis and leaflet is obtained from [1998] 14. Funaria gametophyte is [2001] (a) Cycas (b) coconut milk (a) dioecious (c) Pinus (d) mango (b) heteroecious 6. Which is a saprophytic angiosperm? [1998] (c) autoecious (a) Cuscuta (b) Neottia (d) monoecious and autoecious (c) Agaricus (d) Yeast 15. Sometimes, the fern plant arises from fern 7. Bryophytes do not possess [1999] prothallus without fertilization. This is an example (a) vascular tissue of [2001] (a) apospory (b) apogamy (b) gametophyte (c) parthenocarpy (d) gametogenesis (c) alternation of generation 16. Zygospore of Spirogyra at the time of meiosis (d) spores is divided into 4 nuclei. How many nuclei 8. The heart shaped fern prothallus is [1999] degenerate out of these four ? [2001] (a) gametophyte (b) sporophyte (a) One (b) Two (c) saprophyte (d) gamete (c) Three (d) Four IIT JEE NEET eBooks: www.crackjee.xyz

Plant Kingdom B-11

17. Cycas is [2001] 26. In the following table identify the correct (a) monoecious (b) bisexual matching of the crop, its disease and the (c) dioecious (d) hermaphrodite corresponding pathogen [2006] 18. The drug bellodona is obtained from [2001] Crop Disease Pathogen (a) Atropa (b) Opium (a) Citrus Canker Pseudomonas (c) Rauwolfia (d) Solanum rubrilineans 19. Algae are useful because they [2002] (b) Potato Late blight Fusarium udum (a) purify the atmosphere (c) Brinjal Root-knot Meloidogyne (b) are large in number incognita (c) are used in fermentation (d) Pigeon pea Seed gall Phytophthora (d) are used to study respiration infestans 20. The plant body of Funaria is [2002] 27. Plants of which one of the following groups of (a) sporophyte genera are pollinated by the same agency? (b) gametophyte (a) Triticum, Cocos, Mangifera [2006] (c) predominantly sporophyte with (b) Ficus, Kigelia, Casuarina independent gametophyte (c) Salvia, Morus, Euphorbia (d) predominantly gametophyte with (d) Bombax, Butea, Bauhinia dependent sporophyte. 28. Which of the following match is correct? [2007] 21. Elaters help in dispersal of spores of [2002] Disease Pathogen (a) Riccia (b) Marchantia (a) Wilt disease Synchytrium (c) Dryopteris (d) Funaria (b) Citrus canker Xanthomonas 22. The nature of megasporophyll of Cycas is similar (c) Red rot of sugarane Ustilago to [2002] (d) Powdery mildew Fusarium (a) stamen (b) carpel 29. People recovering from long illness are often (c) sepal (d) petal advised to include the alga Spirulina in their 23. Which of the following helps in respiration of diet because it [2003, 2008] lichens? [2002] (a) makes the food easy to digest. (a) Isidia (b) Soredia (b) is rich in proteins. (c) Cyphella (d) Cephalodia (c) has antibiotic properties. 24. In Ulothrix, meiosis takes place in [2004] (d) restores the intestinal microflora. (a) cells of the filament 30. Botanical name of Sanjeevani is [2009] (b) holdfast (a) Selaginella chrysocaulos (c) zygote (b) Selaginella bryopteris (d) zoospores (c) Selaginella chrysorhizos 25. Mosses and ferns are found in moist and shady (d) None of the above places because both [2004] 31. In Pinus, the third tier of embryonal cells from (a) require presence of water for fertilization. below is known as [2009] (b) do not need sunlight for photosynthesis. (a) rosette tier (b) suspensor tier (c) embryonal tier (d) free-nuclear tier (c) depend for their nutrition on micro- organisms which can survive only at low 32. Which of the following is found in algal zone of Cycas coralloid roots? [2007, 2011] temperature. (a) Blue green algae (b) Red algae (d) cannot compete with sun-loving plants. (c) Diatoms (d) Brown algae IIT JEE NEET eBooks: www.crackjee.xyz EBD_7100

B-12 Topicwise AIIMS Solved Papers – BIOLOGY 33. Which among the following is a rootless (a) (i) is correct, but (ii) and (iii) are incorrect plant? [2007, 2011] (b) (i) and (ii) are correct, but (iii) is incorrect (a) Nymphaea (b) Sagittaria (c) (i) and (iii) are correct, but (ii) is incorrect (c) Ceratophyllum (d) Vallisneria (d) (iii) is correct, but (i) and (ii) are incorrect 34. Sporocarp is a reproductive structure of (a) Some algae [2012] TYPE B : ASSERTION REASON QUESTIONS (b) Some aquatic ferns having sori Directions for (Qs. 39-43) : These questions consist (c) Angiosperms having spores of two statements, each printed as Assertion and (d) Bryophytes Reason. While answering these questions, you are 35. Coralloid roots of Cycas is distinguished from required to choose any one of the following five angiosperm roots by [2013] responses. (a) absence of pith (b) having xylem tissue (a) If both Assertion and Reason are correct and (c) absence of algal zone the Reason is a correct explanation of the (d) presence of algal zone Assertion. 36. If the cells of root in wheat plant have 42 (b) If both Assertion and Reason are correct but chromosomes, then the no. of chromosome in Reason is not a correct explanation of the the cell of pollen grain is [2014] Assertion. (a) 14 (b) 21 (c) If the Assertion is correct but Reason is (c) 28 (d) 42 37. Match the following incorrect. Column-I Column-II (d) If both the Assertion and Reason are incorrect. (Classes) (Examples) (e) If the Assertion is incorrect but the Reason is A. Psilotopsida I. Dryopteris, Pteris, correct. Adiantum B. Lycopsida II. Equisetum 39. Assertion : Mosses are evolved from algae. C. Sphenopsida III. Selaginella Reason : Protonema of mosses is similar to some D. Pteropsida IV. Lycopodium green algae. [2001] V. Psilotum 40. Assertion : Coconut tree is distributed in coastal (2016) areas over a large part of the world. (a) A – V; B – III; C – II; D – I Reason : Coconut fruit can float and get (b) A – I; B – II; C – III; D – IV dispersed over thousands of kilometers before (c) A – IV; B – III; C – II; D – I (d) A – III; B – V; C – I; D – II losing viability. [2004] 38. Consider the following statements regarding the 41. Assertion : The fungi are widespread in major pigments and stored food in the different distribution and they even live on or inside other groups of algae and choose the correct option plants and animals. (i) In chlorophyceae, the stored food material Reason : Fungi are able to grow anywhere on is starch and the major pigments are land, water or on other organisms because they chlorophyll-a and d. have a variety of pigments, including chlorophyll, (ii) In phaeophyceae, laminarian is the stored food and major pigments are chlorophyll-a carotenoids, fucoxanthin and phycoerythrin. and b. [2005] (iii) In rhodophyceae, floridean starch is the 42. Assertion : Algae and fungi are classified as stored food and the major pigments are thallophytes. chlorophyll-a, d and phycoerythrin. [2017] Reason : They both are autotrophs. [2007] IIT JEE NEET eBooks: www.crackjee.xyz

Plant Kingdom B-13

43. Assertion : Conifer trees produce a large 44. Assertion : Red algae contributes in producing quantity of wind borne pollen grains. coral reefs. [2004, 2011] Reason : The pollen grains have wings. [2007] Reason : Some red algae secrete and deposit Directions for (Qs. 44-46) : Each of these questions calcium carbonate over their walls. contains an Assertion followed by Reason. Read them 45. Assertion: The peristome is a fringe of teeth- carefully and answer the question on the basis of like projections found at the mouth of the following options. You have to select the one that capsule. [2004, 2011] best describes the two statements. Reason: It may be of two types nematodontous (a) If both Assertion and Reason are correct and and orthodontus. Reason is the correct explanation of Assertion. 46. Assertion : Members of phaeophyceae vary in (b) If both Assertion and Reason are correct, but colour from olive green to various shades of Reason is not the correct explanation of brown Assertion. (c) If Assertion is correct but Reason is incorrect. Reason : Phaeophceae possess chlorophyll a, (d) If both the Assertion and Reason are incorrect. c, carotenoids and xanthophysis. [2016] IIT JEE NEET eBooks: www.crackjee.xyz EBD_7100

B-14 Topicwise AIIMS Solved Papers – BIOLOGY

Type A : Multiple Choice Questions The sperms of Cycas are also the largest (300 µm) in the biological world. 1. (a) In sporangium, 5. (b) Coconut milk is rich in compounds Meiosis chemically similar to cytokinin and promote Spore mother cell (2n) o callus growth by inducing cell division. dispersed by spores (n) o germination 6. (b) Neottia grows on soil rich humus. It has an underground stem with a cluster of roots.  o Prothallus(n)  o gamete fusion The roots are associated with endotrophic mycorrhiza. The fungus absorbs its food  o zygote  o Main plant body from humus and the cortical cells of the (2n). roots of Neottia absorb food from humus. 2. (c) Conjugation is exchange of genetic material Neottia is actually a parasite on fungus. i.e. mode of sexual reproduction in lower 7. (a) Bryophytes are terrestrial plants found in organisms. Conjugation occurs by two moist places and shady localities .They are methods– rightly called the amphibians of the plant (i) Scalariform conjugation kingdom .Since fertilization does not occur It takes place during the night and without water. Being in moist places, water between the recently divided cells of and mineral salts are being absorbed by the conjugating participating rhizoids that have the main function of filaments. Opposite cells of two fixing the plant. Absorption also occurs due filaments develop conjugation tube to diffusion. Moreover, these plants are by lying side by side and parallel to very small. Hence, Bryophytes do not each other. The conjugation tube possess the vascular tissue. between two filament looks like a 8. (a) The heart shaped prothallus is the ladder, through which gamete from characteristic feature of ferns and mainly one gametangia passes through to belong to polypodiaceae.They show fuse with the passive gamete of polarity and dorsoventrality. The another filament. prothallus contains both the antheridia and (ii) Lateral conjugation archegonia which are embedded in the prothallus. Since it forms the male and Two adjacent cells of same filament female gametes the prothallus is function as male and female cells. It is considered as the gametophyte. commonly reported in S. affinis and 9. (b) When the capsule of the sporangium in S. fenuissima. Dryopteris matures, about 4 lower median 3. (d) Pteridophytes are seedless vascular plants. cells of the jacket stretch tangentially. The They are also called vascular cryptogams two median ones identify the place from as they possess xylem and phloem. where the capsule opens. This is called as 4. (b) Largest ovule in plant kingdom is found in stomium. The other cells of the jacket Cycas. Female cone consists of develop a thickening . At maturity, the megasporophylls. Each megasporophyll indusium dries exposing the sorus. Due to bears 2-12 reddish ovules in the middle loss of water in the annulus, the upper walls fertile part. Ovules of Cycas are the largest, contract and inner ones straighten, the 6-7 cm in diameter. annulus coils. This feature exerts pressure IIT JEE NEET eBooks: www.crackjee.xyz

Plant Kingdom B-15

on the wall resulting in the breaking of the to germination, the diploid zygospore capsule, between the cells of the stomium, nucleus undergoes meiosis and forms four thereby, releasing the spores. nuclei. Three of the four nuclei degenerate 10. (a) The structure and development of male since they are smaller in size. The last cell gametophyte was described by Slagg is bigger and grows using the reserve food (1932). The 13 celled male gametophyte has substances. 12 antheridial cells and one prothallial cell 17. (c) Cycas is dioecious in nature since it as a result of segmentation. This is to produces micro and megaspores in the increase the number of biflagellate micro and megasporangia. The micro and antherozoids. megasporangia are born on 11. (a) The algal zone consists of blue green algae microsporophyll and megasporophyll in the coralloid roots. The relationship is a respectively. mutual relationship. The algae get the 18. (a) Atropa bellandona produces the drug dwelling place, while Cycas can use the belladona. It is an alkaloid which is atropine food produced by the algae. and hyoscyamine. The leaves and root 12. (d) Sex organs in Funaria develop at the tip of contain the alkaloid. It is used to cure and gametophore since the plant itself relieve pain. constitutes the gametophytic phase. After 19. (a) Algae is a collective term for all those fertilization develops the capsule from chlorophyll bearing organisms which are which protonema develops from the spores thalloid. So, during photosynthesis by that dehisces from the capsule. The capsule taking in CO2 and giving out O2, they purify constitutes the sporophytic phase. The the atmosphere. plant shows an alternation of generation, 20. (d) In the life cycle of Funaria, two distinct between the sporophytic and individuals occur. One of these is haploid gametophytic phase. and is independent leafy plant. The other 13. (d) The rachis and leaflet have diploxylic is diploid, leafless sporogonium. It is vascular bundles. At the region of the stem partially dependent on the leafy cortex, the leaf trace has only centrifugal gametophyte for its nutrition. xylem, hence, it is endarch. These traces 21. (b) The diploid bispiral elaters are hygroscopic enter the leaf as the rachis separates, the and they help in the dispersal of spores. centripetal xylem appears and it is in the exarch condition. The leaflet has a single 22. (b) Megasporophyll of Cycas bears ovules midrib bundle and is in the diploxylic similar to carpel. condition. 23. (c) Cyphella are aerating organs in the form of 14. (d) Funaria is both monoecious and organised breaks, which occur in the lower autoecious. The antheridium (male sex cortex and appear as concave circular organ) are born on different heads of the depression where white medulla is exposed. same plant. Hence, it is both autoecious 24. (c) Ulothrix being an haploid organism, shows and monoecious. zygotic meiosis. Zygote is the only diploid 15. (b) Apogamy is the formation of sporophyte structure which undergoes meiosis to form from a gametophytic cell other than egg four cells. Each develops into Ulothrix. without fertilization. The prothallus is only 25. (a) Mosses and ferns are called the amphibians a part of gametophytic cell. Hence, the of plants which are land plants but require development is apogamy. a film of water for antherozoids to reach 16. (c) The united protoplasmic mass of two archegonium. gametes is called zygote or zygospore. Prior IIT JEE NEET eBooks: www.crackjee.xyz EBD_7100

B-16 Topicwise AIIMS Solved Papers – BIOLOGY

26. (c) Root knot/root gall in brinjal is caused by 32. (a) The algal zone consists of blue green algae nematode Meloidogyne incognita. in the coralloid roots. The relationship is a 27. (c) Salvia, Morus, Euphorbia are pollinated mutual relationship. The algae get the by insects (entomophily). dwelling place, while cycas can use the 28. (b) Citrus canker is the disease caused by an food produced by the algae. aerobic, rod shaped bacterium, 33. (c) Nymphaea, Sagittaria, Ceratophyllum Xanthomonas citri. and Vallisneria are hydrophytes which 29. (b) Some dieters say that blue green algae grow in water. Nymphaea, Sagittaria and helps satisfy appetite and supplies good Vallisnaria are rooted hydrophytes while Ceratophyllum is a submerged floating nutrition while dieting. Some dieters say hydrophyte. It remains completely under that blue-green algae Spirulina, a well- water but is not rooted in mud. known example, is a group of 1,500 species Ceratophyllum lacks roots even in of microscopic aquatic plants. The two embryonic stage. most common species used for human consumption are Spirulina maxima and 34. (b) Sporocarp is a reproductive structure of Spirulina platensis. Spirulina is particularly some aquatic ferns like Marsilea. rich in protein and also contains Sporocarps are functionally and carotenoids, vitamins, minerals, and developmentally modified leaf which bears essential fatty acids. Blue-green algae several sori. Each sori consists of (Spirulina) is a concentrated source of sporangia of two types -megasporangia nutrients compared to most foods, but it is producing simple large female megaspore expensive compared to other supplemental and microsporangia producing many small male microspore. sources of protein, vitamins, and minerals. 30. (b) Selaginella bryopteris (L.), popularly 35. (d) In Cycas, coralloid roots are bluish green known as Sanjeevani, is a plant with in colour and have blue-green algae in their medicinal properties. Sanjeevani grows on cortex. This character distinguishes them the hills of tropical areas, particularly the from angiosperms. Arawali mountain terrains from east to west 36. (b) Number of chromosome in root cells of in India. Traditionally plants have been used wheat plant is to: (i) relief from heat stroke and the burning 2n = 42 sensation during urination; (ii) restorating menstrual irregularities to normal and applied 42 n = = 21 topically to pregnant women, aids easy 2 delivery; and (iii) for treating jaundice. The number of chromosome in the cell of 31. (a) In Pinus, the third tier of embryonal cells pollen grain is 21 because pollen grains are from below is known as rosette tier. The haploid (n) in nature. four lower most cells which are far from the 37. (a) micropolar end are called the embryonal tier. These develop into an embryo. The cells 38. (d) In chlorophyceae, the stored food material above the embryonal tier called the is starch and the major pigments are suspensor tier and the third tier from below chlorophyll- a and b. In phaeophyceae, laminarian is the stored food and major is called the rosette tier. The free nuclei tier pigments are chlorophyll a, c and are formed of 4 nuclei. It is the fourth upper most tier. fucoxanthin. IIT JEE NEET eBooks: www.crackjee.xyz

Plant Kingdom B-17 Type B : Assertion Reason Questions 44. (a) Red algae generally grow attached to rocky stones. Some deep water red algae are 39. (a) Mosses that belong to the bryophytes calcareous and build up hard stony thalii have evolved from algae. The fact that responsible for the production of lime protonema has a thallus like body shows stones and coral reefs. that mosses have evolved from algae. 45. (b) The peristome teeth are present at the 40. (a) Coconut tree is distributed in coastal areas mouth of capsule. The teeth may be solid since it floats on saline water because salt is more denser than freshwater, so it floats. cellular tissue or composed only of the thickened portions of the cell walls of 41. (c) Fungi lack pigments and are therefore, adjacent cells. When the teeth of peristome heterotrophic. The mode of nutrition is are solid structures composed of bundles either parasitic/saprophytic. of dead cells, it is termed nematodontous 42. (c) Thallophyte includes plants in which body peristome are solid structures composed is not differentiated into root, stem and of bundles of dead cells, it is termed leaves. Algae and fungi have thallose plant nematodyntous peristome. It is found in body. Algae are autotrophs i.e. they can polytrichum, Pogonatum and Tetraphis. If prepare their own food by photosynthesis. peristome composed of thin, membranous, while fungi are heterotrophs. transversely barred teeth, each tooth is 43. (a) In pinus, a conifer tree, the microspores are made up of the thickened portions of the produced by microsporogenesis in cell walls of adjacent cells. Such a peristome microsporangium. Each microsporangium is called orthodontous. has an inner nourishing layer known as tapetum. A large number of dusty and two 46. (a) Phaeophyceae possesses chlorophyll a, c, winged microspores are present. On carotenoids and xanthophylls. Members of maturity the microsporangium wall bursts phaeophyceae show variations in colour and microspores are released in the air which from olive green to different shades of is called "shower of sulphur."They are brown depending upon the amount of dispersed by wind due to presence of wings. xanthophyll pigments. IIT JEE NEET eBooks: www.crackjee.xyz EBD_7100

B-18 Topicwise AIIMS Solved Papers – BIOLOGY 4 Kingdom

TYPE A : MULTIPLE CHOICE QUESTIONS 10. The long bones are hollow and connected by air passage. They are characteristic of [1998] 1. Pouched are known as [1997] (a) aves (b) mammalia (a) prototherians (b) metatherians (c) reptilia (d) sponges (c) eutherians (d) therians 11. Aristotle’s lantern is found in [1998] 2. Sponges capture food with the help of [1997] (a) Asteroidea (b) Echinoidea (a) pinacocytes (b) choanocytes (c) Holothuroidea (d) Ophiuroidea (c) trophocytes (d) theocytes 12. Sympathetic nerves in mammals arise from 3. Chloragogen cells of earthworm are similar to (a) sacral region [1998] which organ of vertebrates ? [1997] (a) Liver (b) Lung (b) cervical region (c) Kidney (d) Spleen (c) 3rd, 7th, 9th, 10th cranial nerves 4. Haemocoel is found in [1997] (d) thoracico-lumbar region (a) Hydra and Aurelia 13. Which of the following statement is true (b) Taenia and Ascaris regarding corals ? [1999] (c) Cockroach and Pila (a) Form branch colonies. (d) Herdmania and Balanoglossus (b) Solitary or colonial. 5. Aquatic reptiles are [1997] (c) Grow as massive bodies. (a) ureotelic (d) All of the above (b) ureotelic on land 14. Water current in Leucosolenia is produced (c) ammonotelic by [1999] (d) uricotelic in water (a) pinacocytes (b) choanocytes 6. Larva of house-fly lacks [1997] (c) archaeocytes (d) collencytes (a) eyes (b) wings 15. Which is the example of platyhelminthes ?[1999] (c) spiracles (d) none of these (a) Entamoeba (b) Plasmodium 7. Single filament of Nostoc without mucilage (c) Wuchereria (d) Schistosoma sheath is called as [1998] 16. Paired spermathecae occur in Pheretima in (a) colony (b) mycelium which of the following segments ? [1999] (c) trichome (d) hyphae (a) 4, 5, 6, 7 (b) 5, 6, 7, 8 8. Organisms, attached to substratum generally possess [1998] (c) 6, 7, 8, 9 (d) 3, 4, 5, 6 (a) radial symmetry 17. Weberian ossicles are found in [1999] (b) asymmetrical body (a) frog (b) snakes (c) single opening of digestive canal (c) fishes (d) birds (d) cilia to create water current 18. The vertebrae in birds are mostly [1999] 9. Hydra recognizes its prey by [1998] (a) procoelous (b) heterocoelous (a) nematocyst (c) amphicoelous (d) acoelous (b) special organ 19. Basket star belongs to class [1999] (c) chemical stimulus (a) Ophiuroidea (b) Echinoidea (d) mechanical stimulus (c) Crinoidea (d) Asteroidea IIT JEE NEET eBooks: www.crackjee.xyz

Animal Kingdom B-19 20. The egg case in female cockroach is formed by 33. What is left, when bath sponges dries up?[2002] secretion of [2000] (a) Spicules (b) Hold fast (a) collaterial gland (b) mushroom gland (c) Spongin fibres (d) Tentacles (c) conglobate gland (d) prothoracic gland 34. Hydra receives impulses and stimuli 21. Power of regeneration in sponges is due to through [2002] [2000] (a) nerve cells (b) sensory cells (a) theocytes (b) archaeocytes (c) neuron cell (d) nematocysts (c) amoebocytes (d) sclerocytes 35. Which of the following are uricotelic animals? 22. The poisonous fluid present in nematocyst of (a) Rohu, Frog [2002] Hydra is [2000] (b) Camel, Frog (c) Lizard, Crow (a) toxin (b) venom (d) Eagles, Earthworm (c) hematin (d) hypnotoxin 36. In Entamoeba histolytica, the presence of 23. Life cycle of Taenia is [2000] chromatid bodies is characteristic of [2002] (a) monogenetic (b) digenetic (a) precystic stage (c) polygenetic (d) hexogenetic (b) trophozoite stage 24. Pigment haemocyanin is found in [2000] (c) mature binucleate stage (a) chordata (b) annelida (d) both (a) and (b) (c) porifera (d) mollusca 37. Just as Xenopsylla is a Yersenia pestis, so is 25. Antedon belongs to which of the following [2003] class? [2000] (a) Glossina palpalis to Wuchereria bancrofti (a) Asteroidea (b) Ophiuroidea (b) Culex to Plasmodium falciparum (c) Crinoidea (d) Echinoidea (c) Homo sapiens to Taenia solium 26. Scales in Chondrichthyes are [2000] (d) Phlebotomus to Leishmania donovani (a) placoid (b) ctenoid 38. Which of the following feature is common to (c) cycloid (d) all of these leech, cockroach and scorpion? [2004] 27. Which of the following snake is not poisonous? (a) Nephridia (b) Ventral nerve cord (a) Naja- Naja (b) Python [2000] (c) Cephalization (d) Antennae (c) Hydrophis (d) Bungarus 39. Which one of the following statements is correct 28. Birds are [2000] with respect to salt water balance inside the body (a) cold blooded (b) homeothermal of living organisms? [2005] (c) poikilothermal (d) homeopoiesis (a) When water is not available camels do not 29. Which of the following substances is at its produce urine but store urea in tissues. lowest level in fish food ? [2000] (b) Salmon fish excretes lot of stored salt through gill membrane in fresh water. (a) Actin (b) Myosin (c) Paramecium discharges concentrated salt (c) Cholesterol (d) Tissue fluid solution by contractile vacuoles. 30. How many ovaries are found in birds ? [2000] (d) The body fluids of fresh water animals are (a) One (b) Two generally hypotonic to surrounding water. (c) Three (d) Many 40. Which one of the following groups of 31. Gemmule formation in sponges are useful in structures/organs have similar function? (a) asexual reproduction [2001] [2005] (b) sexual reproduction (a) Typhlosole in earthworm, intestinal villi in (c) parthenogenesis rat and contractile vacuole in Amoeba. (d) parthenocarpy (b) Nephridia in earthworm, Malpighian 32. The places of first, second and third moulting of tubules in cockroach and urinary tubules Ascaris larva are [2002] in rat. (a) soil, alveoli, lung (c) Antennae of cockroach, tympanum of frog (b) liver, soil, stomach and clitellum of earthworm. (c) soil, lung, liver (d) Incisors of rat, gizzard (proventriculus) of (d) soil, intestine, lung cockroach and tube feet of starfish. IIT JEE NEET eBooks: www.crackjee.xyz EBD_7100

B-20 Topicwise AIIMS Solved Papers – BIOLOGY

41. Which one of the following features is common (c) Morphallaxis - Healing up of a wound in silverfish, scorpion, dragonfly and prawn? in the skin. [2005] (d) Epimorphosis - Regeneration of (a) Three pairs of legs and segmented body. crushed and filtered (b) Chitinous cuticle and two pairs of antennae. out pieces of a (c) Jointed appendages and chitinous Planaria into as many exoskeleton. new Planarians. (d) Cephalothorax and trachae. 47. Which statement best explains why 42. Which one of the following animals is correctly invertebrates regenerate lost tissue more readily matched with its one characteristic and the than most vertebrates do? [2009] taxon? [2006] (a) Invertebrates contain specialized cells that Animal Characteristic Taxon produce the hormones necessary for this (a) Millipede Ventral nerve Arachnida process. cord (b) Invertebrate cells exhibit a higher degree (b) Duckbill Oviparous Mammalia of uncontrolled cell division than vertebrate cells do. platypus (c) Invertebrate animals reproduce asexually, (c) Silverfish Pectoral & Chordata but vertebrate animals reproduce sexually. Pelvic fins (d) Invertebrate animals have more (d) Sea anemone Triploblastic Cnidaria undifferentiated cells than vertebrate 43. All mammals without any exception are animals have. characterized by [2006] 48. Which of the following is correctly matched? (a) viviparity and biconcave red blood cells. (a) Human -Renal portal system [2009] (b) extra-abdominal testes and a four- (b) Earthworm - Closed circulatory system chambered heart. (c) Cockroach - Nephridia (c) heterodont teeth and 12 pairs of cranial (d) None of the above nerves. 49. Which set of terms would most likely be used in (d) a muscular diaphragm and milk producing a description of the nervous system of glands. ? [2009] 44. Which of the following does not come under the class mammals? [2007] (a) Brain, dorsal nerve cord, highly developed (a) Flying fox (b) Hedgehog receptors (c) Manatee (d) Lamprey (b) Brain, fused ganglia, ventral nerve cord 45. Which one feature is common to leech, (c) No brain, fused ganglia, tympana cockroach and scorpion? [2008] (d) No brain, nerve net, modified neurons (a) Nephridia (b) Ventral nerve cord 50. Which one of the following statements is not (c) Cephalization (d) Antennae correct? [2010] 46. Based on cellular mechanisms there are two major (a) All echinoderms are viviparous. types of regeneration found in the animals. (b) Roundworm has no circulatory system. Which one of the following is the correct example of the type mentioned? [2008] (c) In bony fishes, swim bladder is usually (a) Morphallaxis - Regeneration of two present. transversely cut equal (d) In cartilaginous fishes, fertilization is pieces of a Hydra into internal. two small hydras 51. Ticks and mites are actually [2010] (b) Epimorphosis - Replacement of old and (a) arachnids (b) crustaceans dead erythrocytes by (c) insects (d) myriapods the new ones. IIT JEE NEET eBooks: www.crackjee.xyz

Animal Kingdom B-21

52. "Portuguese man of war" is [2012] 55. Cockroaches are brown or black bodied animals (a) Soldier of world war I that are included in class ______of phylum ______. [2014] (b) Portuguese soldier (a) Reptilia; Annelida (c) A sponge (b) Insecta; Arthropoda (d) A polymorphic, colonial, coelenterata (c) Insecta; Annelida 53. Which are exclusively viviparous ? [2012] (d) Reptilia; Arthropoda (a) Bony fishes Ectoderm 56. Mesoglea (b) Cartilagenous fishes Endoderm (c) Sharks (d) Whales 54. Given are the four matches of phyla with their characteristic cells [2013]

A. Coelenterata - Nematocytes Mesoderm B. Porifera - Choanocytes A B C. Ctenophora - Solenocytes The above diagram shows the germs layer. The animals having structures shown in the figures A D. Platyhelminthes - Nephrocytes and B are respectively called [2014] Mark the option that has both correct matches (a) Diploblastic, Triploblastic (a) A and B (b) B and C (b) Triploblastic, Diploblastic (c) C and D (d) B and D (c) Diploblastic, Diploblastic (d) Triploblastic, Triploblastic

57. Given below are pie diagrams A, B and C related to proportionate number of species of major taxa of invertebrates, vertebrates and plants respectively. Critically study and fill in the blanks I, II, III and IV

Other animal groups Mosses Mammals Ferns and Crustaceans Fishes Birds allies I III Insects IV II Reptiles Lichens

A B Algae C Invertebrates Vertebrates Plant

[2015] (a) I- Molluscs, II-Amphibians, III-Fungi, IV-Angiosperms (b) I- Molluscs, II-Amphibians, III-Angiosperms, IV-Fungi (c) I- Hexapoda, II-Amphibians, III-Fungi, IV-Angiosperms (d) I- Turtles, II-Amphibians, III-Fungi, IV-Angiosperms IIT JEE NEET eBooks: www.crackjee.xyz EBD_7100

B-22 Topicwise AIIMS Solved Papers – BIOLOGY

58. Which of the following statements are true/false? TYPE B : ASSERTION REASON QUESTIONS (i) In Torpedo, the electric organs are capable of generating strong electric shock to Directions for (Qs. 61-69) : These questions consist paralyze the prey. of two statements, each printed as Assertion and (ii) Bony fishes use pectoral, pelvic, dorsal anal Reason. While answering these questions, you are and caudal fins in swimming. required to choose any one of the following five (iii) Amphibian skin is moist and has thick scales. responses. (iv) Birds are poikilothermic animals. (a) If both Assertion and Reason are correct and (v) The most unique mammalian characteristic the Reason is a correct explanation of the is the presence of milk producing mammary Assertion. glands by which the young ones are nourished. [2015] (b) If both Assertion and Reason are correct but (a) (i), (ii) and (iii) are true; (iv), (v) are false Reason is not a correct explanation of the (b) (i), (ii) and (v) are true; (iii) and (iv) are false Assertion. (c) (i), (iv) and (v) are true; (ii) and (iii) are false (c) If the Assertion is correct but Reason is (d) (i), (ii) and (iv) are false; (iii) and (v) are true 59. Column-I contains organisms and column-II incorrect. contains their excretory structures. Choose the (d) If both the Assertion and Reason are incorrect. correct match form the options given below. (e) If the Assertion is incorrect but the Reason is Column- I Column -II correct. (Organism) (Excretory structures) 61. Assertion : Cold blooded animals do not have A. Cockroach I. Nephridia fat layer. B. Cat fish II. Malpighian Reason : Cold blooded animals use their fat for tubules metabolic process during hibernation. [1997] C. Earthworm III. Kidneys 62. Assertion : Birds have one ovary. D. Balanoglossus IV. Flame cells E. Flatworm V. Proboscisgland Reason : This reduces the body weight for flight. [2017] [1999] (a) A – I; B – III; C – II; D – IV; E – V 63. Assertion: Lateral line system is found in fishes (b) A – III; B – I; C – II; D – V; E – IV and aquatic larval amphibians. (c) A – II; B – I; C – III; D – V; E – IV Reason: Lateral line system has receptor of (d) A – II; B – III; C – I; D – V; E – IV sensory cells derived from ectoderm. [2002] 60. In which one of the following the genus name, 64. Assertion : Bats and whales are classified as its two characters and phylum are not correctly matched ? mammals. Reason : Bats and whales have four-chambered Genus Two characters Phylum heart. [2003] name 65. Assertion : Holoblastic cleavage with almost (i) Body segmented (a) Pila Mollusca equal sized blastomeres is a characteristic of Mouth with radula placental animals. (ii) Spiny skinned Reason : Eggs of most mammals, including (b) Asterias Echinodermata Water vascular system humans, are of centrolecithal type. [2003] (iii) Pore bearing 66. Assertion : All birds, except the ones like koel (c) Sycon Porifera Canal system (cuckoo) build nests for retiring and taking rest during night time (day time for nocturnal). (iv) Jointed appendages (d) Periplaneta Arthropoda Reason : Koel lays its eggs in the nests of tailor Chitinous exoskeleton bird. [2003] [2017] IIT JEE NEET eBooks: www.crackjee.xyz

Animal Kingdom B-23

67. Assertion : The honey bee queen copulates 71. Assertion : Acraniata is a group of organisms only ones in her life time. which do not have distinct cranium. Reason : The honey bee queen can lay fertilized Reason : It includes small marine forms without as well as unfertilized eggs. [2004] head. [1997, 2012] 68. Assertion : Torsion can be seen in ctendium. Reason : Ctenidium acts as the respiratory 72. Assertion : Sponges belong to Porifera. organ. [2007] Reason : Sponges have canal system. 69. Assertion : Tapeworm, roundworm and pinworm are endoparasites of human intestine. [1998, 2014] Reason : Improperly cooked food is the source 73. Assertion : The duck-billed Platypus and the of intestinal infections. [2004, 2008] spiny ant-eater, both are egg-laying animals yet Directions for (Qs. 70-76) : Each of these questions they are grouped under mammals. contains an Assertion followed by Reason. Read them Reason : Both of them have seven cervical carefully and answer the question on the basis of vertebrae and 12 pairs of cranial nerves. following options. You have to select the one that best describes the two statements. [2005, 2015] 74. Assertion : Typhlosole increases the effective (a) If both Assertion and Reason are correct and area of absorption in the intestine. Reason is the correct explanation of Assertion. (b) If both Assertion and Reason are correct, but Reason : Typhlosole, present in the intestine, is Reason is not the correct explanation of the characteristic feature of cockroach. Assertion. [2016] (c) If Assertion is correct but Reason is incorrect. 75. Assertion : Ambulacral system plays a major (d) If both the Assertion and Reason are incorrect. role in locomotion of echinoderm. 70. Assertion : Sponges have body organization of Reason : Hydraulic pressure of fluid and "cellular level". Reason : There is some physiological division contraction of muscle of tube feet make possible of labour. [2009] movement of echinoderm. [2010, 2017] IIT JEE NEET eBooks: www.crackjee.xyz EBD_7100

B-24 Topicwise AIIMS Solved Papers – BIOLOGY

Type A : Multiple Choice Questions 11. (b) Aristotle’s lantern is a chewing toothed apparatus in class echinoidea of phylum 1. (b) Pouched mammals are metatherians or echinodermata. marsupials because their female has a pouch 12. (d) Sympathetic nerves in mammals arise from or marsupium containing the teats for thoracico–lumbar region. Sympathetic rearing the young ones. e.g., Kangaroo and nervous system is represented by a chain Wallaby. of 21 sympathetic ganglion on either side 2 (b) Sponges capture food with the help of of spinal cord. It receives preganglionic flagellated cells/collar cells/choanocytes. sympathetic fibres from the spinal cord These cells maintain a flow of water through which make these exit along with thoracic the body and improves both respiratory and lumbar region. and digestive functions, pulling in oxygen 13. (b) Corals belong to class anthozoa which exist and nutrients and allowing a rapid as solitary or colonial. Corals is the rock expulsion of CO and other waste products. like external skeletons. Coral animals secrete 2 external skeletons of calcium carbonate. 3. (a) Chloragogen cells of earthworm are similar 14. (b) Water current in Leucosolenia is produced to the liver of vertebrates because of the by flagellated choanocytes. connection with storage and synthesis of 15. (d) Platyhelminthes are flat worms, e.g. blood glycogen and fat, deamination and urea fluke, Schistosoma. formation. 16. (c) Four pairs of small spermathecae are 4. (c) Presence of haemocoel (a blood filled body present on the intersegmental groove of 5 cavity) is a characteristic of arthropods and & 6, 6 & 7, 7 & 8 and 8 & 9 segments. They molluscs, e.g. cockroach and Pila. receive sperms from another worm during 5. (c) Aquatic reptiles are ammonotelic. Their copulation. main excretory product is nitrogenous 17. (c) Weberian ossicles are the chain of 4 small ammonia. Availability of water makes them bones that connect air bladder and internal ammonotelic. ear of teleost fishes. They serve to enhance 6. (b) Larva of housefly is legless, headless and hearing by conducting pressure changes wingless maggot. produced by externally originating sound 7. (c) Nostoc is a colonial cyanobacterium. It waves from the swim bladder to the ear. contains a number of interwined filaments 18. (b) The vertebrae in birds are of heterocoelous on the periphery, a mucilage filled hollow type. interior and a dense mucilage covering on 19. (a) Basket star (Astropecten) belongs to class the outside. ophiuroidea of phylum echinoderm. 8. (a) The organisms attached to substratum 20. (a) A pair of unequal, branched collaterial glands (opening separately into genital generally possess radial symmetry. chamber) form an egg case ootheca. Radial animals are usually sessile, freely 21. (b) In sponges, archaeocytes are totipotent floating or weakly swimming. cells which can transform into sclerocytes, 9. (c) Hydra recognizes its prey by a chemical spongocytes or collencytes. They also stimulus. have a role in nutrient transport and sexual 10. (a) Presence of air passages in the bones reproduction. (pneumatic bones) is a characteristic of 22. (d) Penetrant, the largest nematocysts in Hydra birds. produce hypnotoxin. IIT JEE NEET eBooks: www.crackjee.xyz

Animal Kingdom B-25 23. (b) Life cycle of Taenia is digenetic. Their 36. (a) Chromatid bodies are present in the primary host is pig and secondary is man. precystic stage in Entamoeba histolytica. 24. (d) Copper containing pigment, hemocyanin These bodies disappear as the cyst mature. is found in molluscs. 37. (d) Phlebotomus (sand fly) transmits 25. (c) Antedon (sea lily/feather star) is found in Leishmania donovani which causes dum the class crinoidea of phylum echino- dum fever/kala azar. dermata. 38. (b) In leech, cockroach and scorpion, double ventral nerve cord is present. 26. (a) Chondrichthyes have placoid scales and teleosts have cycloid and ctenoid type of 39. (a) When water is not available, the camels scales. produce dry faeces and concentrated urine. One of the best sources of metabolic water 27. (b) Python is the largest non-poisonous in camels is oxidation of food in the hump. snake. 40. (b) Nephridia in earthworm, malpighian 28. (b) Birds are warm blooded/homeothermal tubules in cockroach and urinary tubules animals. Their body temperature is in the rat are excretory in function. independent of that of their external 41. (c) Silverfish, scorpion, dragonfly and prawn environment. are arthropods. They are characterised by 29. (c) Fish meal has good nutritive value because jointed appendages and chitinous it contains easily digestive proteins (55 – exoskeleton. 77%), minerals (10 – 20%) and moisture (6 42. (b) Millipede is myriapod, silver fish is insecta – 12%). and sea anemone is diploblastic. 30. (a) Birds have only one ovary (left) which Duck bill platypus is a small semiaquatic reduces their body weight. It is an oviparous . adaptation for flight. Ornithorhynchus anatinus, the Duckbill 31. (a) Gemmule is a type of internal bud formed platypus, is a unique mammal native to in sponges to pass the unfavourable Queensland, New South Wales, Victoria, season. It is helpful in asexual reproduction. South Australia, and Tasmania. This animal 32. (a) The rhabditoid larva moults within egg is about the size of a house cat and is shell in soil to form second stage rhabditoid covered by thick waterproof hair. It has a (capable of infection). Egg hatches in the beak like a duck, webbed forelimbs for intestine, bores through epithelium of swimming, clawed hind feet for aid in intestine to enter the circulatory system burrowing, a common opening for the and then it reaches alveoli where it moults reproductive, excretory and digestive for the second time. After its 3rd moult, it systems, and a broad, flat tail. In addition, leaves the lungs through trachea. the males have a single spur on each hind 33. (c) Spongin is an organic horny elastic ankle that contains venom, and the females substance. They are fibres insoluble, lay eggs. chemically inert and resistant to protein 43. (d) Mammals are characterised by muscular digesting enzymes. diaphragm and milk producing glands. 34. (b) Hydra has diffused type of nervous 44. (d) A lamprey (sometimes also called as system. It receives stimulus through lamprey eel) is a jawless fish that belongs sensory cells scattered in to the class cyclostomata. Lampreys epitheliomuscular layer. possess toothed funnel-like sucking 35. (c) Animals excreting uric acid are called mouth, having scaleless slimy skin with uricotelic. It is least toxic and causes least seven pairs of circular gill slits on each side. loss of body water, e.g. land reptiles, birds They are well known as those species etc. which bore into the flesh of other fish to suck their blood. IIT JEE NEET eBooks: www.crackjee.xyz EBD_7100

B-26 Topicwise AIIMS Solved Papers – BIOLOGY 45. (a) Nephridia are invertebrate organs which systems made up of a brain and a dorsal function similarly to kidneys. They remove (back) nerve cord. We all have highly devel- metabolic wastes from an animal’s body. oped receptors to detect changes in the en- They are of two basic types, metanephridia vironment for us to respond to. Fused and protonephridia. A metanephridium is a ganglia is a primitive brain found in earth- type of excretory gland or nephridium found worms, tympana are primitive ears found in in annelids, arthropods and molluscs. A grasshoppers, and nerve nets are branched protonephridium is a network of dead-end nerves without direction found in the Hydra. tubules lacking internal openings. The ends 50. (a) The animals of phylum echinodermata are are called flame cells or solenocytes; they oviparous. They have the ability of regen- functions in osmoregulation. eration. 46. (a) Morphallaxis refers to the type of 51. (a) Scorpion, spider, tick and mites comes un- regeneration in which lost body parts are der class arachnida of phylum arthropoda. replaced by the remodeling of the remaining tissue. In this type of regeneration, little or 52. (d) Portuguese man of war is a polymorphic, no cellular proliferation takes place during colonial coelenterate Physalia physalis. It the regeneration process. A classic example is named so due to its appearance which of an organism that regenerates using this resembles with portuguese vessels sailing mechanism is the Hydra. When a Hydra is in sea in 15th and 16th centuries cut into two pieces, two Hydra as will be 53. (d) Whales are exclusively viviparous. regenerated, both smaller than the parental Hydra. Once regeneration is completed, the In viviparous animals (majority of mammals two Hydra can continue to grow and reach including human beings), the zygote the size of their original parent. Growth develops into a young one inside the body requires cellular proliferation but during the of the female organism. After attaining a regenerative process very little cellular certain stage of growth, the young ones are proliferation takes place. delivered out of the body of the female 47. (d) Invertebrates, animals without a backbone organism. such as an earthworm, are simpler than 54. (a) 55. (b) vertebrates such as a human. Therefore, their cells are not as complex and they may be 56. (a) Diploblastic animals have two germinal able to regenerate or grow lost body parts layers, outer ectoderm and inner endoderm, easier than a more complex animal. e.g., Porifera and Coelenterate. Triploblastic 48. (b) Earthworm has closed circulatory system. animals have three germinal layers – outer The blood circulates in definite walled blood ectoderm, middle mesoderm and inner vessels. Renal portal system is characteristic endoderm, e.g., Platyhelminthes, of reptiles. In reptiles, renal portal system Aschelminthes, Annelida, Arthropoda, brings blood from internal organs to the Molluscs, Echinodermata and Chordata. kidneys because their digestive tract and urinary tract only have cloaca to eliminate 57. (a) wastes. Humans have hepatic portal system 58. (c) The amphibian skin is moist. that brings all the venous flow from digestive 59. (d) system into the liver. In cockroach, the 60. (a) Molluscans are soft bodied animals. Their excretory organs are malpighian tubules not body is unsegmented with a distinct head, nephridia. muscular foot and visceral hump. In Pila, 49. (a) Chordates are a group of animals such as the buccal cavity contains a rasping organ, fish, frogs, snakes, birds, dogs, monkeys and the radula with transverse rows of teeth. humans. We all have central nervous IIT JEE NEET eBooks: www.crackjee.xyz

Animal Kingdom B-27 Type B : Assertion Reason Questions 69. (b) Tapeworm, roundworm & pinworm are all endoparasites. The main cause of the 61. (a) Cold blooded organisms utilize their stored intestinal infection is improperly cooked food at the time of hibernation and food. However, tapeworm infection occur aestivation. by eating improperly cooked food, 62. (a) Birds have only left ovary which is an roundworm is transmitted by contaminated adaptation to reduce the weight for flight. food & water and pinworm or ringworm is transmitted through food or improper 63. (b) Lateral line system is made up of sensory sanitary condition. cells of ectodermal origin. It is meant for 70. (b) Sponges are multicellular but they have balancing the body while swimming. So, is cellular level of body organization i.e., true found in fishes and larval forms of tissue, movable parts, or appendages are amphibians. not formed. Although, there is some 64. (b) Bats and whales are classified as mammals. physiological division of labour, They have milk secreting mammary glands. accompanied with structural differentiation amongst body cells. But here, similar cells Whales and bats are mammals. Whales are are arranged neither in permanent layer nor warm blooded, breath air through lungs and masses to form tissues. give birth to live young that are suckled on 71. (b) Cephalochordates and urochordates are milk secreted from the mother’s mammary acraniates. These are marine animals glands. Bats have hair, give birth to live without cranium, jaws, vertebral column young and feed these young on milk and paired appendages. Notochord is produced in mammary glands. They are present and they are less developed than the only true flying mammals and are so crainates. unique that they have been placed in an 72. (b) Sponges belong to Porifera and they have order of their own chiroptera. 4-chambered characteristic canal system. hearts in mammals and birds keep fully 73. (b) Both duck billed platypus and spiny ant oxgenated. eaters are mammals because of their 65. (c) Cleavage in placental mammals is constant body temperature and presence holoblastic because of microlecithal eggs. of diaphragm. 66. (c) Koel (Eudynemis) lays eggs in crow’s nest 74. (d) Typhlosole is the characteristic feature of for incubation and rearing. earthworm. It can be defined as an extra flap of tissue or an infolding along the inner 67. (a) Honey bee queen copulates only once in wall of intestine. The typhlosole in her life span (nupital flight) and stores all earthworm increases the surface area of the the sperms in her seminal receptacles. It intestine for efficient secretion and depends on her whether she releases absorption during digestion. sperms while laying eggs or not, so there 75. (a) The water vascular system is a unique are both fertilized and unfertilized eggs. organ system that functions in locomotion, 68. (b) Ctenidium is a gill situated on the right side feeding, respiration and excretion. of the branchial chamber. It helps in Ambulacral canal is connected to outside respiration by beating cilia. During through external tube feet. Hydraulic development, ctenidium shifts from left side pressure of fluid and contraction of muscle of tube feet make possible movement of to right side which is called "torison". It is Echinoderm. characteristic feature of gastropods. IIT JEE NEET eBooks: www.crackjee.xyz EBD_7100

B-28 Topicwise AIIMS Solved Papers – BIOLOGY 5 Morphology of Flowering Plants

TYPE A : MULTIPLE CHOICE QUESTIONS 10. Clove is [2001] (a) flower bud (b) axillary bud 1. Fern character of Cycas is [1997] (c) thalamus (d) ovule (a) coralloid root 11. Pollinia are found in [2001] (b) tap root (a) wheat (b) madar (c) parallel venation (d) circinate venation (c) mango (d) banana 2. The desert plants in order to tolerate water stress 12. Monocarpic plants flower [2001] show [1997] (a) once (b) twice (a) sunken stomata (c) many times (d) never (b) reduced leaves 13. Nodules with nitrogen fixing bacteria are found (c) well developed root system in [2001] (d) all of these (a) cotton (b) gram 3. Perisperm is [1997] (c) mustard (d) wheat (a) remnant of endosperm 14. In which family (9) + 1 androecium condition is (b) persistant nucellus found ? [2001] (c) remnant of embryo (a) Malvaceae (b) Papilionaceae (d) part of endosperm 4. In moss capsule, the number of peristome whorls (c) Solanaceae (d) Poaceae are [1998] 15. Which of following type of anther is found in (a)1 (b) 2 Malvaceae? [2002] (c)3 (d) 4 (a) Monothecous (b) Dithecous 5. A mature ligule, having a prominent basal (c) Polythecous (d) Without thecous portion, is called [1998] 16. Potato and sweet potato [2004] (a) glossopodium (b) rhizophore (a) have edible parts which are homologous (c) trichome (d) None of these organs. 6. Inflorescence of Ficus is [1999] (b) have edible parts which are analogous (a) spike (b) hypanthodium organs. (c) raceme (d) verticillaster 7. The edible part of cauliflower is [2000] (c) have been introduced in India from the (a) inflorescence (b) leaf same place. (c) flower (d) stem (d) are two species of the same genus. 8. Most reduced form of stem is found in [2000] 17. The sugarcane plant has [2004] (a) bulb (b) rhizome (a) dumb-bell shaped guard cells (c) tree (d) stem (b) pentamerous flowers 9. In Opuntia, spines are modification of [2000] (c) reticulate venation (a) stem (b) root (d) capsular fruits (c) leaf (d) flower IIT JEE NEET eBooks: www.crackjee.xyz

Morphology of Flowering Plants B-29

18. The family containing mustard and its main 25. Of the following, which instrument is most characters are [2005] commonly used to observe the external features (a) Brassicaceae - Tetramerous flowers, six of a grasshopper's abdomen? [2009] stamens, bicarpellary gynoecium, siliqua (a) Ultracentrifuge type fruit (b) Microdissection instrument (b) Brassicaceae - Pentamerous flowers, many (c) Dissecting microscope stamens, pentacarpellary gynoecium, (d) Electron microscope capsule type fruit 26. In a bisexual flower, if androecium and (c) Solanaceae - Pentamerous flowers, five gynoecium mature at different times, the stamens, bicarpellary gynoecium, berry phenomenon is known as a [2010] type fruit (a) dichogamy (b) herkogamy (d) Poaceae - Trimerous flowers, three (c) heterogamy (d) monogamy stamens, monocarpellary gynoecium, 27. Which of following type of anther is found in caryopsis type of fruit Malvaceae? [2011] 19. Velamen present in orchids help in [2007] (a) Monothecous (b) Dithecous (a) absorbing water from support (c) Polythecous (d) Without thecous (b) respiration 28. Parachute type dispersal occurs in [2011] (c) absorption of moisture from air (a) tomato (b) mustard (d) synthesizing food (c) pea (d) cotton 20. Composite fruit develops from [2007] 29. Prickles of rose are [2012] (a) single ovary (b) inflorescence (a) Modified leaves (c) apocarpous ovary (d) pericarp (b) Modified stipules 21. If the anthers are fused together forming a tubular structure while the filaments remain free, (c) Exogenous in origin the condition is found in which one of the (d) Endogenous in origin following family? [2009] 30. Which of the following are not characteristic (a) Malvaceae (b) Cucurbitaceae features of fabaceae? [2013] (c) Solanaceae (d) Asteraceae (a) Tap root system, compound leaves and 22. Floral diagram fails to indicate [2009] raceme inflorescence. (a) epiphylly and epipetaly (b) Flowers actinomorphic, twisted aestivation (b) aestivation and placentation and gamopetalous. (c) position of ovary on the thalamus (c) Stamens 10, introrse, basifixed, dithecous. (d) cohesion of carpels and stamens (d) Monocarpellary, ovary superior and bent 23. Aggregate fruit develops from [2009] stigma. (a) syncarpous ovary 31. Which one of the following is correctly (b) multicarpellary, syncarpous ovary matched? [2014] (c) unilocular ovary (a) Onion – Bulb (d) multicarpellary, apocarpous ovary (b) Ginger – Sucker 24. The presence of cilia, an oral groove, and food vacuoles, and the absence of chloroplasts in a (c) Chlamydomonas – Conidia unicellular organism indicate that the organism (d) Yeast – Zoospores carries on [2009] 32. Read the following statements. (a) sexual reproduction (i) Gynoecium is situated in the centre and (b) autotrophic nutrition other parts of the flower are located on the (c) extracellular digestion rim of the thalamus almost at the same level. (d) heterotrophic nutrition IIT JEE NEET eBooks: www.crackjee.xyz EBD_7100

B-30 Topicwise AIIMS Solved Papers – BIOLOGY

(ii) Ovary is half-inferior. C. Indusium III. An unbranched (iii) Examples are plum, rose and peach. columnar stem with a Which condition of flowers is being described crown of leaves. by the above statements ? [2014] D. Caudex IV. Protective covering of (a) Hypogyny (b) Perigyny radicle (c) Epigyny (d) None of these V. Protective structure of a 33. Which one of the option is correct? sorus. [2016] (a) A – V; B – II; C – IV; D – I Seed coat C (b) A – IV; B – I; C – V; D – III Raphe D (c) A – III; B – V; C – II; D – IV A E (d) A – II; B – III; C – I; D – V B 36. Which of the following is a modified stem for the protection of plants from browsing animals? Seed Seed opened (a) Tendrils (b) Thorns [2015] (c) Rhizome (d) Tuber (a) A - Hilum, B - Micropyle, C - Radicle, [2017] D - Cotyledon, E - Plumule 37. Leaves of dicotyledonous plants possess (b) A - Hilum, B - Micropyle, C - Plumule, ______venation, while ______D - Cotyledon, E - Radicle venation is the characteristic of most (c) A - Micropyle, B - Hilum, C - Plumule, monocotyledons. [2017] D - Cotyledon, E - Radicle (a) reticulate and parallel (d) A - Hilum, B - Micropyle, C - Plumule, (b) parallel and reticulate D - Radicle, E - Cotyledon (c) reticulate and perpendicular 34. Seeds are adaptively important because [2015] (d) obliquely and parallel 1. they maintain dormancy TYPE B : ASSERTION REASON QUESTIONS 2. they protect young plants during vulnerable stages Directions for (Qs. 38-40) : These questions consist of two statements, each printed as Assertion and 3. they store food for young plants, and facilitate dispersal Reason. While answering these questions, you are required to choose any one of the following five (a) 1 and 3 (b) 2 and 3 responses. (c) 1 and 2 (d) All of these (a) If both Assertion and Reason are correct and 35. Match the following- the Reason is a correct explanation of the List-I List-II Assertion. A. Coleorhiza I. Development of sporo- phyte directly from ga- (b) If both Assertion and Reason are correct but metophyte without in- Reason is not a correct explanation of the tervention of gametes Assertion. B. Apogamy II. Development of game- (c) If the Assertion is correct but Reason is tophyte directly from incorrect. sporophyte (d) If both the Assertion and Reason are incorrect. without the involvement (e) If the Assertion is incorrect but the Reason is of reduction division. correct. IIT JEE NEET eBooks: www.crackjee.xyz

Morphology of Flowering Plants B-31

38. Assertion : In hemianatropous ovule, the funicle (a) If both Assertion and Reason are correct and lies parallel to body of ovule. Reason is the correct explanation of Assertion. Reason : Here, body of ovule is rotated by 90°. (b) If both Assertion and Reason are correct, but [1999] Reason is not the correct explanation of 39. Assertion : Many plants are propagated Assertion. (c) If Assertion is correct but Reason is incorrect. vegetatively even though they bear seeds. (d) If both the Assertion and Reason are incorrect. Reason : Potatoes multiply by tubers, apple by 41. Assertion : In fabaceae family monocarpellary, cutting etc. [2001] unilocular ovary is present. [2010] 40. Assertion : Ginger has a prostrate growing rhizome. Reason : In fabaceae, placentation is parietal. Reason : Shoot growth is not effected by gravity. 42. Assertion : Apical meristem of root is [2004] subterminal. Directions for (Qs. 41-43) : Each of these questions Reason : At the terminal end of root, root cap is contains an Assertion followed by Reason. Read them present. [2014] carefully and answer the question on the basis of 43. Assertion : A simple leaf has undivided lamina. following options. You have to select the one that Reason : Leaves showing pinnate and palmate best describes the two statements. venation have various types of incisions. [2016] IIT JEE NEET eBooks: www.crackjee.xyz EBD_7100

B-32 Topicwise AIIMS Solved Papers – BIOLOGY

Type A : Multiple Choice Questions and spice has more of the medicinal property at the bud stage. 1. (d) In circinate venation, leaves are coiled 11. (b) Pollinia is generally seen in fused anthers. when immature and gradually rolled with Each anther has pollinia. Madar shows the maturity. This coiling protects the growing characteristic feature of pollinia. point. 12. (a) Monocarpic flowers have a single carpel 2. (d) Desert plants have well developed root that can mature only once in their life time. system so that they can absorb water from Hence, they flower only once. the deeper layers of soil. They have sunken stomata and reduced leaves which reduce 13. (b) Grams are leguminous plants. Nitrogen the rate of water loss through transpiration. fixing bacteria is seen in leguminous plants. They convert atmospheric N to nitrate that 3. (b) The nucellus is generally used up during 2 can easily be absorbed by plants. the development of embryo but in some cases it remains outside the endosperm in 14. (b) In papilionaceae- the androceium is seen in diadelphous condition. 10 stamens are the form of a thin layer, called perisperm. seen in two bundles- 9 + 1. 9 together form 4. (b) In moss, peristome consists of 32 acellular one bundle whereas the single stamen teeth arranged in 2 whorls, outer forms another bundle. hygroscopic and inner hygroscopic. 15. (a) The filament of stamen bears one celled 5. (a) Glossopodium is a mature ligule. It is anther. generally present in grasses. The ligule is 16. (b) Potato is the modified underground stem an outgrowth between leaf base and lamina. whereas sweet-potato is the modified root Leaves with ligule are called ligulates. for storage of food. These are analogous 6. (b) Ficus has hypanthodium type of organs which have different origin but inflorescence i.e. cup shaped, fleshy serve the same functions. receptacle bearing flowers on the inner wall 17. (a) Sugarcane being a monocot plant is of the cavity. characterized by the presence of dumb-bell 7. (a) Cauliflower is a crossbreed between 2 shaped guard cells. In dicots, guard cells are kidney shaped. varieties of cabbage. It is evident that the edible part is the inflorescence due to the presence of small florets. 8. (a) Bulb has the most reduced form of stem since the stem is discoid in nature – a flattened disc. e.g. Allium cepa, Allium Stomata Stomata sativum. Dicot Monocot 9. (c) In Opuntia, the entire leaf is modified into 18. (a) Brassicaceae (cruciferae) the spine, to reduce water loss due to Other members are : Raddish, cabbage, transpiration as Opuntia is a xerophyte. cauliflower. 10. (a) The structure of clove itself resembles the Floral formula : bud where we are, infact, able to see the calyx also. Clove that is used as a condiment Ebr, , K2 + 2, C4 A2 + 4, G(2) IIT JEE NEET eBooks: www.crackjee.xyz

Morphology of Flowering Plants B-33

19. (c) Orchids are the epiphytes that posses aerial manipulate microscopic organelles (such roots. These roots contain a spongy tissue as transferring nuclei), and electron micro- called velamen whose main function is to scopes can magnify an image up to 250,000x absorb moisture from the air. and are used to see microscopic details. 20. (b) Composite fruits develop from the 26. (a) In a bisexual flower, if androecium and complete inflorescence and are also known gynoecium mature at different times, the as multiple fruit. These are of 2 types- phenomenon is known as dichogamy. sorosis and syconus. Sorosis develops 27. (a) The filament of stamen bears one celled from spike, spadix or catkin inflorescence, anther. e.g. Ananas, Arto carpus. Syconus 28. (b) Dispersal is a universal biological need. For develops from hypanthodium non-aquatic, terrestrial plants, the wind is inflorescence, e.g. Ficus. an obvious supplier of energy for 21. (d) This condition is called syngenesious. It movement, and many plant adaptations is found in members of family asteraceae exist that clearly take advantage of this fact. (or compositae), e.g., disc florets of This type of seed dispersal is not efficient, sunflower. but very effective. Appendages of some 22. (c) Floral diagram illustrates the whorls and fruits & seeds act as parachute like seeds number of parts in each of the sets of of cotton possess hair that help in dispersal organs comprising a flower. It shows the of these seeds. Another well-known position of floral parts in relation to mother example is the dandelion. axis but although position of ovary on the 29. (c) Prickles of rose develop only from cortex thalamus is not shown by it. and epidermis and found at the nodes or 23. (d) A muticarpellary ovary may be syncarpous internodes. It helps in climbing. It is (when carpels are fused) or apocarpous exogenous in origin. (when the carpels are free). A syncarpous 30. (b) ovary gives rise to a simple fruit while in an 31. (a) Onion - Bulb - Undeground stem , Ginger - apocarpous ovary, each carpel changes Rhizome, Chlamydomonas - Zoospore, into a fruitlet. The collection or aggregate Yeast - Ascospores of these fruitlets is known as etaerio. 32. (b) 33. (b) 24. (d) Cilia are hair-like bristles on a Paramecium 34. (d) Seeds perform all the given functions. used in locomotion and to find food. An 35. (b) oral groove is a mouth for a Paramecium, 36. (b) Thorn is a stiff, sharp-pointed woody and food vacuoles store food in the cell projection on the stem or other part of a body of the Paramecium. All of these are plant. Thorns are found in many plants cell organelles used to ingest, digest, and such as Citrus, Bougainvillea. They egest preformed food, which is protect plants from grazing animals. heterotrophic nutritrion. An autotroph can 37. (a) Leaves of dicotyledonous plants possess make its own food using chloroplasts. reticulate venation while parallel venation 25. (c) The dissecting microscope allows to view is the characteristics of most 3D images up to 50x magnification. It is monocotyledonous. In reticulate venation, commonly used in dissections. The the main veins of leaf form numerous ultracentrifuge spins liquids and separates irregular branches and as a result a net like the contents by their density, micro- arrangements is formed. Reticulate dissection instruments are used to IIT JEE NEET eBooks: www.crackjee.xyz EBD_7100

B-34 Topicwise AIIMS Solved Papers – BIOLOGY

venation is the most common vein emerges from axils of scaly leaves. formation in leaves. It can be found in the Response to light by plants is called leaves of maple trees, oak trees and rose phototropism. In this sense, shoot shows bushes. In parallel venation, veins are positive phototropism and root shows arranged parallel to each other. negative phototropism. Type B : Assertion Reason Questions 41. (c) In fabaceae, ovary is present. Placentation is marginal with many ovules. 38. (e) In hemianatropus ovule, the funicle lies at 42. (a) right angles to the body of the ovule. The body of the ovule is rotated by 90º. 43. (b) A leaf having a single or undivided lamina is called simple leaf, the lamina can have 39. (c) Plants do propagate more by vegetative different types of incisions, which may means since they multiply faster reach upto half, more than half or near the vegetatively. base or midrib. Depending upon the pinnate 40. (b) Ginger is an example of rhizome (e.g. or palmate venation, the incisions are known prostrate stem creeping horizontally under as pinnatifid, palmatifid, pinnatipartite, soil surface). There is no effect of gravity. palmatipartite, pinnatisect and palmatisect, Rhizome of ginger contains nodes, etc. internodes and scaly leaves. Buds are IIT JEE NEET eBooks: www.crackjee.xyz

6 Anatomy of Flowering Plants

TYPE A : MULTIPLE CHOICE QUESTIONS 10. Passage cells are found in [2002] (a) endodermis (b) pericycle 1. Cycas stem shows [1997] (c) cortex (d) epiblema (a) porous wood (b) manoxylic wood (c) pycnoxylic wood (d) ring porous wood 11. Fascicular cambium is the cambium of vascular 2. Aerenchyma is found in [1997] bundle of [2002] (a) parenchyma (b) xylem (a) monocot stem (b) dicot stem (c) phloem (d) sclerenchyma (c) monocot leaf (d) dicot leaf 3. Which of the following tissue is absent in 12. Mesophyll is usually differentiated in [2002] vascular bundles of monocot stem ? [1997] (a) monocot leaf (b) isobilateral leaf (a) Xylem (b) Phloem (c) dorsiventral leaf (d) both ‘a’ and ‘b’ (c) Cambium (d) All of these 13. In a dicotyledonous stem, the sequence of 4. Cork cambium is a [1999] tissues from the outside to the inside is[2003] (a) lateral meristem (b) apical meristem (a) phellem-pericycle-endodermis-phloem (c) intercalary meristem (b) phellem-phloem-endodermis-pericycle (d) primitive meristem (c) phellem-endodermis-pericycle-phloem 5. Endodermis is a part of [1999] (d) pericycle-phellem-endodermis-phloem (a) cortex (b) pericycle 14. The quiescent centre in root meristem serves as a (c) medulla (d) epidermis (a) site for storage of food which is utilized 6. Lateral root in higher plants arise from [1999] during maturation. [2003] (a) cortex (b) pericycle (b) reservoir of growth hormones. (c) epidermis (d) endodermis (c) reserve for replenishment of damaged cells 7. Cambium of root is an example of [2000] of the meristem. (a) apical meristem (d) region for absorption of water. (b) intercalary meristem (c) primary meristem 15. In a plant organ which is covered by periderm (d) secondary meristem and in which the stomata are absent, some gaseous exchange still takes place through 8. Which of the following is enucleate at maturity? [2004] [2000] (a) aerenchyma (b) trichomes (a) Companion cell (b) Meristematic cell (c) pneumatophores (d) lenticels (c) Parenchyma (d) Sieve tube cell 16. Companion cells in plants are associated with 9. Porous wood contains [2001] [2004] (a) vessels (b) tracheids (a) vessels (b) sperms (c) fibres (d) parenchyma (c) sieve elements (d) guard cells IIT JEE NEET eBooks: www.crackjee.xyz EBD_7100

B-36 Topicwise AIIMS Solved Papers – BIOLOGY

17. Cork cambium results in the formation of cork 23. If a stem is girdled [2012] which becomes impermeable to water due to the (a) Root dies first accumulation of [2004] (b) Shoot dies first (a) resins (b) suberin (c) Both die together (c) lignins (d) tannins (d) None of the above would die 18. Which one of the following statements 24. Which of the following statement(s) is/are true? pertaining to plant structure is correct? [2005] (A) Uneven thickening of cell wall is (a) Cork lacks stomata but lenticels carry out characteristic of sclerenchyma. [2013] transpiration. (B) Periblem forms cortex of the stem and the (b) Passage cells help in transfer of food from root. cortex to phloem. (C) Tracheids are the chief water transporting (c) Sieve tube elements possess cytoplasm but elements in gymnosperms. no nuclei. (d) The shoot apical meristem has a quiescent (D) Companion cell is devoid of nucleus at centre. maturity. 19. In which one of the following would you expect (E) The Commercial cork is obtained from to find glyoxysomes ? [2005] Quercus suber. (a) Endosperm of wheat (a) A and D only (b) B and E only (b) Endosperm of castor (c) C and D only (d) B, C and E only (c) Palisade cells in leaf 25. Sclerenchyma usually______and ______(d) Root hairs protoplasts. [2014] 20. Grafting is successful in dicots but not in (a) live, without (b) dead, with monocots because the dicots have [2006] (c) live, with (d) dead, without (a) vascular bundles arranged in a ring 26. T.S. of dicot leaf passing through the midrib is (b) cambium for secondary growth given below, certain parts have been indicated (c) vessels with elements arranged end to end by alphabets. Choose the correct option. (d) cork cambium 21. In the sieve elements, which one of the following is the most likely function of P-proteins?[2006] (a) Deposition of callose on sieve plates. (b) Providing energy for active translocation. (c) Autolytic enzymes. (d) Sealing mechanism on wounding. 22. Two cross-sections of stem and root appear simple, when viewed by naked eye. But under microscope, they can be differentiated by [2009] (a) exarch condition of root and stem [2015] (b) endarch condition of stem and root (a) A – Epidermis, B – Spongy parenchyma, (c) endarch condition of root and exarch C – Palisade parenchyma, D – Stomata, E – condition of stem Guard cells, F – Phloem, G – Metaxylem, H (d) endarch condition of stem and exarch – Protoxylem condition of root IIT JEE NEET eBooks: www.crackjee.xyz

Anatomy of Flowering Plants B-37

(b) A – Epidermis, B – Palisade parenchyma, (a) If both Assertion and Reason are correct and C – Spongy parenchyma, D – Sub-stomatal the Reason is a correct explanation of the cavity, E – Stoma, F – Phloem, G – Assertion. Metaxylem, H – Bundle sheath (b) If both Assertion and Reason are correct but (c) A – Epidermis, B – Palisade parenchyma, Reason is not a correct explanation of the C – Spongy parenchyma, D – Stomata, E – Assertion. Guard cells, F – Epidermis, G – Xylem, H – (c) If the Assertion is correct but Reason is Phloem incorrect. (d) A – Epidermis, C – Palisade parenchyma, (d) If both the Assertion and Reason are incorrect. C – Spongy parenchyma, D – Stomata, E – (e) If the Assertion is incorrect but the Reason is Guard cells, F – Phloem, G – Metaxylem, H correct. – Protoxylem 30. Assertion : Thick cuticle is mostly present in 27. Contractile tissues have the following features disease resistant plants. (i) Mesodermal in origin Reason : Disease causing agents cannot grow (ii) They contain stretch receptors. (iii) Rhythmic contractions are seen in them on cuticle and cannot invade the cuticle. [1997] (iv) They do not fatigue during the life of the 31. Assertion: Cambium is a lateral meristem and animal cause growth in width. Which of the above are characteristics of Reason: Cambium is made up of fusiform and sphincters? [2015] ray initials in stem. [1998] (a) All the four (b) Only (i), (ii) and (iii) 32. Assertion : Higher plants have meristematic (c) Only (i), (ii) and (iv) regions for indefinite growth. (d) Only (i), (iii) and (iv) Reason : Higher plants have root and shoot 28. Meristematic tissue responsible for increase in apices. [1999] girth of tree trunk is [2016] 33. Assertion : In collateral vascular bundles, (a) Apical meristem phloem is situated towards inner side. (b) Intercalary meristem Reason : In monocot stem, cambium is present. (c) Lateral meristem [2000] (d) Phellogen 34. Assertion : Collenchyma is thick walled dead 29. In stems, the protoxylem lies towards the tissue. ______Reason : Collenchymatous cells show and the metaxylem lies towards the thickenings of pectin. [2002] ______of the organ. 35. Assertion: The two cotyledons in seed are (a) centre; periphery embryonic leaves. (b) periphery; centre Reason: The embryo contains radicle and (c) periphery; periphery plumule. [2002] (d) centre; centre [2017] 36. Assertion : In angiosperms, the conduction of TYPE B : ASSERTION REASON QUESTIONS water is more efficient because their xylem has vessels. [2006] Directions for (Qs. 30-38) : These questions consist Reason : Conduction of water by vessel of two statements, each printed as Assertion and elements is an active process with energy Reason. While answering these questions, you are supplied by xylem parenchyma rich in required to choose any one of the following five mitochondria. responses. IIT JEE NEET eBooks: www.crackjee.xyz EBD_7100

B-38 Topicwise AIIMS Solved Papers – BIOLOGY

37. Assertion : In woody stems, the amount of heart (b) If both Assertion and Reason are correct, but wood continues to increase year after year. Reason is not the correct explanation of Reason : The cambial activity continues Assertion. uninterrupted. [2007] (c) If Assertion is correct but Reason is incorrect. (d) If both the Assertion and Reason are incorrect. 38. Assertion : Petroplants produce large amount 39. Assertion : Vessels are more efficient for water of latex. conduction as compared to tracheids. [2010] Reason : The latex contains long chain Reason : Vessels are dead lignified. hydrocarbons. [2007] 40. Assertion: Bulliform cells are useful in the Directions for (Qs. 39-41) : Each of these questions unrolling of leaf. contains an Assertion followed by Reason. Read them carefully and answer the question on the basis of Reason: Bulliform leaves store water. [2011] following options. You have to select the one that 41. Assertion : In stem, pericycle take active part in best describes the two statements. secondary growth. (a) If both Assertion and Reason are correct and Reason : In dicots, pericycle has the capacity to Reason is the correct explanation of Assertion. produce lateral roots. [2013] IIT JEE NEET eBooks: www.crackjee.xyz

Anatomy of Flowering Plants B-39

Type A : Multiple Choice Questions 7. (d) The cambium is secondary in its functional aspect since it forms secondary tissues like 1. (b) Cycas stem shows monoxylic wood with the secondary xylem and secondary broad parenchymatous rays and often phloem. It is, however, primary in origin. contain abundant resinous cells and resin 8. (d) Sieve tube cell is enucleate at maturity due canals. Towards the periphery of the stem, to the degeneration of its nucleus during cycads produce a 'leaf armour' consisting its developmental process. The companion of the tightly packed, helically arranged leaf cell-that develops from the same initial as bases. the sieve tube cell, possesses the nucleus 2. (a) Aerenchyma is the modification of throughout its life. The companion cell parenchyma tissue in which cells are carries out the function of the sieve tube arranged in such a way that - large air filled cell in the event of its degeneration. spaces are formed. Aerenchyma is found 9. (a) Porous wood contains vessels, with sieve in aquatic plants to produce buoyancy. cells, which contains passages for 3. (c) movement of substances. Monocot vascular Dicot vascular bundle bundle 10. (a) Passage cells are found in endodermis which allow a limited transfer of materials between the cortex and the vascular Phloem Phloem cylinder. Cambium 11. (b) In dicot stem, fascicular cambium and Xylem Xylem interfascicular cambium join to form a No secondary growth, secondary growth, complete ring which helps in secondary cambium absent V.B. is open. growth. i.e. V.B. is closed 12. (c) In a dorsiventral leaf, mesophyll is 4. (a) Cork cambium (phellogen) is a secondary differentiated into two layers i.e. palisade lateral meristem which develops from parenchyma and spongy parenchyma. permanent tissues in the region of 13. (c) In a dicotyledonous stem, the sequence of epidermis, hypodermis, cortex and even in tissues from outside to the inside is outer layers of phloem. phellem-endodermis-pericycle-phloem. 5. (a) Endodermis is the inner most layer of cortex. 14. (c) The concept of Quiescent Centre was The cells are characterized by the presence proposed by Clowes in 1961. On the basis of casparian strips. of autoradiographic studies of DNA 6. (b) The lateral roots arise from the cell of synthesis in the root tip of zea, he found a pericycle and hence, the root branches are reservoir of cells having low DNA, RNA said to be endogenous in origin (arising and protein concentration. He called it as from a layer inner to endodermis). The Quiescent Centre. They may or may not lateral roots help in absorption of water and divide. It is resistant to damages. mineral salts from the soil. The meristematic 15. (d) During secondary growth, where epidermis cells of the lateral root push through the is replaced by tough and hard periderm, endodermis and cortex and then pierce the cracked/ruptured epidermis forms the through the epidermis to come out to form small holes called lenticels which help in the lateral root. gaseous exchange. IIT JEE NEET eBooks: www.crackjee.xyz EBD_7100

B-40 Topicwise AIIMS Solved Papers – BIOLOGY

16. (c) Companion cells are long elongated living 23. (a) If a stem is girdled, root dies first, as the cells, that lie on the sides of the sieve tubes food synthesized by leaves is not able to in phloem. Companion cells control the reach to the roots. activities of the sieve tube through 24. (d) plasmodesmata. 17. (b) Phellogen produces cork or phellem on the 25. (d) Sclerenchyma consists of long, narrow cells outer side. It consists of dead and with thick and lignified cell walls having a compactly arranged rectangular cells that few or numerous pits. They are usually possess suberised cell walls. dead and without protoplasts. Lenticels 26. (b) 27. (b) 28. (c) 29. (a) The first formed primary xylem elements are called protoxylem and the later formed Cork cells primary xylem is called metaxylem. In stems, (suberized) the protoxylem lies towards the centre Phellogen (pith) and the metaxylem lies towards the Secondary periphery of the organ. This type of cortex primary xylem is called endarch. 18. (c) Sieve tube elements possess cytoplasm but Type B : Assertion Reason Questions lack nucleus at maturity. Its metabolic activities are regulated by the nucleus of a 30. (e) Plant cuticles are a protective waxy closely associated cell called companion covering produced only by the epidermal cell. cells of leaves, young shoots and all other 19. (b) Glyoxysomes are found in the plant cells aerial plant organs. In addition to its particularly in the cells of germinating fatty function as a permeability barrier for water seeds, e.g. endosperm of castor. and other molecules, the micro and nano- 20. (b) Grafting is a horticultural technique structure of the cuticle confer specialized whereby tissues from one plants are surface properties that prevent inserted into those of another so that the contamination of plant tissues with external sets of vascular tissues may join together. water, dirt and micro-organisms. The waxy Grafting is successful in dicots because sheet of cuticle also functions in defence, vascular bundles are arranged in a ring and forming a physical barrier that resists have cambium for secondary growth. penetration by virus particles, bacterial 21. (a) In the sieve elements, P-proteins deposit cells, and the spores or growing filaments callose on sieve plates. of fungi. 22. (d) The cross sections of stem and root appear 31. (b) Fusiform initials are vertically elongated cells simple, when viewed by naked eye but that produce xylem and phloem elements. under microscope they can be Ray initials are isodiametric and produce differentiated as endarch condition in stem parenchymatous rays in secondary xylem and exarch condition in root. In endarch, and phloem. protoxylem is present towards the center 32. (a) The root apex and shoot apex are of stem while metaxylem towards the meristematic in nature. These meristematic pericycle. In the exarch condition, tissues are embryonic in origin. They are protoxylem is present towards pericycle primary in origin because it develops from and metaxylem towards the center of the embryonic tissues and primary in function root. IIT JEE NEET eBooks: www.crackjee.xyz

Anatomy of Flowering Plants B-41

because they form the primary structure of 38. (a) Petroplants are plants having large amount the plant cell, the root apex and shoot apex, of latex with long chain hydrocarbons. Latex that live till the death of the whole plant. of these plants are a good substitute for Hence, plants have the feature of indefinite liquid fuels or petroleum. Cultivation of growth. petroplants is a part of energy- cropping. 33. (d) Collateral vascular bundles have the xylem Dr. Calvin was the scientist who identified pointing towards the inner side of the petrocrops. They have property of phloem. In the same way in monocots, converting large amount of their cambium is absent. Collateral vascular photosynthates into latex along with bundles are present in stems and leaves of hydrocarbons. Some important petrocrops angiosperms and gymnosperms. are Euphorbia antisyphilitica, E. lathyris 34. (e) Collenchyma is made up of living cells with Calotropis procera etc. unevenly thickened cell wall. Their cell wall 39. (b) Vessels are more efficient for water is made up of cellulose and pectin. conduction as compared to tracheids. Collenchyma are present beneath the Vessels resemble tracheids very much in epidermis of young stem, petioles and structure and function. But unlike tracheids midrib of leaves etc. These are absent in these are like long tubes arranged in vertical underground tissues and leaves and stems row formed of cylindrical cells arranged to of monocots. end with their end walls completely 35. (b) During epigeal germination, cotyledons dissolved. These are also dead and come out of the soil. The green cotyledons lignified. function as leaves of the seedling. They 40. (b) In isobilateral leaves, the upper epidermis manufacture food and sustain the young seedling till the plumule gives rise to new contains specialized cells, i.e., bulliform or leaves. motor cells. They are highly vacuolate and can store water, if available. However, in 36. (d) Xylem is the water conducting tissue. It consists of living cells like parenchyma and case of water deficiency, the bulliform cells dead cells like tracheary elements. lose water and become flaccid. As a result, the leaf gets rolled up to reduce the exposed 37. (a) In woody trees, the central portion of stem is dark in colour. It is hard and tough due surface. The bulliform cells are also useful to deposition of resins, tannins, gums and in the unrolling of leaf during its formation of tyloses. This central hard development. portion is called heart wood. It is formed 41. (c) Pericycle is the outermost layer of stele. In by secondary growth. Due to cambial dicot stems, pericycle strengths the stem activity secondary xylem becomes non- and provides protection to the vascular functional and forms heart wood or bundles. In angiosperms (dicots), pericycle duramen. It is more durable and little gives rise to lateral roots and contribute to susceptible to attack of pathogens. The the vascular cambium often diverging into cambial activity continues in this region. a work cambium. IIT JEE NEET eBooks: www.crackjee.xyz EBD_7100

B-42 Topicwise AIIMS Solved Papers – BIOLOGY 7 Structural Organisation in Animals

(b) administered large amounts of thyroxine. TYPE A : MULTIPLE CHOICE QUESTIONS (c) reared on a diet rich in egg yolk. 1. In frog, gastrulation process involves [1997] (d) reared on a diet rich in both egg yolk and (a) epiboly (b) emboly glucose. (c) invagination (d) all of these 10. Which of the following type of cell junction is 2. Structure which remains unchanged during not found in animal tissues ? [2013] metamorphosis of frog's tadpole is [1997] (a) Adhering junction (b) Tight junction (a) lung (b) heart (c) Gap junction (d) Plasmodesmata (c) nervous system (d) intestine 11. Identify the figure with its correct function 3. Which gland plays a key role in metamorphosis of frog ? [1999] (a) Adrenal (b) Thyroid (c) Thymus (d) Pancreas 4. Major protein of connective tissue is [2001] (a) myosin (b) collagen (c) melanin (d) keratin 5. Outer covering of cartilage is called [2001] (a) perichondrium (b) periosteum (c) endosternum (d) peritoneum 6. The protoplasmic segment of a striated muscle Fig :. Adipose connective tissue fibre is termed as [2001] (a) Areolar connective tissue – Serves as a support framework for epithelium (a) sarcoplasm (b) sarcomere (b) Adipose tissue – Store fats and act as heat (c) neuromere (d) metamere insulators 7. Sharpey’s perforating fibres are related with (c) Dense regular tissue – Provide flexibility (a) heart contraction [2002] (d) Dense irregular tissue – Provide strength (b) muscle relaxation and elasticity [2014] (c) fixing of teeth 12. Which of the following statement about cell (d) none of these junctions is false? [2015] 8. The type of epithelial cells which line the inner surface of fallopian tubes, bronchioles and small (i) All the cells of the epithelium are held bronchi are known as [2006] together with little intercellular materials. (a) squamous epithelium (ii) In almost all animal tissues specialized (b) columnar epithelium junction provide both structural and (c) ciliated epithelium functional link between its individual cells. (d) cubical epithelium (iii) Tight junctions help to stop substances 9. Tadpoles of frog can be made to grow as giant from leaking across a tissue. sized tadpoles, if they are [2006] (iv) Adhering junctions provide cementing to (a) administered antithyroid substance like keep neighbouring cells together. thiourea. IIT JEE NEET eBooks: www.crackjee.xyz

Structural Organisation in Animals B-43

(v) Gap junctions provide cytoplasmic (a) smell (b) hearing channels between cells for passage of ions, (c) respiration (d) touch small molecules and sometimes big 17. In earthworms, setae are present in all segments molecules. except [2017] (a) (ii) and (iii) (b) (i) and (ii) (a) first and the last segments (b) first segment and the clitellum (c) Only (v) (d) None of these (c) first segment 13. i. The shape of the cells may vary with the (d) clitellum and last segments function they perform TYPE B : ASSERTION REASON QUESTIONS ii. Human RBC is about 7.0 Pm in diameter iii. Cytoplasm is the main arena of cellular Directions for (Q. 18) : These questions consist of activities two statements, each printed as Assertion and Reason. iv. Various chemical reactions occur in While answering these questions, you are required to choose any one of the following five responses. cytoplasm to keep the cell in the living state (a) If both Assertion and Reason are correct and [2015] the Reason is a correct explanation of the (a) All are correct Assertion. (b) Only I and II are correct (b) If both Assertion and Reason are correct but (c) Only IV is correct Reason is not a correct explanation of the (d) All are wrong Assertion. 14. The figure given below shows the head region (c) If the Assertion is correct but Reason is of cockroach. Identify A to F. incorrect. (d) If both the Assertion and Reason are incorrect. Antennae A B (e) If the Assertion is incorrect but the Reason is correct. D 18. Assertion : Cartilage and bone are rigid C connective tissues. E F Reason : Blood is a connective tissue [2001] [2016] Directions for (Q. 19) : Each of these questions (a) A- Compound eye, B-Ocellus, C-Maxilla, contains an Assertion followed by Reason. Read them D-Mandible, E-Labrum, F-Labium carefully and answer the question on the basis of (b) A- Ocellus, B-Compound eye, C-Mandible, following options. You have to select the one that D-Maxilla, E-Labrum, F-Labium best describes the two statements. (c) A- Ocellus, B-Compound eye, C-Mandible, (a) If both Assertion and Reason are correct and D-Maxilla, E-Labium, F-Labrum Reason is the correct explanation of Assertion. (d) A- Ocellus, B-Compound eye, C-Maxilla, (b) If both Assertion and Reason are correct, but D-Mandible, E-Labrum, F-Labium Reason is not the correct explanation of 15. Male cockroach can be identified from the Assertion. female by the presence of [2017] (c) If Assertion is correct but Reason is incorrect. (a) long antennae (d) If both the Assertion and Reason are incorrect. (b) wingless body 19. Assertion : The squamous epithelium is made (c) elongated abdomen of a single thin layer of flattened cells with irregular boundaries. (d) anal styles Reason : They are found in walls of blood 16. The sensory papillae in frogs are associated vessels and air sacs of wings. [2017] with [2017] IIT JEE NEET eBooks: www.crackjee.xyz EBD_7100

B-44 Topicwise AIIMS Solved Papers – BIOLOGY

Type A : Multiple Choice Questions Endosternum is a collective name for the apodemes or interior processes of the 1. (d) Gastrulation is the process of formation of sternum in the thoracic of an insect. The three layers i.e. ectoderm, endoderm and peritoneum is thin membrane that lines the mesoderm. In frog, these layers are formed abdominal and pelvic cavities and covers by the processes of epiboly, emboly and most abdominal viscera. invagination. 6. (b) Sarcomere is the smallest contractile unit 2. (c) Frog’s nervous system once differentiated of striated muscle fibre. Sarcomere occurs remains as such throughout life. as repeating units along the length of a 3. (b) Thyroxine hormone produced by thyroid myofibril, occupying the region between Z gland plays a very important role in the lines of the myofibril. metamorphosis of frog. Sarcoplasm is the cytoplasm of a muscle 4. (b) Collagen is the major protein of connective fibre. It is a water solution containing ATP tissue. Collagen, in the form of elongated and phosphogens, as well as the enzymes fibrils is mostly found in fibrous tissues of intermediate and product molecules such as tendon, ligament and skin, and is involved in many metabolic reactions. also abundant in cornea cartilage, bone , Neuromeres is a metameric segment of CNS. blood vessels, the gut, and intervertebral Metamere is a linear series of primitively disc. Collagen performs a very important similar segments into which the body of role in ageing processes. higher invertebrate and vertebrate is Myosin, commonest protein in muscle cells, divisible. is responsible for the elastic and contractile 7. (c) Sharpey's fibres are collagenous fibres that properties of muscle. It combines with actin pass from the periodontal membrane into to form actomyosin. Melanin is a black or the cementum and the jaw bones, fixing the dark brown pigment that is responsible for teeth firmly in the sockets. the dark colour of the skin, hair, scales, 8. (c) Columnar ciliated epithelium posses fine feathers and eyes of animals. Keratin is a hair like ongrowths, cilia on thier free fibrous scleroprotein that occurs in the surfaces. This epithelium lines the nasal outer layer of the skin and in horny tissues passages, oviduct (fallopion tube), terminal such as hair, feathers, nails and hooves. bronchiole etc. Its major function is 5. (a) Cartilage is a type of connective tissue protection and movement of mucus, urine consisting of cells (called chondrocytes) and egg in a particular direction. and though flexible matrix made of collagen, protein, and sugar. The cartilage is covered Squamous epithelium forms the lining of on the outside by death of white fibrous cavities such as the mouth, blood vessels, connective tissue called perichondrium. heart and lungs and make up the outer layers of the skin. Columnar epithelium Periosteum is a membrane that lines the forms the lining of the stomach and outer surface of all bones, except at the intestines. Some columnar cells are joints of long bones. It contains the blood specialized for sensory reception such as vessels and nerves that provide in the nose, ears and the taste buds of the nourishment and sensation. IIT JEE NEET eBooks: www.crackjee.xyz

Structural Organisation in Animals B-45

tongue. Cubical epithelium is found in 16. (d) Frog has different types of sense organs glands and in the lining of the kidney like organs of touch (sensory papillae), tubules as well as in the ducts of the glands. taste (taste buds), smell (nasal epithelium), They also constitute the germinal vision (eyes) and hearing (tympanum with epithelium which produces the egg cells in internal ears). the female ovary and the sperm cells in the 17. (d) Except the first, the last and clitellar segment male testes. in each segment bear a ring of tiny curved, 9. (b) Thyroxine helps in the metamorphosis of chitinous structure known as setae. Setae tadpole. helps in locomotion and copulation. 10. (d) Type B : Assertion Reason Questions 11. (b) Adipose tissue is another type of 18. (e) Cartilage is a semi rigid connective tissue connective tissue located mainly beneath that is weaker than bone, but more flexible the skin. The cells of this tissue are resilient. Cartilage serves to provide specialised to store fats. structure and support to the body's other 12. (d) All the given statements about cell tissues and also provide a cushioning effect functions are true. in points. Bone is rigid connective tissue and forms the skeleton of the body. It is 13. (a) All the given statements are correct. composed chiefly of calcium phosphate 14. (b) A - Ocellus; B - Compound eye; and calcium carbonate. It also serves as a C - Mandible; D - Maxilla; E - Labrum; storage area for calcium, playing a large F - Labium. role in calcium balance in the blood. Blood 15. (d) Both the sexes of cockroach have anal cerci is a fluid connecting tissue. Blood delivers necessary substances, such as nutrients which are jointed structures. But in the and oxygen, to the body's cells (in animals) male, in addition, there is a paired unjointed and transports waste products away from needle-like anal style, which serve to those same cells. distinguish between the male and the female. 19. (b) IIT JEE NEET eBooks: www.crackjee.xyz EBD_7100

B-46 Topicwise AIIMS Solved Papers – BIOLOGY 8 Cell : The Unit of Life

10. Chromosomes with equal arms are called [2000] TYPE A : MULTIPLE CHOICE QUESTIONS (a) metacentric (b) telocentric 1. In many bacteria, cell membrane is invaginated (c) acentric (d) polycentric and folded to form [1997] 11. A prokaryotic cell lacks [2001] (a) pili (b) cristae (a) true nucleus (c) flagella (d) mesosome (b) nuclear membrane 2. Cristae helps in [1998] (c) membrane bound organelles (a) respiration (b) photosynthesis (c) transpiration (d) guttation (d) all of the above 3. In nucleoplasm, a spherical body attached to a 12. Which of the following is a single membranous particular chromosome on a definite position is structure ? [2001] called [1998] (a) Lysosome (b) Nucleus (a) nucleolus (b) karyolymph (c) Mitochondria (d) Chloroplast (c) plasmid (d) reticulum 13. In bacteria, site of respiration is [2001] 4. Which of the following is responsible for (a) mesosome (b) episome mechanical support and enzyme transport ? (c) plasmid (d) cytoplasm [1999] 14. Which of the following organelle is related with (a) Dictyosome (b) Cell membrane photorespiration? [2002] (c) E. R. (d) Mitochondria (a) Peroxisome (b) Nucleus 5. Which of the following is present between cell (c) Cell wall (d) Lysosome walls of the plant cells ? [1999] 15. The phagocytosis was first of all seen by (a) Lomasome (b) Microsome (c) Lysosome (d) Middle lamella (a) Huxley (b) Haeckel [2002] 6. Rough E. R. differs from smooth E. R. due to the (c) Metchnikoff (d) Darwin presence of [2000] 16. A chromosome with centromere at one end is (a) DNA (b) nucleus called [2002] (c) ribosome (d) enzyme (a) telocentric (b) metacentric 7. Electron microscope was invented by [2000] (c) excentric (d) apocentric (a) Robert Hooke 17. Plasmodesmata connections help in [2003] (b) Knoll and Ruska (a) cytoplasmic streaming. (c) Pasteur (b) synchronous mitotic divisions. (d) Schwann and Schleiden (c) locomotion of unicellular organisms. 8. Double membrane structure of cell are [2000] (d) movement of substances between cells. (a) nucleus (b) chloroplast 18. DNA is present in [2004] (c) mitochondria (d) all of these (a) chromosomes and dictyosomes 9. Hydrolytic enzymes are found in [2000] (b) chloroplasts and lysosomes (a) peroxisomes (b) lysosomes (c) mitochondria and chloroplasts (c) lepdosomes (d) lomasomes (d) mitochondria and endoplasmic reticulum IIT JEE NEET eBooks: www.crackjee.xyz

Cell : The Unit of Life B-47

19. Three of the following statements regarding cell (b) lysosomes and peroxisomes organelles are correct while one is wrong. Which (c) Golgi bodies and' smooth endoplasmic one is wrong? [2005] reticulum (a) Lysosomes are double membraned vesicles (d) plastids and inherited via male gamete budded off from Golgi apparatus and 25. What is common between chloroplasts, contain digestive enzymes. chromoplasts and leucoplasts ? [2008] (b) Endoplasmic reticulum consists of a (a) Presence of pigments. network of membranous tubules and helps (b) Possession of thylakoids and grana. in transport, synthesis and secretion. (c) Storage of starch, proteins and lipids. (c) Leucoplasts are bound by two membranes, (d) Ability to multiply by a fission-like process. lack pigment but contain their own DNA 26. Molecules that are too large to pass through and protein synthesizing machinery. the pores of a cell membrane may enter the cell (d) Sphaerosomes are single membrane bound by a process known as [2009] and are associated with synthesis and (a) hydrolysis (b) pinocytosis storage of lipids. (c) cyclosis (d) synthesis 20. What is common between chloroplasts, 27. Three morphological forms of golgi complex are chromoplasts and leucoplasts? [2006] (a) Lamellae, tubules and vesicles [2012] (a) Presence of pigments. (b) Cisternae, tubules and vesicles (b) Possession of thylakoids and grana. (c) Cisternae, tubules and lamellae (c) Storage of starch, proteins and lipids. (d) Granum, thalykoids and vesicles (d) Ability to multiply by a fission-like process. 28. Which chromosome may lost during cell 21. In prokaryotes, chromatophores are [2006] division? [2012] (a) specialized granules responsible for (a) Giant chromosome colouration of cells. (b) Acentric chromosome (b) structures responsible for organizing the (c) Polycentric chromosome shape of the organism. (d) Telocentric chromosome (c) inclusion bodies lying free inside the cells 29. Choose the incorrect match [2013] for carrying out various metabolic (a) Nucleus — RNA activities. (b) Lysosome — Protein synthesis (d) internal membrane systems that may become extensive and complex in (c) Mitochondria — Respiration photosynthetic bacteria. (d) Cytoskeleton — Microtubules 30. Which of the following statements are correct ? 22. Which of the following is responsible for the mechanical support, protein synthesis and (i) In prokaryotic cells, a special membranous enzyme transport? [2007] structure formed by the extension of the plasma membrane into the cell is known as (a) Cell membrane polysome. (b) Mitochondria (ii) The smooth endoplasmic reticulum is the (c) Dictyosome major site for synthesis of glycoproteins. (d) Endoplasmic reticulum (iii) RuBisCO is the most abundant protein in 23. “Omnis-cellula-e-cellula” was given by[2007] the whole biosphere. (a) Virchow (b) Hooke (iv) Mitochondria, chloroplasts and (c) Leeuwenhoek (d) Brown peroxisomes are not considered as part of 24. Genes present in the cytoplasm of eukaryotic endomembrane system. [2016] cells, are found in [2005, 2008] (a) (iii) and (iv) (b) (i) and (ii) (a) mitochondria and inherited via egg (c) (ii) and (iii) (d) (i) and (iv) cytoplasm IIT JEE NEET eBooks: www.crackjee.xyz EBD_7100

B-48 Topicwise AIIMS Solved Papers – BIOLOGY

TYPE B : ASSERTION REASON QUESTIONS 36. Assertion : Eukaryotic cells have the ability to adopt a variety of shapes and carry out directed Directions for (Qs. 31-38) : These questions consist movements. of two statements, each printed as Assertion and Reason : There are three principal types of Reason. While answering these questions, you are protein filaments-microfilaments, microtubules required to choose any one of the following five and intermediate filaments, which constitute the responses. cytoskeleton. [2006] (a) If both Assertion and Reason are correct and Directions for (Qs. 37-40) : Each of these questions the Reason is a correct explanation of the contains an Assertion followed by Reason. Read them Assertion. carefully and answer the question on the basis of (b) If both Assertion and Reason are correct but following options. You have to select the one that Reason is not a correct explanation of the best describes the two statements. Assertion. (a) If both Assertion and Reason are correct and (c) If the Assertion is correct but Reason is Reason is the correct explanation of Assertion. incorrect. (b) If both Assertion and Reason are correct, but (d) If both the Assertion and Reason are incorrect. Reason is not the correct explanation of (e) If the Assertion is incorrect but the Reason is Assertion. correct. (c) If Assertion is correct but Reason is incorrect. (d) If both the Assertion and Reason are incorrect. 31. Assertion : Power house of cell is mitochondria. 37. Assertion : Lipids present in the outer and inner Reason : ATP is produced in mitochondria. side of the bilayer membrane are commonly [2001] different. 32. Assertion : Cell wall is not found in animal cell. Reason : Oligosaccharides are attached to Reason : Animal cells are covered by cell external surface as well as inner surface of a membrane. [2001] biomembrane. [2009] 33. Assertion: Organisms are made up of cells. 38. Assertion : Mitochondria and chloroplasts are Reason: Cells are structural unit of living semi autonomous organelles. organisms. A cell keeps its chemical composition Reason : They are formed by division of pre- steady within its boundary. [2002] existing organelles as well as contain DNA but 34. Assertion: Specialization of cells is useful for lack protein synthesizing machinery. organism. [2005, 2014] 39. Assertion : A cell membrane shows fluid Reason: It increases the operational efficiency behaviour. of an organism. [2002] Reason : A membrane is a mosaic or composite 35. Assertion: The number of cells in a multicellular of diverse lipids and proteins. organism is inversely proportional to size of [2003, 2008, 2015] body. 40. Assertion : Centrosomes and centrioles are Reason: All cells of biological world are alive. related to each other. [2002] Reason : Centrosome usually contains two cylindrical structures called centrioles. [2016] IIT JEE NEET eBooks: www.crackjee.xyz

Cell : The Unit of Life B-49

Type A : Multiple Choice Questions 12. (a) Lysosome consists of hydrolytic enzymes, enclosed in a unit membrane. 1. (d) In prokaryotic cell, cell membrane 13. (a) In bacteria, mesosome helps in cell invaginate to form mesosomes. They have respiration. enzymes, which are useful for respiration. 14. (a) Leaf peroxisomes are associated with 2. (a) Cristae form a part of mitochondria which endoplasmic reticulum, chloroplast and helps in cellular respiration. mitochondria and are involved in 3. (a) A small spherical body attached to the photorespiration. Photorespiration is light particular chromosome (nucleolar induced CO liberation from a C chromosome) in the nucleoplasm is 2 2 compound (glycolic acid) of dark phase of nucleolus. Nucleolus is the principal or active site for the development of ribosomal photosynthesis. RNAs & it is essential for spindle formation 15. (c) The phagocytosis was first of all seen by of ribosomes. Metchnikoff in 1893. Phagocytosis is a 4. (c) ER provides mechanical support and process whereby certain cells & unicellular enzyme transport in a cell. organisms are capable of ingesting and 5. (d) Adjacent cells in a plant tissue are held digesting solid material. together by a thin, sticky, amorphous layer 16. (a) When centromere is located at the tip of of cementing tissue called middle lamella. the chromosome, it is said to be telocentric. It is made up of Ca and Mg pectate. 17. (d) Plasmodesmata are small fuses that 6. (c) Rough endoplasmic reticulum is rough due connect plant cells to each other, providing to the presence of ribosomes at its surface. living bridges between cells. Their attachment to the ER is by means of Plasmodesmata brings the exchange of protein ribophorin I and II. substances between cells. 7. (b) Electron microscope was invented by Knoll 18. (c) DNA is present in mitochondria and and Ruska in 1932. chloroplasts (extranuclear DNA). So these 8. (d) Double membrane organelles of the cell are organelles are self replicating. mitochondria, chloroplast and nucleus. 19. (a) Lysosomes are single membrane bound 9. (b) Lysosomes are spherical, single membrane organelles rich in hydrolytic enzymes. limited vesicles, containing hydrolytic 20. (a) C pathway/Hatch and Slack pathway enzymes working at acidic pH. Lysosome 4 ensures the Calvin cycle to be operated is also known as suicidal bag/sac as they contain hydrolytic enzymes. only in bundle sheath cell. It is an adaptation to photorespiratory loss. 10. (a) Metacentric chromosomes have equal Therefore, C plants are adapted to sized chromatids i.e. they have centromere 4 in the centre. photorespiratory loss. 11. (d) Prokaryotic cells, among the tiniest of all 21. (d) In prokaryotes, chromatophores are cells, has one envelope system with no internal membrane system that may membrane lined internal organelles except become extensive and complex in thylakoid if present. A prokaryotic cell photosynthetic bacteria. It is structurally lacks membrane bound organelles, nuclear and functionally similar to eukaryotic membrane and histone proteins. chloroplast. IIT JEE NEET eBooks: www.crackjee.xyz EBD_7100

B-50 Topicwise AIIMS Solved Papers – BIOLOGY

22. (d) Endoplasmic reticulum (ER) is a system of molecules. Hydrolysis, is the process of flattened membranes running through the using water to split molecules apart. cytoplasm. Rough ER containing Cyclosis is a mechanism for transporting ribosomes is mainly concerned with materials within a cell, by the cytoplasm protein synthesis and transport of swirling. Synthesis is the process of enzymes. It also functions as cytoskeleton building up molecules within the cell. by giving mechanical support to the 27. (b) Three morphological forms of golgi cytoplasm. complex are cisternae, tubules and vesicles. Varied number of cisternae are present in a 23. (a) The cell is the basic structural and Golgi complex. functional unit of living organisms. In 1855, Rudolf Virchow showed that all cells arise The Golgi cisternae are concentrically from the pre-existing cells by cell division arranged near the nucleus with distinct or Omnis-cellula-e-cellula. Robert Hooke convex cis or the forming face and concave trans or the maturing face. was the first one to find out the basic units of life and termed them as cells. Anton van Tubules are long flattened structure while Leeuwenhoek was the one who observed vesicles are round or oval structure. unicellular organisms including bacteria. 28. (b) Acentric chromosome may be lost during Robert Brown described the nucleus as a cell division as centromere is absent in characteristic spherical body in plant cells. them. During metaphase and anaphase, spindle fibres are not attached with them 24. (a) Genes present in the cytoplasm of and so they are not able to reach to the eukaryotic cells are found in mitochondria poles. and inherited via egg cytoplasm. 29. (b) Protein synthesis takes place in ribosomes, 25. (c) Chromoplasts are plastids responsible for which are attached to surface of pigment synthesis and storage. They, like endoplasmic reticulum by ribophorin-I and all other plastids (including chloroplasts ribophorin-II. About 50 hydrolytic enzymes and leucoplasts), are organelles found in are found in the lysosome. They include specific photosynthetic eukaryotic proteases, nucleases, glycosidases, lipases, species. Chloroplasts conduct phospholipases, phosphatases and photosynthesis. Chloroplasts absorb light sulphatases. All lysosomal enzymes are acid and use it in conjunction with water and hydrolases and optimally active at pH-5.0 carbon dioxide to produce sugars. 30. (a) The special membranous structure formed Leucoplasts are non-pigmented, in contrast by the extension of prokaryotic plasma to other plastids such as the chloroplast. membrane is known as mesosome while Lacking pigments, leucoplasts are not polysome is structure formed by green, so they are predictably located in combination of many ribosomes. roots and non-photosynthetic tissues of SER is the major site of synthesis of lipids. plants. They may become specialized for The site of protein synthesis is RER. bulk storage of starch, lipid or protein and are then known as amyloplasts, elaioplasts, Type B : Assertion Reason Questions or proteinoplasts respectively. 31. (b) Mitochondria are called power house of a 26. (b) Pinocytosis, or pinching in of the cell cell because they produce large amount of membrane, allows cells, such as the energy in the form of ATP. Paramecium, to capture larger food IIT JEE NEET eBooks: www.crackjee.xyz

Cell : The Unit of Life B-51

32. (a) Animal cells are covered by semipermeable 38. (c) plasma membrane. Cell wall is absent in 39. (a) The cell membrane also called the plasma animal because cell wall is incompatible with membrane, plasmalemma, or “phospholipid the way in which an animal moves and bilayer” is a selectively permeable lipid grows. bilayer found in all cells. It contains a wide 33. (a) Cells are the basic structural and functional variety of biological molecules, primarily unit of organism. proteins and lipids, which are involved in a 34. (a) Specialization of the cell increases the vast array of cellular processes such as cell efficiency of the cell for a particular adhesion, ion channel conductance and function. cell signaling. The plasma membrane also 35. (d) The size and shape of the cell in serves as the attachment point for both the multicellular organism depends upon the intracellular cytoskeleton and, if present, location and function performed by them. the extracellular cell wall. 36. (b) Eukaryotic cells contain three types of 40. (a) The centrosome is the main place where filaments as microtubules, microfilament cell microtubules get organized. and intermediate filament which give Centrosome usually contains two definite shape to the cell and also helps in cylindrical structure called centrioles. directional movement. Centrioles are composed of grouping of 37. (c) Lipids present in the outer and inner side of microtubules arranged in 9 + 3 pattern. The the bilayer are commonly different, e.g., pattern is so named because a ring of 9 lecithin on the outer side and cephalin on microtubule "triplets" are arranged at right the inner side of erythrocyte membrane. angles to one another. Centrioles, found Oligosaccoharides are attached to external in animal cells, help to organize the surface of lipids and proteins of a bio-membrane. They are absent on the inner assembly of microtubules during cell side. division. Centrioles replicate during the interphase stage of mitosis and meiosis. IIT JEE NEET eBooks: www.crackjee.xyz EBD_7100

B-52 Topicwise AIIMS Solved Papers – BIOLOGY 9 Biomolecules

TYPE A : MULTIPLE CHOICE QUESTIONS 10. Which of the following set of three items are not true as each set belongs to the category 1. Which is the derivative of amino acid ? [1999] mentioned against them [2005] (a) Epinephrine (b) Estrogen (a) Lysine, glycine, thiamine - Amino acids (c) Progesterone (d) All of these (b) Myosin, oxytocin and gastrin - Hormones 2. High energy bond of ATP are present in between (c) Rennin, helicase and hyaluronidase - [1999] Enzyme (a) C – C (b) C – O (d) Optic nerve, occulomotor, vagus - Sensory (c) C – N (d) O – P nerves 3. Who coined the term zymase? [1999] 11. The figure given below show three velocity- (a) Pasteur (b) Buchner substrate concentration curves for an enzyme reaction. What do the curves a, b and c depict (c) Kuhne (d) Sumner respectively? [2006] 4. Apoenzyme is [2000] (a) protein (b) lipid (c) sugar (d) vitamin a b 5. Gamma globulins are synthesized inside 0 c V y t (a) liver [2000] i c o l e

(b) kidney v l a i t

(c) bone marrow i n I (d) lymph and lymphoid tissues Substrateconcentration[S] 6. Proteins are [2002] (a) a - normal enzyme reaction, (a) polysaccharides (b) polyamides b - competitive inhibition, (c) polynucleotides (d) polyglycol c - non-competitive inhibition. 7. Which of the following gives Fehling’s test? (b) a - enzyme with an allosteric modulator (a) Pectin (b) Sucrose [2002] added, (c) Cellulose (d) Glucose b - normal enzyme activity, 8. The nicotinamide is synthesized in our body c - competitive inhibition. from [2002] (c) a - enyzme with an allosteric stimulator, (a) tryptophan (b) tryosine b - competitive inhibitor added, (c) valine (d) alanine c - normal enzyme reaction. 9. An example of competitive inhibition of an (d) a - normal enzyme reaction, enzyme is the inhibition of [2003] b - non-competitive inhibitor added, (a) succinic dehydrogenase by malonic acid c - allosteric inhibitor added. (b) cytochrome oxidase by cyanide 12. Which of the following contain E-l, 4 linkage? (c) hexokinase by glucose-6-phosphate (a) Maltose (b) Sucrose [2007] (d) carbonic anhydrase by carbon dioxide (c) Lactose (d) Fructose IIT JEE NEET eBooks: www.crackjee.xyz

Biomolecules B-53 13. Which statement is true? [2007] 20. The Km value of the enzyme is the value of (a) Adenine has 4 nitrogen atoms. the substrate concentration at which the (b) Cytosine has 3 nitrogen atoms. reaction reaches to [2014] (c) Guanosine has 3 nitrogen atoms. (a) Zero (b) 2 Vmax (d) Uracil has 5 nitrogen atoms. (c) ½ Vmax (d) ¼ Vmax 14. Michaelis constant Km is equal to [2010] 21. The following diagrams represent the K1 KK23 nitrogenous bases of nucleic acid molecules. (a) KK– (b) K 23 1 Identify the correct combination KK23– KK12u (c) (d) K1 K3 15. Alpha-keratin is a protein present in [2010] (a) blood (b) skin (c) lymph (d) eggs 16. Which one of the following statements regarding starch and cellulose is not correct? [2010] (a) Both of them are of plant origin. (b) Both of them are polymers. (c) Both of them give colour with iodine. (d) Both of them are made up of glucose molecules. 17. Which of the following type of enzyme is not matched correctly with the molecule that it breaks [2015] down? [2013] (a) A-uracil, B-adenine, C-thymine, D-guanine, (a) Amylase–starch (b) Lipase–starch E- cytosine (c) Protease–proteins (b) A - uracil, B-guanine, C-cytosine, (d) Disaccharidase–sugars D-adenine, E-thymine 18. The diagram illustrates energy changes in an (c) A-uracil, B - guanine, C-thymine, enzyme controlled reaction. [2013] D-adenine, E-cytosine (d) A-thymine, B-guanine, C-uracil, D-adenine,

y Z E-cytosine. g reactants r

e Y

n 22. The given graph shows the effect of substrate e products X concentration on the rate of reaction of the reaction enzyme green -gram -phosphatase. What does Which of the following represents the lowering the graph indicate ? of the activation energy?

(a)X (b) Y ) v ( (c)Z (d) Z – Y y t i

19. Which one of the following is a non - reducing c o l

carbohydrate? [2014] e (a) Maltose V (b) Sucrose O (c) Lactose Substrate Concentration (s) (d) Ribose 5 - phosphate [2015] IIT JEE NEET eBooks: www.crackjee.xyz EBD_7100

B-54 Topicwise AIIMS Solved Papers – BIOLOGY

(a) The rate of enzyme reaction is directly (c) If the Assertion is correct but Reason is proportional to the substrate concentration incorrect. (d) If both the Assertion and Reason are incorrect. (b) Presence of an enzyme inhibitor in the (e) If the Assertion is incorrect but the Reason is reaction mixture correct. (c) Formation of an enzyme-substrate complex 26. Assertion : Enzymes have active sites and (d) At higher substrate-concentration the pH substrates have reactive sites on their surface increases. respectively. 23. Inorganic catalyst work efficiently at Reason : Active and reactive sites push the ______temperature and ______pressure. enzyme and substrate molecules away from each [2016] other. [1999] (a) high, low (b) low, low 27. Assertion : Vegetable oils are fats which are (c) low, high (d) high, high present in plant cells in soluble form. 24. Refer the given structure of adenylic acid. In Reason : Vegetable oils occur only in cells of this identify A. embryo. [2007] O Directions for (Qs. 28-37) : Each of these questions || O contains an Assertion followed by Reason. Read them HO – P – OCH N-base | 2 carefully and answer the question on the basis of OH A following options. You have to select the one that best describes the two statements. (a) If both Assertion and Reason are correct and [2016] Reason is the correct explanation of Assertion. (a) Glycosidic bond (b) Phosphate bond (b) If both Assertion and Reason are correct, but (c) Ester bond (d) Ionic bond Reason is not the correct explanation of 25. Nucleotides are building blocks of nucleic acids. Assertion. Each nucleotide is a composite molecule formed (c) If Assertion is correct but Reason is incorrect. by [2017] (d) If both the Assertion and Reason are incorrect. (a) base-sugar-phosphate. 28. Assertion : Human diet should compulsorily (b) base-sugar-OH. contain glycine, serine and tyrosine. [2010]

(c) (base-sugar-phosphate)n. Reason : Essential amino acids can not be (d) sugar-phosphate. synthesized in the human body. TYPE B : ASSERTION REASON QUESTIONS 29. Assertion : Unsaturated fats are more reactive compared with the saturated fats. [2010] Directions for (Qs. 26-27) : These questions consist Reason : Unsaturated fats have only single of two statements, each printed as Assertion and bonds in their structure. Reason. While answering these questions, you are 30. Assertion : The amino acid glycine comes under required to choose any one of the following five the category of nonessential amino acids. responses. Reason : This is due to the fact that it can not be (a) If both Assertion and Reason are correct and synthesised in the body. [2011] the Reason is a correct explanation of the Assertion. 31. Assertion : Allosteric enzymes show feed back inhibition. (b) If both Assertion and Reason are correct but Reason is not a correct explanation of the Reason : The inhibitor is competitive. [2012] Assertion. IIT JEE NEET eBooks: www.crackjee.xyz

Biomolecules B-55

32. Assertion : Coenzymes serve as co-factors in a Reason : A complete, catalytically active enzyme number of different enzyme catalyzed reactions. together with its bound prosthetic group is Reason : Coenzymes and prosthetic groups are called apoenzyme. [2016] cofactors. [2013] 36. Assertion : Glycosidic bonds are formed by 33. Assertion : Enzymes lower the activation dehydration. energy. Reason : In polysaccharides, individual Reason : A substrate molecule can be acted upon monosaccharide is linked by glycosidic bond. by a particular enzyme. [2014] [2016] 34. Assertion : Comparative biochemistry provides 37. Assertion : In a DNA molecule, A–T rich parts a strong evidence in favour of common ancestory melt before G–C rich parts. of living beings. Reason: In between A and T there are three Reason : Genetic code is universal. [2015] H–bond, whereas in between G and C there are 35. Assertion : A co-enzymes or metal ions that is two H-bonds. [2017] very tightly bound to enzyme protein is called prosthetic group. IIT JEE NEET eBooks: www.crackjee.xyz EBD_7100

B-56 Topicwise AIIMS Solved Papers – BIOLOGY

Type A : Multiple Choice Questions competitive inhibitor decreases the vmax of the reaction i.e. it can not be overcome by 1. (a) Epinephrine is derived from tyrosine amino increase in substrate concentration. acid. 12. (c) Lactose or milk sugar, found exclusively in 2. (d) High energy bonds of ATP are between milk, contain E-1,4 linkage. It is a O~P. disaccharide formed by combination of 3. (b) Buchner coined the term zymase for the galactose and glucose by means of a complex of biocatalysts extracted from chemical reaction called as condensation yeast and taking part in alcoholic reaction. fermentation. 13. (a) Nucleotides contain carbon, hydrogen, 4. (a) Apoenzyme is the protein part of oxygen, nitrogen and phosphorous. holoenzyme. Nucleotides are either purines or 5. (a) Gamma globulins (J) are synthesized by pyrimidines. Adenine and guanine are the B-lymphocytes and stem cells found in the two purines which are the 9-membered liver during foetal stage and bone marrow double ringed compound where each ring cells in the adult stage. possesses four nitrogen atoms. Thymine, 6. (b) Proteins are the polymers of basic units uracil, and cytosine are the pyrimidines amino acids and hence are polyamides. which are single ringed nitrogenous compounds. 7. (d) Glucose is an aldehydic sugar and their free –CHO part converts Cu+2 to Cu+1 14. (b) Michaels constant Km is equal to

(Fehling's reagent). KK23 . 8. (a) The enzyme nicotinamide can be K1 synthesized in small quantities from amino 15. (b) Alpha-keratin is present in high quantity in acid, tryptophan. skin and epidermal appendages like hair and 9. (a) Enzyme inhibition caused by a substance nail. resembling substrate molecule through 16. (c) Starch is the reserve substance in plant cells blocking its active site is competitive whereas cellulose is the most important inhibition. Malonate closely resembles structural component of the cell wall of succinate in structure that inhibits the plants. Both starch and cellulose are action of succinic dehydrogenase. polymers of glucose. Starch is a branched 10. (c) Thiamine is a nitrogen base; myosin is a polymers of D-D glucose units which are muscle protein; occulomotor nerve is motor linked by D–1, 4 glycosidic bonds (but at and vagus is mixed type. branching, by D–1, 6 glycosidic bonds). 11. (a) The effect of a competitive inhibitor is Cellulose is a linear polymer of E–D– reversed by increasing substrate glucose units connected through E-1, 4 concentration. At a sufficiently high glycosidic bonds. In contrast to starch and substrate concentration, the reaction glycogen, cellulose is insoluble in ordinary solvents and is not hydrolysed by boiling velocity reaches the vmax observed in the absence of inhibitor whereas non- dilute acids. It gives no colour with iodine. IIT JEE NEET eBooks: www.crackjee.xyz

Biomolecules B-57

17. (b) Lipases are enzymes found in the small 27. (d) Vegetable oils and fats are present in plants intestine of humans that help in the in insoluble form. They are extracted mostly breakdown of fats. from seeds. In several cereals, they are 18. (d) The energy required to initiate a reaction is obtained from embryo. Olive and palm oils known as the activation energy, Ea. The are obtained from flesly pericarp of the fruit. bold curve shows the uncatalysed reaction Sometimes oils are also extracted from with Ea = Z, whilst Y represents the Ea of roots, stem and leaves. the catalysed reaction (dotted curve). The 28. (d) Essential amino acids are those which are Ea is thus lowered by (Z – Y). taken from food and not synthesized in the 19. (b) Sucrose is classified under non-reducing body whereas non-essential amino acids sugar because it does not have any free need not be supplied in the diet and are aldehyde or keto group. synthesized in the body. Glycine, serine and tyrosine are non-essential amino acids. 20. (c) The concentration of substrate at which 29. (c) Compound having double bond in their velocity of enzymatic action reaches half structure are more unstable compound in of its maximum value, is called Km value or comparison to single bond holder Michaelis constant. compounds. Unsaturated fats those have 21. (a) double bonds in their structures are more 22. (b) Even though the substrate concentration reactive than saturated fats. increases the velocity is decreasing there by showing a presence of inhibitor. 30. (c) Non-essential amino acids are those amino acids which need not be supplied in the diet 23. (d) Inorganic catalyst work efficiently at high because they can be synthesised by the temperature and high pressure. Inorganic body, particularly from carbohydrate catalysts speed up reactions, but they do metabolites, Glycine is one such non not have carbon-hydrogen atoms. An essential amino acid. On the contrary, example of this is magnesium sulphate, essential amino acids are those amino acids which is a compound used to speed up which can not be synthesised in the animal some reactions in the chemistry lab. body and must be supplied with food in 24. (c) Phosphate is bound to pentose sugar by adequate amounts. Out of twenty amino ester bond. acids, eight are considered essential in 25. (a) Nucleotides are the building blocks of human diet. nucleic acid. Each nucleotide consists of 31. (c) Feed back inhibition is a type of reversible three parts: a sugar (ribose for RNA and inhibition found in allosteric enzymes. The deoxyribose for DNA), a phosphate, and a inhibitor is noncompetitive and is usually a nitrogenous base. low molecular intermediate or product of Type B : Assertion Reason Questions metabolic pathway having a chain of reactions involving a number of enzymes. 26. (d) Enzymes are biocatalyst that increases the 32. (b) Cofactor may be inorganic or organic in speed of a chemical reaction without nature. Organic cofactors are of two types, themselves undergoing any permanent coenzymes and prosthetic groups. chemical change. Enzymes have active Coenzymes are easily separable nonprotein sites to bind the substrate during catalyzed organic cofactors. Prosthetic groups are reaction. IIT JEE NEET eBooks: www.crackjee.xyz EBD_7100

B-58 Topicwise AIIMS Solved Papers – BIOLOGY

non-protein organic cofactors firmly a protein that forms an active enzyme system attached to apoenzymes (protein part of by combination with a coenzyme and enzyme). determines the specificity of this system for 33. (b) Activation energy is an external supply of a substrate. energy which is needed for the initiation of 36. (b) In polysaccharides, individual the chemical reaction. Activation energy monosaccharide is linked by glycosidic required for such a large number of reactions bond. This bond is formed between two cannot be provided by living systems. carbon atoms of two adjacent Enzymes lower the activation energy monosaccharides. A glycosidic bond is a required for a reaction. Enzymes are type of covalent bond that joins a generally specific for their substrates. carbohydrate molecule to another group, 34. (b) Comparative biochemistry provides a strong which may or may not be another evidence for common ancestors of living carbohydrate. Glycosidic bonds are formed beings (e.g. proteins lymph, enzymes, by dehydration. hormones, blood groups etc.) 37. (c) In a DNA molecule, A-T rich parts melt 35. (c) Prosthetic group is an organic substance before G-C rich parts because there are two which is thermostable and firmly attached H-bond between A and T whereas in to the protein or apoenzyme portion during between G and C, there are three H-bond. the complete catalytic cycle. Apoenzyme is IIT JEE NEET eBooks: www.crackjee.xyz

Cell Cycle and Cell Division B-59 10 Cell Cycle and Cell Division

7. Many cells function properly and divide TYPE A : MULTIPLE CHOICE QUESTIONS mitotically even though they do not have 1. The correct sequence in cell cycle is [1999] [2011] (a) S - G - G - M (b) S - M - G - G (a) plasma membrane 1 2 1 2 (b) cytoskeleton (c)G - S - G - M (d) M - G - G - S 1 2 1 1 2 (c) mitochondria 2. Colchicine prevents the mitosis of cells at which (d) plastids of the following stage? [2000] 8. The stage of meiosis where centromere separate (a) Anaphase (b) Metaphase [2013] (c) Prophase (d) Interphase (a) metaphase I (b) metaphase II 3. Spindle fibres of mitotic cells are made up of (c) anaphase I (d) anaphase II (a) tubulin (b) actin [2001] 9. During meiosis I, the chromosomes start pairing (c) myosin (d) collagen at [2014] 4. When synapsis is complete all along the (a) Leptotene (b) Zygotene chromosome, the cell is said to have entered a (c) Pachytene (d) Diplotene 10. How many mitotic divisions are needed for a stage called [2005] single cell to make 128 cells? [2016] (a) zygotene (b) pachytene (a)7 (b) 14 (c) diplotene (d) diakinesis (c) 28 (d) 64 5. Which one of the following precedes re- formation of the nuclear envelope during M- 11. Match the description (given in column I) with phase of the cell cycle? [2008] correct stage of prophase I (given column II) (a) Decondensation from chromosomes and and choose the correct option. [2017] reassembly of the nuclear lamina. Column I Column II (b) Transcription from chromosomes and A. Chromosomes are I. Pachytene reassembly of the nuclear lamina. moved to spindle (c) Formation of the contractile ring and equator formation of the phragmoplast. B. Centromere splits and II. Zygotene (d) Formation of the contractile ring and chromatids move apart transcription from chromosomes. C. Pairing between III. Anaphase 6. During which stages (or prophase I substages) homologous of meiosis do you expect to find the bivalents chromosomes takes and DNA replication respectively? [2009] place (a) Pachytene and interphase (between two D. Crossing between IV. Metaphase meiotic divisions) homologous (b) Pachytene and interphase (just prior to chromosomes prophase I) (a) A – I; B – II; C – III; D – IV (c) Pachytene and S phase (of interphase just (b) A – II; B – III; C – IV; D – I prior to prophase I) (d) Zygotene and S phase (of interphase prior (c) A – IV; B – III; C – II; D – I to prophase I) (d) A – III; B – I; C – IV; D – II IIT JEE NEET eBooks: www.crackjee.xyz EBD_7100

B-60 Topicwise AIIMS Solved Papers – BIOLOGY

TYPE B : ASSERTION REASON QUESTIONS (a) If both Assertion and Reason are correct and Reason is the correct explanation of Assertion. Directions for (Q. 12) : These questions consist of (b) If both Assertion and Reason are correct, but two statements, each printed as Assertion and Reason. Reason is not the correct explanation of While answering these questions, you are required to Assertion. choose any one of the following five responses. (c) If Assertion is correct but Reason is incorrect. (a) If both Assertion and Reason are correct and (d) If both the Assertion and Reason are incorrect. the Reason is a correct explanation of the 13. Assertion : Meiosis II is known as equational Assertion. or homotypic division. [2010] (b) If both Assertion and Reason are correct but Reason :Meiosis II produces same number of Reason is not a correct explanation of the chromosome in cell. Assertion. 14. Assertion : Interphase is resting stage. (c) If the Assertion is correct but Reason is Reason : The interphase cell is metabolically incorrect. inactive. [2012] (d) If both the Assertion and Reason are incorrect. 15. Assertion : During zygotene, chromosomes show bivalent stage. (e) If the Assertion is incorrect but the Reason is correct. Reason : Bivalent is half the number of chromosomes. [2013] 12. Assertion: Meiosis results in production of haploid cells. 16. Assertion : The stage between two mitotic Reason: Synapses occurs during leptotene. divisions is called interkinesis. [1998] Reason : Interkinesis is generally short lived. Directions for (Qs. 13-17) : Each of these questions [2016] contains an Assertion followed by Reason. Read them 17. Assertion : Diplotene is characterized by the carefully and answer the question on the basis of presence of chiasmata. following options. You have to select the one that Reason : Diplotene can last for months and years best describes the two statements. in oocytes of some vertebrates. [2016] IIT JEE NEET eBooks: www.crackjee.xyz

Cell Cycle and Cell Division B-61

Type A : Multiple Choice Questions phenomenon is called synapsis and it occurs during zygotene stage. DNA 1. (c) The correct sequence of cell cycle is G , S, 1 replication occurs during S phase or G and M. 2 synthetic phase which is the second phase 2. (c) Colchicine is an alkaloid derived from of interphase. (colchicum autumnale) arrests the spindle 7. (d) Many cells function properly and formation at the end of prophase. Hence, it divide mitotically even in the absence of is also called 'Mitotic Poison'. plastids. 3. (a) Spindle fibres of mitotic cells are made up 8. (d) of microtubules. It consists of protein 9. (b) During zygotene, a substage of Prophase I tubulin that maintain cell shape, serve a of meiosis I, where chromosomes start tracks for organelle movement & help in pairing together is called synapsis. Such cell division by getting attached to the paired chromosomes are called as centromeres of bivalents. They are homologous chromosomes. A complex arranged on equator due to congression structure i.e., synaptonemal complex is movements. The movement of bivalents or formed by a pair of synapsed homologous chromosomes occur towards the poles as chromosomes called a bivalent or a tetrad. a result of spindle fibres contraction. 10. (a) In mitosis, a single cell divides to form two 4. (b) Synapse stabilizes the paired condition of daughter cells. So, the number of mitotic chromosomes in zygotene stage. After this divisions can be calculated by 2n. the cell enters the pachytene stage. where, n is the number of division of cell. 5. (a) In most eukaryotes, the nuclear envelope 2n = 128 that separates the DNA from the cytoplasm 2n = 27 disassembles. The chromosomes align themselves in a line spanning the cell. n = 7 Microtubules, essentially miniature strings, Thus, 7 mitotic divisions are needed for a pulls out from opposite ends of the cell single cell to make 128 cells. and shorten, pulling apart the sister 11. (c) A – IV; B – III; C – II; D – I chromatids of each chromosome. As a Metaphase – Chromosomes are moved matter of convention, each sister chromatid to spindle fibre. is now considered a chromosome, so they Anaphase – Centromere splits and are renamed to sister chromosomes. As the chromatids move apart. cell elongates, corresponding sister Zygotene – Pairing between chromosomes are pulled toward opposite homologous chromosomes ends. A new nuclear envelope forms takes place. around the separated sister chromosomes. Pachytene – Crossing between 6. (d) In bivalent formation of chromosomes homologous during meiosis, the homologous chromosomes occurs. chromosomes are arranged in pairs. The IIT JEE NEET eBooks: www.crackjee.xyz EBD_7100

B-62 Topicwise AIIMS Solved Papers – BIOLOGY Type B : Assertion Reason Questions 15. (b) During zygotene, because of the pairing of the homologues, the nucleus contains half 12. (c) Synapsis occurs during zygotene stage. the number of chromosomes. Each unit is a Synapsis is the pairing of homologous bivalent composed of two homologous chromosomes which leads to formation of chromosomes. bivalents. 16. (d) Interkinesis or interphase II is a period of 13. (a) Meiosis II is known as equational or rest that cells of some species enter during homotypic division like mitosis. It ensures meiosis, between meiosis I and meiosis II. the maintenance of constant number of No DNA replication occurs during chromosomes from generation to interkinesis however it does occur during generation on a species. the interphase I stage of meiosis. 14. (c) Previously interphase is called resting stage Interkinesis is generally short lived. because there is no apparent activity 17. (b) Diplotene is the longest and most active related to cell division. The interphase cell subphase of prophase I of meiosis. The is metabolically quite active. Interphase beginning of diplotene is recognized by consist of three subphases (G1 , G2 and S). the dissolution of synaptonemal complex Synthesis of DNA occurs in S phase. G1 is and the tendency of the recombined the period between the end of mitosis and homologous chromosomes of the the start of S phase. G2 is the phase bivalents to separate from each other between S phase and the next mitosis. As except at the sites of crossovers. These the synthesis of DNA and proteins occurs X-shaped structures are called chiasmata. in interphase so, it is considered as Diplotene can last for months and years in metabolically active phase. oocytes of some vertebrate. IIT JEE NEET eBooks: www.crackjee.xyz

11 Transport in Plants

TYPE A : MULTIPLE CHOICE QUESTIONS 8. Water potential of pure water and its solution are [2013] 1. Translocation of organic materials in plants is (a) 0 and 1 explained by [1997] (b) 0 and 0 (c) 0 and more than one (a) active transport (d) 0 and less than 1. (b) transpiration pull 9. In which method of transport in plasma (c) inhibition theory membrane does not require carrier molecule? (d) mass flow hypothesis [2014] 2. The main function of phloem is translocation of (a) Active transport (b) Facilitated diffusion (a) food (b) water [1998] (c) Simple diffusion (c) mineral (d) air (d) Na+ – K+ pump 3. In rainy season, door gets swelled due to [2001] 10. Seed increase in its volume by the adsorption of (a) imbibition (b) diffusion water through [2014] (c) transpiration (d) respiration (a) Osmosis (b) Plasmolysis (c) Imbibition (d) Diffusion 4. Which of the following helps in ascent of sap? 11. Minerals are known to enter the plant root by [2007] means of a number of mechanisms, including all (a) Root pressure (b) Transpiration except one of the following. Which one of the ( c) Capillarity (d) All of these following is NOT a mechanism for moving 5. Hydroponics is [2007] minerals into roots? [2015] (a) Foliar feeding (b) Active transport (a) nutrient less culture (c) Proton (H+) pump (d) Cation exchange (b) water less culture 12. A botanist discovered a mutant plant that was (c) soilless culture unable to produce materials that form casparian (d) none of these strip. This plant would be [2015] 6. During Na+ – K+ pump [2010] (a) unable to transport water or solutes to the (a) 3Na+ and 2K+ are transported leaves. (b) unable to use its sugar as a sugar sink. (b) 1Na+ and 2K+ are transported (c) able to exert greater root pressure than the + + (c) 3 Na and 3K are transported normal plant. (d) Depends on requirement of cell (d) unable to control amounts of water and solutes 7. Excessive loss of water causes wilting of leaves, it absorbs. it can be prevented by : [2012] 13. If a cell A with D.P.D. 4 bars is connected to cell B, C, D whose O.P. and T.P. are respectively 4 (a) Keeping the plant in bright light and 4, 10 and 5 and 7 and 3 bars, the flow of (b) Spraying the plant with alcohol water will be [2015] (c) Applying vaseline on the leaf surface (a) A and D to B and C (d) Adding high amounts of fertilizers to the (b) A to B, C and D soil (c) B to A, C and D (d) C to A, B and D IIT JEE NEET eBooks: www.crackjee.xyz EBD_7100

B-64 Topicwise AIIMS Solved Papers – BIOLOGY

14. A boy is studying transport of a certain type of (d) If both the Assertion and Reason are incorrect. molecules into cell. He finds that transport slows (e) If the Assertion is incorrect but the Reason is down when the cells are poisoned with a correct. chemical that inhibits energy production. Under 16. Assertion : Waxy and cutin coating on plant normal circumstances, the molecules studied by parts reduce the transpiration. the boy is probably transported by [2016] Reason : These adaptation are found in (a) simple diffusion xerophytes. [1999] (b) osmosis 17. Assertion : Water and mineral uptake by root (c) active transport hairs from the soil occurs through apoplast until (d) facilitated diffusion it reaches endodermis. 15. Which of the following statements is/are not Reason : Casparian strips in endodermis are incorrect? [2017] suberized. [2003] (i) Water and minerals, and food are generally 18. Assertion :When the ambient temperature is moved by a mass or bulk flow system. high and soil contains excess of water, the plants (ii) Bulk flow can be achieved either through a tend to lose water in the form of droplets from positive hydrostatic pressure gradient or a lenticels. negative hydrostatic pressure gradient. Reason : Root pressure regulates the rate of loss (iii) The bulk movement of substances through of water form lenticels. [2006] the conducting tissues of plants is called Directions for (Qs.19-21) : Each of these questions translocation. contains an Assertion followed by Reason. Read them (iv) Xylem translocates organic and inorganic carefully and answer the question on the basis of solutes, mainly from roots to the aerial parts following options. You have to select the one that of the plants. best describes the two statements. (v) Phloem translocates water, mineral salts, some organic nitrogen and hormones, from (a) If both Assertion and Reason are correct and the leaves to other parts of the plants. Reason is the correct explanation of Assertion. (b) If both Assertion and Reason are correct, but (a) (ii), (iii) and (v) Reason is not the correct explanation of (b) (ii), (iii) and (iv) Assertion. (c) (iv) and (v) (c) If Assertion is correct but Reason is incorrect. (d) (ii) and (v) (d) If both the Assertion and Reason are incorrect. TYPE B : ASSERTION REASON QUESTIONS 19. Assertion : Upward movement of water is called ascent of sap. Directions for (Qs. 16-18) : These questions consist Reason : Upward movement of water occurs of two statements, each printed as Assertion and through xylem and phloem. [2013] Reason. While answering these questions, you are required to choose any one of the following five 20. Assertion : Long distance flow of photo responses. assimilates in plants occurs through sieve tubes. Reason : Mature sieve tubes have parietal (a) If both Assertion and Reason are correct and cytoplasm and perforated sieve plates. the Reason is a correct explanation of the [2012, 2015] Assertion. 21. Assertion : Light is very important factor in (b) If both Assertion and Reason are correct but transpiration. Reason is not a correct explanation of the Reason : Light induces stomatal opening and Assertion. darkness closing of stomata. Therefore, (c) If the Assertion is correct but Reason is transpiration increases in light and decreases in incorrect. dark. [1999, 2015] IIT JEE NEET eBooks: www.crackjee.xyz

Transport in Plants B-65

Type A : Multiple Choice Questions 9. (c) 1. (a) Active transport is the mediated transport 10. (c) Imbibition is the process of adsorption of of biochemicals, and other atomic/ water by hydrophilic surfaces of a molecular substances, across membranes. substance without forming a solution. It is Unlike passive transport, this process a type of diffusion by which movement of requires chemical energy in the form of water takes place along a diffusion gradient. adenosine triphosphate(ATP). In this form The solid particles which adsorb water or of transport, molecules move against either any other liquid are called Imbibants. The an electrical or concentration gradient liquid which is imbibed is known as (collectively termed an electrochemical Imbibate. Examples are absorption of water gradient). by seeds and dry wood. 2. (a) Phloem is the chief food conducting tissue 11. (a) Potassium is accumulated by passive of vascular plants responsible for transport. Some solutes are pumped across translocation of organic solutes. membranes using active transport. The role 3. (a) In rainy season, door gets swelled due to of proton pumps in the transport process the phenomenon of imbibition. It is the of plant cells is a specific application of process of absorption of water without chemiosmosis, a transmembrane proton forming a solution. gradient that links energy-releasing 4. (d) Upward movement of water (sap) through processes to energy-consuming ones like xylem against the force of gravity is called active transport. ascent of sap. All three help in ascent of 12. (d) The casparian strips function in regulation sap. of the flow of water between outer tissues 5. (c) Cultivation of plants by placing the roots and the vascular cylinder. in the nutrient solution without any soil is 13. (c) DPD = OP – TP. called hydroponics. It is also known as soilless culture/ water culture/ solution DPD (A) = 4 bar. culture. It is used to determine which DPD (B) = 0 bar. elements are essential for plant growth and DPD (C) = 10 – 5 = 5 bars. what symptoms are produced by the DPD (D) = 7 – 3 = 4 bars. absence or deficiency of essential elements. Water always flows from lower DPD 6. (a) During sodium-potassium pump, the (Diffusion Pressure Deficit) to higher DPD. concentration of sodium ions will be about Since the DPD of cell B is the lowest (O) the 14 times more in extra cellular fluid (outside) water will flow from B to A and then to C. and concentration of potassium ions will 14. (c) Active transport uses energy (ATP) to be about 28-30 times more in axoplasm pump molecules against a concentration + + (inside). Thus, 3Na and 2K are transported gradient. Cells undergoing active transport during the process. bear abundant mitochondria to provide 7. (c) Excessive loss of water from the leaves can ATP, needed to power active transport. So, be prevented by applying vaseline on leaf the production of ATP is blocked or surface. It will close the stomata and check decreased and active transport is blocked transpiration. or slow down. 8. (d) 15. (c) Statements (iv) and (v) are not correct. IIT JEE NEET eBooks: www.crackjee.xyz EBD_7100

B-66 Topicwise AIIMS Solved Papers – BIOLOGY

(iv) Xylem is associated with the sap from cut stems and guttation of water translocation of mainly water, mineral from leaves. The pressure is generated by salts, some organic nitrogen and the concentration of solutes in the xylem hormones from roots to the aerial parts of the root and stem which is then causes of the plants. water to move into the xylem by osmosis. 19. (c) Sap is water with dissolved ingredients. (v) Phloem translocates a variety of The upward movement of water from roots organic and inorganic solutes mainly towards the tips of stem branches and their from the leaves to other parts of the leaves is called ascent of sap. It occurs plants. through the tracheary elements of xylem. Type B : Assertion Reason Questions 20. (a) The parietal cytoplasm and perforated sieve plates help in the transport of 16. (a) Waxy and cutin coating does reduce photoassimilates that are required for transpiration. This adaptive feature is seen photosynthesis. The parietal cytoplasm is in xerophytic and in plants of semi-arid the streaming of cytoplasm. The streaming region. This adaptive feature is seen to reduce water loss by transpiration. cytoplasm moves throughout the cell, thus, 17. (a) The radial and the inner walls of the cells helping in the transport of photoassimilates of the endodermis are greatly thickened. from one cell to another through the sieve These are called as casparian strips and plates. these are impervious to water. Apart from 21. (a) Light is an important factor in transpiration. the casparian strips, and suberization also The stomata opens well on days when light does not allow the water to reach the endodermis. Hence, water moves through is brighter. It is also evident on cloudy days the apoplast which are passage cells. that the stomata does not open well. Hence, 18. (d) Root pressure is a pressure produced in light induces stomatal opening and the roots of plants, causing exudation of darkness closing. IIT JEE NEET eBooks: www.crackjee.xyz

12 Mineral Nutrition

TYPE A : MULTIPLE CHOICE QUESTIONS D. Chlorine IV. Required to activate 1. Leghaemoglobin helps in [2007] respiratory (a) nitrogen fixation enzyme. (b) protecting nitrogenase from O2 E. Manganese V. Required for (c) destroys bacteria synthesis (d) transport of food in plants of mitotic spindle. 2. Which of the following is correct set of (a) A – 1; B – 2; C – 3; D – 4; E – 5 micronutrient for plants? [2007, 2011] (b) A – 5; B – 4; C – 3; D – 2; E – 1 (a) Mg, Si, Fe, Cu, Ca (b) Cu, Fe, Zn, B, Mn (c) A – 4; B – 1; C – 5; D – 3; E – 2 (c) Mg, Fe, Zn, B, Mn (d) A – 5; B – 3; C – 2; D – 1; E – 4 (d) Mo, Zn, Cl, Mg, Ca 7. Which of the following is an INCORRECT 3. In glycolysis, glucose molecule is converted into: match of essential element and function? [2011] [2014] (a) Manganese - structural component of (a) PEP (b) RuBP chlorophyll. (c) Acetyl CoA (d) Pyruvic acid (b) Calcium - component of the middle lamella. 4. Which one of the following elements is not an (c) Zinc - enzyme activator. essential micronutrient for plant growth? [2012] (d) Iron - component of ferredoxin. (a) Ca (b) Mn 8. Which of the following is the mismatched pair? (c) Zn (d) Cu Mineral Form that is 5. Which element plays an important role in elements absorbed nitrogen fixation? [2012] by plant – (a) Mn (b) Mo (a) Nitrogen NO3 – (c) Zn (d) Cu (b) Phosphorus H2PO4 6. Match column-I and Column-II and choose the (c) Sulphur H2SO4 correct option given below the columns.[2013] (d) Iron Fe3+ Column-I Column-II [2016] (Element) (Function) TYPE B : ASSERTION REASON QUESTIONS A. Calcium I. Required for ionic- balance. Directions for (Qs. 9-10) : These questions consist B. Boron II. Essential for of two statements, each printed as Assertion and constitution Reason. While answering these questions, you are of nucleic acid required to choose any one of the following five responses. C. Phosphorus III. Required for absorption of (a) If both Assertion and Reason are correct and calcium. the Reason is a correct explanation of the Assertion. IIT JEE NEET eBooks: www.crackjee.xyz EBD_7100

B-68 Topicwise AIIMS Solved Papers – BIOLOGY

(b) If both Assertion and Reason are correct but Directions for (Q.11) : Each of these questions Reason is not a correct explanation of the contains an Assertion followed by Reason. Read them Assertion. carefully and answer the question on the basis of (c) If the Assertion is correct but Reason is following options. You have to select the one that incorrect. best describes the two statements. (d) If both the Assertion and Reason are incorrect. (a) If both Assertion and Reason are correct and (e) If the Assertion is incorrect but the Reason is Reason is the correct explanation of Assertion. correct. (b) If both Assertion and Reason are correct, but 9. Assertion : Plants lack excretory organs. Reason is not the correct explanation of Reason : Plant usually absorb essential nutrients Assertion. and lead a passive life. [1997] (c) If Assertion is correct but Reason is incorrect. (d) If both the Assertion and Reason are incorrect. 10. Assertion : Plants absorb sulphur in the form of 11. Assertion : Deficiency of sulphur causes sulphate ions. chlorosis in plants. Reason : Sulphur bacteria are required for the Reason : Sulphur is a constituent of chlorophyll, formation of sulphate. [2007] protein and nucleic acids. [2004, 2014] IIT JEE NEET eBooks: www.crackjee.xyz

Mineral Nutrition B-69

Type A : Multiple Choice Questions Molybdenum availability varies with soil type, being highest in organic soil, less in 1. (b) The root nodules of leguminous plants clay, least in sandy soil. contain a symbiotic nitrogen fixing bacteria Rhizobium. Root nodules are small irregular 6. (d) outgrowth of the roots which are pinkish 7. (a) Magnesium is a constituent of the ring internally due to presence of a pigment structure of chlorophyll. Function of called leghaemoglobin. It is related to blood manganese is to activate many enzymes pigment haemoglobin. The cells of root involved in photosynthesis, respiration and nodules are tetraploid and contain nitrogen metabolism. The best defined function polyhedral bacteria called bacteroids. of manganese is in the splitting of water to Leghaemoglobin is an oxygen scavenger liberate oxygen during photosynthesis. and protects the nitrogen fixing enzyme 8. (c) Plants obtain sulphur in the form of 2– nitrogenase of bacteroids. sulphate (SO4 ). 2. (b) The essential elements are divided into Type B : Assertion Reason Questions macroelements and microelements based on the quantity in which they are required 9. (b) Plants do lack excretory organs, but it is by the plants. Mn, Cu, Mo, Zn, B, and Cl not due to absorption pattern or passive are the micronutrients needed in very small life. The carbon dioxide during respiration quantities by the plants. C, H, O, N, P, S, K, passes out through the stomata. The other Ca, Mg, Fe are the macronutrients required waste materials comes out in the form of alkaloids, gums and resins. in more quantity. 10. (a) Sulphur is a constituent of amino acids 3. (d) In this process, one molecule of glucose (cystein and cystine and methionine). undergoes partial oxidation to form 2 Sulphur is present in the soil in the form of molecules of pyruvic acid. oxides. Sulphur bacteria convert them into sulphate ions.The plants absorb sulphur 4. (a) Micronutrient elements are those element in the form of sulphate ions. which is required in less quantity. These 11. (c) Due to deficiency of sulphur plant shows are Cu, Zn, Mn, B, Cl, Mo and Ni. Some chlorosis (i.e., yellowing due to degradation physiologist consider Fe as micronutrient. of chlorophyll) followed by anthocyanin 5. (b) Molybdenum (Mo) is required for development. The younger leaves show symbiotic nitrogen fixation by legumes. chlorosis before older ones. Sulphur is not Plants requires molybdenum 0.1 to 2.5 ppm the constituent of chlorophyll. The main in their tissue for normal growth. constituent of chlorophyll is magnesium. IIT JEE NEET eBooks: www.crackjee.xyz EBD_7100

B-70 Topicwise AIIMS Solved Papers – BIOLOGY 13 Photosynthesis in Higher Plants

8. Select the incorrect statement [2011] TYPE A : MULTIPLE CHOICE QUESTIONS (a)C pathway for CO fixation were 1. In C plants, the first carbon dioxide acceptor is 4 2 4 discovered by Hatch and Slack (a) pyruvate [1997] (b) CO is essential for photosynthesis (b) phosphoenol pyruvate 2 (c) Addition of sodium carbonate in water (c) ribulose biphosphate retards photosynthetic rate in vallisneria (d) ribulose 5, phosphate 2. In photosynthesis, splitting of water and release (d) Phloem is the principal pathway for of oxygen occurs during [1998] translocation of solutes (a) photolysis (b) red drop 9. The family in which many plants are C4 type (c) Pasteur effect (d) Calvin cycle [2012] 3. Blackman’s law of limiting factor is applied (a) Malvaceae (b) Solanaceae to [2001] (c) Crucifereae (d) Graminae (a) respiration (b) transpiration 10. In the electron transport chain during terminal (c) photorespiration (d) photosynthesis oxidation, the cytochrome, which donates 4. Hill reaction occurs in [2003] electrons to O2 is [2012] (a) high altitude plants (a) Cytochrome-b (b) Cyto-C (b) total darkness (c) Cyto-a3 (d) Cyto-f (c) absence of water 11. Which one does not differ between a C3 and a (d) presence of ferricyanide C4 plant? [2013] 5. Which one of the following categories of I. Initial CO2 acceptor. organisms do not evolve oxygen during II. Extent of photorespiration. photosynthesis? [2004] III. Enzyme catalyzing reaction that fixes CO2. (a) Red algae IV. Presence of Calvin cycle. (b) Photosynthetic bacteria V. Leaf anatomy. (a) I and V (b) IV (c)C4 plants with Kranz anatomy (d) Blue green algae (c) II and III (d) II 12. The total requirement of ATP & NADPH for each 6. What is PAR range? [2007] molecule of CO2 fixed & reduced in (a) 200 nm - 800 nm (b) 400 nm - 700 nm photosynthesis in the Calvin cycle is [2014] (c) 350 nm - 550 nm (d) 600 nm - 100 nm (a) 2 ATP & 2 NADPH 7. Through the use of oxygen-18 (heavy oxygen), (b) 2 ATP & 3 NADPH scientists have found that the oxygen released (c) 3 ATP & 2 NADPH (d) 4 ATP & 3 NADPH during photosynthesis comes from molecules of 13. Consider the following statements regarding [2009] photosynthesis. (a) carbon dioxide (b) water [2015] (c) glucose (d) chlorophyll (A) ATP formation during photosynthesis is termed as photophosphorylation. IIT JEE NEET eBooks: www.crackjee.xyz

Photosynthesis in Higher Plants B-71 (B) Kranz anatomy pertains to leaf. Reason : Oxygen started accumulating in the (C) Reduction of NADP+ to NADPH occurs atmosphere after the non-cyclic pathway of during Calvin cycle. photosynthesis evolved. [2004] (D) In a chlorophyll molecule, magnesium is 19. Assertion : C4 photosynthetic pathway is more present in phytol tail. Of the above efficient than the C3 pathway. statements [2015] Reason : Photorespiration is suppressed in C4 (a) A and B are correct plants. [2005] (b) C and D are correct 20. Assertion : The atmospheric concentration of (c) A and C are correct CO2 at which photosynthesis just compensates (d) A and D are correct for respiration is referred to as CO2 compensation point. TYPE B : ASSERTION REASON QUESTIONS Reason : The CO2 compensation point is Directions for (Qs. 14-23) : These questions consist reached when the amount of CO2 uptake is less of two statements, each printed as Assertion and than that generated through respiration because Reason. While answering these questions, you are the level of CO2 in the atmosphere is more than that required for achieving CO compensation required to choose any one of the following five 2 point. [2005] responses. 21. Assertion : Under conditions of high light (a) If both Assertion and Reason are correct and intensity and limited CO2 supply, the Reason is a correct explanation of the photorespiration has a useful role in protecting Assertion. the plants from photo-oxidative damage. (b) If both Assertion and Reason are correct but Reason : If enough CO2 is not available to utilize Reason is not a correct explanation of the light energy for carboxylation to proceed, the Assertion. excess energy may not cause damage to plants. (c) If the Assertion is correct but Reason is [2006] incorrect. 22. Assertion : Photosynthetically C4 plants are less (d) If both the Assertion and Reason are incorrect. efficient than C3 plants. [2006] Reason : The operation of C pathway requires (e) If the Assertion is incorrect but the Reason is 4 the involvement of only bundle-sheath cells. correct. 23. Assertion : Dark reaction is purely enzymatic 14. Assertion: C4 pathway of CO2 fixation is reaction. [2007] found in some tropical plants. Reason : It occurs only in absence of light. Directions for (Qs.24-30) : Each of these questions Reason: In this pathway, CO2 is fixed by 3C contains an Assertion followed by Reason. Read them compound. [1998] carefully and answer the question on the basis of 15. Assertion : Mitochondria helps in following options. You have to select the one that photosynthesis best describes the two statements. Reason : Mitochondria have enzymes for dark reaction. [1999] (a) If both Assertion and Reason are correct and 16. Assertion: Bacterial photosynthesis occurs by Reason is the correct explanation of Assertion. utilizing wavelength longer than 700 nm. (b) If both Assertion and Reason are correct, but Reason: Here reaction centre is B-890. [2002] Reason is not the correct explanation of 17. Assertion : Rhoeo leaves contain anthocyanin Assertion. pigments in epidermal cells. (c) If Assertion is correct but Reason is incorrect. Reason : Anthocyanins are accessory (d) If both the Assertion and Reason are incorrect. photosynthetic pigments. [2003] 24. Assertion : Dark reaction occurs only at night 18. Assertion : Cyclic pathway of photosynthesis in the stroma of chloroplast. first appeared in some eubacterial species. Reason : CO2 fixation occurs only during C3 cycle. [2009] IIT JEE NEET eBooks: www.crackjee.xyz EBD_7100

B-72 Topicwise AIIMS Solved Papers – BIOLOGY 25. Assertion : Amaranthus and sugarcane are Reason : Oxygen started accumulating in the called as Hatch & Slack plants. atmosphere after the non-cyclic pathway of Reason : One glucose is formed by fixation of 6 photosynthesis evolved. [2012] CO2 in the plants. [2010] 29. Assertion : Each molecule of ribulose-1, 26. Assertion : D.C.M.U. is a photosynthetic 5-bisphosphate fixes one molecule of CO2. inhibitor. [2010] Reason : Three molecules of NADPH and two Reason : D.C.M.U. inhibits a photolysis of water. ATP are required for fixation of one molecule of 27. Assertion: The stromal thylakoids are rich in CO2. [2013] both PS I and PS II. 30. Assertion: Six molecules of CO2 and twelve Reason: The stroma membranes are rich in ATP molecules of NADPH+ + H+ and 18 ATP are synthetase. [2011] used to form one hexose molecule. 28. Assertion : Cyclic pathway of photosynthesis Reason: Light reaction results in formation of first appeared in some eubacterial species. ATP and NADPH2. [2002, 2015] IIT JEE NEET eBooks: www.crackjee.xyz

Photosynthesis in Higher Plants B-73

Type A : Multiple Choice Questions 9. (d) C4 cycle occurs in 1500 species of 19 angiospermic families but most of the plants 1. (b) In C4 plants, the first CO2 acceptor is 3C are monocots which belong to graminae compound, phosphoenol pyruvate. and cyperaceae family. 2. (a) In photosynthesis, release of oxygen 10. (c) In ETS or respiratory chain, there are five occurs during photolysis of water in light cytochromes cyto-b, cyto-c , cyto-c, cyto- reaction. 1 a, cyto-a3. Cytochrome a3 is last 3. (d) Blackman’s law is applicable to cytochrome of ETS which denotes photosynthesis. It states that when a electrons to O due to this metabolic water process is conditioned as to its rapidity by 2 is formed. number of factors, its rate is limited to the pace of slowest factor. Taking temperature 11. (b) into consideration, rate of photosynthesis 12. (c) For every CO2 molecule entering the Calvin is maximum at optimum temperature. cycle, 3 molecules of ATP and 2 molecules 4. (a) Hill reaction or light reaction depends upon of NADPH are required. light. It involves photolysis of water and 13. (a) During Calvin cycle, NADPH is oxidised production of assimilatory power in the to NADP. In a chlorophyll molecule, form of NADPH and ATP. magnesium is present in the porphyrin ring. Type B : Assertion Reason Questions 5. (b) Photosynthetic bacteria do not use H2O as electron donor but some other 14. (b) C pathway is an adaptation of tropical 2+ 4 compound like Fe , H2S. plants to reduce/avoid the photorespiratory In photosynthesis, splitting of water and loss. In C4 pathway, first acceptor of CO2 is liberation of O2 by chlorophyll in presence a 3 carbon compound - phosphoenol of light and hydrogen acceptor is called pyruvate. photolysis of water. 15. (d) Mitochondria helps in cellular respiration 1 by transferring energy from organic H OZZX 2H 2e O 22YZZ 2 compounds to ATP. Chloroplast helps in [Photolysis of water] photosynthesis. Dark reaction takes part 6. (b) PAR (Photosynthetically active radiations) in the stroma of the chloroplast. ranges between the wavelength 400- 16. (b) In bacteria, photosynthesis utilizes light 700nm. wavelength more than 700 nm and their 7. (b) Oxygen which is liberated during reaction centre is B-890. photosynthesis comes form water.In 17. (c) Anthocyanin pigments only give photosynthesis, the light energy is colouration since the epidermal cells mainly captured by chlorophyll in the chloroplasts have potential colouring pigments. It is in plant leaves. This energy is used to split responsible of blue, red, pink and purple water apart in a process called photolysis. colours, observed in different parts of plants +- such as petals, stamens and fruits etc. 4H2 OZ ZX 4H+ 4OH YZZ Anthocyanin are also important for --Mn+- ,Cl 4OH¾¾¾¾® 2H22 O + O •+ 4e attracting insects for pollination and seed 8. (c) C4 pathway for CO2 fixation were discovered dispersal. Hence, Anthocyanin pigments by Hatch and Slack. This pathway has more are not accessory photosynthetic effective method of CO2 fixation and is seen pigments. in plants like sugarcane. CO2 is essential for 18. (b) Photosynthetic bacteria have a substance photosynthesis. It is the source of carbon. called bacteriochlorophyll. The bacterio- The phloem or leptome is the pathway for chlorophyll pigment absorbs light in the movement of solutes. extreme UV and infra-red parts of the IIT JEE NEET eBooks: www.crackjee.xyz EBD_7100

B-74 Topicwise AIIMS Solved Papers – BIOLOGY

spectrum which is outside the range used CO2 is added by the enzyme, RuBisco to a by normal chlorophyll, seen in plants. 5 carbon compound RuBP that is converted 19. (a) C4 pathway/Hatch and Slack pathway to 2 molecules of 3-carbon PGA. In C4 cycle, ensures the Calvin cycle to be operated the first product of CO2 fixation (takes only in bundle sheath cell. It is an place in mesophyll) is a 4-carbon adaptation to photorespiratory loss. compound, oxaloacetic acid. It is seen in Therefore, C4 plants are adapted to some tropical plants. photorespiratory loss. 25. (b) Amaranthus sp and sugarcane are known 20. (c) Compensation point is that value or point as Hatch and Slack plants. In Hatch and in the light intensity and atmospheric CO2 Slack pathway, one glucose molecule is concentration when the rate of formed by fixation of 6CO2 in the plants. photosynthesis is just equivalent to the 26. (a) DCMU (Dichlorophenyl dimethyl urea) is rate of respiration in the photosynthetic a herbicide that can prevent non cyclic organ. So that there is not net gaseous photophosphorylation and oxygen exchange. production. It inhibits photolysis of water. 21. (c) Photorespiration is the uptake of O2 and 27. (d) The grana stacks of membranes are release of CO2 in light and results from the enriched in PS II and LHC (Light harvesting biosynthesis of glycolate in chloroplasts centre), while there is little ATP synthetase. and subsequent metabolism of glycolate On the other hand, a fraction of stroma acid in the same leaf cell. During thylakoids is rich in PS I and ATPase and photorespiration loss of carbon takes place poor in PS II and LHC. in the form of CO2. 28. (b) Cyclic pathway of photosynthesis is 22. (d) C4 plants are more efficient in picking up appeared first in some eubacterial species. CO2 even when it is found in low It is supposed to be the first evidence of concentration because of its high affinity production of ATP in the presence of light. for PEP. They show kranz anatomy i.e. During non-cyclic photophosphorylation vascular bundle is surrounded by bundle photolysis of water takes place. Under the sheath and mesopyll cells. influence of light energy and the catalytic 23. (e) During photosynthesis, assimilatory power action of chlorophyll, water is split up into ATP and NADPH2 are produced which oxygen and hydrogen. Non-cyclic require light. This reaction is called light photophosphorylation is the only natural reactions or Hill's reaction. Assimilatory process which adds molecular oxygen to powers are required for the reduction of the atmosphere. CO2. This reaction is enzymatic and 29. (c) Each molecule of ribulose-1, 5-biphosphate independent of light. It is called dark fixes one molecule of carbon dioxide with reaction which takes place in stroma of the addition of water, thereby resulting in chloroplast. So, dark reaction is the formation of two molecules of independent of presence or absence of 3-phosphoglyceric acid (3-PGA). The light. fixation and reduction of one molecule of 24. (d) Dark reaction is also known as light- CO2 requires three molecules of ATP and independent phase. Unlike, light reaction, two of NADPH, coming from the it does not require light as an essential photochemical reactions. factor. Thus, can take place both in the 30. (b) Light reaction or Hill reaction results in the presence or absence of light. The term dark formation of ATP and NADPH2, 6CO2, reaction does not mean that it takes place 6H2O, ATP and NADPH2 are utilised to only in dark period or at night. CO2 fixation produce one molecule of glucose. occurs in both C3 and C4 cycle. In C3 cycle, IIT JEE NEET eBooks: www.crackjee.xyz

14 Respiration in Plants

TYPE A : MULTIPLE CHOICE QUESTIONS (c) Isocitrate dehydrogenase (d) Malate dehydrogenase 1. Glycolysis occurs in [2000] 8. Which of the metabolites is common to (a) mitochondria (b) chloroplast respiration mediated breakdown of fats, (c) cytoplasm (d) peroxisome carbohydrates and proteins? [2014] 2. Anaerobic respiration, after glycolysis is also (a) Fructose 1, 6 - bisphosphate called as [2002] (b) Pyruvic acid (a) fermentation (b) fragmentation (c) Acetyl CoA (c) restoration (d) multiplication (d) Glucose - 6 - phosphate 3. In glycolysis, glucose molecule is converted into 9. Which of the following representation correctly [2002] explain the function of mitochondrion? [2015] (a) PEP (b) RuBP (c) acetyl CoA (d) pyruvic acid O2 H2O 4. Photorespiration in C3 plants starts from [2003] (a) phosphoglycerate ADP ATP (a) M (b) phosphoglycolate Phosphate CO (c) glycerate 2 (d) glycine 5. Which of the following is the connecting link O H2O between glycolysis and Krebs cycle? [2007] 2 ADP M AMP (a) Acetyl CoA (b) (b) Oxalosuccinic acid Phosphate CO2 (c) Pyruvic acid (d) Citric acid 6. Pyruvate kinase enzyme catalyses [2010] CO2 H2O (a) first irreversible step of glycolysis (c) ADP M ATP (b) second irreversible step of glycolysis Phosphate O (c) third irreversible step of glycolysis 2 (d) fourth irreversible step of glycolysis 7. An enzymes of TCA cycle are located in the CO H O mitochondrial matrix except one which is located 2 2 in inner mitochondrial membrane in eukaryotes ADP M AMP and in cytosol in prokaryotes. This enzyme is (d)

[2012] Phosphate O2 (a) Succinate dehydrogenase (b) Lactate dehydrogenase IIT JEE NEET eBooks: www.crackjee.xyz EBD_7100

B-76 Topicwise AIIMS Solved Papers – BIOLOGY

10. Refer the figure and answer the question. (iii) Glucose is phosphorylated to give rise to glucose - 6 - phosphate by the activity of Glucose the enzyme phosphofructokinase. ATP (iv) The scheme of glycolysis was given by ADP Gustav Embden, Otto Morrison, and J. Glucose-6-Phosphate Parnas and is often referred to as the EMP pathway. P Q R + (v) ATP is utilized at two steps: first in the NAD + conversion of glucose into glucose 6- NADH + H phosphate & second in the conversion of S fructose - 6- phosphate to fructose 1, 2 × 3 PGA 6-disphosphate. [2016] (a) (i), (iv) and (v) (b) (iii) and (v) 2 × 2 PGA (c) (iv) and (v) (d) (ii) and (iv) 13. Refer the given equation and answer the HO 2 question. PEPA ADP 2(C51 H 98 O 6 ) 145O 2  o 102CO 2 ATP 98H2 O Energy 2× CHO 3 43 The R.Q of above reaction is [2017] (a)1 (b) 0.7 Choose the correct names of P, Q, R, and S. (c) 1.45 (d) 1.62 [2015] 14. In alcoholic fermentation, NAD+ is produced during the [2017] P Q R S (a) reduction of acetyldehyde to ethanol. (a) 1,3 di PGA 3 PGAld Fr.1,6 di P Fr. 6 P (b) oxidation of glucose. (b) 3 PGA1d 1,3 di PGA Fr. 1,6 di P Fr.6 P (c) oxidation of pyruvate to acetyl coA. (c) Fr. 1,6 di P Fr. 6 P 3 PGAld 1,3 di PGA (d) hydrolysis of ATP to ADP. (d) Fr.6 P Fr. 1,6 di P 3 PGAld 1,3 di PGA TYPE B : ASSERTION REASON QUESTIONS 11. By which of the following complex, proton is Directions for (Q. 15) : These questions consist of pumped to reach ATP synthase to participate in two statements, each printed as Assertion and Reason. ATP synthesis? While answering these questions, you are required to (a) Cytochrome b6f choose any one of the following five responses. (b) Cytochrome c oxidase (a) If both Assertion and Reason are correct and (c) Cytochrome a - a3 the Reason is a correct explanation of the (d) Cytochrome bc Assertion. [2016] (b) If both Assertion and Reason are correct but 12. Which of the following statements (i to v) Reason is not a correct explanation of the regarding glycolysis are correct. Assertion. (i) It is ten enzymatic reactions that convert a (c) If the Assertion is correct but Reason is six-carbon molecule to a three carbon incorrect. pyruvate and result in a net gain of 2 ATP (d) If both the Assertion and Reason are incorrect. molecules. (e) If the Assertion is incorrect but the Reason is (ii) Glucose undergoes partial oxidation to form correct. one molecule of pyruvic acid. IIT JEE NEET eBooks: www.crackjee.xyz

Respiration in Plants B-77

15. Assertion : Stomata are absent in submerged 16. Assertion : Glycolysis is the first step of hydrophytes. respiration in which glucose completely breaks

Reason : Respiration occurs by means of air into CO2 and H2O. chambers in submerged plants. [1997] Reason : In this process, there is net gain of Directions for (Qs.16-18) : Each of these questions twenty four molecules of ATP. [2009] contains an Assertion followed by Reason. Read them 17. Assertion : The inner membrane of carefully and answer the question on the basis of mitochondria contains systems involving following options. You have to select the one that electron transport. best describes the two statements. Reason : The mitochondrial matrix contains (a) If both Assertion and Reason are correct and enzymes of Kreb's cycle. [2013] Reason is the correct explanation of Assertion. 18. Assertion : Glycolysis occurs in cytoplasm. (b) If both Assertion and Reason are correct, but Reason : Enzymes for glycolysis are found in Reason is not the correct explanation of cytoplasm. It is common in aerobic/anaerobic Assertion. respiration. [2002, 2014] (c) If Assertion is correct but Reason is incorrect. (d) If both the Assertion and Reason are incorrect. IIT JEE NEET eBooks: www.crackjee.xyz EBD_7100

B-78 Topicwise AIIMS Solved Papers – BIOLOGY

Type A : Multiple Choice Questions 9. (a) The main function of mitochondrion is the 1. (c) All the enzymes required for the different generation of ATP from ADP and inorganic steps of glycolysis (Ist step in cellular phosphate during cellular respiration. respiration) are present in the cytoplasm. 10. (d) 2. (a) Glycolysis is a common step for both 11. (a) With the help of cytochrome b6 f, proton is aerobic and anaerobic respiration. Next step pumped to reach ATP synthetase to after glycolysis is fermentation in anaerobic participate in ATP synthesis. respiration which leads to the formation of 12. (a) Glucose undergoes partial oxidation to form ethanol or lactic acid. two molecules of pyruvic acid, Glucose is 3. (d) In glycolysis, one molecule of glucose phosphorylated to give rise to glucose-6- undergoes partial oxidation to form 2 phosphate by the activity of the enzyme hexokinase. The scheme of glycolysis was molecules of pyruvic acid. given by Gustav Embden, Otto Meyerhof, 4. (a) In C3 plants, photorespiration starts from and J. Parnas, and is often referred to as 3-phosphoglycerate. Photorespiration is the EMP pathway. also called glycolate pathway. 13. (b) The ratio of the volume of CO2 liberated to 5. (a) Acetyl CoA is the connecting link between the volume of oxygen absorbed per glycolysis and Krebs cycle. Acetyl CoA molecule during respiration is called combines with oxalo acetic acid to form Respiratory Quotient (RQ). The value of citric acid during Krebs cycle. RQ indicates the types of respiratory 6. (c) Pyruvate kinase enzyme catalyses third substrate. irreversible step of glycolysis. Volume of CO 2 evolved 2 Phosphoenol pyruvic acid + 2ADP RQ = Volume of O 2 consumed Pyruvate kinase 2 Pyruvic acid + ATP.. 102 Mg ,K RQ 0.7 145 7. (a) All the oxidative enzymes of TCA are located in matrix except succinic dehydrogenase. 14. (a) Alcoholic fermentation is a process in This enzyme catalysed the conversion of which molecules such as glucose etc. are succinic acid into fumaric acid. The enzyme converted into cellular energy and thereby is an integral protein complex that is tightly produce ethanol and carbon dioxide as bound to the inner mitochondrial metabolic waste products. During alcoholic membrane. Infact this enzyme is the fermentation, NAD+ is produced when preferred marker enzyme for inner acetaldehyde is reduced to ethanol. membranes when doing mitochondrial fractionations. Type B : Assertion Reason Questions 8. (c) Acetyl CoA is common to respiration mediated breakdown of fats, carbohydrates 15. (b) Stomata are absent since gaseous exchange and proteins. Fats are broken down to fatty takes place through diffusion in submerged acid and glycerol and again fatty acid plants. degraded to acetyl CoA . Protein first 16. (d) Glycolysis is the process of breakdown of degraded by proteases to indiviual amino glucose or similar hexose sugar into two acids which deaminated to pyruric acid and molecules of pyruvic acid through a series further decarboxylised to acetyl CoA. of enzyme mediated reactions, releasing IIT JEE NEET eBooks: www.crackjee.xyz

Respiration in Plants B-79 energy (ATP) and reducing power 17. (b) Electron transport system is a series of co- (NADH2). It is the first step of respiration, enzymes and cytochromes that takes part which occurs inside the cytoplasm and is in the passage of electrons from a chemical independent of O2. In glycolysis, two to its ultimate receptor. The mitochondrial molecules of ATP are consumed during matrix contains all the soluble enzymes of double phosphorylation of glucose to form the citric acid or Kreb's cycle and those fructose 1, 6 diphosphate. Four molecules involved in the oxidation of fatty acids. of ATP are produced in the conversion of 1, 18. (a) Glycolysis occurs in cytoplasm as all 3-diphosphoglycerate to 3-phospho- necessary enzymes are found in it. This glycerate and phosphenol pyruvate to process is common in aerobic/anaerobic pyruvate whereas, two molecules of respiration. In this process, one glucose NADH2 are formed during oxidation of molecule is converted into 2 molecules of glyceraldehyde 3-phosphate to pyruvic acid. 1,3-diphosphoglycerate. Since, each NADH is equivalent to 3 ATP, so net gain in glycolysis is 8 ATP. IIT JEE NEET eBooks: www.crackjee.xyz EBD_7100

B-80 Topicwise AIIMS Solved Papers – BIOLOGY 15 Plant Growth and Development

TYPE A : MULTIPLE CHOICE QUESTIONS 12. Induction of cell division and delay in senescence is done by [2011] 1. In plant, auxin synthesis occurs in [1997] (a) Cytokinins (b) Auxins (a) cortex (c) GA (d) CoA (b) xylem 13. Gibberellins can promote seed germination (c) phloem because of their influence on [2005, 2012] (d) root and shoot tips (a) Rate of cell division 2. Which of the following hormone induces cell division ? [1997] (b) Production of hydrolyzing enzymes (a) Auxin (b) Gibberellin (c) Synthesis of abscisic acid (c) Cytokinin (d) Trypsin (d) Absorption of water through hard seed 3. Which is a weedicide ? [1998] coat. (a) IAA (b) 2,4-D 14. Study the following statements. [2013] (c) IBA (d) NAA I. “X” hormone promotes root growth and 4. Which of the following is gaseous hormone ? root hair formation thus helping the plants to increase their absorption surface. (a) Auxin (b) Ethylene [1998] II. “Y” hormone induces flowering in mango (c) Cytokinin (d) GA and also promotes rapid internode/petiole 5. A plant cell has potential to develop into a full elongation in deep plants and hence plant. This is called [1998] helping leaves or upper parts of shoot (a) totipotency (b) gene cloning above water. (c) tissue culture (d) regeneration III. “Z” hormone inhibits the seed germination, 6. Which of the following induces dormancy ? increase the tolerance of plant to various (a) Auxin (b) Cytokinin [1999] stresses, play import in seed development, (c) Both (a) and (b) (d) Abscisic acid maturation and dormancy. 7. The plant hormone controlling fruit ripening is Identify the correct names of hormones marked (a) IAA (b) GA [1999] as ‘X’, ‘Y’ & ‘Z’. (c) ABA (d) Ethylene (a) Y = ABA; X = Auxin; Z = GA 8. Induction of cell division and delay in (b) Z = GA; X = Auxin; Y = C2H4 senescence is done by [2001] (c) Y = Auxin; X = C H ; Z = GA (a) cytokinins (b) auxins 2 4 (d) Y = C2H4; X = C2H4; Z = ABA (c) GA (d) CoA 15. Identify the correct and incorrect statements from 9. Curling of tendrils is due to [2001] the following. (a) thigmotropism (b) phototropism (i) 17,500 new cells are produced per hour by (c) chemotropism (d) nyctinasty a single maize root apical meristem. 10. Bud dormancy can be induced by [2002] (ii) With the help of length, growth of pollen tube (a) IAA (b) GA is measured. (c) ABA (d) ethylene (iii) The growth of the leaf is measured in term 11. Avena curvature test is a bioassay for examining of volume. the activity of [2006] (iv) Cells in a watermelon may increase in size (a) auxins (b) gibberellins by upto 3,50,000 times. [2016] (c) cytokinins (d) ethylene IIT JEE NEET eBooks: www.crackjee.xyz

Plant Growth and Development B-81 (a) (i), (ii), (iii) are correct and (iv) is incorrect. 16. Assertion : Dark period plays more important (b) (i), (ii), (iv) are correct and (iii) is incorrect. part in flowering than light period. (c) (ii), (iii) are correct and (i), (iv) are incorrect. Reason : Flowering occurs in short-day plant if (d) (i), (iv) are correct and (ii), (iii) are incorrect. the dark period is interrupted by light break. TYPE B : ASSERTION REASON QUESTIONS [2013] 17. Assertion : Photomodulation of flowering is a Directions for (Qs. 16-19) : These questions consist phytochrome regulated process. of two statements, each printed as Assertion and Reason : Active form of phytochrome (PFR) Reason. While answering these questions, you are directly induces floral induction in shoot buds. required to choose any one of the following five [2015] responses. 18. Assertion (A) : Vernalization is acceleration of (a) If both Assertion and Reason are correct and subsequent flowering by low temperature the Reason is a correct explanation of the treatment. Assertion. Reason (R) : Site of vernalization is apical (b) If both Assertion and Reason are correct but meristem. Reason is not a correct explanation of the Assertion. [2015] 19. Assertion : Auxins help to prevent fruit and leaf (c) If the Assertion is correct but Reason is drop at early stages. incorrect. Reason : Auxins promote the abscission of older (d) If both the Assertion and Reason are incorrect. mature leaves and fruits. [2017] IIT JEE NEET eBooks: www.crackjee.xyz EBD_7100

B-82 Topicwise AIIMS Solved Papers – BIOLOGY

Type A : Multiple Choice Questions 10. (c) Abscisic acid (ABA) is a plant hormone 1. (d) Auxin is a plant hormone which promote which promotes dormancy in seeds and and regulate its growth and development. buds. Auxin are produced in the meristem of 11. (a) Went (1928) performed Avena curvature shoot tips and move down the plant test for auxins. causing various effects. 12. (a) Cytokinins promote cell division and inhibit 2. (c) Cytokinins are substances which act the degradative reactions in detached primarily on cell division and have little or leaves and slow down senescence in intact no effect on extension/growth. It delays leaves. The effect of cytokinin in retarding the senescence of leaves. ageing is called the Richmond Lang Effect. 3. (b) 2, 4-D (dichlorophenoxyacetic acid) is a common systemic pesticide/herbicide. It is 13. (b) Gibberellins can promote seed germination a synthetic auxin (plant hormone) and is in cereals due to production of hydrolyzing used for killing broad leaved weeds enzymes like D amylase and proteases. (generally dicot). 14. (d) Gibberellins stimulate the production of 4. (b) Ethylene is the only gaseous natural plant some mRNAs and then hydrolytic enzymes growth regulator produced by all plant like amylase, lipase ribonuclease and organs but its maximum production occurs proteases. The enzymes solubilise the in ripening fruits and during senescence. reserve food of the seeds and the same is 5. (a) Any cell which has an ability to develop transferred to embryo axis for its growth. into a complete organism is totipotent. 15. (b) The growth of the leaf is measured in term 6. (d) Abscisic acid (ABA) induces dormancy. of surface area. Robinson and Warming (1964), isolated a substance responsible for dormancy in Type B : Assertion Reason Questions Acer pseudoplatanus, and named it as dormin. This hormone was similar to 16. (c) It has been demonstrated that flowering in absicissin. It is a naturally occurring plant is more of a response to the dark hormone that is present in all vascular period than to the light period. In short day plants and some mosses but is not present plants, the plants can flower in complete in bacteria, algae, fungi and liverworts. This hormone is responsible for dormancy. darkness if supplied with exogenous 7. (d) Fruit ripening is controlled by hormone, nutrients. Flowering is prevented in them ethylene. Ethylene is a gaseous hormone. if dark period below the critical level is It affects the growth, development, ripening interrupted by a flash of light. Interruption and senescence (aging) of all plants. of light by dark inhibits flowering under 8. (a) Cytokinins promote cell division and inhibit normal photoperiods. the degradative reactions in detached leaves and slow down senescence in intact 17. (a) Active form of PFR is responsible for leaves. The effect of cytokinin in retarding inducing flowering. Phytochrome, protein ageing is called the Richmond Lang Effect. pigment, exists in two inter convertible 9. (a) The curling of tendrils is a growth forms. movement induced due to contact or touch. Such a movement is called Red thigmotropic movement. When the tip of Pr Pfr the tendril comes in contact with an uneven Far red surface, it coils around the support. This is due to the differential growth in the tendril. Dark IIT JEE NEET eBooks: www.crackjee.xyz

Plant Growth and Development B-83

18. (b) The physiological mechanism of flowering 19. (b) Auxin delays abscission of young leaves in plants is controlled by two factors__ light and fruits. Its effect is through non- period and low temperature. The cold formation of abscission zone below a leaf treatment of plants to induce flowering is or fruit. Abscission zone cuts off nutrients called vernalization. Term vernalization was and water supply. However, auxin promotes first given by T.D.Lysenko(1928). As a the abscission of mature or older leaves result of vernalization a flowering hormone and fruits. called vernaline is formed. Site of vernalization is apical meristem. IIT JEE NEET eBooks: www.crackjee.xyz EBD_7100

B-84 Topicwise AIIMS Solved Papers – BIOLOGY 16 Digestion and Absorption

TYPE A : MULTIPLE CHOICE QUESTIONS 11. Wharton’s duct is the duct of [2002] (a) Parotid gland 1. Tocopherol stands for [1997] (a) Vitamin A (b) Vitamin E (b) Sublingual gland (c) Vitamin C (d) Vitamin K (c) Submaxillary gland 2. Scurvy is caused due to deficiency of vitamin (d) Pancreatic gland (a)A (b) B [1997] 12. Continued consumption of a diet rich in butter, (c)E (d) C red meat and eggs for a long period may lead to 3. The contraction of gall bladder is due to [1998] (a) vitamin A toxicity [2003] (a) gastrin (b) secretin (b) kidney stones (c) cholecystokinin (d) enterokinase (c) hypercholesterolemia 4. The function of rennin is [1999] (d) urine laden with ketone bodies (a) vasodilation 13. Which one of the following pairs of the cells (b) reduce blood pressure (c) degradation of angiotensinogen with their secretion is correctly matched? [2006] (d) none of the above 5. Liver in our body stores [1999] (a) Oxyntic cells - A secretion with pH between (a) Vitamin A (b) Vitamin D 2.0 and 3.0 (b) Alpha cells of Islets of Langerhans - (c) Vitamin B12 (d) All of these 6. Vitamin C is also called as [2000] Secretion that decreases blood sugar level. (a) ascorbic acid (b) glutamic acid (c) Kupffer cells - A digestive enzyme that (c) aspartic acid (d) enolic acid hydrolysis nucleic acids. 7. Brunner’s glands are present in [2001] (d) Sebaceous glands - A digestive enzyme (a) duodenum (b) oesophagus that hydrolysis nucleic acids (c) ileum (d) stomach 14. Which match is true? [2007] 8. Curdling of milk in small intestine takes place Vitamin deficiency Vitamin Source due to [2002] disease (a) rennin (b) trypsin (a) Severe bleeding Tocopherol Milk, (c) chymotrypsin (d) ptyalin egg 9. Which of the following has minimum pH?[2002] (b) Anaemia Ascorbic Lemon, (a) Bile (b) Saliva acid orange (c) Night blindness Retinol Carrot, (c) Gastric juice (d) Pancreatic juice milk 10. Which of following teeth are lophodont? [2002] (d) Sterility Calciferol Milk, (a) Incisor and canine butter (b) Premolar and molar 15. A child took sugar cane and sucked its juice. (c) Canine and premolar Regarding this which of the following match is (d) Premolar and incisor correct? [2007] IIT JEE NEET eBooks: www.crackjee.xyz

Digestion and Absorption B-85

Substrate Enzyme Site of Products (a) polyunsaturated fatty acids of enzyme secretion formed (b) saturated fatty acids (a) Proteins Pepsin Duodenum Polypeptides (c) essential vitamins (b) Starch Amyase Salivary Glucose (d) more carbohydrates and proteins glands 21. Endoscopy, a technique used to explore the (c) Lipids Lipase Pancreas Fat globules stomach or other inner parts of the body, is based (d) Sucrose Invertase Duodenum Glucose + on the phenomenon of [2010] Fructose (a) total internal reflection 16. Which one of the following pairs of the kind of (b) interference cells and their secretion are correctly matched? (c) diffraction [2008] (d) polarization (a) Oxyntic cells – A secretion with 22. Lathyrism is caused by excessive consumption pH between 2.0 of [2010] and 3.0 (a) khesari dal (b) mustard oil (b) Alpha cells – Secretion of (c) polished rice (d) mushrooms of (Nutrition) Langerhans that 23. The normal temperature of human body on the islets decreases blood Kelvin scale is [2010] sugar level (a) 280 (b) 290 (c) Kupffer cells – A digestive (c) 300 (d) 310 enzyme that 24. Parotid salivary glands are present [2012] hydrolyses (a) Below the tongue (b) Below the cheeks nucleic acids (c) In the angle between two jaws (d) Sebaceous – A secretion that (d) Below the eye orbits glands evaporates for 25. If for some reason the parietal cells of the gut cooling epithelium become partially non-funcitonal, what 17. Fat present below the skin surface in our body, is likely to happen? [2015] acts as a barrier against [2010] (a) The pancreatic enzymes and specially the (a) loss of heat from the body trypsin and lipase will not work efficiently (b) loss of essential body fluids (b) The pH of stomach will fall abruptly (c) loss of salts from the body (c) Steapsin will be more effective (d) entry of harmful micro-organisms from the (d) Proteins will not be adequately hydrolysed environment by pepsin into proteoses and peptones 18. The nutritional deficiency condition that needs 26. A healthy person eats the following diet - 5 gm to be given top priority for remedial action in raw sugar, 4 gm albumin, 10 gm pure buffalo ghee India today is [2010] adultrated with 2 gm vegetable ghee (hydrogenated vegetable oil) and 5 gm lignin. (a) scurvy (b) rickets How many calories he is likely to get? (c) xerophthalmia (d) pellagra [2014, 2016] 19. What is the average fat content of buffalo milk? (a) 144 (b) 126 (a) 7.2% (b) 4.5% [2010] (c) 164 (d) 112 (c) 9.0% (d) 10.9% 27. Which of the following statement is true ? [2017] (a) Pepsin cannot digest casein. 20. Consumption of fish is considered to be healthy (b) Trypsin can digest collagen. when compared to flesh of other animals because (c) Pepsin cannot digest collagen. when compared to flesh of other animals, fish (d) Chymotrypsin can digest casein. contains [2010] IIT JEE NEET eBooks: www.crackjee.xyz EBD_7100

B-86 Topicwise AIIMS Solved Papers – BIOLOGY

TYPE B : ASSERTION REASON QUESTIONS 29. Assertion : In the condition of obstructive jaundice, large amounts of unabsorbed fats are Directions for (Q. 28) : These questions consist of eliminated out of the body. two statements, each printed as Assertion and Reason. Reason : Entry of bile into the small intestine is While answering these questions, you are required to prevented during obstructive jaundice. [2009] choose any one of the following five responses. 30. Assertion : Pancreatic amylase digest starch to (a) If both Assertion and Reason are correct and maltose. [2010] the Reason is a correct explanation of the Reason : Pancreatic amylase breaks the peptide Assertion. bond of protein. (b) If both Assertion and Reason are correct but 31. Assertion : Trypsin helps in blood digestion of Reason is not a correct explanation of the predator animals. Assertion. Reason : Trypsin hydrolyzes fibrinogen. (c) If the Assertion is correct but Reason is [2011] incorrect. 32. Assertion : Lipases of bile help in the (d) If both the Assertion and Reason are incorrect. emulsification of fats. (e) If the Assertion is incorrect but the Reason is Reason : Lipases can break large fat droplets correct. into smaller ones. [2011] 28. Assertion : Scurvy is caused by deficiency of 33. Assertion : In the condition of obstructive vitamin. jaundice, large amounts of unabsorbed fats are Reason : Deficiency of ascorbic acid causes scurvy. eliminated out of the body. [2001] Reason : Entry of bile into the small intestine is Directions for (Qs.29-35) : Each of these questions prevented during obstructive jaundice. contains an Assertion followed by Reason. Read them carefully and answer the question on the basis of [2015] following options. You have to select the one that 34. Assertion : Starch is hydrolysed by ptyalin to best describes the two statements. maltose. (a) If both Assertion and Reason are correct and Reason : Sucrase hydrolyses sucrose to lactose. Reason is the correct explanation of Assertion. [2016] (b) If both Assertion and Reason are correct, but 35. Assertion : Water and electrolytes are almost Reason is not the correct explanation of fully absorbed in the large intestine. Assertion. Reason : In large intestine, haustral contractions (c) If Assertion is correct but Reason is incorrect. (slow segmenting movements) roll the forming (d) If both the Assertion and Reason are incorrect. faeces over and over, causing absorption of water and electrolytes. [2017] IIT JEE NEET eBooks: www.crackjee.xyz

Digestion and Absorption B-87

Type A : Multiple Choice Questions from dudonum's glands, called the crypts 1. (b) Tocopherol or vitamin E is fat soluble of Lieberkuhn, it converts inactive vitamin. It is antisterility factor and trypsinogen into active trypsin. antioxidant for membrane lipids, skin, and 4. (d) Rennin (also called rennet or chymosin) is hair etc. It reduces atherosclerosis and an coagulating enzyme produced from inhibits oxidation of vitamin A and stomach of human body. It catalyzes the unsaturated fatty acid. Retinol, ascorbic coagulation of milk by converting milk with acid and phylloquinone stands for vitamin soluble protein caesin into insoluble semi A, C and K respectively. fluid calcium paracaesinate. This is called 2. (d) Deficiency of vitamin C (which is necessary curdling of milk. Rennin produced in the for collagen production and iron infants immediately after birth. As the child absorption) causes scurvy. It is grows, rennin production goes down and characterised by bleeding of gums, is replaced by pepsin digestive enzymes. disturbance of protein metabolism and Renin is an enzyme which acts as hormone increased susceptibility to infections. secreted by juxtaglomerular cells. It converts Scurvy is most frequently seen in order, angiotensinogen into angiotensin. malnourished adults. Scurvy commonly is 5. (d) Liver is the largest organ in the body. It associated with sailors in the 16th to 18th helps in digestion and removes waste centuries who navigated long voyages products and worn-out cells from the without enough vitamin C frequently blood. Liver performs many functions, like perished from the condition. it forms and secretes bile that contains bile Deficiency of vitamin A, B and E results in acids to aid in the intestinal absorption xeropthalmia, beri-beri and miscarriage (taking in ) of fats and the fat-soluble respectively. vitamins as A, D, E, K and B12. Hence, it stores vitamins as A, D, E, K and B . 3. (c) The contraction of gall bladder occurs due 12 to hormone, cholecystokinin. 6. (a) Vitamin C (also called as ascorbic acid) is a water soluble vitamin. It helps the body to Cholecystokinin is a gastrointestinal make collagen, an important protein used hormone that is secreted by cells in the to make skin, cartilage, tendons, ligaments duodenum. Its release is stimulated by the and blood vessels. Vitamin C is essential presence of fatty acids and amino acids in for healing wounds, and for repairing and the small intestine. It stimulates the release maintaining bones and teeth. of bile into the intestine by contracting gall bladder and the secretion of pancreatic 7. (a) Brunner's glands are compound tubular enzymes. Gastrin and secretin are glands found only in the sub-mucosa of polypeptide hormones. Gastrin, secreted by duodenum. They produce mucoid fluid certain cells of the pyloric glands, which protects the duodenal mucosa from stimulates secretion of gastric juice into the acidic chyme coming from the stomach. bloodstream and churning movement of 8. (a) Trypsin changes chymotrypsinogen to stomach. Secretin is secreted by the chymotrypsin and procarboxypeptidase to mucosa of the duodenum and jejunum. It carboxypeptidase. Chymotrypsin changes stimulates secretion of water and caesin of milk into paracaesin. Ptyalin or bicarbonates in bile and activates pancreas salivary amylase converts starch and to secrete pancreatic juice. Enterokinase of glycogen into limit dextrins, maltose and enteropeptidase is an enzyme, secreted isomaltose. IIT JEE NEET eBooks: www.crackjee.xyz EBD_7100

B-88 Topicwise AIIMS Solved Papers – BIOLOGY 9. (c) pH of bile is 8; pH of saliva is 6.7, pH of 14. (c) Retinol is the chemical name of the vitamin pancreatic juice is 8.8 and that of stomach A, which is mostly found in carrots, milk, is 2. cheese, fish etc. Retinol is well adapted for 10. (b) Premolar and molar are lophodont teeths. light absorption in animals where it is Lophodont teeth with the cusps elongated converted into the light-absorbing to form narrow ridges. The molars in molecule called retinal. Deficiency of elephants and have cusps fused by vitamin A mostly affects the rods containing retinal and leads to a disease means of intermediate masses of dentine called as night blindness or poor night to form ridges or lophs. vision. Tocopherol is the chemical name of 11. (c) Wharton's duct is the duct of the vitamin E, found mostly in wheat germ oil, submaxillary or submandibular gland that brown flour etc. Its deficiency causes occurs in the mouth on a papilla at the side sterility in rats. Vitamin C, also called as of the frenulum of the tongue. The purpose ascorbic acid, is found mostly in citrus of this duct is to drain the saliva. fruits, potatoes, tomatoes etc. Its deficiency causes painful disease of the joints and 12. (c) Continued consumption of fat rich gums called scurvy. diet causes hypercholesterolemia. 15. (d) Sugarcane and sugarbeet are the richest Hypercholesterolemia is the presence of sources of sucrose, a disaccharide. It is high levels of cholesterol in the blood. High most commonly found in plants, where it is cholesterol raises your risk for heart transported in large quantity by phloem disease, heart attack, and stroke. Kidney tissue. In humans, enzyme invertase stones are solid mass made up of tiny (sucrase) present in duodenum of the small crystals. There are different types of kidney intestine hydrolyses sucrose into one stones. The exact cause depends on the molecule of glucose and one molecule of type of stone like, calcium stones, uric acid fructose. Pepsin is proteolytic enzyme that stone etc. Vitamin A toxicity or hydrolyses many proteins into smaller hypervitaminosis A is having too much of molecules of peptones. Saliva of humans vitamin A in the body. Ketonuria is contains salivary amylase (ptyalin) that condition in which ketone bodies are hydrolyses starch into maltose, isomaltose present in urine. Body productes excess and small dextrins. Lipases hydrolyse ketone bodies as an alternate source of triglyceride fat into diglycerides, and then energy during starvation or diabetes into monoglycerides alongwith fatty acids mellitus (type 1). at each step. 13. (a) Oxyntic cells or Parietal cells, are the 16. (a) Parietal cells also called oxyntic cells are the stomach epithelium cells that secrete gastric stomach epithelium cells that secrete gastric acid intrinsic factor. These cells secrete acid and intrinsic factor. A cell of the gastric hydrochloric acid (HCl) which makes the glands that secretes hydrochloric acid. gastric juice acidic.(pH = 2.0-3.0). 17. (a) Fats present below the skin surface in our Alpha cells of islets of Langerhans secretes body, is called subcutaneous fat deposition. glucagon hormone which increase the It acts as insulator of body and prevent loss glucose level in the blood by converting of heat from the body. glycogen to glucose in liver cells. Kupffer's 18. (c) Xerophthalmia is caused by vitamin A cells are specialized cells in the liver that deficiency. This generally occurs in poorer destroy bacteria, foreign proteins, and section of the society because often they do worn-out blood cells. Sebaceous glands not have adequate amount of fruits in diet. and microscopic glands in the skin that 19. (a) The average fat content in buffalo milk is secrete an oily/waxy matter (called sebum) 7.2% which is higher than human milk. to lubricate the skin and hair of mammals. Lactose is higher in human milk than cow and buffalo’s milk. IIT JEE NEET eBooks: www.crackjee.xyz

Digestion and Absorption B-89 20. (a) Fish has more polyunsaturated fatty acids 30. (c) Pancreatic amylase is a starch splitting which act as natural antioxidant. enzymes similar to ptyalin by hydrolysing 21. (a) Endoscopy, a technique used to explore the starch and glycogen to maltose, isomaltose stomach or other inner parts of the body, is and limit dixtrins. based on total internal reflection. 31. (a) Trypsin is protein digesting enzyme present 22. (a) Lathyrism is caused by excessive in the intestine of animals. Though it cannot consumption of khesari dal. digest casein (a milk protein), in predator 23. (d) Normal body temperature is 37 degree cen- animals drinking the blood of their prey, tigrade but in Kelvin scale 0ºC = 0ºC + 273 = trypsin hydrolyses fibriongen of blood into 273ºK. When we convert 37ºC into Kelvin, fibrin, leading to blood coagulation thus it becomes 37 + 273 = 310ºK. help in blood digestion. It also activates 24. (b) There are three pairs of salivary glands. other pancreatic proteases. These are parotids (below the cheeks), sub maxillary/ or sub mandibular (lower jaw) and 32. (d) It is not lipases but the bile salts which are the sublinguals (below the tongue) responsible for the emulsification of fats. Bile salts are steroids secreted by the liver 25. (d) in the bile. In the intestinal lumen, they 26. (a) Physiological value of carbohydrates is 4.0 reduce the surface tension of fat dropletes, kcal/g, proteins 4.0 kcal/g and of fats is 9.0 causing their breakdown into many smaller kcal/g. Hence, ones. A stable fine emulsion of fat is thereby 5 g raw sugar will yield formed. On the other hand, lipases are the 5 × 4.0 = 20.0 kcal enzymes which hydrolyse fats and oils. 4 g albumin (protein) will yield Lipases can digest fat in significant amounts 4 × 4.0 = 16.0 kcal only when large fat droplets are broken into 10 + 2 g of fat will yield tiny droplets to form a fine emulsion. 12 × 9.0 = 108.0 kcal Emulsificaion of fats by bile salts thus, increases the lipase action on fats. Total yield = 144 kcal. 33. (a) In the condition of obstructive jaundice the 27. (d) Milk protein can be digested by pepsin and entry of bile into the small intestine is chymotrypsin prevented due to an obstruction in the bile Type B : Assertion Reason Questions duct. As we know that bile salts help in the digestion of fats by emulsification and also 28. (a) Deficiency of ascorbic acid/vitamin C in their absorption by the formation of causes scurvy. water soluble droplets called micelles from whom fatty acids, glycerides, sterols and 29. (a) In the condition of obstructive jaundice the fat soluble vitamins are absorbed into the entry of bile into the small intestine is intestinal cells. Therefore, in the absence prevented due to an obstruction in the bile of bile, the fats remain unabsorbed and duct. As we know that bile salts helps in consequently are eliminated out of the the digestion of fats by emulsification and body in the faeces. also in their absorption by the formation of water soluble droplets called micelles from 34. (c) Sucrase hydrolyses sucrose to glucose and whom fatty acids, glycerides, sterols and fructose. fat soluble vitamins are absorbed into the 35. (a) intestinal cells. Therefore, in the absence of bile, the fats remain unabsorbed and consequently are eliminated out of the body in the faeces. IIT JEE NEET eBooks: www.crackjee.xyz EBD_7100

B-90 Topicwise AIIMS Solved Papers – BIOLOGY Breathing and Exchange 17 of Gases (a) Pulmonary ventilation of ‘A’ and ‘B’ is TYPE A : MULTIPLE CHOICE QUESTIONS same. 1. Oxidative breakdown of respiratory substrates (b) Alveolar ventilation of ‘A’ and ‘B’ is same. (c) Pulmonary ventilation of ‘A’ is greater than with the help of O2 is called as [1997] (a) fermentation ‘B’. (b) anaerobic respiration (d) Alveolar ventilation of ‘A’ is greater than (c) R. Q. ‘B’. (d) aerobic respiration 6. Oxyhaemoglobin can transport [2010] (a) 8 ml of CO /100 ml blood 2. Severe Acute Respiratory Syndrome (SARS) 2 (b) 5 ml of CO /100 ml blood (a) is caused by a variant of Pneumococcus 2 pneumoniae. [2004] (c) 3 ml of CO2/100 ml blood (b) is caused by a variant of the common cold (d) 2 ml of CO2/100 ml blood virus (corona virus). 7. Which of the following match is correct?[2011] (c) is an acute form of asthma. (a) Emphysema: reduction of surface area of (d) affects non-vegetarians much faster than alveoli and bronchi the vegetarians. (b) Pneumonia: occupational disease with 3. The diagram below represents part of a capillary asbestos in a specific region of the human body. The (c) Silicosis: inflammation of alveoli region labeled X represents part of [2009] (d) Asthma: excessive secretion of bronchial mucus 8. Volume of air breathed in and out during normal breathing is called [2012] (a) Vital capacity (b) IRV (c) ERV (d) Tidal volume 9. Much developed larynx of human male is called (a) Aristole's lanttern (b) Syrinx [2012] (c) Adam's apple (d) Muller's organ (a) a glomerulus (b) an alveolus 10. The presence of CO2 in blood will lower pH (c) a villus (d) the liver because CO2 combines with____, with the rate 4. In humans, the concentration of carbon dioxide of reaction increased by_____ . [2013] + – in the plasma [2009] (a)H2O to form H and HCO3 , carbonic (a) causes increased production of anhydrase – hydrochloric acid. (b) H2O to form only HCO3 , carbonic anhydrase (b) regulates gastric acid production by + forming carbonic acid. (c)H2O to form only H , carbonic ions + – (c) regulates breathing rate by its effect on the (d) H to form HCO3 , oxyhaemoglobin medulla. 11. Approximately seventy percent of carbon- (d) causes inflammation of the tissues of the dioxide absorbed by the blood will be bronchial tubes. transported to the lungs [2014] (a) as bicarbonate ions 5. If the respiratory rate of ‘A’ is 35 breaths/min and tidal volume 185 cc/breath and of ‘B’ is (b) in the form of dissolved gas molecules 25 breaths/min and tidal volume 259 cc/breath (c) by binding to RBC then [2009] (d) as carbamino - haemoglobin IIT JEE NEET eBooks: www.crackjee.xyz

Breathing and Exchange of Gases B-91 12. During oxygen transport the oxyhaemoglobin 14. Assertion : Severe Acute Respiratory Syndrome at the tissue level liberates oxygen to the cells (SARS) is originated in China. because in tissue [2016] Reason : China is the most populated country (a)O concentration is high and CO is low 2 2 of the world. [2003] (b) O concentration is low and CO is high 2 2 Directions for (Qs.15-17) : Each of these questions (c)O tension is high and CO tension is low 2 2 contains an Assertion followed by Reason. Read them (d) O tension is low and CO tension is high 2 2 carefully and answer the question on the basis of TYPE B : ASSERTION REASON QUESTIONS following options. You have to select the one that best describes the two statements. Directions for (Qs. 13-14) : These questions consist of two statements, each printed as Assertion and (a) If both Assertion and Reason are correct and Reason. While answering these questions, you are Reason is the correct explanation of Assertion. required to choose any one of the following five (b) If both Assertion and Reason are correct, but responses. Reason is not the correct explanation of (a) If both Assertion and Reason are correct and Assertion. the Reason is a correct explanation of the (c) If Assertion is correct but Reason is incorrect. Assertion. (d) If both the Assertion and Reason are incorrect. 15. Assertion : Oxyhaemoglobin dissociates near (b) If both Assertion and Reason are correct but the organ tissue due to Bohr effect and oxygen Reason is not a correct explanation of the is released. [2010] Assertion. Reason :Increased CO concentration reduces (c) If the Assertion is correct but Reason is 2 the affinity of haemoglobin for oxygen. incorrect. 16. Assertion: In mammals, complex respiratory (d) If both the Assertion and Reason are incorrect. system has developed. [2011] (e) If the Assertion is incorrect but the Reason is Reason: Mammalian skin is impermeable to gases correct. 17. Assertion : Inspiration occurs due to muscular 13. Assertion : Many visitors to the hills suffer from relaxation. skin and respiratory allergy problems. Reason : During inspiration, the diaphragm and Reason : Conifer trees produce a large quantity external intercostal muscle contract of wind-borne pollen grains. [2003] simultaneously. [2012] IIT JEE NEET eBooks: www.crackjee.xyz EBD_7100

B-92 Topicwise AIIMS Solved Papers – BIOLOGY

Type A : Multiple Choice Questions 8. (d) Volume of air breathed in and out during normal breathing is called tidal volume. It 1. (d) The oxidative breakdown of respiratory is approximately 500 ml i.e., a healthy substrates (like carbohydrates, fats, person can expire or inspire approximately proteins) into CO and water, occurs in the 2 6000 to 8000 ml of air per minute as presence of oxygen. It is called aerobic breathing rate is 12-16 times/minute. respiration. 2. (b) Severe acute respiratory syndrome (SARS) 9. (c) Larynx is a cartilaginous box which helps is caused by mutant of Influenza/common in sound production and hence is called cold virus (Corona virus). the sound box. Much developed larynx of 3. (b) An alveolus is a tiny air sac in the lung. It human male is called Adam's apple. is the actual part of the lung that exchanges 10. (a) Carbon dioxide combines with H2O in the + – atmospheric oxygen with carbon dioxide plasma to form H and HCO3 . The enzyme from the blood. A glomerulus is a ball of carbonic anhydrase catalyzes the reaction. capillaries found in the nephrons of the kidneys (to filter nitrogenous wastes), a 11. (a) CO2 from the respiratory tissues to the villus is a tiny projection of a capillary in lungs is transported by the blood in 3 ways: the small intestine (to absorb digested (i) In dissolved state or as a physical food), and the liver is the site of bile solution: Very small amount is physically production and breakdown of amino acids. dissolved in plasma (7% i.e. 0.3 ml of CO2 4. (c) The medulla oblongata at the base of the by each 100 ml of blood). brain increases breathing rate if the amount (ii) Bicarbonate ions: About 70% (i.e. @ of carbon dioxide increases. It does not 2.5 ml per 100 ml of blood) CO diffuses in regulate breathing rate by checking oxygen 2 plasma & then into RBCs where it (in the content. presence of carbonic anhydrase) combines 5. (c) The process by which a continuous exchange of gases is maintained across with H2O to form carbonic acid which is respiratory surface is often called external almost spontaneously dissociated into respiration. The ventilation rate of an hydrogen ion and bicarbonate ions. animal is the volume of air breathed per (iii) Carbaminohaemoglobin : 23% minute i.e tidal volume x number of (i.e. 1 ml of CO2 per 100 ml of blood) breathes per minute. It can be measured combines with haemoglobin forming an with the aid of a respirometer. unstable compound. 6. (c) Oxyhaemoglobin can transport about 3ml 12. (d) of carbon dioxide per 100 ml of blood. 7. (a) Cigarette smoking leads to the disease Type B : Assertion Reason Questions emphysema. In this disease, terminal 13. (b) The skin problem could be due to pollen bronchioles get obstructed. This reduces allergy and respiratory problem could be the ventilation of the alveoli connected to due to the decrease in oxygen content, them. Many alveoli coalesce together to since the atmosphere becomes thin as one form large chambers due to destruction of goes up the hill. their walls. This change of smaller alveoli 14. (b) The first patient of SARS was reported in to large chambers reducing the area of alveolar surface across which gases are February 2003 in China. Its causing agent exchanged. All these changes reduce both is human corona virus (type of Influenza oxygen uptake and carbon dioxide virus) which spreads through contact, elimination. respiratory secretions and cockroaches. IIT JEE NEET eBooks: www.crackjee.xyz

Breathing and Exchange of Gases B-93

, 15. (a) Bohr s effect is the effect of CO2 on animals; so they need a more extensive oxyhaemoglobin. Body tissues obtain respiratory surface. Thus, a complex oxygen from oxyhaemoglobin because of mammalian respiratory system consists of its dissociation caused by low O2 and high the nasal cavity, nasopharynx, larynx, CO2 concentration. The increased CO2 trachea, bronchi, bronchiole and lungs. concentration reduces the affinity of 17. (c) Inspiration is the result of muscular haemoglobin for oxygen. contraction. The diaphragm and external 16. (b) Mammalian skin is impermeable so that intercostal muscles contract simultaneously. water loss through it is minimised. But The lateral thoracic wall moves outward and mammals need far more oxygen to maintain their high metabolic rates than lower upward. IIT JEE NEET eBooks: www.crackjee.xyz EBD_7100

B-94 Topicwise AIIMS Solved Papers – BIOLOGY 18 Body Fluids and Circulation

TYPE A : MULTIPLE CHOICE QUESTIONS (b) It is coronary artery which is blocked by a plaque and the same is being cracked. 1. Anti-serum contains [1997] (c) It is coronary vein in which the defective (a) antigens (b) antibodies valves are being opened. (c) leucocytes (d) RBCs (d) It is coronary vein blocked by a parasite 2. Which enzyme induces lysis of fibrinogen to (blood fluke) that is being removed. fibrin during fibrinolysis ? [1997] 8. Hirudin is [2006] (a) Plasmin (b) Thrombin (a) A protein produced by Hordeum vulgare, (c) Fibrin (d) Trypsin which is rich in lysine. 3. Which of the following blood group can be given (b) A toxic molecule isolated from Gossypium to any patient ? [1999] hirusutum, which reduces human fertility. (a)A (b) B (c) A protein produced from transgenic (c)O (d) AB Brassica napus, which prevents blood 4. Which of the following chamber of heart has clotting. the thickest muscular wall ? [1999] (d) An antibiotic produced by a genetically (a) Left auricle (b) Left ventricle engineered bacterium, Escherichia coli. (c) Right ventricle (d) Right auricle 9. The component of blood which prevents its 5. In which of the following pairs the two items coagulation in the blood vessels is [2007] mean one and the same thing? [2004] (a) haemoglobin (b) plasma (a) Malleus - Anvil (c) thrombin (d) heparin (b) SA node - Pace maker 10. Thickening of arteries due to cholesterol (c) Leucocytes - Lymphocytes deposition is [2007] (d) Haemophilia - Blood cancer (a) arteriosclerosis (b) rheumatic heart 6. An artificial pace maker is implanted (c) blood pressure (d) cardiac arrest. subcutaneously and connected to the heart in 11. Which one of the following is a matching pair? [2003, 2008] patients [2004] (a) Lubb - Sharp closure of AV valves at the (a) having 90% blockage of the three main beginning of ventricular systole. coronary arteries. (b) Dup - Sudden opening of semilunar valves (b) having a very high blood pressure. at the beginning of ventricular diastole. (c) with irregularity in the heart rhythm. (c) Pulsation of the radial artery valves in the (d) suffering from arteriosclerosis. blood vessels. 7. The figure below shows an angioplasty of the (d) Purkinje fibres-Initiation of the heart beat. coronary blood vessel. Which one of the 12. A malfunction of the lymph nodes would most following statements correctly describes, what likely interfere with the [2009] is being done? [2006] (a) release of carbon dioxide into the lymph (b) filtering of glucose from the lymph (c) release of oxygen into the lymph (d) filtering of bacteria from the lymph 13. Arteries supplying blood to the heart are called [2010] (a) It is coronary artery which has a cancerous (a) carotid arteries (b) hepatic arteries growth that is being removed. (c) coronary arteries (d) pulmonary arteries IIT JEE NEET eBooks: www.crackjee.xyz

Body Fluids and Circulation B-95 14. A man whose blood group is not known meets (ii) RBC, WBC and blood platelets are with a serious accident and needs blood produced by bone marrow. transfusion immediately, which one of the blood (iii) Neutrophils bring about destruction and groups readily available in the hospital will be detoxification of toxins of protein origin. safe for transfusion? [2010] (iv) The important function of lymphocytes is – + (a) O, Rh (b) O, Rh to produce antibodies. – + (c) AB, Rh (d) AB, Rh (a) (i) and (ii) only (b) (i) and (iv) only 15. With reference to the blood in a normal (c) (i) and (iii) only (d) (ii) and (iii) only person, which one of the following statements is correct? [2010] 20. The diagram given here is the standard ECG of (a) Compared to arteries, veins are less a normal person, the P-wave represents the : numerous and hold less of the body’s R blood at any given time. (b) Blood cells constitute about 70 percent of the total volume of the blood. PQ S T (c) White blood cells (WBC) are made by lymph nodes only. (a) Initiation of the ventricular contraction (d) The blood has more platelets than WBC. (b) Beginning of the systole 16. Which of the following organs is the blood (c) End of systole bank? [2011] (d) Contraction of both the atria [2014] (a) Heart (b) Lungs 21. Bulk of carbon dioxide (CO ) released from body (c) Spleen (d) Liver 2 tissues into the blood is present as [2015] 17. Which one of the following is a matching pair of a certain body feature and its value/count in a (a) 70% carbamino-haemoglobin and 30% as normal human adult? [2003, 2008, 2011] bicarbonate (a) Urea 5-10 mg / 100 ml of blood (b) carbamino-haemoglobin in RBCs (b) Blood sugar (fasting) - 70-100 mg/100 ml (c) bicarbonate in blood plasma and RBCs (c) Total blood volume - 5-6 (d) free CO2 in blood plasma (d) ESR in Wintrobe method - 9-15 mm in males 22. Given below is the ECG of a normal human. and 20-34 mm in females Which one of its components is correctly 18. Given below is the ECG of a normal human. Which one of its components is correctly interpreted below? interpreted below? [2013] R R [2015] PQ S T PQ S T

(a) Peak P and Peak R together - systolic and (a) Peak P and Peak R together - systolic and diastolic blood pressures diastolic blood pressures (b) Peak P - Initiation of left atrial contraction (b) Peak P - Initiation of left atrial contraction only only (c) Complex QRS - One complete pulse (c) Complex QRS - One complete pulse (d) Peak T - Initiation of total cardiac (d) Peak T - Initiation of total cardiac contraction contraction 19. Which of the following statements are wrong? TYPE B : ASSERTION REASON QUESTIONS [2013] Directions for (Qs. 23-29) : These questions consist (i) Leucocytes disintegrate in the spleen and of two statements, each printed as Assertion and liver. Reason. While answering these questions, you are IIT JEE NEET eBooks: www.crackjee.xyz EBD_7100

B-96 Topicwise AIIMS Solved Papers – BIOLOGY

required to choose any one of the following five 29. Assertion : Smaller the organism higher is the responses. rate of metabolism per gram weight. (a) If both Assertion and Reason are correct and Reason : The heart rate of a six month old baby is the Reason is a correct explanation of the much higher than that of an old person. [2007] Assertion. Directions for (Qs.30-34) : Each of these questions (b) If both Assertion and Reason are correct but contains an Assertion followed by Reason. Read them Reason is not a correct explanation of the carefully and answer the question on the basis of Assertion. following options. You have to select the one that (c) If the Assertion is correct but Reason is best describes the two statements. incorrect. (a) If both Assertion and Reason are correct and (d) If both the Assertion and Reason are incorrect. Reason is the correct explanation of Assertion. (e) If the Assertion is incorrect but the Reason is (b) If both Assertion and Reason are correct, but correct. Reason is not the correct explanation of 23. Assertion : Muscle fibres of SA node possess Assertion. the lowest rhythmicity among all cardiac (c) If Assertion is correct but Reason is incorrect. muscles. (d) If both the Assertion and Reason are incorrect. Reason : Due to this fact, it can initiate excitatory 30. Assertion : Prothrombinase enzyme act as waves at the highest rate. [1999] 24. Assertion : Saline water is not given to patients antiheparin. [2010] of hypertension. Reason : Heparin prevent coagulation of blood Reason : Saline water can cause vomiting and in blood vessels. may drop blood pressure suddenly causing 31. Assertion : Blood is coloured in the insects. cardiac arrest. [2000] Reason : Insect blood has no role in O2 transport. 25. Assertion : Blood pressure is arterial blood [2012, 2013] pressure. 32. Assertion : When there is a fall in the blood Reason : Blood pressure is measured by pressure due to loss of blood volume, this is sphygmomanometer. [2000] compensated by vasoconstriction of veins. 26. Assertion: WBCs accumulate at site of wounds Reason : Veins hold the extra amount of blood by diapedesis. which can be shifted to the arteries as required. Reason: It is squeezing of leucocytes from [2010, 2015] endothelium. [2002] 33. Assertion : Lub is a heart sound which is 27. Assertion : Persons suffering from haemophilia produced during each cardiac cycle. fail to produce blood clotting factor VIII. Reason : It is associated with the closure of the Reason : Prothrombin producing platelets in tricusp and bicuspid valves. such persons are found in very low [2016] concentration. [2005] 28. Assertion (A) : Blood coagulates in uninjured blood vessels. Reason (R) : Uninjured blood vessels release an anticoagulant heparin. [2007] IIT JEE NEET eBooks: www.crackjee.xyz

Body Fluids and Circulation B-97

Type A : Multiple Choice Questions walls of blood vessels. This causes the arteries to lose their elasticity and get 1. (b) Anti-serum is a blood serum that contains stiffened. This is called arteriosclerosis or antibodies. hardening of arteries. 2. (b) During blood clotting, lysis of fibrinogen 11. (a) Lubb sound is caused partly by the closure to fibrin occurs by plasma thrombin. Later of the bicuspid and tricuspid valves and on fibrin entangles corpuscles to form a partly by the contraction of the muscles in clot. Thrombin is not mormally found in the ventricles. Lubb is the first heart sound. circulating blood, but instead it is repressed 12. (d) The lymphatic system not only regulates by its inactive precursor prothrombin. The fluid amounts in the blood but also helps conversion of prothrombin to thrombin to fight infection. The lymph nodes requires blood platelets, calcium ions and produce white blood cells and filter harmful thromboplastin. bacteria, that is why they tend to enlarge 3. (c) Blood group ‘O’ is universal donor. Group when we have an infection. ‘O’ blood, with no antigens, can be given 13. (c) Coronary arteries supply the blood to the heart. Carotid arteries supply the blood to to anyone. head region. Hepatic arteries supply the 4. (b) Left ventricle of the heart has the thickest blood to liver while the pulmonary artery wall because it has to pump the blood to supplies blood to the lungs. the farthest end of the body. 14. (b) According to ABO system of blood 5. (b) Malleus is hammer shaped; lymphocytes grouping, O type blood can be given to are type of leucocytes and hemophilia is person of all types of blood, i.e., O, A, B bleeder’s disease (genetic) disorder). SA and AB. Hence, a person with O type blood node acts as pacemaker because it is called universal donor. functions as originator of cardiac impulse. According to Rh system of blood 6. (c) Artificial pace maker is implanted to the grouping, most of the people have blood patients where heart beat level drops in which there is a substance called Rh abnormally low (30-40) due to disease or factor. Rh stands for rhesus. This type of operations. blood can be donated only in rarest of rare 7. (b) In this figure, coronary angioplasty is being chances. In India, about 97% of people are done where atherosclerotic plaque, which Rh+. So, if an Rh+ person is transfused with builds up and clogs the coronary arteries, Rh+ blood, then it is safe. But if an Rh is compressed against the vessel wall by negative (Rh–) person is transfused with + expanding a balloon like device inserted Rh blood then he/she will develop anti- through a catheter that has been threaded Rh factor i.e., antibodies in his/her blood, through the artery. and there might beno harm is, but may kill the recipient if a second Rh+ transfusion is 8. (c) Hirudin is an anticlotting agent produced done. by transgenic Brassica napus. 9. (d) Blood contains an anticoagulant called 15. (d) The number of blood platelets per cubic mm in human blood is 3 lacs while the heparin, which prevents the activation of number of WBCs are 5000/cubic mm of prothrombin. The latter is an inactive blood. Veins are as complex as the arteries. globulin which is required during blood Veins and arteries both are types of blood coagulation. Heparin is released from the vessels. Arteries carry blood from heart to mast-cell granules. different organs while vein carries blood 10. (a) High proportion of cholesterol in blood from different organs to heart. At any given leads to deposition of cholesterol on the IIT JEE NEET eBooks: www.crackjee.xyz EBD_7100

B-98 Topicwise AIIMS Solved Papers – BIOLOGY time in a healthy human, the blood amount 24. (c) Saline water increases the blood pressure is same in both, as the circulation of blood due to the presence of NaCl in it. Hence, never stops. the reason for the assertion is false. Blood consists of two parts: 25. (b) Blood pressure is the arterial pressure of The plasma (water, proteins, inorganic salts blood exerted on the wall of arteries with and other elements) constitutes 55-60% of each heart beat. It is measured form the blood while cellular part constitutes 40- brachial artery in the elbow pit. It is 45% of total blood. WBCs are produced in expressed as red bone marrow, lymph nodes and systolic pressure (mm/Hg) sometimes even in liver and spleen. diastolic pressure (mm/Hg) 16. (c) Spleen is referred to as a blood bank of the body because it is the organ having the Arterial (superficial; arteries) blood function of making and storing pressure is measured by lymphocytes and red corpuscles. These are sphygmomanometer. squeezed out into the blood-stream when 26. (b) WBC’s are wandering cells capable of the body needs more in circulation, as for coming out of blood capillaries by instance in haemorrhage or shock. amoeboid movement called diapedesis. 17. (b) Fasting glucose level is 70-110 mg/decilitre. 27. (a) Haemophilia is caused by lack of activity Total blood volume in normal adult human of blood clotting factor VIII or IX and they is 5-6 litres. Blood is a fluid connective show platelet function disorder. tissues. Its cells consists of corpuscles. 28. (d) When an injury is caused to a blood vessel, Plasma represents matrix of blood. Blood bleeding starts which is stopped by blood is mesodermal in origin and salty in taste. clotting. At the site of injury blood platelets Its pH is 7.3 – 7.4. release platelet factor - 3 and injured tissues 18. (c) 19. (c) release thromboplastin. The two combine to form prothrombinase enzyme which 20. (d) The P-wave represents the electrical exci- converts prothrombin to thrombin. The tation (or depolarisation) of the atria, which latter stimulates formation of fibrin thread leads to the contraction of both the atria. or clot. Blood contains an anticoagutant The QRS complex represents the heparin which prevents blood clotting in depolarisation of the ventricles, which ini- uninjured vessels. tiates the ventricular contraction. The con- traction starts shortly after Q and marks 29. (b) The basal metabatic rate is defined as the the beginning of the systole. energy requirement of human body at rest. 21. (c) BMR of smallest animals are generally 22. (c) higher then larger animals. Peoples with higher metabolism means that they have Type B : Assertion Reason Questions higher heart rate. Heart rate of baby is 23. (e) The sinoatrial or sinoauricular node 70-190 times/minute, whereas adults determines the rate of heart beat by (including serious) is 60-100 times/ minute. determining the rate of discharge of cardiac 30. (b) Prothrombinase enzyme is necessary for impulse. It is called the pace maker. It is blood clotting. It acts as antiheparin. formed of specialized cardiac muscles and Coagulation of blood in vessels is is located in the right atrial wall near the prevented by heparin, a quick acting opening of superior venacava. These anticoagulant. It inhibits conversion of muscles are self excitable. Since it is self prothrombin to thrombin and is used in excitable, it can produce waves at highest open-heart surgery. rate. Hence, the assertion is incorrect. IIT JEE NEET eBooks: www.crackjee.xyz

Body Fluids and Circulation B-99 31. (b) Insect blood is colourless and does not 33. (b) Lub and dub are two heart sounds, which play any role in transport of oxygen. Insects occurs due to the closure of cuspid valves have tracheal respiration. and semilunar valves respectively. Lub is 32. (a) When the blood pressure of an individual the first heart sound which is formed due decreases due to loss of blood volume, then to closure of atrioventricular valves at the vasoconstriction of veins occurs. This beginning of ventricular systole. It is low shifts the little amount of blood from veins pitched of long duration (0.15 sec). to arteries. IIT JEE NEET eBooks: www.crackjee.xyz EBD_7100

B-100 Topicwise AIIMS Solved Papers – BIOLOGY Excretory Products and their 19 Elimination

TYPE A : MULTIPLE CHOICE QUESTIONS 8. If Henle’s loop were absent from mammalian nephron which of the following is to be expected 1. Podocytes occur in [1998] [2015] (a) large intestine (a) there will be no urine formation (b) glomerulus of kidney (b) there will be hardly any change in the (c) wall of capillaries quality and quantity of urine formed (d) neck region (c) the urine will be more concentrated 2. The end product of ornithine cycle is [1999] (d) the urine will be more dilute. (a) urea (b) uric acid 9. Which blood vessel in mammals would normally (b) NH (d) CO 3 2 carry the largest amount of urea? [2016] 3. Reabsorption in tubules of nephrons occurs by (a) Dorsal aorta (b) Hepatic vein [2000] (c) Hepatic portal vein (d) Renal vein (a) osmosis (b) diffusion 10. In ornithine cycle, enzyme arginase breaks down (c) active transport (d) both (b) & (c) arginine into [2016] 4. Toxic substances are detoxified in human body (a) Citrulline and ammonia in [2001] (b) Ornithine and ammonia (a) kidney (b) lungs (c) Ornithine and urea (c) liver (d) stomach (d) Citrulline and urea. 5. In which of the following minimum content of TYPE B : ASSERTION REASON QUESTIONS urea is present ? [2012] (a) Hepatic portal vein Directions for (Qs. 11-13) : These questions consist (b) Portal vein of two statements, each printed as Assertion and (c) Renal vein Reason. While answering these questions, you are (d) Vena cava required to choose any one of the following five 6. Duct of Bellini is concerned with [2012] responses. (a) Filtration of urine (a) If both Assertion and Reason are correct and (b) Purification of urine the Reason is a correct explanation of the (c) Conduction of urine Assertion. (b) If both Assertion and Reason are correct but (d) All the above Reason is not a correct explanation of the 7. Which one of the following statements in regard Assertion. to the excretion by the human kidneys is (c) If the Assertion is correct but Reason is correct? [2013] incorrect. (a) Ascending limb of Loop of Henle is (d) If both the Assertion and Reason are incorrect. impermeable to electrolytes (e) If the Assertion is incorrect but the Reason is (b) Descending limb of Loop of Henle is correct. impermeable to water 11. Assertion : During physiology of excretion, (c) Distal convoluted tubule is incapable of deamination does not take place in liver. – reabsorbing HCO3 Reason : Deamination is a process to make use (d) Nearly 99 per cent of the glomerular filtrate of excess of amino acids which cannot be is reabsorbed by the renal tubules incorporated into protoplasm. [2001] IIT JEE NEET eBooks: www.crackjee.xyz

Excretory Products and their Elimination B-101 12. Assertion : Secreting hypotonic urine is 14. Assertion : Ultrafiltration takes place in presence effective in reducing urinary loss of water. of effective filtration pressure. Reason : Hypotonic urine is more concentrated Reason : In ultrafiltration process, blood is filtered and higher in osmotic pressure than the blood. in Bowman’s capsule, filtered fluid contain protein [2007] & blood corpuscles also. [2010] 13. Assertion : Aldosterone is a steroid hormone 15. Assertion : In vertebrates, the liver is also and is important in the control of sodium and referred as an accessory excretory organ. potassium ion concentration in mammals. Reason : Liver helps kidneys in the secretion of Reason : It upgrades sodium ion concentration urine. [2012] in the ECF by promoting reabsorption of sodium 16. Assertion : Main constituent of human urine is ions from renal tubules and excretion of ammonia. potassium ions in urine. [2007] Reason : If human urine is allowed to stand for some time, it smells strongly of ammonia. [2013] Directions for (Qs.14-18) : Each of these questions 17. Assertion : Hemodialysis can save and prolong contains an Assertion followed by Reason. Read them the life of uremic patients. carefully and answer the question on the basis of Reason : Waste products like urea can be following options. You have to select the one that removed from the blood by the process of best describes the two statements. hemodialysis. [2014] (a) If both Assertion and Reason are correct and 18. Assertion : In the descending limb of loop of Reason is the correct explanation of Assertion. Henle, the urine is hypertonic, whereas in (b) If both Assertion and Reason are correct, but ascending limb of loop of Henle, the urine is Reason is not the correct explanation of hypotonic. Assertion. Reason : Descending limb is impermeable to (c) If Assertion is correct but Reason is incorrect. sodium, while ascending limb is impermeable to (d) If both the Assertion and Reason are incorrect. water. [2016] IIT JEE NEET eBooks: www.crackjee.xyz EBD_7100

B-102 Topicwise AIIMS Solved Papers – BIOLOGY

Type A : Multiple Choice Questions 12. (d) Hypotonic urine means concentration of urine is less than that of blood. Hypertonic 1. (b) Podocytes are specialized visceral urine is more concentrated than blood and epithelial cell in the Bowman's capsule in has high osmotic pressure than the blood, the kidneys that wraps around the therefore it helps in reducing the loss of capillaries of the glomerulus. It helps filter water with urine. The urine is filtered by blood in the glomerulus of the kidney. the Bowman's capsule. The tubules of 2. (a) The ornithine cycle occurs in the nephrons reabsorb a large quantity of water mitochondria of liver cells. It is meant for making urine more concentrated. urea formation. 13. (a) Aldosterone is one of the important 3. (d) Tubular reabsorption is the second mineralocorticoids in humans secreted by process in urine formation through filtrate. adrenal cortex. Its main function is to In this, most of the filtrate passes out of regulate sodium content of the body . It rephron tubule and returns to the blood increases sodium ion concentration in the through peritubular capillaries. Tubular blood by absorbing sodium ions from renal reabsorption occurs by diffusion and tubules. Excessive production of active transport. aldosterone causes a disease aldosteronism. 4. (c) Liver is the primary site of detoxification Its symptoms include high blood pressure, and elimination of body wastes and high blood volume. poisons. Liver detoxifies endotoxins, e.g. 14. (c) Ultrafiltration takes place in renal corpuscle toxic NH combined with CO to form less 3 2 of uriniferous tubule. It takes place in toxic urea. It also detoxifies alcohol and presence of effective filtration pressure. convert them to acetaldehyde and then During the process, blood is filtered and harmless acetyl CoA. contains only blood plasma – proteins. The 5. (c) Renal veins connects the kidney to inferior filtered blood entering into Bowman’s vena cava. They carry the blood purified by capsule is called glomerular filtrate. the kidney and carry minimum quantity of urea. Glomerular filtrate = Blood – (Blood 6. (c) Renal collecting tubules are also known as corpuscles + plasma proteins) duct of bellini. These are the long narrow 15. (c) In vertebrates, the lungs, liver & skin are tubes in the kidney that conduct urine from referred as accessory excretory organs the nephrons to larger ducts that leads to because besides the urinary system these urinary bladder. organs also participate in the removal of 7. (d) waste products from the body. The liver is 8. (d) Henle's loop is responsible for the the principal organ for the excretion of reabsorption of water and sodium chloride cholesterol, bile pigments (bilirubin and from the urine. Hence, in the absence of biliverdin) and inactivated products of Henle's loop the urine will become more steroid hormones, some vitamins and many dilute. drugs. It secretes these substances in the 9. (b) 10. (c) bile and indirectly helps by formation of urea through amino acids in ornithine cycle. Type B : Assertion Reason Questions It has no role in secretion of urine. 11. (d) Deamination is the process of converting 16. (d) Urea is the chief nitrogenous constituent amino acid to keto acid with the release of of human urine, though it possesses small amount of ammonia. But when the urine is NH . It occurs in the liver. 3 allowed to stand for sometime, bacterial IIT JEE NEET eBooks: www.crackjee.xyz

Excretory Products and their Elimination B-103

degradation occurs and it leads to the hemodialysis saves and prolongs the life production of ammonia from urine. And of many uremic patients. thus it smells strongly. 18. (a) Descending limb of Henle is permeable to 17. (a) The blood urea level rises abnormally water but not to sodium. Consequently (uremia) in patients suffering from renal water moves out into interstitium and failures. In uremia patients an artificial concentration of sodium in tubular filtrate kidney is used for removing accumulated rises making filtrate hypertonic. Ascending waste products like urea from the blood by loop is impermeable to water but permeable to sodium and makes the filtrate hypotonic. a process called hemodialysis. In this way, IIT JEE NEET eBooks: www.crackjee.xyz EBD_7100

B-104 Topicwise AIIMS Solved Papers – BIOLOGY 20 Locomotion and Movement

TYPE A : MULTIPLE CHOICE QUESTIONS (a) tibia and tarsals (b) femur and fibula 1. Cranium of human contains [2000] (c) fibula and phalanges (a) 12 bones (b) 8 bones (d) tarsals and femur (c) 14 bones (d) 20 bones 7. A cricket player is fast chasing a ball in the field. 2. Which of the following is made up of a single Which one of the following groups of bones are bone in mammal ? [2001] directly contributing in this movement?[2006] (a) Dentary (b) Hyoid (a) Femur, malleus, tibia, metatarsals (c) Upper jaw (d) All of these 3. Sella turcica is found [2001] (b) Pelvis, ulna, patella, tarsals (a) near pituitary (b) in bone (c) Sternum, femur, tibia, fibula (c) in joints (d) near thyroid (d) Tarsals, femur, metatarsals, tibia 4. Which one of the following is a sesamoid bone? 8. The shoulder blade is made of [2007] [2003] (a) clavicle (b) humerus (a) Pelvis (b) Patella (c) ilium (d) scapula (c) Pterygoid (d) Pectoral girdle 9. The sensation of fatigue in the muscles after 5. Two of the body parts which do not appear in prolonged strenuous physical work, is caused MRI may be [2005] by [2010] (a) molar teeth and eye lens (a) a decrease in the supply of oxygen (b) scapula and canines (b) minor wear and tear of muscle fibres (c) ligaments and ribs (c) the depletion of glucose (d) tendons and premolars (d) the accumulation of lactic acid 6. Given below is a diagram of the bones of the left 10. Sesamoid bone is derived from- [2012] (a) Cartilage (b) Areolar tissue human hindlimb as seen from front. It has certain (c) Tendon (d) Ligament mistakes in labelling. Two of the wrongly labelled 11. Select the correct matching of the type of the bones are [2005] joint with the example in human skeletal system: [2014] Type of joint Example (a) Cartilaginous between frontal and Femur joint parietal (b) Pivot joint between third and fourth cervical vertebrae Fibula (c) Hinge joint between humerus and Tibia pectoral girdle (d) Gliding joint between carpals 12. Wish bone in birds is formed from the bones of Tarsals [2016] Phalanges (a) Shoulder girdle (b) Hip girdle (c) Keeled sternum (d) Skull bones IIT JEE NEET eBooks: www.crackjee.xyz

Locomotion and Movement B-105

13. The given figure represents the cross bridge 14. Assertion: Fatigue is inability of muscle to relax. cycle in skeletal muscle. What does the step B Reason: It is due to lactic acid accumulation by in the figure represents? [2017] repeated contractions. [1998] Directions for (Qs.15-19) : Each of these questions

Myosin head ADP contains an Assertion followed by Reason. Read them (high-energy P configuration) 1 carefully and answer the question on the basis of A following options. You have to select the one that Thin filament best describes the two statements. (a) If both Assertion and Reason are correct and ADP ATP ADP Thick filament hydrolysis P 1 Reason is the correct explanation of Assertion. (b) If both Assertion and Reason are correct, but D B Reason is not the correct explanation of

ATP Assertion. Myosin head ATP (low-energy (c) If Assertion is correct but Reason is incorrect. configuration) (d) If both the Assertion and Reason are incorrect. C 15. Assertion : Ball and socket joints are the most (a) Attachment of myosin head to actin mobile joints. forming cross bridge. (b) Release of phosphate. Myosin changes Reason : Synovial fluid is present here. shape to pull actin. [2012] (c) Attachment of new ATP to myosin head. 16. Assertion : Intercalated discs are important The cross bridge detaches. regions of cardiac muscle cells. (d) Splitting of ATP into ADP and Pi. Myosin Reason : Intercalated discs function as boosters cocks into its high energy conformation.T for muscle contraction waves. [2012] TYPE B : ASSERTION REASON QUESTIONS 17. Assertion : Arthritis or inflammation of a joint makes the joint painful. Directions for (Qs. 14-15) : These questions consist Reason : Some toxic substances are deposited of two statements, each printed as Assertion and at the joint. [2013] Reason. While answering these questions, you are 18. Assertion : The phase of muscle contraction required to choose any one of the following five occurs when myosin binds and releases actin. responses. Reason : Muscle contraction is initiated by a (a) If both Assertion and Reason are correct and signal sent by the peripheral nervous system the Reason is a correct explanation of the via motor neuron. [2016] Assertion. 19. Assertion : Inflammation of a skeletal joint may (b) If both Assertion and Reason are correct but immobilize the movements of the joint. Reason is not a correct explanation of the Reason :Uric acid crystals in the joint cavity Assertion. and ossification of articular cartilage lead to this. (c) If the Assertion is correct but Reason is [2006, 2017] incorrect. (d) If both the Assertion and Reason are incorrect. (e) If the Assertion is incorrect but the Reason is correct. IIT JEE NEET eBooks: www.crackjee.xyz EBD_7100

B-106 Topicwise AIIMS Solved Papers – BIOLOGY

Type A : Multiple Choice Questions joints allow the bones to glide past one another in any direction along the plane of 1. (b) Cranium of human body consists of 8 bones. the joint - up and down, left and right, and There are 1 frontal, 2 parietal, 2 temporal, 1 diagonally. Many gliding joints are formed occipital, 1 sphenoid and 1 ethmoid bone. in the appendicular skeleton between the 2. (b) Hyoid is a shoe shaped bone present carpal bones of the wrist; between the in neck between lower law and sound box carpals and the metacarpals of the palm; (larynx). It is not articulated to any bone, between the tarsal bones of the ankle; and but is simply suspended, from temporal between the tarsals and the metatarsals of bones by means of ligaments. Hyoid the foot. provides surface for the attachment of 12. (a) Two clavicles fuse with one inter-clavicle tongue muscles. to form ‘Wish bone’ or ‘Bone of merry 3. (b) Sella turcica/turkish saddle/pituitary fossa thought’. is a depression in sphenoid bone of skull in which pituitary gland lies. 13. (b) Step A: Attachment of myosin head to actin 4. (b) Sesamoid bones are formed in the tendons forming cross bridge. at the joints, e.g. patella. Step B: Release of phosphate. Myosin 5. (b) MRI machine does not show face image of changes shape to pull actin. bone and calcium, e.g. scapula, canine. It Step C: Attachment of new ATP to myosin is also not suitable for patients with cardiac head. The cross bridge detaches. pace makers. Step D: Splitting of ATP into ADP and Pi. 6. (c) Fibula and phalanges marked parts are Myosin cocks into its high energy actually tibia and metatarsals. conformation. 7. (d) In a fast chasing cricketer, foot and leg bones contribute directly in this movement Type B : Assertion Reason Questions e.g. femur, tibia, tarsal and metatarsals. 14. (a) Due to the accumulation of lactic acid, 8. (d) Shoulder blade is made up of scapula. It is muscles do not respond to a stimuli. After a flat, triangular-shaped largest bone a prolonged previous activity. present in each half of the pectoral 15. (b) Synovial fluid is a thick sticky fluid secreted (shoulder) girdle. It is joined to the clavicle by synovial membranes into the synovial in front. The clavicle is well developed in cavity. Though the presence of synovial humans that links scapula to the sternum. fluid is one of the reasons behind the 9. (d) The sensation of fatigue in the muscles mobility of the joints, but the most accurate after prolonged strenuous physical work reason is the arrangement of the bones at is caused by the accumulation of lactic the joint, the spheroidal ball-like end of one acid. bone articulates here with the cup-shaped 10. (c) Sesmoid bones are embedded in tendons. depression in another. This allows the bone Sesmoids are found in locations where a with the ball head to be moved freely in tendon passes over a joint, such as the many planes. Shoulder joints and hip joints hand, knee and foot. Functionally they act are the ball-and-socket joints. to protect the tendon and its mechanical 16. (a) Cardiac muscle cells are short cylindrical effect. cells joined end to end and by side 11. (d) A gliding joint is a common type of synovial branching to form a network. Intercalated joint formed between bones that meet at discs are the dense junctions formed in flat or nearly flat articular surfaces. Gliding between the cardiac muscle cells where IIT JEE NEET eBooks: www.crackjee.xyz

Locomotion and Movement B-107 they meet each other. Intercalated discs are 18. (c) The phase of muscle contraction occurs the specialised regions of the cell when myosin binds and releases actin. membranes. As cardiac muscle possesses Muscle contraction is initiated by a signal considerable rhythmicity and generates its sent by the central nervous system via a own wave of excitation, these discs motor neuron. A motor neuron along with function as boosters for muscle contraction the muscle fibres connected to it constitutes wave. a motor unit. 17. (c) Arthritis or inflammation of a joint makes 19. (a) Painful inflammation of the synovial the joint painful and may even immobilise membrane of the joints results in stiffening the movements at the joint. This may result of joints and painful movement Uric acid from a lack of the synovial fluid at the joint. accumulation in the joints can lead to The ossification of the articular cartilage, painful movement of joint. deposition of uric acid crystals in the joint cavity or other changes at the joint. IIT JEE NEET eBooks: www.crackjee.xyz EBD_7100

B-108 Topicwise AIIMS Solved Papers – BIOLOGY 21 Neural Control and Coordination

9. Excessive stimulation of vagus nerve in humans TYPE A : MULTIPLE CHOICE QUESTIONS may lead to [2003] 1. The vagus nerve is the cranial nerve numbering (a) hoarse voice [1997] (b) peptic ulcers (a) 10th (b) 9th (c) efficient digestion of proteins (c) 5th (d) 8th (d) irregular contraction of diaphragm 2. Sensation of stomach pain is due to [1998] 10. A person is wearing spectacles with concave (a) interoceptors (b) exteroceptors lenses for correcting vision. While not using (c) teloreceptors (d) all of these the glasses, the image of a distant object in his 3. Which is the example of conditioned reflex ? case will be formed [2003] [1999] (a) on the blind spot (a) Eyes closed when anything enter into it. (b) behind the retina (b) Hand took up when piercing with needle. (c) in front of the retina (c) Salivation in a hungry dog in response to (d) on the yellow spot ringing of a bell. 11. Unidirectional transmission of a nerve impulse (d) Digestion food goes forward in alimentary through nerve fibre is due to the fact that[2004] canal. (a) nerve fibre is insulated by a medullary 4. Otorhinolaryngology is the study of [1999] sheath. (a) brain cells (b) sodium pump starts operating only at the (b) bird anatomy cyton and then continues into the nerve fibre. (c) locomotary organs (c) neurotransmitters are released by dendrites (d) ENT and not by axon endings. 5. If frog’s brain is crushed, even then its leg moves (d) neurotransmitters are released by the axon on pinpointing . It is called [2001] endings and not by dendrites. (a) simple reflex 12. Examine the diagram of the two cell types A and (b) conditional reflex B given below and select the correct option: (c) neurotransmitter function [2006] (d) autonomic nerve conditions 6. Which of the following is not a mental disorder? (a) Epilepsy (b) Neurosis [2001] (c) Psychosis (d) Plague 7. The 5th cranial nerve of frog is called [1998, 2001] (a) optic nerve (b) vagus nerve A B (c) trigeminal nerve (d) olfactory nerve 8. The crystal of lead zirconate is a key component (a) Cell A is the rod cell found evenly all over of [2003] retina. (a) electroencephalography (b) Cell A is the cone cell more concentrated in (b) electrocardiography the fovea centralis. (c) magnetoencephalography (c) Cell B is concerned with colour vision in bright light. (d) sonography (d) Cell A is sensitive to low light intensities. IIT JEE NEET eBooks: www.crackjee.xyz

Neural Control and Coordination B-109

13. A person, who shows unpredictable moods, 20. Multipolar nerve cells are present in [2012] outbursts of emotion, quarrelsome behaviour (a) Cochlea and conflicts with others, is suffering from (b) Dorsal root ganglia of spinal cord (a) borderline personality disorder (BPD) (c) Retina of eye (b) mood disorder [2006] (d) Brain (c) addictive disorder 21. Neurons receive signals through their (d) schizophrenia ______and send signals to other neurons 14. Which of the following is an eye disease? through their ______. [2013] (a) hepatitis (b) measles (a) dendrites ... receptors (c) glaucoma (d) bronchitis (b) end feet ... cell bodies and dendrites 15. Given below is a table comparing the effects of (c) cell bodies and dendrites ... axons sympathetic and parasympathetic nervous (d) transmitter vesicles ... axons system for four features (1-4). [2006] Which one feature is correctly described? 22. Which of the following ions are required for nerve conduction ? [2016] (a) Ca++, Na+ and K+ (b) Ca++ and Mg++ Sympathetic ++ + + + Parasympahtetic (c) Mg and K (d) Na and K Feature nervous nervous sy stem 23. The following diagram indicates the reflex arc. system Identify the parts labelled as A, B, C, D, E, F and (a) Salivary Stimulates G. Choose the correct option Inhibits secretion glands secretion (b) Pupil of Dilate Constricts the eye

(c) Heart rate Decrease Increase (d) Intestinal Stimulates Inhibits peristalsis

16. Hearing impairment affects which part of [2016] brain? [2007] (a) A = sense organ; B = sensory nerve; (a) Frontal lobe (b) Parietal lobe C = dorsal horn; (c) Temporal lobe (d) Cerebellum D = interneuron; E = ventral horn; 17. The black pigment in the eye which reduces the F = motor nerve; G = effector internal reflection is located in [2007] (b) A = sense organ; B = sensory nerve; (a) retina (b) iris C = ventral horn; (c) cornea (d) sclerotic D = interneuron; E = dorsal horn; 18.. Bipolar nerve cells are present in [2012] F = motor nerve; G = effector (a) Skin tactile corpuscles (c) A = effector; B = motor nerve; (b) Spinal cord C = dorsal horn; (c) Retina of eye D = interneuron; E = ventral horn; (d) All the above F = sensory nerve; G = effector 19. Fenestra ovalis is the opening of [2012] (d) A = effector; B = motor nerve; C = ventral horn; D = interneuron; (a) Cranium (b) Tympanum E = dorsal horn; F = sensory nerve; (c) Tympanic cavity (d) Brain G = sense organ. IIT JEE NEET eBooks: www.crackjee.xyz EBD_7100

B-110 Topicwise AIIMS Solved Papers – BIOLOGY TYPE B : ASSERTION REASON QUESTIONS Directions for (Qs. 27-31) : Each of these questions contains an Assertion followed by Reason. Read them Directions for (Qs. 24-26) : These questions consist carefully and answer the question on the basis of of two statements, each printed as Assertion and following options. You have to select the one that Reason. While answering these questions, you are best describes the two statements. required to choose any one of the following five (a) If both Assertion and Reason are correct and responses. Reason is the correct explanation of Assertion. (a) If both Assertion and Reason are correct and (b) If both Assertion and Reason are correct, but the Reason is a correct explanation of the Reason is not the correct explanation of Assertion. Assertion. (b) If both Assertion and Reason are correct but (c) If Assertion is correct but Reason is incorrect. Reason is not a correct explanation of the (d) If both the Assertion and Reason are incorrect. Assertion. 27. Assertion : The brain stem contains centres for (c) If the Assertion is correct but Reason is controlling activities. incorrect. Reason : Brain stem is very sensitive. [2012] (d) If both the Assertion and Reason are incorrect. 28. Assertion : The chemical stored in the synaptic (e) If the Assertion is incorrect but the Reason is vesicles are termed as neurotransmitters. correct. Reason : Synaptic vesicles release these chemicals in the synaptic cleft. [2013] 24. Assertion : Transmission of nerve impulse 29. Assertion: The imbalance in concentration of across a synapse is accomplished by Na+, K+ and proteins generates resting potential. neurotransmitters. Reason: To maintain the unequal distribution of Reason : Transmission across a synapse usually Na+ & K+, the neurons use electrical energy. requires neurotransmitters because there is a [2002, 2015] small space, i.e., synaptic cleft, that separates 30. Assertion : The axonal membrane of the neuron + one neuron from another. [1999] is more permeable to sodium ion (Na ) and nearly impermeable to potassium (K+). 25. Assertion : Tongue is a gustatoreceptor. Reason : In a resting state neuron does not Reason : Receptors for gustatory sensations are conduct any impulse. [2016] located in taste buds. [2000] 31. Assertion : A cerebellum is related with skillful 26. Assertion : Astigmatism is due to uneven voluntary movement and involuntary activity curvature of lens. like body balance, equilibrium, etc. Reason : It is treated with cylindrical Reason : It is part of hind brain and it is situated lenses. [2007] behind the pons. [2010, 2017] IIT JEE NEET eBooks: www.crackjee.xyz

Neural Control and Coordination B-111

Type A : Multiple Choice Questions 11. (d) Transmission of nerve impulse is always from axon of one neuron to the dendrite of th 1. (a) Vagus is the 10 cranial nerve. Vagus is another neuron i.e. unidirectional because the longest cranial nerve. It has maximum neurotransmitters are produced by axons branches and also called as wandering and not by dendrites. nerve. 12. (b) Cell A is the cone cell more concentrated in 2. (a) The sensation of stomach pain is due to the fovea centralis/yellow spot of the eye. enteroceptors (visceroreceptors). Cone cells are sensitive to bright light hence 3 (c) Conditioned reflexes are acquired reflexes helps in differentiating colours and give and dependent on past experiences, high resolution. These cells are specialized training and learning I.R. Pavlov for colour vision. demonstrated conditioned reflexes in a 13. (a) Borderline personality disorder is an hungry dog. He called food and salivation emotionally unstable personality disorder in response to it as unconditioned stimulus charactarised by impulsivity, unpredictable and sound of bell and salivation in moods, outburst of emotion, behavioural response to bell as conditioned reflexes. explosions, quarrelsome behaviour and 4. (d) Otorhinolaryngology is the study of ENT. conflicts with others. 5. (a) The withdrawl of leg in a decapitated frog 14. (c) Glaucoma is an eye disease characterized when pin pointed is a type of simple-reflex. by increased occular pressure within the 6. (d) Plague is a bacterial disease of rat, caused eye ball. Glaucoma is a group of diseases by Pasteurella pestis. Their vector is rat of the optic nerve involving loss of retinal flea (Xenopsylla cheopsis). Bubonic plague ganglion cells in a characteristic pattern of affects lymph nodes. Pneumonic plague optic neuropathy. Untreated glaucoma affects lungs and septicemic plague causes leads to permanent damage of the optic anaemia. nerve and resultant visual field loss that 7. (c) The 5th cranial nerve in frog is trigeminal. can progress to blindness.Measles is a highly infectious viral disease that usually 8. (d) Lead zirconate is a key component of spread by droplet infection. Bronchitis is sonography. These crystals are housed in the inflammation of the membrane lining of a transducer which gets excited and starts the bronchial tubes. vibrating when an electric potential is applied to it. These vibrations are the 15. (b) Sympathetic nervous system inhibits source of ultrasound. salivary gland secretion, accelerate heart rate, decreases intestinal peristalsis and 9. (d) Vagus nerve is a mixed nerve. It controls dilate pupil of the eye. Whereas the visceral sensations and visceral parasympathetic nervous system movements, e.g. respiratory movements. stimulates salivary gland secretion, slows 10. (c) Concave lenses correct the eye condition heart rate, sitmulates intestinal peristalsis of near sightedness i.e. myopia by bringing and constricts pupil of the eye. The the light rays to a focus on retina. In such sympathetic and the parasympathetic cases light rays converge at a point in front nervous system are parts of what is of the retina. commonly called the autonomic nervous IIT JEE NEET eBooks: www.crackjee.xyz EBD_7100

B-112 Topicwise AIIMS Solved Papers – BIOLOGY

system. These systems work in balance 18. (c) Bipolar nerve cells are present in retina of with each other and directly or indirectly eye. Bipolar nerve cell posses one axon and affect almost every structure in the body one dendron. (e.g. heart frequency, heart capacity, lumbar 19. (c) Fenestra ovalis is membrane covered function, kidneys, blood vessels, stomach opening leading from tympanic cavity into and intestines). The sympathetic nervous the vestibule of the internal ear. system has an active “pushing” function, 20. (b) Multipolar nerve cells are present in dorsal the parasympathetic has mainly a relaxing root ganglia of spinal cord. function. 21. (c) Dendrites generally receive inputs and Sympathetic Structure Parasympathetic conduct signals toward the cell body, Rate increased Heart Rate decreased whereas axons conduct signals away from Force increased Heart Force decreased the cell body. Bronchial muscle Bronchial muscle 22. (a) 23. (a) Lungs relaxed contracted Type B : Assertion Reason Questions Pupil dilation Eye Pupil cons triction Motility reduced Intestine Digestion increased 24. (a) Transmission of nerve impulse access Sphincter closed Bladder Sphincter relaxed synapses is accomplishes by neurotransmitter because synapses Decreased urine Increased urine Kidneys secretion secretion comprises of a synaptic cleft between the end of one nerve fibres and the beginning 16. (c) Forebrain is the largest part of the brain of the next. consisting of two halves called cerebral 25. (a) Gustatoreceptors are chemoreceptors, hemispheres separated by longitudinal enclosed within taste buds. fissures. Each cerebral hemisphere is divided into four lobes-frontal lobe, parietal 26. (b) Astigmatism is a kind of defect of vision in which the image of an object is distorted. It lobe, temporal lobe and occipital lobe. is because all the light rays do not come to Temporal lobe has cells that bring to focus on retina. It is due to abnormal consciousness, the sensations of hearing curvature of the lens . It can be corrected and smell. The frontal lobe has centers that are concerned with voluntary movements by wearing cylindrical lenses. and personality. The parietal lobe is 27. (b) The brain stem consists of pons varoli, concerned with general sensations like medulla oblongata, mid brain and temperature, touch, pressure, pain, and diencephalon. The brain stem is the proprioception. The occipital lobe has connection between brain and spinal cord. centers of visual sense. It contains centres for controlling many vital activities like respiration, body 17. (a) The inner layer of the posterior two-thirds temperature, urge for eating and drinking of the eyeball consists of a light sensitive etc. It also carries nerve tracts between the layer, called retina that possesses two types spinal cord and the higher brain structure. of photoreceptors called the rods and the 28. (b) The axon terminal of the neuron contains cone cells. Retina reduces the internal many membrane bound vesicles called reflection so any damage to it leads to synaptic vesicles, in its cytoplasm. Within greater internal reflection of light often these vesicles, chemical substances such causing an increase in light sensitivity. as adrenaline and acetylcholine remain IIT JEE NEET eBooks: www.crackjee.xyz

Neural Control and Coordination B-113

stored. These chemicals are called nearly impermeable to sodium ions (Na+). neurotransmitters, because they help to In a resting state neuron does not conduct transmit nerve impulses across the any impulse. In the resting state the period synapses. When a nerve impulse passes the during which a neuron is not conducting axon terminal, its synaptic vesicles release the fluids outside the cell membrane carry their stored chemicals to the synaptic cleft. a relatively high positive charge. The fluids These diffuse through the cleft to reach the inside the cell membrane. Carry a less membrane of the next neuron, stimulating positive, or relatively negative, charge. the latter. This causes the nerve impulse to 31. (b) Hind brain consists of cerebellum located be transmitted along the next neuron. dorsally to medulla oblongate and pons 29. (c) Resting potential is due to differential varolii. It contains centres for maintenance distribution of ions on two sides of cell of posture and equilibrium of the body and membrane. for the muscle tone. All activities of the 30. (d) The axonal membrane of neuron is more cerebellum are involuntary but may involve permeable to potassium ions (K+) and learning in their early stages. IIT JEE NEET eBooks: www.crackjee.xyz EBD_7100

B-114 Topicwise AIIMS Solved Papers – BIOLOGY 22 Chemical Co-ordination and Integration

TYPE A : MULTIPLE CHOICE QUESTIONS (a) a reduction in insulin secretion from pancreas 1. To yield more milk, cow is injected with [1997] (b) a reduction in vasopressin secretion from (a) sorbitol (b) prolactin posterior pituitary (c) gonadotrophs (d) stilbesterol (c) a fall in the glucose concentration in urine 2. Treatment with alloxan destroys [1998] (d) an increase in secretion of glucagon (a) stilt cells 10. The source of somatostatin is same as that of (b) E-cells of Langerhans (a) Thyroxine and calcitonin [2003] (c) sertoli cells (b) Insulin and glucagon (d) cells of Leydig (c) Somatotropin and prolactin 3. Addison's disease result from [1998] (a) hyposecretion of gonads (d) Vasopressin and oxytocin (b) hyposecretion of adrenal gland 11. Which one of the following four secretions is (c) hypertrophy of kidney correctly matched with its source, target and (d) hyposecretion of pituitary gland nature of action? [2005] 4. The development of adult characteristics in a Secretion Source Target Action moulting insect is promoted by [1998] (a) Thyroxine (b) Ecdysone (a) Gastrin Stomach Oxyntic Production (c) Pheromone (d) None of these lining cells of HCl 5. Heart beat increases at the time of interview because (b) Inhibin Sertoli Hypotha- Inhibition of of [1999] cells lamus secretion of (a) renin (b) rennin gonadotropin (c) adrenaline (d) Diuretic ADH releasing 6. Conn’s disease is caused by the over secretion hormone of [1999] (c) Entero- Duodenum Gall Release of (a) ADH (b) ACTH kinase bladder bile juice (c) Oxytocin (d) Aldosterone (d) Atrial Sinu atrial Juxta- Inhibition of 7. Acromegaly is due to hypersecretion of [2001] Natriuretic node (SAN) glomerular release of (a) Insulin (b) Thyroxine Factor M-cells of apparatus renin (c) Growth hormone (d) None of these (ANF) Atria (JGA) 8. Steroid hormones are similar in structure to (a) Tryosine [2001] 12. Which of the following match is correct?[2007] (b) Cholesterol Hormone Effect (c) Coenzyme A (a) Oxytocin Milk ejection hormone (d) Glycerol (b) Glucagon Decreases blood sugar 9. A person passes much urine and drinks much level water put his blood glucose level normal. This (c) Adrenaline Decreases heart rate condition may be the result of [2003] (d) Thyroxine Decreases BMR IIT JEE NEET eBooks: www.crackjee.xyz

Chemical Co-ordination and Integration B-115

13. Which of the following statements regarding 17. Which one of the following hormones contains glucagon is false? [2007] iodine? [2010] (a) It is secreted by D-cells of Langerhans. (a) Thyroxine (b) Testosterone (b) It acts antagonistically to insulin. (c) Insulin (d) Adrenaline (c) It decreases blood sugar level. 18. The pituitary gland by virtue of its tropic (d) The gland responsible for its secretion is hormones controls the secretory activity of other heterocrine gland. endocrine glands. Which one of the following 14. Which one of the following four gland is endocrine gland can function independently of correctly matched with the accompanying the pituitary gland? [2010] description? [2005, 2008] (a) Thyroid (b) Gonads (a) Thyroid - Hyperactivity in young (c) Adrenals (d) Parathyroid children causes cretinism 19. Match List-I (Endocrine glands) with List-II (b) Thymus - Starts undergoing (Hormones secreted) and select the correct atrophy after puberty answer using the codes given below [2010] (c) Parathyroid - Secretes para-thormone, List- I List-II which promotes A. Gonads I. Insulin movement of calcium ions B. Pituitary II. Progesterone from blood into bones C. Pancreas III. Growth during calcification hormones (d) Pancreas - Delta cells of the 'islets of D. Adrenal IV. Cortisone Langerhans' secrete a Codes : hormone, which stimulates glycolysis in (a) A –III; B –II; C –IV; D –I liver (b) A –II; B –III; C –IV; D –I 15. Which row in the chart contains the words that (c) A –II; B –III; C –I; D –IV best complete this statement? The (I) glands (d) A –III; B –II; C –I; D –IV produce (II), which are transported by the (III) 20. Which gland is concerned with salt equilibrium system. [2009] in body? [2012] (a) Anterior pituitary (b) Pancreas Row I II III (c) Adrenal (d) Thyroid A digestive hormones circulatory 21. Which of the following hormones have B endocrine enzymes lymphatic antagonistic (opposing) effects? [2013] C endocrine hormones circulatory (a) Thyroxine and calcitonin D digestive enzymes lymphatic (b) Insulin and glucagon (c) Growth hormone and epinephrine (a)A (b) B (d) ACTH and glucocorticoids (c)C (d) D 22. Select the correct option describing 16. The blood glucose level is commonly expressed gonadotropin activity in a normal pregnant as [2010] female: [2014] (a) mm. of Hg (a) High level of FSH and LH stimulate the (b) milligram per deci litre thickening of endometrium. (c) parts per million (b) High level of FSH and LH facilitate (d) gram (mg/dl) per litre implantation of the embryo. IIT JEE NEET eBooks: www.crackjee.xyz EBD_7100

B-116 Topicwise AIIMS Solved Papers – BIOLOGY

(c) High level of hCG stimulates the synthesis Reason : Anti-diuretic hormone (ADH) is of estrogen and progesterone. secreted by the posterior lobe of pituitary gland. (d) High level of hCG stimulates the thickening [2004] of endometrium. 27. Assertion : Our body secretes adrenaline in 23. Which endocrine gland is called ‘the Throne of intense cold. [2006] immunity’? [2016] Reason : Adrenaline raises metabolic rate. (a) Spleen (b) Thymus Directions for (Qs. 28-31) : Each of these questions (c) Pineal (d) Adrenal medulla contains an Assertion followed by Reason. Read them 24. Which of the following hormone acts upon the carefully and answer the question on the basis of renal tubule and blood capillaries ? [2017] following options. You have to select the one that (a) Glucagon (b) Aldosterone best describes the two statements. (c) Vasopressin (d) Glucocorticoids (a) If both Assertion and Reason are correct and TYPE B : ASSERTION REASON QUESTIONS Reason is the correct explanation of Assertion. (b) If both Assertion and Reason are correct, but Directions for (Qs. 25-27) : These questions consist Reason is not the correct explanation of of two statements, each printed as Assertion and Assertion. Reason. While answering these questions, you are (c) If Assertion is correct but Reason is incorrect. required to choose any one of the following five (d) If both the Assertion and Reason are incorrect. responses. 28. Assertion : Mammary glands are apocrine (a) If both Assertion and Reason are correct and glands. [2009] the Reason is a correct explanation of the Assertion. Reason : The distal part containing secretory (b) If both Assertion and Reason are correct but granules break down and leaves as a secretion. Reason is not a correct explanation of the 29. Assertion : Hormone calcitonin has antagonistic Assertion. effect to that of parathormone. [2009] (c) If the Assertion is correct but Reason is Reason : Calcitonin decreases blood calcium incorrect. level while parathormone increases blood (d) If both the Assertion and Reason are incorrect. calcium level. (e) If the Assertion is incorrect but the Reason is 30. Assertion : The person with diabetes insipidus correct. feels thirsty. 25. Assertion: The regulation of RBC production is Reason : A person with diabetes insipidus accomplished by FSH. suffers from excess secretion of vasopressin. [2010] Reason: Erythropoietin hormone circulates to 31. Assertion: Failure of secretion of hormone red bone marrow where it increases stem cell vasopressin causes diabetes mellitus in the mitosis and speed up development of RBCs. patient. [2011] [2002] Reason: Vasopressin increases the volume of 26. Assertion : Diabetes insipidus is marked by urine by increasing the reabsorption of water excessive urination and too much thirst of water. from the urine. IIT JEE NEET eBooks: www.crackjee.xyz

Chemical Co-ordination and Integration B-117

Type A : Multiple Choice Questions 5. (c) Adrenaline (amine hormone) is secreted by adrenal medulla on stimulation of 1. (b) Prolactin (also called leuteotrophic) sympathetic nervous system for meeting hormone is associated with lactation an emergency or stress condition like fear (secretion of milk from the mammary gland). injury accident etc. Hence, it also called as Therefore, to yield more milk cow is injected emergency hormone. It increases blood with prolactin hormone. Sorbitol is a pressure, respiration rate, sugar level in sweetner found in some fruits (like apple, blood etc. It prepares the body to face stress peers etc). Gonadotrophs a type of basophil at the time of interview by increasing heart in the adenohypohysis (anterior pituitary beat. So it is also called stress hormone. gland) whose granules secrete FSH Renin is proteolytic enzyme (follicele-stimulating hormone) and LH synthesized,stored, and secreted by the (luteinizing hormone). Stilbesterol is juxtaglomerular cells of the kidney. It plays synthetic estrogen used in hormone a role in regulation of blood pressure by therapy, as a post-coital contraceptive, and catalyzing the conversion of as a growth-promoting agent for livestock. angiotensinogen to angiotensin I and II 2. (b) Alloxan treatment damages E-cells of Islets which in turn stimulates the release of langerhans which leads to aldosterone from adrenal gland. Rennin, a hyperglycaemia and glycosuria. coagulating enzyme produced from the 3. (b) Addisons disease occurs due to stomach of human body, catalyzes the coagulation of milk by converting milk hyposecretion of both mineralocorticoids protein, caesin into paracaesinate. ADH (aldosterone) and glucocorticoids (cortisol) (antidiuretic hormone) is secreted by the from the layers of adrenal cortex called zona posterior portion of the pituitary gland that glomerulosa and zona fasciculata constricts blood vessels raises blood respectively. The disease is characterized pressure, and reduces excretion of urine. by excessive loss of Na+, Cl– and HCO 3 6. (d) Conn's syndrome/aldosteronism is due to + increased K level in blood, weight loss, hypersecretion of aldosterone. Aldosetrone muscle weakness, fatigue, low blood is secreted from zona glomerulosa layer of pressure, and sometimes darkening of the adrenal cortex. The principle action of the skin in both exposed and nonexposed parts aldosterone is retention of sodium. Conn’s of the body. syndrome is characterized by rise in blood 4. (b) Ecdysone is a moulting hormone of insects. volume and blood pressure; muscular Ecdysone is produced from prothoracic weakness; high sodium and low potassium gland that triggers moulting and level in the blood plasma resulting in kidney metamorphosis. Thyroxine is thyroid damage with polyuria and tetany and hormone that stimulates body metabolism metabolic disorder. and helps regulate body growth and 7. (c) Acromegaly is due to hyposecretion of development. Pheromone is a chemical growth/somatotrophic hormone in adults screted by an animal that influences the after the closure of epiphysed plate at the behavior or development of other members end of long bones. Growth hormone is of the same species. secreted by artesian pituitary gland. IIT JEE NEET eBooks: www.crackjee.xyz EBD_7100

B-118 Topicwise AIIMS Solved Papers – BIOLOGY

It is characterised by abnormal elongation Vasopressin and oxytocin are secreted from of limbs and lower jaw giving gorilla like posterior pituitary gland. appearance. Hypersecretion of insulin 11. (d) Atrial natriuretic factor (ANF) is produced results in hypoglycemia, hunger, sweating by cardiocytes of atria of heart in response and double vision. Hypersecretion of to an increased return of the deoxygenated thyroxine results in Grave's disease (also blood. It inhibits the release of renin from called exopthalmic goiter) which is juxta-glomerular apparatus and thereby, characterised by increased BMR, heart rate, inhibits NaCl reabsorption by the collecting pulse rate; protrusion of eyes etc. duct and reduces aldosterone release from 8. (b) Steroid hormones are fat soluble and have the adrenal cortex. Inhibin is an endocrine sterol group. They are derived from hormone, produced from ovary and testes. cholesterol, e.g. hormones of adrenal When inhibin is secreted, it inhibits the cortex, testis and ovaries. Amine hormones production of follicle stimulating hormone are derived form tyrosine amino acids and (FSH). It also limits the release of have amino group, e.g. thyroxine, gonadotropin releasing hormone. (For epinephrine and nor epinephrine. other hormones refer answer no. 4) Coenzyme A is essential to metabolism of 12. (a) Oxytocin is the hormone secreted by carbohydrates and fats and some amino posterior pituitary that causes contraction acids. Glycerol is a clear, colorless, viscous, of the smooth muscles of myometrium sweet-tasting liquid organic compound of during child birth and ejection of milk from the alcohol family. the mammary glands. Glucagon is secreted 9. (b) When a person passes more urine and by the D-cells of islets of langerhans of drinks more water, he is suffering from pancreas. Its main fuction is to increase diabetes insipidus. It is caused by blood glucose level. Adrenaline reduction in vasopressin (ADH) secretion (epinephrine) is secreted by adrenal from post pituitary gland. medulla. It increases the rate and force of 10. (b) The source of somatostatin is same as that heart beat. Thyroid gland secretes of insulin and glucagons. All these thyroxine that regulates basal metabolic hormones are secreted from endocrine part rate. (called Islets of Langerhans) of pancreas. 13. (c) Pancreas is a heterocrine gland comprising These endocrine parts contain different both endocrine and exocrine parts. Its types of endorcrine cells which secretes endocrine part consists of small masses of different hormones. hormone secreting cells called islets of langerhans. The D-cells of latter secrete Endocrine cells Hormones glucagons and its E-cells secrete insulin. D cell Glucagon These two hormones have antagonistic E cell Insulin effects on the glucose level in the blood J cell Gastrin which means that insulin decreases the blood glucose level while glucagon G cell Somatostatin increases blood glucose level. F cell Pancreatic polypeptides 14. (b) The thymus is an organ located in the upper anterior portion of the chest cavity just Thyroxine and calcitonin are secreted by behind the sternum. The thymus continues thyroid gland. Somatotropin and prolactin to grow between birth and puberty and then are secreted from anterior pituitary gland. begins to atrophy. Proportional to thymic IIT JEE NEET eBooks: www.crackjee.xyz

Chemical Co-ordination and Integration B-119

size, thymic activity is most active before 24. (c) ADH (or vasopressin) is secreted by puberty. Upon atrophy, the size and activity posterior pituitary gland. It acts on kidney are dramatically reduced, and the organ is tubule and blood capillaries and primarily replaced with fat. The atrophy is concentrates the urine by promoting the due to the increased circulating level of sex reabsorption of water and salts into the hormones, and chemical or physical cortical collecting ducts. castration of an adult result in the thymus increasing in size and activity. Type B : Assertion Reason Questions 15. (c) The endocrine glands produce hormones, which are transported by the circulatory 25. (e) The regulation of RBC production is system. The digestive system makes accomplished by erythropoietin hormone enzymes that are secreted via ducts to the (EPO). Erythropoietin, a glycoprotein, is organ that needs them. produced by the kidney when the oxygen 16. (b) Blood glucose level is commonly expressed level of low EPO then stimulates the bone as milligram per deci litre. marrow to produce more red cells and 17. (a) The main secretion of thyroid gland is thereby increase the oxygen-carrying called thyroxine. Thyroxine contains iodine. capacity of the blood. Follicle-stimulating When thyroid gland becomes inactive, the hormone (FSH) is a gonadotropic hormone lack of iodine causes goitre. that is secreted by the anterior pituitary 18. (d) Parathyroid gland secretes parathormone gland. FSH causes gametogenesis and ++ 2 stimulates estrogen production from hormone, which regulates Ca and PO4 ovaries. ion in body. This gland works independently to pituitary gland. 26. (b) Diabetes insipidus (DI) occurs when the 19. (c) Gonads, pituitary, pancreas & adrenal are kidneys are unable to conserve water as all endocrine glands which secretes they perform their function of filtering progresterone, growth hormone, insulin and blood. The amount of water conserved is coltisone hormone respectively. controlled by antidiuretic hormone (ADH) 20. (c) Adrenal glands are concerned with salt also called vasopressin which is secreted equilibrium in the body. Mineralorcoticoids by posterior lobe of pituitary gland. secreted from adrenal cortex. Aldosterone Diabetes inscipidus is characterised by is the main mineralocorticoid in our body. excessive urination and thirst. This Aldosterone acts mainly at the renal problem appears due to the increase in + tubules and stimulates reabsorbtion of Na permeability of collecting tubules. and water and excretion of K+ and 27. (a) Adrenaline is an emergency hormone phosphate ions. whose concentration increases under 21. (b) Insulin lowers blood sugar levels. While stress conditions. e.g. cold, stress. Glucagon raises blood sugar levels. Adrenaline is secreted from adrenal 22. (c) Synthesis of estrogen and progesterone medulla. It initiates many bodily responses, due to high level of hCG is a normal including the stimulation of heart action gonadotropic activity in a normal pregnant female. and an increase in blood pressure, metabolic rate, and blood glucose 23. (b) Thymus stimulates T-cells which regulate the production of antibodies. concentration. IIT JEE NEET eBooks: www.crackjee.xyz EBD_7100

B-120 Topicwise AIIMS Solved Papers – BIOLOGY

28. (a) Based on the mode of secretion, the glands calcitonin hormone increases excretion of are of three types : mesocrine, apocrine and calcium in urine. holocrine. Mammary glands that are present 30. (c) Vasopressin or antidiuretic hormone is in mammals to feed the young ones with secreted by posterior pituitary gland. The milk are the example of apocrine glands. In deficiency of vassopressin results in a apocrine glands, the secretion accumulates disorder known as diabetes insipidus. The as secretory granules in the distal part of main symptoms of diabetes insipidus are the cell. This part later breaks down and increase in thirst and increase in urination. leaves as a secretion. 31. (d) Vasopressin or anti-diuretic hormone 29. (a) Calcitonin or thyrocalcitonin is secreted by (ADH) reduces the volume of urine by parafollicular cells of thyroid stroma. It increasing the reabsorption of water from retards bone dissolution and stimulates the urine in the distal convoluted tubules, excretion of calcium in urine. Thus, it lowers collecting tubules and collecting ducts in calcium level in extra cellular fluid (ECF). the kidney. It does so by rendering the walls Parathormone is secreted by chief cells of of these tubules leads to diabetes insipidus parathyroid gland and is also known as (increased urination). Although the volume Collip's hormone. It maintains blood calcium of urine is increased. No glucose appears in level by increasing its absorption from food the urine of such patients. in intestine and its reabsorption from Diabetes mellitus is a disease which is nephrons in the kidney. Maintenance of caused due to the failure of insulin hormone proper calcium level is in fact, a combined secretion by the pancreatic islets. The function of parathormone and calcitonin. When calcium level falls below normal osmotic effect of glucose in the urine parathormone maintains it by promoting its considerably increases the volume of urine, absorption, reabsorption and also by due to which thirst is also enhanced. In demineralisation of bones. When blood extreme cases, the patient suffers from coma calcium level exceeds above normal then and may die. IIT JEE NEET eBooks: www.crackjee.xyz

23 Reproduction in Organisms

TYPE A : MULTIPLE CHOICE QUESTIONS 4. Identify the events (A, B, D and E) in life of general reproduction- 1. Based on cellular mechanisms there are two major types of regeneration found in the animals. Which one of the following is the correct example of the type mentioned? [2005] (a) Morphallaxis - Regeneration of two transversely cut equal pieces of a Hydra [2015] into two small Hydras. (a) A-Gamete transfer, B-Gametogeneis, D-Zygote formation, E-Embryogenesis (b) Epimorphosis - Replacement of old and dead erythrocytes by the new ones. (b) A-Gametogeneis, B-Gamete transfer, D-Zygote formation, E-Embryogenesis (c) Morphallaxis - Healing up of a wound in (c) A-Gametogeneis, B-Zygote formation, the skin. D-Gamete transfer, E-Embryogenesis (d) Epimorphosis - Regeneration of crushed (d) A-Gametogeneis, B-Gamete transfer, and filtered out pieces of a Planaria into as D-Embryogenesis, E-Zygote formation. many new Planarians. from chapter 25 5. Which of the following statements is incorrect ? 2. Which form of reproduction is correctly (i) Bamboo species flower only once in their matched? [2007] life time, generally after 50-100 years and (a) Euglena o transverse binary fission produce large number of fruits and die. (b) Paramecium o longitudinal binary (ii) In animals, the juvenile phase is followed fission by morphological and physiological (c) Amoeba o multiple fission changes prior to active reproductive (d) Plasmodium o binary fission behaviour. 3. Which reproductive adaptation is characteristic (iii) The reproductive phase is of same duration of most terrestrial vertebrates but not of most in all organisms. aquatic vertebrates? [2009] (iv) Juvenile phase is the period of growth (a) External fertilization between the birth of an individual till it (b) Internal fertilization reaches reproductive maturity. [2016] (c) Motile gametes (a) Only (i) (b) Only (ii) (d) External development (c) Only (iii) (d) Only (iv) IIT JEE NEET eBooks: www.crackjee.xyz EBD_7100

B-122 Topicwise AIIMS Solved Papers – BIOLOGY

6. In a practical test, a student has to identify the carefully and answer the question on the basis of organisms in which syngamy does not occur. In following options. You have to select the one that those organisms the female gamete undergoes best describes the two statements. development to form new organisms without (a) If both Assertion and Reason are correct and fertilization. This phenomenon is called "X". Reason is the correct explanation of Assertion. Identify the organisms and the phenomenon (b) If both Assertion and Reason are correct, but "X". [2017] Reason is not the correct explanation of (a) Frog, Parthenogenesis Assertion. (b) Lizards, Gametogenesis (c) If Assertion is correct but Reason is incorrect. (c) Rotifers, Embryogenesis (d) If both the Assertion and Reason are incorrect. (d) Honeybee, Parthenogenesis 7. Assertion : Leaves of Bryophyllum, Begonia TYPE B : ASSERTION REASON QUESTIONS help in vegetative multiplication. Reason : Leaves of these plants possess Directions for (Q. 7) : Each of these questions adventitious buds. [2014] contains an Assertion followed by Reason. Read them IIT JEE NEET eBooks: www.crackjee.xyz

Reproduction in Organisms B-123

Type A : Multiple Choice Questions sperm directly into the female body. This is advantageous since sperm need liquid to 1. (a) Morphallaxis is the production of an entire swim and the moist female reproductive animal from a small fragment whereas tract provides this. Aquatic vertebrate epimorphosis is the replacement of the lost animals live in water so sperm can easily part. be deposited in the water and swim to the female reproductive tract. 2. (c) Reproduction is the production of a new 4. (b) Gametogenesis leads to production of generation of individuals of the same gametes (sperm and ovum). Male gametes species. It involves transfer of genetic are then transferred to the site of material from one generation to the next. fertilization. Fertilization results in zygote Asexual and sexual are the two types of formation. The zygote then gives rise to reproduction. Fission and budding are two embryo. most common forms of asexual 5. (c) Statement (iii) is not correct. The reproduction in animals. During adverse reproductive phase is not of same duration conditions, amoeba reproduces by multiple in all organisms. fission that gives rise to many amoeba. 6. (d) Parthenogenesis is a form of reproduction Euglena reproduces by longitudinal binary in which an unfertilized egg develops into fission, Paramecium reproduces by a new individual, occurring commonly transverse binary fission and Plasmodium among insects and certain other reproduces by multiple fission. arthropods. 3. (b) Most vertebrate animals that live on land Type B : Assertion Reason Questions have an adaptation that allows internal fertilization, whereby the male penis inserts 7. (a) IIT JEE NEET eBooks: www.crackjee.xyz EBD_7100

B-124 Topicwise AIIMS Solved Papers – BIOLOGY Sexual Reproduction in 24 Flowering Plants

9. Xenia refers to [2002] TYPE A : MULTIPLE CHOICE QUESTIONS (a) effect of pollen on endosperm 1. Asexual reproduction is called as [1997] (b) effect of pollen on stems (a) apomixis (b) fragmentation (c) effect of pollen on taste of fruits (c) self fertilization (d) cross fertilization (d) effect of pollen on vascular tissue 2. Pollination by snail and slug is called as 10. Ploidy of ovum of angiosperms is [2002] [1998] (a) haploid (b) diploid (a) entomophilous (c) triploid (d) polyploid (b) malacophilous 11. Pollen grains are able to withstand extremes of (c) ornithophilous temperature and dessication because their exine (d) chiropterophilous is composed of [2003] 3. In angiosperm, the endosperm is [1998] (a) cutin (b) suberin (a) diploid (b) triploid (c) sporopollenin (d) callose (c) haploid (d) polyploid 12. The pollen tube usually enters the embryo sac 4. Female gametophyte of angiosperm is [1999] (a) between the egg cell and synergid[2004] (b) by directly penetrating the egg (a) 7 celled (b) 8 celled (c) between one synergid and antipodal cell (c) 11 celled (d) 5 celled (d) by knocking off the antipodal cells 5. Anemophilous flowers have [1999] 13. Double fertilization involves [2005] (a) sessile stigma (a) fertilization of egg by two male gametes (b) small, smooth stigma (b) fertilization of two eggs in the same embryo (c) coloured and scented flowers sac by two sperms brought by one pollen tube (d) large feathery stigma (c) fertilization of the egg and the central cell 6. Growth of pollen tube towards embryo is [2000] by two sperms brought by different pollen tubes (a) geotropism (b) chemotaxis (d) fertilization of the egg and the central cell (c) phototaxis (d) thigmotaxis by two sperms brought by the same pollen tube 7. Which of the following statement is true? 14. In which one of the following combinations [2000] (a - d) the number of chromosomes of the present (a) Spores are gametes day hexaploid wheat is correctly represented? (b) Spores and gametes are diploid [2006] (c) Gametes are always haploid Comb- Mono- Haploid Nulli- Tri- (d) Spores are always diploid ination somic somic somic 8. Which part of embryo comes out first during (a) 21 28 42 43 seed germination ? [2001] (b) 7 28 40 42 (c) 21 7 42 43 (a) Radicle (b) Plumule (d) 41 21 40 43 (c) Hypocotyl (d) Epicotyl IIT JEE NEET eBooks: www.crackjee.xyz

Sexual Reproduction in Flowering Plants B-125

15. Apomixis is [2007] 21. Which of the following statement is correct? (a) formation of seeds by fusion of gametes. [2016] (b) formation of seeds without syngamy and (a) Sporopollenin can withstand high meiosis. temperatures but not strong acids. (c) formation of seeds with syngamy but no (b) Sporopollenin can be degraded by meiosis. enzymes. (d) None of the above (c) Sporopollenin is made up of inorganic materials. 16. The plant part which consists of two (d) Sporopollenin can withstand high generations one within the other, is [2008] temperature as well as strong acids and (a) germinated pollen grain alkalis. (b) embryo (c) unfertilized ovule TYPE B : ASSERTION REASON QUESTIONS (d) seed Directions for (Qs. 22-24) : These questions consist 17. Chasmogamy refers to the condition where of two statements, each printed as Assertion and [2012] Reason. While answering these questions, you are (a) Flowers remains closed required to choose any one of the following five (b) Flowers are absent responses. (c) Flowers are open (a) If both Assertion and Reason are correct and (d) Flower are gamopetalous the Reason is a correct explanation of the 18. What is common between vegetative Assertion. reproduction and apomixis? [2013] (b) If both Assertion and Reason are correct but (a) Both are applicable to only dicot plants Reason is not a correct explanation of the (b) Both bypass the flowering phase Assertion. (c) Both occur round the year (c) If the Assertion is correct but Reason is (d) Both produces progeny identical to the incorrect. parent (d) If both the Assertion and Reason are incorrect. 19. Emasculation is not required when flowers are (e) If the Assertion is incorrect but the Reason is [2013] correct. (a) bisexual (b) intersexual 22. Assertion : If pollen mother cells has 42 (c) unisexual (d) either (a) or (b) chromosomes, the pollen has only 21 20. Geitonogamy involves: chromosomes. (a) fertilization of a flower by the pollen from Reason : Pollens are formed after meiosis in another flower of the same plant. pollen mother cell. [1997] (b) fertilization of a flower by the pollen from 23. Assertion: The megaspore mother cell divide the same flower. mitotically to produce four spores. (c) fertilization of a flower by the pollen from a Reason: Megaspore mother cells are diploid and flower of another plant in the same megaspore is haploid. [2002] population. 24. Assertion : Insects visit flower to gather honey. (d) fertilization of a flower by the pollen from a Reason : Attraction of flowers prevents the flower of another plant belonging to a insects from damaging other parts of the plant. distant population. [2014] [2004] IIT JEE NEET eBooks: www.crackjee.xyz EBD_7100

B-126 Topicwise AIIMS Solved Papers – BIOLOGY

Directions for (Qs. 25-28) : Each of these questions 26. Assertion : Chasmogamous flowers require contains an Assertion followed by Reason. Read them pollinating agents. carefully and answer the question on the basis of Reason : Cleistogamous flowers do not expose following options. You have to select the one that their sex organs. [2012] best describes the two statements. 27. Assertion : Double fertilization is characteristic (a) If both Assertion and Reason are correct and feature of angiospersms. Reason is the correct explanation of Assertion. Reason : Double fertilization involves two (b) If both Assertion and Reason are correct, but fusions. [2016] Reason is not the correct explanation of Assertion. 28. Assertion : Endosperm is a nutritive tissue and (c) If Assertion is correct but Reason is incorrect. it is triploid. (d) If both the Assertion and Reason are incorrect. Reason: Endosperm is formed by fusion of 25. Assertion : Pollen mother cells (PMCs) are the secondary nucleus to second male gamete. It is first male gametophytic cells. [2009] used by developing embryo. [1998, 2017] Reason : Each PMC gives rise to two pollens. IIT JEE NEET eBooks: www.crackjee.xyz

Sexual Reproduction in Flowering Plants B-127

Type A : Multiple Choice Questions 8. (a) The radicle comes out first since it grows towards the earth. During seed germination 1. (c) In asexual reproduction, single parent is the radicle comes out first due to involved. It usually includes amitosis or gravitational force and further more it mitotic division. results in a differential growth. 2. (b) Pollination by insects is entomophily, 9. (a) Xenia is the effect of pollen genes on the pollination by birds is ornithophily, development of the fruit or seed. pollination by bats is chiropterophily and 10. (a) Ovum is a female gamete and is always pollination by molluscs (snow, slugs) is haploid. malacophily. 11. (c) Sporopollenin, which is the hardest 3. (b) Endosperm is formed as a result of triple fusion male gamete (n) + seconday nucleus substance, helps the pollen grains to (2n) = Primary endosperm nucleus (3n) withstand extremes of temperatures. It Secondary nucleus is formed by the fusion avoids transpiration or water loss. This of 2 polar nuclei. hard proteinaceous substance present in 4. (a) The female gametophyte of angiosperms the exine makes it also spinous in nature. is eight nucleated and seven celled. The 12. (a) The synergids direct the growth of pollen organized embryo sac comprises a 3 celled tube by secreting some chemical egg apparatus, three antipodal cells and a substances. The tip of pollen tube enters bipolar central cell. The embryo sac into one synergid. although eight nucleated has only seven 13. (d) Double fertilization involves fertilization of cells. the egg/oosphere (2n) and that of 5. (d) Anemophilous flowers have feathery secondary nucleus (3N) by two different stigma. It is the characteristic feature of sperms produced in the same pollen tube. Gramineae family (grass). In grasses, the stigma, that is plumose, works as an 14. (d) 1n = 21; monosomic (2n – 1) = 42 – 1 = 41; efficient pollen catcher. Hence, nullisomic (2n – 2) = 42 – 2 = 40. Trisomic anemophilous flowers have feathery or (2n + 1) = 42 +1 = 43 plumose stigma. 15. (b) In plants, normal sexual reproduction 6. (b) Growth of pollen tube towards embryo is includes meiosis and fertilization . It is chemotaxis due to the stimulus being called amphimixis. But in some plants chemical in nature. The chemical stimulus abnormal sexual reproduction called is supplied in the form of Ca++ ions. apomixis has been observed. Apomixis 7. (c) Gametes are always haploid in order to includes abnormal sexual reproduction in preserve the species genetically, anatomically which egg or other cells like synergids and and morphologically also. The embryo or antipodals develop into embryo without zygote is formed due to the union of male fertilization and meiosis. The term apomixis and female gametes. (n + n = 2n). Hence, any was given be Winkler (1908) eg. Citrus, species which is 2n is diploid in nature. Rannunculus. IIT JEE NEET eBooks: www.crackjee.xyz EBD_7100

B-128 Topicwise AIIMS Solved Papers – BIOLOGY

16. (b) The plant which consists of two Type B : Assertion Reason Questions generations one within the other is embryo. In botany, a seed plant embryo is part of a 22. (a) Pollen mother cells undergo meiosis and produce pollen grains. The pollen grains seed, consisting of precursor tissues for have haploid number of chromosomes. the leaves, stem and root as well as one or 23. (e) Megaspore mother cell is a prominent cell more cotyledons. Once the embryo begins in the nucellus. It divides by meiosis and to germinate, grow out from the seed, it is forms a row of four haploid megaspores. called a seedling. Plants that do not produce 24. (d) Honey bee visit flowers to gather nectar seeds, but do produce an embryo, include and turn it into honey. Visiting of insects the bryophytes and ferns. In these plants, for nectar helps in pollination. the embryo is a young plant that grows 25. (d) Primary sporogenous cell gives rise to attached to a parental gametophyte. microspore mother cells or pollen mother 17. (c) Chasmogamous flowers are always open. cells (PMCs). They are sporophytic in nature In same bisexual plants like Commelina & i.e., diploid. These cells undergo meiosis Viola. Chasmogamous and cleistogamous (reduction division) which gives rise to 4 microspores or pollens and this formation flowers (which never opens throughout the of microspores or pollens is called life) are found. microsporo-genesis. Microspores represent 18. (d) Vegetative reproduction and apomixis both the beginning of the gametophytic phase are asexual methods of reproduction, and they are haploid in nature. which gives the progeny genetically similar 26. (b) The majority of angiosperms bear to parent. chasmogamous flowers, which means the flowers expose their mature anthers and 19. (c) In unisexual flowers, the female flower buds stigma to the pollinating agents. There is are bagged before the flowers open. When another group of plants which set seeds the stigma become receptive, pollination is without exposing their sex organs. Such carried out using the desired pollen and flowers are called cleistogamous and the the flower rebagged. Hence, there is no phenomenon is cleistogamy. need of emasculation in these flowers. 27. (b) Double fertilization is a characteristic 20. (a) Geitonogamy is the transfer of pollen grains feature of angiosperms. It involves two in different flowers of same plant. fusions in which one female gametes fuse with egg cell to form zygote and other male 21. (d) Pollen grains are generally spherical and gamete fuses with the diploid secondary prominent two-layered wall. The hard outer nucleus to produce triploid primary layer (called the exine) is made up of endosperm nucleus. sporopollenin which is one of the most 28. (a) Male gamete (n) + secondary nucleus (2n) resistant organic material known. It can = primary endosperm nucleus which withstand high temperatures and strong develops into endosperm (3n) acids and alkali. Endosperm is the reserve food used by developing embryo. IIT JEE NEET eBooks: www.crackjee.xyz

25 Human Reproduction

TYPE A : MULTIPLE CHOICE QUESTIONS 10. Which one of the following statements with regard to embryonic development in humans is 1. Prostate gland secretion helps in formation of correct? [2003] (a) larva (b) semen [1997] (a) Cleavage divisions bring about (b) cocoon (c) none of these considerable increase in the mass of 2. Immediately after ovulation, the mammalian egg protoplasm. is covered by a membrane called as [1998] (b) In the second cleavage division, one of the (a) chorion (b) corona radiata two blastomeres usually divides a little (c) zona pellucida (d) none of these sooner than the second. 3. The extra-embryonic membranes of mammalian (c) With more cleavage divisions, the resultant embryo are derived from [1999] blastomeres become larger and larger. (a) trophoblast (b) follicle cells (c) inner cell mass (d) formative cells (d) Cleavage division results in a hollow ball of cells called morula. 4. Acrosome of sperm is formed by [1999] (a) nucleus (b) golgi bodies 11. Women who consumed the drug thalidomide (c) lysosome (d) E. R. for relief from vomiting during early months of 5. Cumulus covers [1999] pregnancy gave birth to children with [2004] (a) ovary (b) ovum (a) no spleen (c) embryo (d) sperm (b) hare-lip 6. Cessation of menstrual cycle in women is called (c) extra fingers and toes (a) menopause (b) lactation [2001] (d) under developed limbs (c) ovulation (d) parturition 12. A cross section at the midpoint of the middle 7. Both corpus luteum and macula lutea are [2003] piece of a human sperm will show [2005] (a) found in human ovaries (a) centriole, mitochondria and 9 + 2 (b) a source of hormones arrangement of microtubules. (c) characterized by a yellow colour (b) centriole and mitochondria. (d) contributory in maintaining pregnancy (c) mitochondria and 9 + 2 arrangement of microtubules. 8. The early human embryo distinctly possesses (d) 9 + 2 arrangement of microtubules only. (a) gills [2003] 13. Which one of the following events is correctly (b) gill slits matched with the time period in a normal (c) external ear (pinna) menstrual cycle? [2005] (d) eyebrows (a) Release of egg : 5th day 9. The phase of menstrual cycle in humans that (b) Endometrium regenerates : 5-10 days lasts for 7-8 days, is [2003] (c) Endometrium secretes nutrients for (a) follicular phase (b) ovulatory phase implantation : 11-18 days (c) luteal phase (d) menstruation (d) Rise in progesterone level : 1-15 days IIT JEE NEET eBooks: www.crackjee.xyz EBD_7100

B-130 Topicwise AIIMS Solved Papers – BIOLOGY

14. Which of the following is true regarding 22. All of the following statements concerning sperm? [2007] pregnancy are accurate EXCEPT [2015] (a) Fertilizin: For penetrating egg membrane (a) the detection of human chorionic (b) Hyalurodinase: For penetrating egg gonadotropin in the urine forms the basis membrane (c) Acrosin: Dissolves corona radiata for pregnancy tests. (d) Capacitation: Takes place in penis (b) the cyclic release of pituitary 15. Both corpus luteum and macula lutea are [2008] gonadotropins and ovarian steroids is (a) found in human ovaries continued. (b) a source of hormones (c) the mammary gland tissue of the pregnant (c) characterized by a yellow colour woman is stimulated to develop by (d) contributory in maintaining pregnancy placental hormones. 16. In humans, what is the ratio of the number of (d) the corpus luteum of pregnancy maintains gametes produced from one male primary sex the uterus until the placenta is well cell to the number of gametes produced from established. one female primary sex cell? [2009] (a) 1:3 (b) 1:4 23. The following graph of relative concentrations (c) 3:1 (d) 4:1 of the four hormones present in the blood plasma 17. Corpus luteum is a mass of cells found in[2010] of a woman during her menstrual cycle. Identify (a) brain (b) ovary the hormones. [2015] (c) pancreas (d) spleen 18. Cells of leydig are found in [1997, 2011]

n o i (a) Testes of frog (b) Testes of rabbit t C D a r t n

(c) Kidney of frog (d) Kidney of rabbit e

c B n

o A 19. Meroblastic cleavage refers to which type of c

g n i

division of egg [2001, 2011] of hormone s a e r

(a) Complete (b) Spiral c n i (c) Incomplete (d) Horizontal day 20. Which of the following organ is differentiated first during development? [2012] A B C D (a) Heart (b) Skin (a) FSH Progesterone LH Oestrogen (c) Brain (d) Neural tube (b) LH Progesterone FSH Oestrogen 21. The correct sequence of spermatogenetic stages (c) FSH Oestrogen LH Progesterone leading to the formation of sperms in a mature human testis is: [2013] (d) LH Oestrogen FSH Progesterone (a) spermatogonia-spermatid-spermatocyte- 24. Ejaculation of human male contains about sperms 200 – 300 million sperms, of which for normal (b) spermatocyte-spermatogonia-spermatid- fertility ____ % sperms must have normal shape sperms and size and at least ____% must show (c) spermatogonia-spermatocyte-spermatid- energetic motility. [2016] sperms (a) 40, 60 (b) 50, 50 (d) spermatid-spermatocyte-spermatogonia- sperms (c) 60, 40 (d) 30, 70 IIT JEE NEET eBooks: www.crackjee.xyz

Human Reproduction B-131

25. The given figure shows the human foetus within (c) C: Grafian follicle, mature follicle which the uterus with few structures marked as A, B, C ruptures to release secondary oocyte. and D. (d) D: Tertiary follicle, a large number of this B follicle degenerates during the phase from birth to puberty. Cavity of uterus TYPE B : ASSERTION REASON QUESTIONS A C Embryo Directions for (Qs. 27-31) : These questions consist of two statements, each printed as Assertion and Reason. While answering these questions, you are required to choose any one of the following five D responses. Which of the following options shows the (a) If both Assertion and Reason are correct and correct labeling? [2016] the Reason is a correct explanation of the (a)AoUmbilical cord with its veins, Bo Assertion. Chorionic villi, CoAntrum, DoPlug of (b) If both Assertion and Reason are correct but mucus in cervix Reason is not a correct explanation of the (b) AoUmbilical cord with its vessels, Assertion. BoFimbriae, Co Oocyte, DoPlug of (c) If the Assertion is correct but Reason is mucus in vagina incorrect. (c)AoUmbilical cord with its vessels, (d) If both the Assertion and Reason are incorrect. BoPlacental villi, CoYolk sac, DoPlug (e) If the Assertion is incorrect but the Reason is of mucus in cervix correct. (d) AoUmbilical cord with its veins, 27. Assertion : During fertilization only head of BoPlacental villi, CoTrophoblast, spermatozoa enters egg. DoPlug of mucus in vagina [2016] Reason : If several spermatozoa hit the egg at 26. The figure given below shows the sectional same time, all can enter the egg. [1997] view of ovary. Select the option which gives 28. Assertion : In morula stage, cells divide without correct identification of marked structure (A to D) increase in size. and its feature. [2017] Reason : Zona pellucida remains undivided till

A B cleavage is complete. [1997] 29. Assertion : Death is one of the important C regulatory process on earth. Reason : It avoids over-crowding caused by continuous reproduction. [2002] 30. Assertion : Old age is not an illness. It is a [ 2017] continuation of life with decreasing capacity for adaptation. Reason : Cessation of mitosis is a normal D genetically programmed event. [2003] (a) A: Primary follicle, it is also called gamete 31. Assertion : Senescence is the time when age mother cell. associated defects are manifested. [2005] (b) B: Corpus luteum, it cannot be formed and Reason : Certain genes may be undergoing added after birth. sequential switching on and off during one's life. IIT JEE NEET eBooks: www.crackjee.xyz EBD_7100

B-132 Topicwise AIIMS Solved Papers – BIOLOGY

Directions for (Qs. 32-38) : Each of these questions Reason : The eggs of mammal are microlecithal contains an Assertion followed by Reason. Read them and telolecithal. carefully and answer the question on the basis of 35. Assertion : Head of sperm consists of acrosome following options. You have to select the one that and mitochondria. best describes the two statements. Reason : Acrosome contains spiral row of (a) If both Assertion and Reason are correct and mitochondria. [2014] Reason is the correct explanation of Assertion. 36. Assertion : Females have less stature than males (b) If both Assertion and Reason are correct, but after puberty. Reason is not the correct explanation of Reason : This happens because of the presence Assertion. of hCG in the blood of females. [2014] (c) If Assertion is correct but Reason is incorrect. 37. Assertion : Testicular lobules are the (d) If both the Assertion and Reason are incorrect. compartments present in testis. 32. Assertion : Corpus luteum degenerates in the Reason : These lobules are involved in the absence of fertilization. [2009] process of fertilization. [2016] Reason : Progesterone level decreases. 38. Assertion : Interstitial cell is present in the region 33. Assertion : Clitoris is not remnant of penis in outside the seminiferous tubule called interstitial females. [2009] spaces. Reason : It also has high blood supply and Reason : Interstitial cells provide nutrition to erectile tissue. the sertoli cells. [2016, 2017] 34. Assertion : Mammalian ova produces hyaluronidase. [2009] IIT JEE NEET eBooks: www.crackjee.xyz

Human Reproduction B-133

Type A : Multiple Choice Questions The period of menopause is between 45-55 years. 1. (b) The prostate gland is a chestnut shaped 7. (c) Corpus luteum is the fluid filled yellow gland which lies at the base of bladder and body in the ovary and macula lutea is the surrounds the first part of the urethra. This yellow spot present in the eyes. gland secretes a slightly alkaline fluid 8. (c) which forms the important component of 9. (b) In menstrual cycle, menstrual phase lasts semen. This fluid constitutes the major for 4 days, proliferating/ovulating phase portion of seminal fluid which carries sperm for about 10 days and secretory phase for and sperms move freely in this fluid. 14 days. Prostate gland secretion contains lipids, 10. (a) Repeated cleavage in the zygote brings small amount of citric acid,  ions HCO3 about the distribution of the cytoplasm of and a few enzymes. They activate and the zygote among blastomere and provides nutrition to sperms and neutralise increases mobility of the protoplasm which the acidity of urine which may kill the facilitates morphogenetic movements for sperms. They form about 30% part of cell differentiation. semen. 11. (d) Woman who took the drug thalidomide in 2. (d) Immediately after ovulation, mammalian early pregnancy gave birth to children with eggs are covered by vitelline membrane. severe birth defects such as missing or 3. (a) Trophoblast in mammalian embryo shortened limbs. produces extra embryonic membranes 12. (c) A cross section at the midpoint of the which provide protection and nourishment middle piece of human sperm will show to foetus. These are of 4 types – chorion, mitochondria and 9+2 arrangement of amnion, allantois & yolk sac embryonic microtubules. membranes. 13. (b) The proliferative phase lasts for about 14 4. (b) Acrosome of sperm is formed from Golgi days in which the endometrium becomes bodies and contains hydrolysing enzymes thicker by rapid cell multiplication. for sperm penetration. 5. (b) Cumulus covers the ovum. The ovum at Cycle begins 5 6 7 the matured conditions has a massy cloud 4 8 here 9 formed with a flat base and rounded 3 2 10 outlines piled up like a mountain. A Menstrual 1 11 granulosa cell is a somatic cell found Phase 28 12 closely associated with the developing Proliferative female gamete (oocyte or egg) in the ovary 27 Phase 13 Secretory of mammals. Granulosa cells form a single 26 14 Phase flattened layer around the oocyte in the 25 15 primordial ovarian follicle and later in follicle 16 24 development they advance to form a multi 17 23 18 layered cumulus surrounding the oocyte. 22 19 Corpus luteum 21 20 Ovulation 6. (a) Menopause is the period when ovulation degenerates and menstrual cycle stop in human females. IIT JEE NEET eBooks: www.crackjee.xyz EBD_7100

B-134 Topicwise AIIMS Solved Papers – BIOLOGY

14. (b) Sperm is the male gamete that fuses with during pregnancy inhibit the cyclic release female gamete and produce a diploid cell of pituitary gonadotropins and prevent the called zygote. During fertilization, menstrual cycles. Pregnancy hormones acrosome of the sperm releases some include hCG, which maintains the corpus enzymes, particularly hyaluronidase, that luteum of pregnancy and forms the basis facilitates the penetration of the sperm into for pregnancy diagnosis urine tests. ovum. These enzymes dissolve the Ovarian and placental estrogen and membrane enveloping the ovum and help progesterone, human placental lactogen, the sperm head to enter the ovum. and pituitary prolactin stimulate 15. (c) Both are characterized by a yellow colour. development of the ducts and alveoli in The corpus luteum is a temporary the mammary glands. endocrine structure in mammals, involved 23. (c) in production of progesterone, which is 24. (c) Ejaculation of human male contains about needed to maintain pregnancy. The macula 200 – 300 million sperms, of which for or macula lutea is an oval yellow spot near normal fertility 60 % sperms must have the center of the retina of the human eye. normal shape and size and at least 40 % 16. (d) Four viable sperm cells are produced from must show energetic motility. one primary sex cell, whereby only one 25. (c) A–Umbilical cord with its vessels, B– viable egg cell is produced, due to the Placental villi, C–Yolk sac, D–Plug of unequal division of cytoplasm and the mucus in cervix formation of polar bodies, which wither and 26. (c) Oogonia are called as gamete mother cell. die. Corpus luteum is formed as a temporary 17. (b) Corpus luteum is a yellow coloured mass endocrine structure after the ovulation. It of cells found in ovary. Corpus luteum is involved in the production of relatively secretes progesterone hormone, which is high levels of progesterone and moderate essential for maintaining pregnancy and levels of estradiol and inhibin A to maintain therefore also called as “pregnancy pregnancy. A large number of primary hormone.” follicles degenerate during the phase from 18. (b) Leydig cells are the characteristic of birth to puberty. mammalian testis. They produce hormone, testosterone meant for developement of Type B : Assertion Reason Questions secondary sexual characters in males. 27. (c) Fertilization is the fusion of male and female 19. (c) Zygote divides partially in meroblastic gametes to form zygote. During fertilization cleavage. only head of the sperm enters egg. After 20. (a) In human beings, after one month of that polyspermy is avoided by fertilization pregnancy, the embryo's heart is formed. membrane. By the end of second month of pregnancy 28. (a) Morula involves cleavage of cells till 32 limbs and digits are developed. By the end cell stage is formed. It is still surrounded of 12 weeks (first trimester) most of the by Zona pellucida. major organ systems are formed. 29. (a) Death is the ultimate goal of every 21. (c) organism. This is caused by the wear and 22. (b) The high levels of estrogen and tear of organs which constitute the body progesterone in the maternal circulation of a living being. IIT JEE NEET eBooks: www.crackjee.xyz

Human Reproduction B-135

30. (c) Old age is the progressive deterioration in 35. (c) Head of a sperm has acrosome but the the structure and functioning of cells, spiral row of mitochondria are present in tissues and organs and cessation of the mid (connecting) piece of the sperm. immune system. 36. (c) Males have more stature than females 31. (a) According to programmed senescence because of the action of male sex hormone- theory of ageing, ageing is a result of testosterone which is secreted by testis in switching on and off of certain genes. B males. Body starts secreting testosterone and T- lymphocytes undergo programmed from the age of puberty. Its secretion is cell death called apoptosis. under the influence of Luteinizing Hormone 32. (b) In female, Graafian follicle forms corpus (LH) of the anterior lobe of pituitary gland. luteum after ovulation. The cells of corpus Testosterone controls the development of luteum are called luteal cells. The cytoplasm secondary sexual characters in males like of luteal cells have yellow granules called hoarseness of voice, development of facial lutein which secrete the hormone hairs, bone growth, calcium retention, progesterone to maintain pregnancy if closing of epiphysial cartilage. The total fertilization takes place. In the absence of quantity of bone matrix increases. The fertilization, corpus luteum degenerates and pelvic outlet is narrowed and lengthened. forms corpus albicans and there is decrease The strength of the pelvic bones increases in progesterone level as well. to carry more loads. That is why, males have 33. (c) Clitoris is a female reproductive organ. It is more stature than females after puberty homologous to penis of males. It is not when this hormone is present in the blood. remnant of penis. It is devoid of erectile hCG (Human Chorionic Gonadotropin) is tissue and high blood supply as in penis, the hormone secreted by human placenta penis is the copulatory organ of males. during pregnancy. hCG enlarges the corpus 34. (d) Hyaluronidase, a hydrolytic enzyme is an luteum in the mother's ovary and stimulates acrosomal content in mammalian sperm. It it to secrete progesterone. helps at the time of fertilization during the 37. (d) Testicular lobules are the compartments penetration of the sperm into the ovum. Based on the amount of yolk mammalian present in the testes, are not involved in eggs are alecithal means egg without yolk. the process of fertilization as whole. Microlecithal eggs contain very little yolk Fusion of male and female gametes is e.g., sea urchin, starfish. On the basis of called fertilization. distribution of yolk telolecithal eggs are 38. (c) Leydig cells, also known as interstitial cells, those eggs in which the yolk is are found adjacent to the seminiferous concentrated towards the vegetal pole and tubules in the testicle. They produce cytoplasm and nucleus lie near the animal testosterone in the presence of luteinizing pole, e.g., birds and reptiles. hormone (LH). IIT JEE NEET eBooks: www.crackjee.xyz EBD_7100

B-136 Topicwise AIIMS Solved Papers – BIOLOGY 26 Reproductive Health

5. Match Column -I with Column - II. [2015] TYPE A : MULTIPLE CHOICE QUESTIONS Column I Column II 1. GIFT is [2009] Method Mode of Action (a) transfer of a sperm in fallopian tube of a A. The pill I. Prevents sperms female with the help of injections. reaching cervix (b) transfer of a zygote fertilized in vitro in the B. Condom II. Prevents implantation fallopian tube of female incapable to C. Vasectomy III. Prevents ovulation conceive. D. Copper T IV. Semen contains no (c) transfer of an ovum collected from a donor sperms into another females fallopian tube who (a) A – III; B – I; C – IV; D – II can’t produce an ovum but can provide a (b) A – IV; B – I; C – II; D – III good environment for further development. (c) A – III; B – IV; C – I; D – II (d) embryo is developed in vitro and then (d) A – II; B – III; C – I; D – IV 6. Select the correct match of the techniques given transferred into female’s tract. in column I with its feature given in column II. 2. What is the function of copper-T ? Column I Column II (a) Checks mutation [2012] A. ICSI I Artificially introduction of (b) Stops fertilization semen into the vagina or uterus. (c) Stops zygote formation B. IUI II Transfer of ovum collected (d) Stops oblituation of blastocoel from a donor into the fallopian 3. Progestasert and LNG-20 are [2013] tube where fertilization occur (a) Implants C. IUT III Formation of embryo by (b) Copper releasing IUDs directly injecting sperm into the ovum (c) Non-medicated IUDs D. GIFT IV Transfer of the zygote or early embryo (with upto 8 (d) Hormone releasing IUDs blastomeres) into a fallopian 4. What is the figure given below showing in tube. particular ? [2014] E. ZIFT V Transfer of embryo with more than 8 blastomeres into the uterus [2016, 2017] (a) A – V; B – IV; C – I; D – III; E – IV (b) A – I; B – II; C – III; D – IV; E – V (a) Ovarian cancer (b) Uterine cancer (c) A – III; B – V; C – II; D – IV; E – I (c) Tubectomy (d) Vasectomy (d) A – III; B – I; C – V; D – II; E – IV IIT JEE NEET eBooks: www.crackjee.xyz

Reproductive Health B-137

TYPE B : ASSERTION REASON QUESTIONS 7. Assertion : Cu-T and Cu-7 do not suppresses sperm-motility. Directions for (Qs. 7-9) : Each of these questions Reason : Hormones released by them affect contains an Assertion followed by Reason. Read them sperm motility. [2009] carefully and answer the question on the basis of 8. Assertion : HIV infection can be avoided by following options. You have to select the one that use of condoms. best describes the two statements. Reason : Condoms secrete anti-viral interferons. (a) If both Assertion and Reason are correct and [2014] Reason is the correct explanation of Assertion. (b) If both Assertion and Reason are correct, but 9. Assertion : Copper-T is an effective Reason is not the correct explanation of contraceptive device in human females. Assertion. Reason : Copper-T prevents passage of sperms (c) If Assertion is correct but Reason is incorrect. from vagina upwards into fallopian tubes. (d) If both the Assertion and Reason are incorrect. [2011, 2014] IIT JEE NEET eBooks: www.crackjee.xyz EBD_7100

B-138 Topicwise AIIMS Solved Papers – BIOLOGY

Type A : Multiple Choice Questions IUI (intrauterine insemination) - Artificial introduction of semen into the vagina or 1. (c) GIFT is an ovum donation programme and uterus its purpose is to provide ovum to the women who suffer from infertility due to IUT (Intra uterine transfer) - Transfer of primary and premature ovarian failure embryo with more than 8 blastomeres into which is incurable. A woman in the uterus reproductive age denotes ova to a woman GIFT (Gamete intra fallopian transfer) - which cannot produce ova but she could Transfer of ovum collected from a donor provide good environment for embryo into the fallopian tube where fertilization development. The ovum from donor mother occurs is transferred to the recepient or would be mother’s fallopian tube where it is fertilized ZIFT (Zygote intra fallopian transfer) - by sperm and develops into embryo. Transfer of the zygote or early embryo 2. (b) Copper-T is copper releasing intra uterine (with upto 8 blastomeres) into a fallopian devices (IUD). It increases phagocytosis tube. of sperms within the uterus and suppress sperm motility and fertilising capacity of Type B : Assertion Reason Questions sperms. 3. (d) 7. (c) Cu-7 and Cu-T are intrauterine contraceptive devices for females. They do 4. (c) The figure shows the tubectomy. This is a not suppress sperm motility. Their mode of surgical method to prevent pregnancy in action is different. Cu-T and Cu-7 discharge women. In tubectomy, small part of the 50-75 micrograms of ionic copper into the fallopian tube is removed or tied through a uterus daily. These copper ions interfere small cut in the abdomen or through with life-sustaining functions that regulate vagina. It is very effective method but implantation in the uterus. No any hormone reversibility is very poor. is released by them. 5. (a) A. The pill — Prevents ovulation 8. (c) The use of condoms has been shown to B. Condom — Prevents sperm decrease the transmission of AIDS because reaching cervix condoms is contraceptic. C.Vasectomy — Semen contains no sperms 9. (c) Intra-uterine device (IUD) Copper-T is D. Copper-T — Prevent implantation. plastic or metal object placed in the uterus 6. (d) ICSI (Intracytoplasmic sperm injection) - by a doctor. Copper-T prevent the Formation of embryo by directly injecting fertilization of the egg or implantation of sperm into the ovum the embryo. IIT JEE NEET eBooks: www.crackjee.xyz

Principles of Inheritance and Variation B-139 Principles of Inheritance and 27 Variation

TYPE A : MULTIPLE CHOICE QUESTIONS (a) essential features (b) acquired characters 1. The formation of multivalents at meiosis in (c) non-essential changes diploid organism is due to [1998] (a) monosomy (d) mutations (b) deletion 8. Mirabilis jalapa shows [2001] (c) inversion (a) codominance (d) reciprocal translocation (b) incomplete dominance 2. If a homozygous tall plant is crossed with (c) dominance homozygous dwarf plant, the offsprings will be (d) complementary genes [1999] 9. Frame shift mutation occurs when [2002] (a) all tall plants (b) all dwarf plants (a) base is added (c) half tall plants (d) half dwarf plants (b) base is deleted 3. XO chromosomal abnormality in humans causes (c) base is added or deleted [1999] (d) none of the above 10. Pure line breed refers to [2002] (a) Turner’s syndrome (a) homozygosity (b) heterozygosity (b) Down’s syndrome (c) linkage (d) both b & c (c) Patau’s syndrome 11. If a homozygous red flowered plant is crossed (d) Klinefelter’s syndrome with a homozygous white flowered plant, the 4. Polygenic genes show [2000] offsprings would be [2002] (a) similar genotype (a) all red flowered (b) different phenotype (b) half red flowered (c) different karyotype (c) half white flowered (d) different genotype (d) all white flowered 5. Which disease has XXY chromosome constitution? 12. Genes of which one of the following is present (a) Down’s syndrome [2000] exclusively on the X-chromosome in humans? (b) Turner’s syndrome [2003] (c) Klinefelter’s syndrome (a) Baldness (d) Okazaki syndrome (b) Red-green colour blindness 6. Barr-body in mammals represents [2001] (c) Facial hair/moustaches in males (a) One of the two X chromosomes in somatic (d) Night blindness cells of females. 13. Given below is a pedigree chart of a family with (b) All heterochromatin of male & female cells. five children. It shows the inheritance of attached earlobes as opposed to the free ones. The squares (c) Y chromosomes of male. represent the male individuals and circles the (d) All heterochromatin of female cells female individuals. Which one of the following 7. Discontinuous variations are [2001] conclusions drawn is correct? [2004] IIT JEE NEET eBooks: www.crackjee.xyz EBD_7100

B-140 Topicwise AIIMS Solved Papers – BIOLOGY 17. Primary source of allelic variation is [2005] (a) independent assortment (b) recombination (c) mutation (d) polyploidy 18. Given below is a pedigree chart showing the inheritance of a certain sex-linked trait in humans.

Generation 1 Free Attached

Ear-lobes Ear-lobes (a) The parents are homozygous recessive. Generation 2 (b) The trait is Y-linked. (c) The parents are homozygous dominant. (d) The parents are heterozygous. Generation 3 14. Given below is a representation of a kind of Key: chromosomal mutation. What is the kind of mutation represented? Unaffected femalemale Unaffected mfemaleale Affected male Affected male A B C D E F G H The trait traced in the above pedigree chart is [2005] (a) dominant X-linked A E F G H (b) recessive X-linked (c) dominant Y-linked D C B (d) recessive Y-linked 19. The "Cri-du-Chat" syndrome is caused by [2004] change in chromosome structure involving (a) deletion [2005] (b) duplication (a) deletion (b) duplication (c) inversion (c) inversion (d) translocation (d) reciprocal translocation 20. Given below is a highly simplified representation 15. How many different types of gametes can be of the human sex chromosomes from a formed by F1 progeny, resulting from the karyotype. [2006] following cross Tt × Rr? [2004] (a)4 (b) 8 (c) 27 (d) 64 Genea 16. Grain colour in wheat is determined by three pairs Geneb of polygene. Following the cross AABBCC (dark colour) × aabbcc (light colour), in F2 generation what proportion of the progeny is likely to resemble either parent? [2005] (a) Half (b) Less than 5 percent (c) One third (d) None of these IIT JEE NEET eBooks: www.crackjee.xyz

Principles of Inheritance and Variation B-141 The genes a and b could be of 27. Bird females have chromosome arrangement as (a) colour blindness and body height [2009] (b) attached ear lobe and Rhesus blood group (a ) XY (b) XO (c) haemophilia and red-green colour (c) WZ (d) WW blindness 28. Gene pool is referred to [2009] (d) phenylketonuria and haemophilia (a) the genetic drift caused in a population 21. In India, we find mangoes with different flavours, (b) aggregate of all genes and their alleles in a colours, fibre content, sugar content and even population. shelf-life. The large variation is on account of (c) deletion of non essential genes. [2006] (d) induce cell division (a) species diversity 29. Mother and father both have blood group ‘A’. (b) induced mutations They have two children one with blood group (c) genetic diversity ‘O’ and second one with blood group ‘A’. (d) hybridization They have [2009] 22. Which one of the following pairs of features is a (a) mother has homozygotic gene father has good example of polygenic inheritance? [2006] heterozygote IcA IA. (a) Human height and skin colour. (b) both are homozygotic (IAIA). (b) ABO blood group in humans and flower (c) mother is heterozygotic (IAi) and father is colour of Mirabilis jalapa. homozygotic (IAIA). (c) Hair pigment of mouse and tonque rolling (d) both are heterozygotic (IAi). in humans. 30. When one gene controls two or more different (d) Human eye colour and sickle cell anaemia. characters simultaneously, the phenomenon is 23. Gene which suppresses other gene’s activity called [2010] but does not lie on the same locus is called as (a) apomixis (b) pleiotropy [2007] (c) polyploidy (d) polyteny (a) epistatic (b) supplementary 31. Three children in a family have blood types O, (c) hypostatic (d) codominant AB and B respectively. What are the genotypes of their parents? [2013] 24. XO-chromosomal abnormality in human beings (a) IA i and IBi (b) IAIB and i i causes [2007] (c)IBIB and IAIA (d) IAIA and IBi (a) Turner’s syndrome 32. If both parents are carriers for thalassemia, which (b) Down’s syndrome is an autosomal recessive disorder, what are the (c) Klinefelter’s syndrome chances of pregnancy resulting in an affected (d) none of the above child? [2014] 25. A normal woman whose father was colour (a) 50% (b) 25% blind, is married to a normal man. The sons (c) 100% (d) no chance would be [2008] 33. In Huntington’s disease, the unaffected persons (a) 75% colour blind are homozygous for normal allele h. The (b) 50% colour blind following is erroneous because [2015] (c) all normal (d) all colour blind 26. Mating of an organism to a double recessive in 1 2 order to determine whether it is homozygous or heterozygous for a character under consideration is called [2008] 3 4 5 6 7 (a) reciprocal cross (b) test cross (c) dihybrid cross (d) back cross 8 9 10 11 IIT JEE NEET eBooks: www.crackjee.xyz EBD_7100

B-142 Topicwise AIIMS Solved Papers – BIOLOGY

(a) it shows both male and female affected by (a) 98.7% and 62.8% Huntingtons disease (b) 1.3% and 37.2% (b) either person 6 or 7 should have the (c) 37.2 and 1.3% disease, if individual 11 shows the disease. (d) 62.8% and 98.7% (c) at least one of the 2 children (8, 9) should have the disease TYPE B : ASSERTION REASON QUESTIONS (d) all of these Directions for (Qs. 35-38) : These questions consist 34. The experiment shown in the given figure has of two statements, each printed as Assertion and been carried out by Morgan to show the Reason. While answering these questions, you are phenomenon of linkage and recombination. If in required to choose any one of the following five cross I, genes are tightly linked and in cross II, responses. genes are loosely linked then what will be the (a) If both Assertion and Reason are correct and percentage of recombinants produced in cross I the Reason is a correct explanation of the and cross II respectively? Assertion. (b) If both Assertion and Reason are correct but Reason is not a correct explanation of the Cross I Assertion. (c) If the Assertion is correct but Reason is Parents incorrect.

Yellow White Wild type (d) If both the Assertion and Reason are incorrect. body eyes (e) If the Assertion is incorrect but the Reason is correct. 35. Assertion : The genetic complement of an F generation 1 organism is called genotype. Reason : Genotype is the type of hereditary Wild type Yellow White properties of an organism. [1999] body eyes 36. Assertion : Phenylketonuria is a recessive hereditary disease caused by body’s failure to oxidize an amino acid phenylalanine to tyrosine, because of a defective enzyme. Cross II Reason : It results in the presence of phenylalanine acid in urine. [2000] 37. Assertion : In humans, the gamete contributed White Miniature Wild type by the male determines whether the child body wings produced will be male or female. Reason : Sex in humans is a polygenic trait depending upon a cumulative effect of some genes on X-chromosome and some on Y-chromosome. [2005] Wild type White Miniature 38. Assertion : Haemophilia is a recessive sex linked body wings disease. Reason : Haemophilia occurs due to mutation [2016] of a structural gene on chromosome 15. [2007] IIT JEE NEET eBooks: www.crackjee.xyz

Principles of Inheritance and Variation B-143

Directions for (Qs.39-44) : Each of these questions 41. Assertion: Aneuploidy may be of hypoploidy contains an Assertion followed by Reason. Read them or hyperploidy type. carefully and answer the question on the basis of Reason: Monosomy lacks one pair of chromo- following options. You have to select the one that somes. [2011] best describes the two statements. 42. Cross of F individual with recessive (a) If both Assertion and Reason are correct and Assertion : 1 homozygous parent is test cross. Reason is the correct explanation of Assertion. (b) If both Assertion and Reason are correct, but Reason : No recessive individual are obtained Reason is not the correct explanation of in the monohybrid test cross. [2012]

Assertion. 43. Assertion : In Mirabilis, selfing of F1 pink (c) If Assertion is correct but Reason is incorrect. flower plants produces same phenotypic & (d) If both the Assertion and Reason are incorrect. genotypic ratio. 39. Assertion : Persons suffering from haemophilia Reason : Flower colour gene shows incomplete fail to produce blood clotting factor VIII. dominance. [2014] Reason : Prothrombin producing platelets in 44. Assertion : In humans, the gamete contributed such persons are found in very low by the male determines whether the child concentration. [2008] produced will be male or female. 40. Assertion : In case of incomplete linkage, linked Reason : Sex in humans is a polygenic trait gene show new combination along with parental depending upon a cumulative effect of some combination. genes on X-chromosome and some on Reason : In case of incomplete linkage, linked Y-chromosome. [2015, 2017] genes are separated by crossing over. [2010] IIT JEE NEET eBooks: www.crackjee.xyz EBD_7100

B-144 Topicwise AIIMS Solved Papers – BIOLOGY

Type A : Multiple Choice Questions 10. (a) The self pollinated progeny of a homozygous plant constitutes a pure line. 1. (d) Translocation is the separation of a chromosome segment and its union to a 11. (a) As per Mendel’s law in F1 generation only non homologous chromosome. In dominant phenotypes appear. reciprocal translocation 2 non-homologous chromosomes exchange segments among Parents R R X r r (White) themselves. Reciprocal translocation (Red) produces duplication. Gametes R R r r 2. (a) Homozygous Tall × Homozygous dwarf Rr (Heterozygous Red) TT tt F1 generation Gamets T T t t 12. (b) Red and green colourblindness is a sex Tt All Tall linked inheritance. Its genes are present on X-chromosome. 3. (a) XO chromosomal abnormality in humans 13. (d) Such types of results are obtained only if is due to monosomy of sex chromosome. parents are heterozygous for the trait. Their total number of chromosomes are 45. This condition is found in Turner’s 14. (c) The kind of mutation is paracentric syndrome. inversion. In this a segment of a 4. (b) Polygenic genes show intermediate chromosome separates and rejoins in an phenotypes, e.g. skin colour. inverted position. 5. (c) Down’s syndrome is due to trisomy of 21st 15. (a) F1 generation is always heterozygous, e.g. chromosome; Turner’s syndrome having TtRr, so there are 4 types of gamete XO genotype is caused by the absence of formation i.e. TR, Tr, tR, tr. X chromosome in females; klinefelter’s syndrome (XXY) is due to trisomy of sex 16. (b) AABBCC aabbcc chromosome. 6. (a) Barr body is the inactive one X- abc Gametes chromosome in somatic cells of female ABC (Dosage compensation). The number of Barr bodies are always one less than the AaBbCc F 2 generation total number of X-chromosome. 7. (d) Discontinuous variations lead to The F2 generation will show the intermediate mutations. colour because of quantitative inheritence. In 8. (b) Mirabilis jalapa (4O’ clock plant) shows case of crossing between AA BB CC (dark incomplete dominance because the genes colour) and aa bb cc (light colour), in F2 for red and white colour do not mix in the generation seven phenotypes will be obtained F1 pink hybrids as both the pure characters with ratio 1 : 6 : 15 : 20 : 15 : 6 : 1. The total number reappear in the F2 plants. of progeny is 64, out of which only two will be 9. (c) A mutation in which there is deletion or likely resemble with either parents. Hence, their insertion of one or few nucleotides is called percentage in F2 generation would be 3.12 i.e frameshift mutation. less than 5%. IIT JEE NEET eBooks: www.crackjee.xyz

Principles of Inheritance and Variation B-145

generation : 17. (c) Primary source of allelic variation is F2 mutation. Gametes CA Ca cA ca 18. (b) The genes for such traits are recessive and CCAA CCAa CcAA CcAa located on the X-chromosome. The CA Agouti Agouti Agouti Agouti character appears more often in males in CCAa CCaa CcAa Ccaa hemizygous condition, but also in females Ca Agouti Coloured Agouti Coloured with homozygous condition. Affected CcAA CcAa ccAA ccAa cA males receive their defective gene from Agouti Agouti Albino Albino CcAa CcAa ccAa ccaa carrier mothers who may have affected ca Agouti Coloured Albino Albino father. These exhibit criss- cross inheritance. Agouti - 9 19. (a) Cri-du-chat/cat cry syndrome is due to the Coloured - 3 deletion of large part of the small or one of Albino - 4 the 5th chromosome. Ratio - 9 : 3 : 4 20. (c) Genes a and b lie very close to each other. 24. (a) Turner's syndrome is caused due to the So, they are representing linked genes. e.g. missing X chromosome. The genotype is genes of haemophilia and colour blindness therefore XO instead of the normal XX and show sex linked inheritance. the sufferer person possesses 45 21. (c) Genetic diversity describes an attribute chromosomes instead of 46. Patients with this which is commonly held to be condition can best be described as advantageous for survival that there are incompletely developed females, although there are often no obvious external many different versions of otherwise similar differences compared with normal females. organisms, e.g. different varieties of Affected females lack ovaries so they are mangoes. infertile. 22. (a) Polygenic inheritance is the trait under the control of more than one pair of genes, e.g. 25. (b) skin colour (trigenic) and human height. Genotype of normal woman with Genotype of 23. (a) When one gene masks the effect or activity colourblind father normal man h X Y of another gene which does not lie on the X X same locus, it is called epistasis. Epistasis refers to non- allelic interactions. Like coat h X X XY colour in mice is controlled by epistatic h Colourblind X Y XX Normal son daughter gene. When coloured (CCaa) mouse is Colourblind Normal crossed with albino (ccAA), agouti mice son daughter

(ccAa) appeared in F1 generation. Agouti, ? 50% son would be colourblind. coloured and albino mice are obtained in 26. (b) In genetics, a test cross, first introduced 9 : 3 : 4 ratio in F2 generation. by Mendel, is used to determine if an individual exhibiting a dominant trait is Parent : CCaa ccAA homozygous or heterozygous for that trait. Test crosses involve breeding the Coloured× Albino individual in question with another Gametes : Ca cA individual that expresses a recessive version of the same trait. If all offspring

F1 generation : CcAa display the dominant phenotype, the Agouti individual in question is homozygous IIT JEE NEET eBooks: www.crackjee.xyz EBD_7100

B-146 Topicwise AIIMS Solved Papers – BIOLOGY dominant; if the offspring display both that superficially resembles normal sexual dominant and recessive phenotypes, then reproduction but there is no fusion of the individual is heterozygous. In some gametes. sources, the “test cross” is defined as 31. (a) being a type of back cross between the 32. (b) Genotype of carrier parents is – Aa (male recessive homozygote and F1 generation. 27. (c) In birds sex is determined by parent) × Aa (female parent) morphologically dissimilar pair of chromosomes called sex chromosomes. Z A a and W are two sex chromosomes of birds. A AA Aa A male bird has ZZ (homogenetic sex) a Aa *aa arrangement while a female bird has ZW (heterogenetic sex) arrangement of AA= normal child (25%) chromosomes. Number of chromosomes in Aa = carriers child (50%) birds is 69. 28. (b) Gene pool is the sum total of genes and their aa = affected child (25%) alleles in the reproductive gametes of a 33. (b) Since the offspring number 11 is affected population. The gene pool is transferred (diseased) either of the two parents 6 or 7 from one generation to the other using have to be affected. gametes from genetic pool. These gametes 34. (b) The percentage of recombinants produced will form zygotes of next generation. Gene in cross I and cross II are respectively 1.3% pool of a population will consist of a large and 37.2%. number of genes which vary in their frequencies. Type B : Assertion Reason Questions 29. (d) In the mentioned case, one child with blood 35. (a) Genotype of the organism include all group ‘O’ and second with blood group dominant and recessive characters. ‘A’ are born to parents with heterozyous 36. (b) Phenylketonuria is an recessive autosomal condition of genes for blood group A . i.e., gene disorder. It occurs due to the absence of enzyme phenylalanine hydroxylase IAI0. Both mother and father have blood which changes phenylalanine to tyrosine. group A but their genotypes indicate 37. (c) In humans, sex of a child depends upon heterozygotic condition. the gametes produced by the male (X, Y). 38. (c) Haemophilia also known as bleeder disease A 0 A 0 II II is an example of recessive sex linked inheritance in human beings. It is masked Mother Father in heterozygous condition. The person suffering from this disease lack factors VIII A A A 0 A 0 0 0 and IX responsible for blood clotting. A Children o II II II II small cut may lead to bleeding till death. Blood group o AAAO Men are affected by this disease while 30. (b) Pleiotropy is the condition in which a single women are the carriers. gene influences more than one trait. Mutation of a structural gene on Polyploidy is a condition in which individuals chromosome number 15 causes Marfan have more than two complete sets of syndrome. This disease results in formation chromosomes. of abnormal form of connective tissues and Apomixis is a reproductive process in plants characteristic extreme loosseness of joints. IIT JEE NEET eBooks: www.crackjee.xyz

Principles of Inheritance and Variation B-147 39. (c) Haemophilia bleeding disorder is a group due to addition of one or more of hereditary genetic disorders that impair chromosomes to complete chromosome the body’s ability to control blood clotting complement (hyperploidy). Hypopliody is or coagulation. In its most common form, mainly due to loss of a single chromosomes, Hemophilia A, clotting factor VIII is absent. monosomes (2n – 1) or due to loss of one In Haemophilia B, factor IX is deficient. pair of chromosomes, nullisomes (2n – 2). Factor VIII participates in blood 42. (c) In the monohybrid test-cross both coagulation; it is a cofactor for factor IXa dominant and recessive traits are obtained +2 which, in the presence of Ca and in 1 : 1 ratio. phospholipids forms a complex that 43. (a) F phenotypic and genotypic ratio in converts factor X to the activated form Xa. 2 monohybrid cross involving incomplete Defects in this gene results in hemophilia dominance is A, a common recessive X-linked coagulation disorder. Prothrombin 1 : 2 : 1 producing platelets in such persons are not RR Rr rr found in very low concentration. (red) (pink) (white) 40. (a) In case of incomplete linkage, the linked 44. (c) In human, the gamete contributed by the gene shows new combination along with male determines whether the child pro- parental combination due to crossing over duced will be male or female. Sex in between chromatids. humans is a polygenic trait depending 41. (c) Aneuploidy can be either due to loss of upon cumulative effect of some genes one or more chromosomes (hypoploidy) or present on Y-chromosome. Only sex in human is amonogenic trait. IIT JEE NEET eBooks: www.crackjee.xyz EBD_7100

B-148 Topicwise AIIMS Solved Papers – BIOLOGY 28 Molecular Basis of Inheritance

TYPE A : MULTIPLE CHOICE QUESTIONS 8. Okazaki fragments form [2000] (a) leading strand (b) lagging strand 1. The process through which the amount of DNA, (c) non-sense strand (d) senseful strand RNA and protein can be known at a time is called 9. Wobble hypothesis was given by [2002] [1997] (a) F.H.C. Crick (b) Nirenberg (a) autoradiography (c) Holley (d) Khorana (b) tissue culture 10. Which one of the following pairs of terms/names mean one and the same thing? [2003] (c) cellular fractioning (a) Gene pool-genome (d) phase contrast microscopy (b) Codon-gene 2. Balbiani rings are found in [1997] (c) Cistron-triplet (a) polysomes (d) DNA fingerprinting - DNA profiling (b) polytene chromosomes 11. What is true about t-RNA? [2003] (c) autosomes (a) It binds with an amino acid at it 3' end. (d) nonsense chromosomes (b) It has five double stranded regions. 3. In DNA helix, cytosine is paired with guanine (c) It had a codon at one end which recognizes by [1997] the anticodon on messenger RNA. (a) covalent bond (d) It looks like clover leaf in the three (b) phosphate bond dimensional structure. (c) three hydrogen bonds 12. Which one of the following codons codes for the same information as UGC? [2003] (d) two hydrogen bonds (a) UGU (b) UGA 4. Which RNAs pick up specific amino acid from (c) UAG (d) UGG amino acid pool in the cytoplasm to ribosome 13. During protein synthesis in an organism, at one during protein synthesis ? [1998] point the process comes to a halt. Select the (a) tRNA (b) mRNA group of the three codons from the following, (c) rRNA (d) hnRNA from which anyone of the three could bring about 5. The structure of DNA is [1998] this halt. [2006] (a) linear (b) double helix (a) UUU, UCC, UAU (b) UUC, UUA, UAC (c) single helix (d) triple helix (c) UAG, UGA, UAA (d) UUG, UCA, UCG 6. Transposon was discovered by [1998] 14. The total number of nitrogenous bases in human (a) Sutton (b) Strassburger genome is estimated to be about [2004, 2008] (c) Fischer (d) B.Mc Clintock (a) 3.5 million (b) 35 thousand (c) 35 million (d) 3.1 billion 7. Root cell of wheat has 42 chromosomes. What 15. Which one of the following pairs is correctly would be the number of chromosomes in the matched with regard to the codon and the amino synergid cell ? [1999] acid coded by it ? [2004, 2008] (a)7 (b) 14 (a) UUA-valine (b) AM-lysine (c) 21 (d) 28 (c) AUG-cysteine (d) CCC-alanine IIT JEE NEET eBooks: www.crackjee.xyz

Molecular Basis of Inheritance B-149 16. 5c 3' 5'

3c 5 3c (d) ' 5c 3' 3' 5c 5 3c ' 19. Select the correct option: [2014] What is the error in above diagram? [2009] Direction of RNA Direction of reading of (a) Arrows are wrongly depicted. synthesis the template DNA strand (b) Polarity is incorrect. (a) 5´—3´ 3´—5´ (c) Both arrows and polarity are incorrect. (b) 3´—5´ 5´—3´ (d) None of the above. (c) 5´—3´ 5´—3´ 17. TATA box of eukaryotic promotor lies [2010] (d) 3´—5´ 3´—5´ (a) about 25 bp upstream of the transcription 20. Which one of the following represents a start site. palindromic sequence in DNA? [2014] (b) about 50 bp upstream of the transcription (a) 5' - GAATTC - 3' start site. 3' - CTTAAG - 5' (c) about 75 bp upstream of the transcription (b) 5' - CCAATG - 3' start site. 3' - GAATCC - 5' (d) about 200 bp upstream of the transcription (c) 5' - CATTAG - 3' start site. 3' - GATAAC - 5' 18. Which one of the following correctly represents (d) 5' - GATACC - 3' the manner of replication of DNA ? [2003, 2012] 3' - CCTAAG - 5' 21. Thirty percent of the bases in a sample of DNA 5' extracted from eukaryotic cells is adenine. What 3' percentage of cytosine is present in this DNA? (a) 3' [2015] 5' (a) 10% (b) 20% 5' (c) 30% (d) 40% 3' 22. There are three genes a, b, c. Percentage of crossing over between a and b is 20%, b and c is 3' 28% and a and c is 8%. What is the sequence of 5' genes on chromosome? [2015] (a) b, a, c (b) a, b, c (b) 5' 3' (c) a, c, b (d) None of these 23. Which one of the following group of codons is 3' 5' called as degenerate codons? [2016] (a) UAA, UAG and UGA

3' (b) GUA, GUG, GCA, GCG and GAA 5 ' (c) UUC, UUG, CCU, CAA and CUG (d) UUA, UUG, CUU, CUC, CUA and CUG (c) 5' 3' 24. The given figure shows the structure of nucleosome with their parts labelled as A, B & 5' 3' C. Identify A, B and C. [2017] IIT JEE NEET eBooks: www.crackjee.xyz EBD_7100

B-150 Topicwise AIIMS Solved Papers – BIOLOGY

(c) If the Assertion is correct but Reason is A B incorrect. (d) If both the Assertion and Reason are incorrect.

C (e) If the Assertion is incorrect but the Reason is correct. 26. Assertion : Histones are basic proteins of major

Core of histone molecules importance in packaging of eukaryotic DNA. DNA and histones comprise chromatin forming (a) A – DNA; B – H1 histone; the bulk of eukaryotic chromosome. C – Histone octamer Reason : Histones are of five major types (b) A – H1 histone; B – DNA; C – Histone octamer andH,BHAH,H H43221 . [2000] (c) A – Histone octamer; B – RNA; 27. Assertion: mRNA attaches to ribosome through C – H1 histone its 3' end. (d) A – RNA; B – H1 histone; Reason: The mRNA has F-capsular nucleotide C – Histone octamer and bases of lagging sequence. [2002] 25. Match the codons given incolumn I with their 28. Assertion : Replication and transcription occur respective amino acids given in column II and in the nucleus but translation in the cytoplasm. choose the correct answer. [2017] Reason : m-RNA is transferred from the nucleus into the cytoplasm where ribosomes and amino Column -I Column -II acids are available for protein synthesis. [2005] (Codons) (Amino acids) 29. Assertion: An organism with lethal mutation A UUU I. Serine may not even develop beyond the zygote. B GGG II. Methionine [2006] C UCU III. Phenylalanine Reason: All types of gene mutations are lethal. D CCC IV. Glycine 30. Assertion: Polytene chromosomes have a high E AUG V. Proline amount of DNA. Reason: Polytene chromosomes are formed by (a) A – III; B – IV; C – I; D – V; E – II repeated replication. [2006] (b) A – III; B – I; C – IV; D – V; E – II Directions for (Qs.31-35) : Each of these questions (c) A – III; B – IV; C – V; D – I; E – II contains an Assertion followed by Reason. Read them (d) A – II; B – IV; C – I; D – V; E – III carefully and answer the question on the basis of TYPE B : ASSERTION REASON QUESTIONS following options. You have to select the one that best describes the two statements. Directions for (Qs. 26-30) : These questions consist (a) If both Assertion and Reason are correct and of two statements, each printed as Assertion and Reason is the correct explanation of Assertion. Reason. While answering these questions, you are (b) If both Assertion and Reason are correct, but required to choose any one of the following five Reason is not the correct explanation of responses. Assertion. (a) If both Assertion and Reason are correct and (c) If Assertion is correct but Reason is incorrect. the Reason is a correct explanation of the (d) If both the Assertion and Reason are incorrect. Assertion. 31. Assertion : DNA is associated with proteins. (b) If both Assertion and Reason are correct but Reason : DNA binds around histone proteins Reason is not a correct explanation of the that form a pool and the entire structure is called Assertion. a nucleosome. [2013] IIT JEE NEET eBooks: www.crackjee.xyz

Molecular Basis of Inheritance B-151 32. Assertion : The uptake of DNA during 34. Assertion : In a DNA molecule, A–T rich parts transformation is an active, energy requiring melt before G–C rich parts. process. Reason : In between A and T there are three Reason : Transformation occurs in only those H–bond, whereas in between G and C there are bacteria, which possess the enzymatic two H-bonds. [2010, 2015] machinery involved in the active uptake and 35. Assertion : Replication and transcription occur recombination [2014] in the nucleus but translation takes place in the 33. Assertion : UAA, UAG and UGA terminate cytoplasm. protein synthesis. Reason : mRNA is transferred from the nucleus into cytoplasm where ribosomes and amino acids Reason : They are not recognised by tRNA. are available for protein synthesis. [2011, 2014] [2008, 2015, 2017] IIT JEE NEET eBooks: www.crackjee.xyz EBD_7100

B-152 Topicwise AIIMS Solved Papers – BIOLOGY

Type A : Multiple Choice Questions of a triplet codon on mRNA have a precise 1. (a) Autoradiography is the use of X-ray films pairing with the bases of the tRNA to detect radioactive material. It produces anticodon. a permanant record of positions and relative 10. (d) Gene pool is the sum total of genes present intensities of radiolabeled bands in a gel or in the inbreeding population whereas slot. Typically, biomolecules are labeled genome is a complete set of genes in the with 32P or 35 S, and detected by over night haploid case of chromosomes. Codon is a film exposure. triplicate of nucleotide whereas gene is a Autoradiography technique may be used linear segment of DNA. Cistron is a segment to determine the tissue localization of a of the DNA molecule carrying information radioactive substance, either introduced for the production of one polypeptide into a metabolic pathway, bound to a chain. DNA fingerprinting is also known receptor or enzyme, or hybridized to a as DNA profiling or DNA typing. nucleic acid. 11. (d) t-RNA has a clover leaf like structure. 2. (b) Balbiani rings are temporary swellings in 12. (a) UGC and UGU codes for cysteine. polytene chromosome having uncoiled 13. (c) Protein termination occurs by nonsense active DNA that transcribes RNA. codons i.e. UAA, UGA ,UAG. 14. (d) Genome size is usually stated as the total 3. (c) In DNA helix, cytosine always pairs with number of base pairs; the human genome guanine by three hydrogen bonds. contains roughly 3.1 billion base pairs 4. (a) There are specific types of tRNA (because organized into 24 distinct, physically of anticodon) for specific amino acid. They separate microscopic units called transport the amino acids from the chromosomes. All genes are arranged cytoplasm to the site of protein synthesis. linearly along the chromosomes. The 5. (b) DNA is double helical in structure. Its both complete set of instructions for making an strands are complementary to each other. organism is called its genome. 6. (d) Mc Clintock discovered transposon or 15. (a) Valine is one of 20 proteogenic amino acids. (jumping genes) in maize plants in 1983. Its codons are GUU, GUC, GUA, and GUG. Cysteine codons are UGU and UGC. With a 7. (c) The root cell in wheat is a somatic cell and, thiol side chain, cysteine is classified as a hence, has 2n number of chromosomes hydrophilic amino acid. Alanine codons are which is a diploid condition. The synergid GCU, GCC, GCA, and GCG. It is classified as a cell that is formed after meiosis along with non-polar amino acid. L-alanine is second the ovum has 21 chromosomes which is only to leucine. Lysine codons are AAA and the haploid condition. AAG. Lysine is a base, as are arginine and 8. (b) Okazaki fragments are short pieces of newly histidine. synthesized DNA, which are generated 16. (b) The figure below is the replicating fork of during the normal process of DNA DNA. The DNA replication takes place in replication. They are linked up by the 5c to 3c direction always. On the leading enzyme DNA ligase after replacing RNA strand DNA replication is continuous while primers with deoxribonucleotides. This will on lagging strand DNA replication is form lagging strand. discontinuous. The polarity of lagging 9. (a) Wobble hypothesis was given by F.H.C. strand is incorrect in the given figure. The Crick. It states that the first two positions correct figure should be IIT JEE NEET eBooks: www.crackjee.xyz

Molecular Basis of Inheritance B-153

d 20% 8% ran b ac st 5c 22. (a) ing gg 28% La 23. (a) Degenerate codons (also called as non - 3c sense codons or terminator codons) do not 3c 5c code for any amino acids. Three types of degenerate codons are UAG (amber), UAA Leading (ochre) and UGA (opal). strand 5c 3c 24. (a) Nucleosome is a structural unit of a eukaryotic chromosome which consists of Both the strands are antiparallel. In one a length of DNA coiled around a core of strand carbon of sugar are in 3c – 5c histones and are thought to be present only direction and in other the carbon of sugar during interphase of the cell cycle. In the are in 5c – 3c direction. given figure of nucleosome structure, the 17. (a) TATA box of eukaryote promoter lies about parts marked as A, B and C are respectively 25 bp upstream of the transcription start site. DNA, H1 histones and histone octamer. 18. (d) The new strands of DNA are formed in the 25. (a) UUU – Phenylalanine GGG – Glycine 5'o 3' direction from the 3'o 5' template DNA by the addition of UCU – Serine deoxyribonucleotides to the 3' end of primer CCC – Proline RNA. AUG – Methionine 19. (a) Synthesis of RNA exhibits several features Type B : Assertion Reason Questions that are synonymous with DNA replication. RNA synthesis requires accurate and 26. (a) Histones are basic proteins found in the efficient initiation, elongation proceeds in eukaryotic chromosomes. These are rich the 5'–3' direction (i.e. the polymerase in the amino acids lysine and arginine. moves along the template strand of DNA Histonie proteins are basic proteins in the 3'–5' direction), and RNA synthesis consisting of 5 types – H1, H2A, H2B, H3, requires distinct and accurate termination. H4. DNA is coiled around it. It exists as Transcription exhibits several features that octamers linked with H1. are distinct from replication. 27. (d) mRNA is attached to the ribosome by 20. (a) A palindromic sequence is a nucleic acid means of protein ribophorin I & II. The sequence (DNA or RNA) that is the same sequence of nucleotides on mRNA is called whether read 5' (five-prime) to 3' (three codon. prime) on one strand or 5' to 3' on the 28. (a) DNA is the master copy which transcribes complementary strand with which it forms to form working copy in the form of mRNA a double helix. which translates in the form of peptide 5 - GAATTC - 3 chain in the cytoplasm. It is also called 3 - CTTAAG - 5 central dogma. It is a palindromic sequence of DNA cut by 29. (c) Organisms with lethal mutation bear lethal restriction enzyme EcoRI. genes that result in the death of the 21. (b) If 30 percent of DNA is adenine, then by individual which carries them. The Chargaff’s rule 30 percent will be thymine. completely lethal genes usually cause death The remaining 40 percent of the DNA is of the zygote. Mutation is a sudden heritable cytosine and guanine. Since the ratio of change in the sequence of gene occuring cytosine to guanine must be equal, then on the chromosomes. Mutation may be each accounts for 20 percent of the bases. beneficial, normal, sublethal or lethal. IIT JEE NEET eBooks: www.crackjee.xyz EBD_7100

B-154 Topicwise AIIMS Solved Papers – BIOLOGY 30. (a) Polytene chromosomes show endomitosis 33. (a) Synthesis of polypeptide terminates when and they have high DNA content. a nonsense codon of mRNA reaches the 31. (a) A chain of DNA has 140 base pairs, make A– site. There are three nonsense codons– 3 UAA, UAG and UGA. These codons are 1 turns and twist around a histone 4 not recognised by any of the tRNAs. octamer forming nucleosome. The core of Therefore, no more aminoacyl tRNA nucleosome consists of 4 histones H2A, reaches the A site. The P– site tRNA is H2B, H3 and H4. hydrolysed and the completed polypeptide 32. (a) Transformation does not involve passive is released in the presence of a release entry of DNA molecules through permeable factor. cell walls and membranes. It does not occur 34. (c) In a DNA molecule, A-T rich parts melt 'naturally' in all species of bacteria, only in before G-C rich parts because there are two those species possessing the enzymatic H-bond between A and T whereas in machinery involved in the active uptake between G and C, there are three H-bond. and recombination processes. Even in 35. (a) In eukaryotes, the replication and these species, all cells in a given population transcription takes place in the nucleus. are not capable of active uptake of DNA. mRNA comes out from the nucleus through Only competent cells, which possess a so the nuclear pore. In cytoplasm, translation called competence factor are capable of occurs. In prokaryote, there is no nuclear serving as recipients in transformation. membrane, so replication, transcription and translation all occur in the cytoplasm. IIT JEE NEET eBooks: www.crackjee.xyz

29 Evolution

TYPE A : MULTIPLE CHOICE QUESTIONS 9. Wings of pigeon, mosquito and bat show [1999, 2007, 2011] 1. The branch of science dealing with process of (a) divergent evolution improvement of human race by selective (b) atavism breeding is called [1997] (a) Eugenics (b) Euthenics (c) convergent evolution (c) Euphenics (d) Obstetrics (d) all of these 2. The connecting link between annelida and 10. Which of the following cannot be explained by mollusca is [1998] Lamarckism? [2012] (a) Neoplina (b) Nautilus (a) Absence of lips in snakes (c) Glochidium (d) Velliger larva (b) Long neck of giraffe 3. Theory of Pangenesis was given by [1998] (c) Degeneration of visual apparatus in cave (a) Lamarck (b) Oparin dwellers (c) Darwin (d) De Vries (d) Dull progeny of noble laureate 4. The cranial capacity was largest among the 11. Thorn of Bougainvillea and tendril of Cucurbita [2002] are example of [2013] (a) Peking man (b) African man (a) analogous organs (c) Java Ape man (d) Neanderthal man (b) homologous organs 5. A baby has been born with a small tail. It is the (c) vestigial organs case exhibiting [2004] (a) retrogressive evolution (d) retrogressive evolution (b) mutation 12. Forelimbs of cat, lizard used in walking; forelimbs (c) atavism of whale used in swimming and forelimbs of bats (d) metamorphosis used in flying are an example of [2014] 6. “Homo sapiens” implies [2007] (a) Analogous organs (a) human race (b) human beings (b) Adaptive radiation (c) modern man (d) none of these (c) Homologous organs 7. The study of homologous structures in mature (d) Convergent evolution organisms provides evidence for the 13. The diagram represents Miller's experiment. evolutionary relationships among certain groups Choose the correct combination of labelling. of organisms. Which field of study includes this [2015] evidence of evolution? [2009] (a) Comparative cytology (b) Biochemistry (c) Geology (d) Comparative anatomy 8. Darwin’s finches were a good example of [2009] (a) convergent evolution (b) adaptive radiation (c) mutation (d) none of the above IIT JEE NEET eBooks: www.crackjee.xyz EBD_7100

B-156 Topicwise AIIMS Solved Papers – BIOLOGY

(a) A–electrodes, B– NH3 + H2 + H2O + CH4, (b) If both Assertion and Reason are correct but C–cold water, D–vacuum, E–U trap Reason is not a correct explanation of the (b) A–electrodes, B–NH4 + H2 + CO2 + CH3, Assertion. C–hot water, D–vacuum, E–U trap (c) If the Assertion is correct but Reason is (c) A–electrodes, B–NH3 + H2O, C–hot water, incorrect. D–tap, E–U trap (d) If both the Assertion and Reason are incorrect. (d) A–electrodes, B–NH3 + H2 + H2O + CH4, C–steam, D– vacuum, E–U trap (e) If the Assertion is incorrect but the Reason is 14. A population is in Hardy- weinberg equilibrium correct. for a gene with only two alleles. If the gene 17. Assertion: We have lost all the direct evidence frequency of an allele A is 0.7, the genotype of origin of life. frequency of Aa is [2014, 2016] Reason: The persons responsible for protecting (a) 0.21 (b) 0.42 evidences were not skilled. [1998] (c) 0.36 (d) 0.7 18. Assertion : Ginkgo biloba is a . 15. According to Hardy-Weinberg principle, allele and genotype frequencies in a population will Reason : Organism which have persisted and remain constant from generation to generation remain unchanged for the past several million in the absence of other evolutionary influences. years while their relatives disappeared. [2000] It makes several assumptions which were given 19. Assertion : Among the primates, chimpanzee is below. [2017] the closest relative of the present day humans. i. Random Mating Reason : The banding pattern in the autosome numbers 3 and 6 of man and chimpanzee is ii. Sexual Reproduction remarkably similar. [2004] iii. Non-overlapping Generations 20. Assertion : From evolutionary point of view, iv. Occurrence of Natural Selection human gestation period is believed to be v. Small size of population shortening. Identify two assumptions which do not meet for Reason : One major evolutionary trend in a population to reach Hardy-Weinberg humans has been the larger head undergoing Equilibrium? relatively faster growth rate in the foetal stage. (a) iv and v (b) ii and iv [2004] 21. Assertion : Coacervates are believed to be the (c) iii, iv and v (d) i, ii and iii precursors of life. 16. Which of the following was most similar to Reason : Coacervates were self-duplicating modern man? [2017] aggregates of proteins surrounded by lipid (a) Java man molecules. [2004] (b) Neanderthal man 22. Assertion : Human ancestors never used their (c) Homo habilis tails and so the tail expressing gene has (d) Cro-Magnon man disappeared in them. TYPE B : ASSERTION REASON QUESTIONS Reason : Lamarck's theory of evolution is Directions for (Qs. 17-26) : These questions consist popularly called theory of continuity of germ plasm. [2005] of two statements, each printed as Assertion and Reason. While answering these questions, you are 23. Assertion : Comparative biochemistry provides a strong evidence in favour of common ancestory required to choose any one of the following five of living beings. responses. Reason : Genetic code is universal. [2005] (a) If both Assertion and Reason are correct and 24. Assertion : Darwin's finches show a variety of the Reason is a correct explanation of the beaks suited for eating large seeds, flying Assertion. insects and cactus seeds. IIT JEE NEET eBooks: www.crackjee.xyz

Evolution B-157 Reason : Ancestral seed-eating stock of Darwin's 27. Assertion : The primitive atmosphere was finches radiated out from South American reducing one i.e., without oxygen. [2009] mainland to different geographical areas of the Reason : In the primitive atmosphere, oxygen Galapagos Islands, where they found was involved in forming ozone. competitor-free new habitats. [2005] 28. Assertion : Jave Ape-man, Peking man and 25. Assertion : The earliest organisms that appeared Heidelberg man are the of Homo erectus. on the earth were non-green and presumably anaerobes. Reason : Homo erectus evolved from Homo habilis. [2009] Reason :The first autotrophic organisms were the chemoautotrophs that never released 29. Assertion : Natural selection is the outcome of oxygen. [2006] difference in survival and reproduction among individuals that show variation in one or more 26. Assertion : The earliest fossil form in the traits. phylogeny of horse is . Reason : Adaptive forms of a given trait tend to Reason : Eohippus lived during the early become more common; less adaptive ones epoch. [2007] become less common or disappear. [2004, 2012] Directions for (Qs.27-30) : Each of these questions 30. Assertion : Organic compounds first evolved contains an Assertion followed by Reason. Read them in earth required for origin of life were protein carefully and answer the question on the basis of and nucleic acid. following options. You have to select the one that Reason : All life forms were in water environment best describes the two statements. on ly. [2016] (a) If both Assertion and Reason are correct and Reason is the correct explanation of Assertion. (b) If both Assertion and Reason are correct, but Reason is not the correct explanation of Assertion. (c) If Assertion is correct but Reason is incorrect. (d) If both the Assertion and Reason are incorrect. IIT JEE NEET eBooks: www.crackjee.xyz EBD_7100

B-158 Topicwise AIIMS Solved Papers – BIOLOGY

Type A : Multiple Choice Questions birds of that Island. He studied the finches 1. (c) Eugenics deals with factors related to the there, which were called Darwin’s finches. improvement of human race by controlled The finches showed differences in their selective breeding. It is the improvement beaks based on their different feeding of human race by improving body habits and are examples of natural selection. functions and treatment of defective Darwin’s Finches are good examples of heredity by genetic engineering. Euthenics adaptive radiation ( also called divergent deals with the improvement of human race evolution). Different types of beaks in these by providing better conditions of life. Finches show adaptive radiation. Obstetrics is the study of the reproductive 9. (c) Wings of birds, insects, mammals are process within the female body including analogous organs i.e. have same function fertilization, pregnancy and childbirth. and different structure. The similarity 2. (a) Neoplina is a connecting link between developed in distantly related groups is an annelids and molluscs. adaptation for the same function. So it is analogy or convergent evolution. 3. (c) The theory of Pangenesis was given by Darwin. 10. (d) According to Lamarck, during the lifetime of an organism new characters are 4. (d) The cranial capacity of Neanderthal man developed due to internal vital forces, effect was the largest. The cranial capacity was of environment, new needs, use and disuse about 1450 cc, roughly equal to that of of organs. These acquired characters are modern man. inherited from one generation to another. 5. (c) Atavism is the reappearance of ancestral According to him, intelligence should also and not parental characters in an organisms be inherited and so it fails to explain dull which do not occur normally. A baby that progeny of nobel laureate . has been born with a small tail it is exhibiting 11. (b) Thorn of Bougainvillea and tendril of the case of atavism. Atavism is the sudden Cucurbita are examples of homologous appearance of some ancestor characters chromosomes. which are lost during course of evolution. 12. (c) Organs which have a common fundamental 6. (b) Homo sapiens imply human beings. anatomical plan and similar embryonic Humans belong to an order of mammals origin whatever various functions they called primates and placed in the family may perform are regarded as homologous hominidae. Hominidae includes modern organs. For example, the flippers of a whale, and archaric human beings and also a bat's wing, fore-limb of a horse, a bird's consists of neanderthal, a sub species of wing and forelimbs of human are Homo sapiens. Humans and their ancestors structurally as well as functionally are more closely related to the apes. different. 7. (d) Comparative cytology is the field of study 13. (a) involving examination of similarities in cells 14. (b) For a gene with two alleles, A (dominant) of different organisms Biochemistry and a (recessive), if the frequency of A is p compares DNA and proteins made from and the frequency of a is q, then the DNA. Geology is the study of the earth. frequencies of the three possible 8. (b) During voyage on the ship M.S. Beagle in genotypes (AA, Aa, and aa) can be 1931, Darwin visited Galopagos Islands of expressed by the Hardy-Weinberg South America and studied the climate and IIT JEE NEET eBooks: www.crackjee.xyz

Evolution B-159 equation : Complete development will not take place 22 before the gestation period of 270 – 290 p21 pqq  days in humans. where, p2 = frequency of AA (homozygous 21. (d) Coacervates are large colloidal aggregates dominant) individuals, 2pq = frequency of formed due to intermolecular attraction from Aa (heterozygous) individuals and large organic molecules synthesized q2 = frequency of aa (homozygous abiotically on primitive earth. It mainly recessive) individuals. The equation can consists of proteins, polysaccharides and be used to calculate allele frequencies if water. They do not fulfil the requirement the numbers of homozygous recessive for probable precursors of life. individuals in the population is known. A coacervate is a tiny spherical droplet of Here, p = 0.7 and q = 0.3 (given) assorted organic molecules which is held ? 2pq (frequency of heterozygote) together by hydrophobic forces from a = 2 × 0.7 × 0.3 = 0.42 surrounding liquid. Coacervates possess 15. (a) Occurrence of natural selection and small osmotic properties and form size of population do not meet the criteria spontaneously from certain dilute organic for a population to reach Hardy-Weinberg solutions. They were even once suggested Equilibrium. For Hardy-Weinberg to have played a significant role in the equilibrium to be reached, natural selection evolution of cells and, therefore, of life should not be occurring. If populations are itself. They are interesting not only in that undergoing natural selection at the locus they provide a locally segregated under consideration, allele frequencies will environment but also in that their be continuously changing in a specific boundaries allow the selective absorption direction and Hardy-Weinberg Equilibrium of simple organic molecules from the predicts that allele frequencies will stay surrounding medium. Coacervates do not constant. It assumes that population size have lipid outer membrane, hence they is very large. cannot reproduce. 16. (d) The skeleton of Cro-Magnon was almost 22. (c) According to Lamarck’s theory, continuous identical to the modern man. disuse of organs makes them weak. The Type B : Assertion Reason Questions theory of continuity of germplasm was given by Weismann. 17. (c) We have lost all the direct evidences of If humans share ancestry with other origin of life only due to destruction of primates such as prosimian, monkeys etc, fossils on account of climatic changes and then remnants of that common ancestory not due to any person/(s). should be present in our genes. 18. (a) Ginkgo biloba, belong to the Ginkgoales evolved during the period and is 23. (b) Comparative biochemistry provides a still existing without any morphological or strong evidence for common ancestors of anatomical changes. Hence, both assertion living beings (e.g. proteins lymph, and reason are correct. enzymes, hormones, blood groups etc.) 19. (a) The banding pattern seen on stained 24. (a) Darwin finches found on Galapagos chromosomes from humans and islands differ primarily in body size, feather chimpanzee show striking similarities which colour, bill shape as adaptation to type of indicates that they have evolutionary food available. It is a type of divergent relationships (cytogenetic evidence). evolution. 20. (d) The human gestation period cannot be 25. (b) The earliest organisms that appeared on shortened unless there is really a problem. the earth were heterotrophic because of IIT JEE NEET eBooks: www.crackjee.xyz EBD_7100

B-160 Topicwise AIIMS Solved Papers – BIOLOGY reducing atmosphere and the first 28. (b) The fossil of Java Ape-man was discovered autotrophs were chemoautotrophs. from pleistocene rocks in central Java. The fossil of Peking man was discovered from An anaerobic organism does not require the lime stone caves of Choukoution near oxygen for growth and may even die in its Peking while that of Heidelberg man was presence. Chemotrophs are the first discovered in mid pleistocene. All these three organisms that appeared on earth & obtain fossils come under the category of Homo energy by the oxidation of electron erectus. Homo erectus appeared about 1.7 donating molecules in their environments. million years ago in the middle pleistocene. These molecules can be organic H. erectus evolved from Homo habilis. He (organotrophs) or inorganic (lithotrophs). was about 1.5-1.8 metres tall. He had erect The chemotrophs utilize solar energy and posture. His skull was flat than that of the can be either autotrophic or heterotrophic. modern man. He had protruding jaws, 26. (c) Eohippus is the earliest fossil form in the projecting brow ridges, small canines and phylogeny of horse. Origin of horse took large molar teeth. He made more elaborate place in the period. The first fossil tools of stones and bones, hunted big of horse was found in North America which animals and perhaps knew the use of fire. was named Eohippus or "Dawn Horse. ", 29. (a) The Darwin's theory of Natural Selection that later renamed as hyracotherium. can be generalised as the change in species It was found during eocene period not by the survival of an organism exhibiting a during pliocene. natural variation that gives it an adaptive 27. (c) The lightest atoms of nitrogen, carbon etc. advantage in an environment. Thus, formed the primitive atmosphere. Hydrogen leading to a new environmental equilibrium. atoms were most numerous and most The idea of the survival of the fittest reactive in primitive atmosphere. Hydrogen explains the above evolution by natural atoms combined with all oxygen atoms to selection. According to survival of the form water leaving no free oxygen. Thus fittest, some of the variations exhibited by primitive atmosphere was reducing (without living things make it easier for them to free oxygen) unlike the present oxidizing survive and reproduce. Thus, more atmosphere (with free oxygen). adaptive forms increase. Those which are not fit (or less adaptive) are eliminated. Formation of ozone layer is the consequence of modern oxidizing 30. (b) Organic compounds that first evolved in atmosphere having plenty of free oxygen. earth which required for origin of life were As more oxygen accumulated in the protein and nucleic acid. All life forms were atmosphere (due to photosynthesis) ozone in aquatic environment only. began to appear in the top layers. IIT JEE NEET eBooks: www.crackjee.xyz

30 Human Health & Diseases

TYPE A : MULTIPLE CHOICE QUESTIONS 10. Sporogony of malarial parasite occurs in [1999] (a) liver of man 1. Gambusia fish has been introduced in lakes and (b) RBCs of man ponds of India to control a deadly disease. It (c) stomach wall of mosquito feeds on larva of [1997] (d) salivary glands of mosquito (a) nepenthes (b) anopheles 11. Malignant tertain malaria is caused by [2000] (c) dragon fly (d) house-fly 2. Anti-viral substance is [1997] (a) P. vivax (b) P. malariae (a) antigen (b) antibody (c) P. ovale (d) P. falciparum (c) interferon (d) antibiotic 12. HIV has a protein coat and genetic material 3. Which malarial parasite has longest incubation [2000] period? [1997] (a) ss RNA (b) ds RNA (a) Plasmodium vivax (c) ss DNA (d) ds DNA (b) Plasmodium falciparum 13. Cyclosporine is used [2002] (c) Plasmodium malariae (a) For allergy (d) Plasmodium ovale (b) As immunodepressent 4. The type of antibodies present in colostrum (c) Prophylactic for virus secreted from mammary gland is [1997] (d) None of the above (a) IgM (b) IgD 14. Lysis of foreign cell is mediated through [2002] (c) IgE (d) IgA (a) IgM (b) IgA 5. Which of the following disease is due to an (c) IgE (d) IgM & IgG allergic reaction ? [1998] 15. The treatment of snake-bite by antivenom is an (a) Goitre (b) Hay fever example of [2004] (c) Skin cancer (d) Rheumatic fever (a) artificially acquired active immunity 6. Influenza is caused by [1998] (b) artificially acquired passive immunity (a) virus (b) bacteria (c) naturally acquired passive immunity (c) alga (d) fungus (d) specific natural immunity 7. Which type of cancer is found in lymph nodes and spleen? [1998] 16. Electron beam therapy is a kind of radiation (a) Carcinoma (b) Sarcoma therapy to treat [2004] (c) Lymphoma (d) Leukemia (a) enlarged prostate gland 8. Amoebiasis is caused by [1999] (b) gall bladder stones by breaking them (a) Entamoeba histolytica (c) certain types of cancer (b) Taenia solium (d) kidney stones (c) Plasmodium vivax 17. A young drug addict used to show symptoms (d) E. coli of depressed brain activity, feeling of calmness, 9. Inflammatory response, in allergy is caused by relaxation and drowsiness. Possibly he was the release of [1999] taking [2005] (a) antigen (b) histones (a) Amphetamine (b) Marijuana (c) histamines (d) antibodies (c) Pethadine (d) Valium IIT JEE NEET eBooks: www.crackjee.xyz EBD_7100

B-162 Topicwise AIIMS Solved Papers – BIOLOGY 18. When children play bare footed in pools of dirty 25. Which of the following is a pentameric water and flood water, they may suffer from immunoglobulin and is produced first in a diseases like [2006] primary response to an antigen? [2010] (a) leptospirosis and bilharizia (a)Ig G (b) Ig M (b) malaria, amoebic dysentery and (c)IgA (d) Ig E leptospirosis 26. Cattle fed with spoilt hay to sweet clover which (c) bilharizia, infective hepatitis and diarrhoea contains dicumarol [2011] (d) guinea worm infection, elephantiasis and (a) are healthier due to a good diet amoebic dysentery (b) catch infections easily 19. Which one of the following is not a matching (c) may suffer vitamin K deficiency and pair of a drug and its category ? [2004, 2008] prolonged bleeding (a) Amphetamines - stimulant (d) may suffer from beri-beri due to deficiency (b) Lysergic acid - narcotic of vitamin-B dimethyl amide 27. Opium is obtained from [2011] (c) Heroin - psychotropic (a) Oryza sativa (d) Benzodiazepam - pain killer (b) Coffea arabica 20. An insect bite may result in inflammation of that (c) Thea sinensis spot. This is triggered by the alarm chemicals (d) Papaver somniferum such as [2005, 2008] 28. Match the following bacteria with the diseases (a) histamine and dopamine Column-I Column-II (b) histamine and kinins A. Treponema pallidum I. Plague (c) interferons and opsonin B. Yersinia pestis II. Anthrax (d) interferons and histones C. Bacillus anthracis III. Syphilis 21. Antigen binding site in an antibody is found D. Vibrio IV. Cholera between [2005, 2008] [2012] (a) two light chains (a) A – III; B – I; C – II; D – IV (b) two heavy chains (b) A – IV; B – I; C – II; D – III (c) one heavy and one light chain (c) A – III; B – II; C – I; D – IV (d) either between two light chains or between (d) A – I; B – III; C – II; D – IV one heavy and one light chain depending 29. Which one of the following is a correct match? upon the nature of antigen [2013] 22. The antigen-binding site are present where on (a) Bhang – Analgesic the antibody molecule [2009] (b) Cocaine – Opiate narcotics (a) on light chain as well as on heavy chain. (c) Morphine – Hallucinogen (b) on light chain only. (d) Barbiturate – Tranquiliser (c) on variable region and constant region of 30. Which of the following is an autoimmune light chain. disorder? [2013] (d) on heavy chain only. 23. Which one of the following antimicrobial drugs (a) Myasthenia gravis is suitable for treatment of both tuberculosis and (b) Osteoporosis leprosy? [2010] (c) Muscular dystrophy (a) Isoniazid (d) Gout (b) R-aminosalicylic acid 31. Which of the following is based upon the (c) Streptomycin principle of antigen-antibody interaction? (d) Rifampicin [2014] 24. Antigen is a substance which [2010] (a) PCR (a) lowers body temperature (b) ELISA (b) destroys harmful bacteria (c) r-DNA technology (c) triggers the immune system (d) RNA (d) is used as an antidote to poison IIT JEE NEET eBooks: www.crackjee.xyz

Human Health & Diseases B-163 32. Identify the molecules (A) and (B) shown below 36. Which one of the following pairs of diseases is and select the right option giving their source viral as well as transmitted by mosquitoes? and use. [2014,2015] [2017] (a) Elephantiasis and dengue CH3 N O (b) Yellow fever and sleeping sickness CH (c) Encephalitis and sleeping sickness (A) 3 O (d) Yellow fever and dengue H O TYPE B : ASSERTION REASON QUESTIONS H Directions for (Qs. 37-44) : These questions consist O of two statements, each printed as Assertion and Reason. While answering these questions, you are OH required to choose any one of the following five responses. (B) (a) If both Assertion and Reason are correct and O H the Reason is a correct explanation of the Assertion. (b) If both Assertion and Reason are correct but Molecule Source Uses Reason is not a correct explanation of the (a) (A) Cocaine Erythroxylum Accelerates Assertion. coca the transport (c) If the Assertion is correct but Reason is of dopamine incorrect. (b) (B) Heroin Cannabis Depressant (d) If both the Assertion and Reason are incorrect. sativa and slows (e) If the Assertion is incorrect but the Reason is down body correct. functions (c) (B) Cannabinoid Atropa Produces 37. Assertion: There is no chance of malaria to a man on the bite of male Anopheles mosquito. belladona hallucinations (d) (A) Morphine Papaver Sedative and Reason: It carries a non-virulant strain of Plasmodium. [1998] somniferum pain killer 38. Assertion : Rabies is an infection of mammals, 33. Pasteurella/Yersinia pestis (causal agent of it involves central nervous system which may Bubonic Plague) is transmitted by [2016] result in paralysis and finally death. (a) Cimex (b) Xenopsylla Reason : This is caused by neurotropic bacteria (c) Pediculus (d) Aedes in saliva of rabies animal. [2000] 34. Which of the following pairs is not correctly 39. Assertion : Plasmodium vivax is responsible matched ? [2016] for malaria. (a) Cholera – Vibrio cholerae Reason : Malaria is caused by polluted water. (b) German measles – Rubella virus [2001] (c) Whooping cough – Bordetella pertussis 40. Assertion: Histamine is related with allergic and (d) Tetanus – Pasteurella pestis inflammatory reactions. 35. Human immuno deficiency virus (HIV) has a Reason: Histamine is a vasodilator. [2002] protein coat and a genetic material which is 41. Assertion : Organ transplantation patients are [2017] given immunosuppressive drugs. (a) Single stranded DNA. (b) Single stranded RNA. Reason : Transplanted tissue has antigens (c) Double stranded RNA. which stimulate the specific immune response (d) Double stranded DNA. of the recipient. [2005] IIT JEE NEET eBooks: www.crackjee.xyz EBD_7100

B-164 Topicwise AIIMS Solved Papers – BIOLOGY 42. Assertion: LSD and marijuana are clinically used Reason : Allergens in the environment on as a analgesics. [2006] reaching human body stimulate mast cells in Reason:Both these drugs suppress brain certain individuals. [2003, 2008] function. 49. Assertion : Cannabis sativa is a powerful 43. Assertion : A person who has received a cut anti-depressant. and is bleeding needs to be given anti-tetanus Reason : Hashish and Marijuana are derived treatment. [2006] from it. [2009] Reason : Anti-tetanus injection provides 50. Assertion: Epstein– Barr virus is an oncovirus. immunity by producing antibodies for tetanus. 44. Assertion (A) : Antigen can be easily recognized Reason: It stimulates the growth of cancer. because it has antigenic determinants. [2011] Reason (R) : The recognition ability is innate. 51. Assertion: HIV infected person are prone to [2007] oppurtunistic diseases. Directions for (Qs.45-57) : Each of these questions Reason: Immune system weakens during HIV contains an Assertion followed by Reason. Read them infection. [2011] carefully and answer the question on the basis of 52. Assertion : Histamine is involved in allergic and following options. You have to select the one that inflammatory reactions. best describes the two statements. Reason : Histamine is a vasodilator. [2012] (a) If both Assertion and Reason are correct and 53. Assertion : The antibodies separted from serum Reason is the correct explanation of Assertion. are homogenous. (b) If both Assertion and Reason are correct, but Reason : Monoclonal antibodies are Reason is not the correct explanation of homogenous immunological reagents. [2013] Assertion. 54. Assertion : Interferons are effective against (c) If Assertion is correct but Reason is incorrect. viruses. (d) If both the Assertion and Reason are incorrect. Reason : Proteins which can be synthesized only 45. Assertion : Escherichia coli, Shigella sp. and by genetic engineering are effective against Salmonella sp. are all responsible for diarrhoeal viruses. [2013, 2014] diseases. 55. Assertion : Cancer cells are virtually immortal Reason : Dehydration is common to all types of until the body in which they reside dies. diarrhoeal diseases and adequate supply of Reason : Cancer is caused by damage to genes fluids and electrolytes should be ensured. regulating the cell division cycle. [2006, 2015] [2008] 56. Assertion : Cocaine has a potent stimulating 46. Assertion : Dope test is used to estimate the action on central nervous system, producing a level of blood alcohol by analyzing the breath sense of euphoria and increased energy. of persons drinking alcohol. Reason : Injecting the microbes intentionally during immunisation or infectious organisms Reason : A drunken person usually feels tense gaining access into body during natural infection and less talkative. [2004, 2008] induces active immunity. [2016] 47. Assertion : Interferons are a type of'antibodies 57. Assertion: Artificially acquired passive immunity produced by body cells infected by bacteria. results when antibodies or lymphocytes produced Reason : Interferons stimulate inflammation at outside the host are introduced into a host. the site of injury. [2004, 2008] Reason: A bone marrow transplant given to a 48. Assertion : Mast cells in the human body release patient with genetic immunodeficiency is an excessive amounts of inflammatory chemicals, example of artificially acquired passive immunity. which cause allergic reactions. [2017] IIT JEE NEET eBooks: www.crackjee.xyz

Human Health & Diseases B-165

Type A : Multiple Choice Questions 14. (c) IgE immunoglobulin acts as mediator in allergic response. 1. (b) Gambusia fish is larvicidal i.e. feed on insect 15. (b) Artificial passive immunity is the resistance larva. They are introduced into lakes and passively transferred to a recipient by ponds to eat mosquito larvae (e.g. administration of antibodies, e.g. Anopheles) and control malaria. administration of antivenom. 2. (c) Interferons are glycoproteins produced by 16. (c) Electron beam therapy is a kind of radiation viral infected cell. They make the cells therapy to treat certain types of cancer. It resistant to viral infection. is used as a therapeutic treatment for cancer. 3. (c) Incubation period of Plasmodium vivax is 17. (d) Valium is the anti depressant used by 14 days, P. malariae is 30 days, P. ovale is addicts that produces feelings of calmness, 14 days, and P. falciparum is 12 days. relaxation and drowsiness. 4. (d) Colostrum (also called first milk) is the thin 18. (a) In leptospirosis and bilhariziasis diseases, yellowish fluid secreted by the mammary cercaria of the fluke penetrates the human glands at the time of parturition and which body through the skin during bath in rivers precedes the production of true milk. It and canals. provides a nursing infant with essential 19. (d) Amphetamine is pap pill/antisleep drug nutrients and infection-fighting antibodies which stimulates CNS. Lysergic acid (called immunoglobulin). I A is the major g diethylamide (LSD) is the most potent immunoglobulin in colostrum. It provides hallucinogen. Benzodiazepam is a sedative protection from inhaled and ingested which switches of the CNS and is pathogens. antianxiety. Heroin is a psychotropic drug 5. (b) Hay fever is due to allergic reaction. which acts on the brain and alters behavior, 6. (a) Influenza is caused by virus Myxovirus consciousness and capacity of perception. influenzae. It is an acute respiratory tract infection. 20. (b) An insect bite may result in inflammation of 7. (c) Cancer of lymphatic tissue i.e. lymph that spot. This is triggered by the alarm nodes, spleen, is lymphoma. chemicals such as histamine and kinins. 8. (a) Amoebiasis or amoebic dysentry is caused Histamine is a biogenic amine involved in by Entamoeba histolytica. local immune responses as well as regulating 9. (c) Inflamatory response is due to the release physiological function in the gut and acting of histamines by the damaged mast cells. as a neurotransmitter. It is found in virtually The vascular fluid comes out of the blood all animal body cells. New evidence also vessels causing swelling of the region. indicates that histamine plays an important 10. (d) Sporogony of Plasmodium occurs in the role in chemotaxis of white blood cells. Kinin salivary glands of female Anopheles. is any of various structurally related 11. (d) Malignant tertain malaria is caused by polypeptides, such as bradykinin and Plasmodium falciparum. kallikrein. They are members of the autacoid 12. (a) HIV (AIDS virus) consists of glycoprotein family. They act locally to induce coat, double layer of lipid membrane of two vasodilation and contraction of smooth protein coats. It contains ss RNA and muscles. reverse transcriptase. 21. (c) Antigen binding site is Y-shaped showing 13. (b) Cyclosporin is used as immunodepressant lock and key pattern made up of one heavy especially for the patients having organ and one light chain. transplantation. It suppresses T-lymphocytes 22. (a) The Y-shaped antibody molecule is activity in the immune response. composed of 4 polypeptide chains - two IIT JEE NEET eBooks: www.crackjee.xyz EBD_7100

B-166 Topicwise AIIMS Solved Papers – BIOLOGY identical light chains and two heavy chains allows for easy visualization of results. held together by disulphide bonds. 32. (d) Molecule (A) represents structure of Antigen-binding site is formed on antibody morphine. Morphine is the most abundant molecule where a heavy chain variable and alkaloid found in Opium, the dried sap a light chain variable come closer. (latex) derived from shallowly slicing the 23. (d) Leprosy is caused by Mycobacterium unripe seedpods of the Opium, or common leprae, while tuberculosis is caused by and/or edible poppy Papaver somniferum. bacteria Streptococcus or Staphylococcus. Morphine is a potent opiate analgesic drug Rifampicin is a common medicine in that is used to relieve severe pain. treatment of both the diseases. 33. (b) 24. (c) Antigen is the only foreign substance 34. (d) Tetanus is caused by Clostridium tetani which triggers immune system by way of while plague is caused by Pasteurella antibody formation. These antibodies pestis. neutralize antigen by antigen-antibody reaction. 35. (b) The human immunodeficiency virus is a lentivirus that causes the acquired 25. (b) I M is a pentameric immunoglobulin that g immunodeficiency syndrome, a condition is produced first in a primary response to in humans in which progressive failure of an antigen. I A, I D and I E consist of g g g the immune system allows life-threatening single monomers. opportunistic infections and cancers to 26. (c) Hemorrhagic syndrome of cattle are thrive. HIV has a protein coat and a genetic associated with the feeding of sweet clover material which is single stranded RNA. hay containing dicumarol. 27. (d) Opium is a narcotic formed from the latex 36. (d) Yellow fever and dengue are viral diseases, released by lacerating the immature seed and they are transmitted by mosquitoes. pods of opium poppies (Papaver Type B : Assertion Reason Questions somniferum). Morphine is the most preva- lent alkaloid in opium, about 10%-16% of 37. (c) Male Anopheles mosquito do not have the total alkaloids. It is responsible for most piercing and sucking type of mouth parts. of the harmful effects such as lung edema, So, they can not inject malarial parasite into respiratory difficulties, coma, or cardiac or man. respiratory collapse. 38. (c) Rabies (hydrophobia) is caused by rabies 28. (a) Plague is a deadly infectious disease that virus. Its vectors are raboid animals is caused by the enterobacteria Yersinia especially dogs. It leads to encephalitis, pestis (formely know as Pasteurella pestis). fear of water (hydrophobia), high fever, Anthrax is an infectious bacterial disease severe headache, spasm of throat & chest, which involves skin, gastrointestinal tract leading to death. or lungs. Syphilis is a sexually transmitted 39. (b) Malaria is caused by Plasmodium whose infection caused by bacterium Treponema sexual phase occurs in the mosquito pallidium. Cholera is an infection in the Anopheles. When female Anopheles feed small intestine caused by the bacterium on blood, they can serve as vector host for Vibrio cholerae that causes a large amount malarial parasite. of watery diarrhea and vomiting. 40. (a) Histamine is produced by mast cells in 29. (d) response to the allergy and inflammatory 30. (a) reaction. When histamine is released from mast cells, it causes vasodilation and an 31. (b) The ELISA is a fundamental tool of clinical increase in permeability of the blood vessel immunology, and is used as an initial screen walls. These effects, in turn cause the for HIV detection. Based on the principle common symptoms of allergy including a of antigen- antibody interaction, this test running nose and watering eyes. IIT JEE NEET eBooks: www.crackjee.xyz

Human Health & Diseases B-167 41. (a) Success of organ transplant depends on 46. (d) Blood alcohol test measures the amount of proper matching of histocompatibility of alcohol (ethanol) in the body. Alcohol is antigens that occurs in all cells of the body. quickly absorbed into the blood and can As there are antigens which are likely to be be measured within minutes of having an attacked by recipient's T-cells and alcoholic drink. The amount of alcohol in antibodies, the recipient of organ transplant the blood reaches its highest level about is always given immunosuppressants to an hour after drinking. But food in the prevent immune rejection of the stomach may increase the amount of time transplanted tissue. it takes for the blood alcohol to reach its 42. (c) LSD and marijuana are anti inflammatory, highest level. About 90% of alcohol is sedative, anticonvulsive and laxative in broken down in the liver. The rest of it is action. passed out of the body in urine and your 43. (c) Anti tetanus is a disinfectant i.e. it prevents exhaled breath. Symptoms of alcohol the infection due to the entry of bacteria intoxication include confusion, lack of through wounds. coordination, unsteadiness that makes it 44. (b) Antigen is a substance that when hard to stand or walk, or erratic or unsafe introduced in the body, stimulate the driving. Dope test is not related to alcohol. production of antibody. They are mostly Dope test is a blood test to know whether proteins but may be carbohydrates, lipids, a person used any drug to improve their nucleic acids etc. One antigen can bind with performance. many antibodies. Every antigen has many 47. (d) Interferons are natural proteins produced antigenic determinants called epitopes. by the cells of the immune system in The recognition ability of antibody is response to challenges by foreign agents innate and develops without exposure to such as viruses, parasites and tumor cells. the antigen. Interferons belong to the large class of 45. (b) Diarrhoeal disease conditions include glycoproteins known as cytokines. frequent and excessive discharge of watery Interferons are produced by a wide variety material from the bowel. Such diseases mostly of cells in response to the presence of result from ingestion of harmful germs with double-stranded RNA, a key indicator of food and water. E. coli, Shigella sp. & viral infection. Interferons assist the immune response by inhibiting viral replication salmonella sp. causes diarrhoea. Diarrhoea within host cells, activating natural killer cells caused by virus, bacteria or parasites and macrophages, increasing antigen possesses two characteristics- firstly, the presentation to lymphocytes, and inducing offending organisms colonise the intestine the resistance of host cells to viral infection. and as a consequence cause inflammation When the antigen is presented to matching of the intestine or enteritis; and secondly, T and B cells, those cells multiply and they upset the balance of intestinal fluid strategically and specifically wipe out the absorption and secretion mechanism, often foreign substance. That is why antigen enhancing the latter very considerably, presentation is so important to the immune which is then manifested as watery stool response. discharged frequently in large volumes. 48. (a) A mast cell contains many granules rich in Shigella sp, Salmonella sp. are quite histamine serotonin and heparin. Although closely related genera that are responsible best known for their role in allergy and for diarrhoeal diseases. Dehydration is anaphylaxis, mast cells play an important common to all types of diarrhoeal diseases protective role as well, being intimately & adequate supply of fluids & electrolytes involved in wound healing and defense that provides ions, should be ensured. against pathogens. The symptoms of an allergic reaction develop in response to IIT JEE NEET eBooks: www.crackjee.xyz EBD_7100

B-168 Topicwise AIIMS Solved Papers – BIOLOGY histamine. Mast cells release a large amount vertebrates. When released, it has the effect of histamine into the blood stream & it also of dilating capillaries and lowering blood act as initiator of the inflammatory response, pressure. Histamine is involved in allergic which aids the arrival of leucocytes at a site and inflammatory reactions also. of infection. Histamine stimulates capillary 53. (d) From hyperimmunized animals, the blood dilation increased capillary permeability, serum may be taken and antibodies may be closure of bronchial tubes, mucus secretion, isolated from this serum. However, the pain & swelling. antibodies, whenever separated from serum 49. (d) Cannabis sativa or Hemp is an annual herb after induction due to an antigen, are of cannabaceae family. It is mainly the source usually heterogenous, because the cells of fibre. Cannabis sativa is not an keep on producing a variety of antibodies. antidepressant. Antidepressant is a Monoclonal antibodies (Mabs), on the psychiatric medication used to alleviate other hand are homogeneous immunological mood disorders such as major depression. reagents of defined specificity so that these Hashish and Marijuana are drugs derived can be utilized for diagnosis and screening from it. Preparations of flowers of Cannabis of disease. is called marijuana while preparations of the 54. (c) Interferons are proteins that are effective resinous extract is called hashish. Both are against most viruses. They are naturally consumed by smoking, vapourising and oral produced by virus infected cells. The ingestion and are hallucinogen drugs that proteins interact with adjacent cells and cause acute panic anxiety reaction. make them resistant to virus attack. Now 50. (a) Oncoviruses are cancer – causing viruses interferons are also being manufactured and may be DNA or RNA virus e.g., Epstein through genetic engineering. – Barr – Virus, Herpes simplex type 2 virus etc. This shows that the development of Interferons control the multiplication of cancerous tumour is associated with virus particles by inhibiting their protein certain genes. synthesis. 51. (a) Usually, person shows symptoms of HIV 55. (b) Cancer cells divide and redivide mitotically infection within 2 to 6 weeks of exposure to and is due to the activation of pro- the virus. But in some persons, the virus oncogenes. may remain silent for long periods (upto 10 56. (c) Cocaine is obtained from the cocaplant, years) before symptoms of full blown AIDS erthyroxylum coca. Cocaine is commonly are observable. The symptoms of HIV called as coke or crack which is usually infection include fever, lethargy, pharyngitis, snored. It has a potent stimulating action nausea, headache, rashes etc. Persons of euphoria and increased energy. It suffering from AIDS have a weakened interferes with the transport of the immune system due to depletion of T- helper neurotransmitter dopamine. Its increased cells. Such persons show opportunistic dosages cause hallucinations. infections, i.e., infection by those fungi, 57. (b) Artificially acquired passive immunity bacteria and viruses to which a person with results when antibodies or lymphocytes normal immune system is expected to be that have been produced outside the host resistant. Therefore, persons prone to are introduced into a host. This type of opportunistic infections may be suspected immunity is immediate short lived, lasting to be infected by HIV, particularly, if the only a few weeks to a few months. An count of T helper cells in 200/ ml or lower. example is bone marrow transplant given 52. (a) Histamine is a derivative of the amino acid to a patient with genetic immunodeficiency. histidine produced by damaged cells of IIT JEE NEET eBooks: www.crackjee.xyz

Strategies for Enhancement in 31 Food Production

TYPE A : MULTIPLE CHOICE QUESTIONS increasing amounts of cholesterol. Results obtained are shown in the given graph. 1. The chemical which are produced by host plants [2005, 2008] due to infection as a defence reaction to pathogen, are called [1997] (a) phytotoxin (b) toxin g

(c) phytotron (d) phytoalexins a v

r 2 a

2. Auxanometer is used to measure [1998] l

t c

(a) length (b) respiration e s n i

(c) transpiration (d) ascent of sap f o

3. Crop rotation is used to increase [1999] t 1 (a) soil fertility W (Khapra beetle) m (b) pore size and soil particle (c) organic content of soil O (d) viscosity of soil water 12345 4. Haploid cultures can be obtained by culturing µg cholesterol/g basal diet (a) pollen grains (b) embryo [2000] The graph indicates (c) shoot apex (d) root apex (a) cholesterol is an essential dietary 5. Azolla is used as a biofertilizer because it[2003] requirement of khapra beetle. (a) multiplies very fast to produce massive biomass (b) growth of khapra beetle is directly (b) has association of nitrogen-fixing Rhizobium proportional to cholesterol concentration. (c) has association of nitrogen-fixing Cyanobacteria (c) cholesterol concentration of 2 µg/g diet is (d) has association of mycorrhiza the optimum level. 6. Pruning of plants promotes branching because (d) growth of khapra beetle is inhibited when the axillary buds get sensitized to [2004] cholesterol concentration exceeds 5 µg/g (a) ethylene (b) gibberellin diet. (c) cytokinin (d) indole acetic acid 10. A scion is grafted to a stock. The quality of 7. Somaclonal variation can be obtained by[2004] fruits produced will be determined by the (a) application of colchicine genotype of [2006] (b) irradiation with gamma rays (a) stock (c) tissue culture (d) hybridisation (b) scion 8. Somaclonal variation appears in [2005] (c) both stock and scion (a) organisms produced through somatic (d) neither stock nor scion hybridization. 11. Cocoa is the plant from which chocolate is made. (b) plants growing in highly polluted Which part is used to extract it? [2007] conditions. (a) Flower (b) Fruit (c) apomictic plants. (c) Seeds (d) Bark (d) tissue culture raised plants. 12. Bean seeds were planted and put on a sunny 9. In an experiment freshly hatched larvae of an windowsill. As the plants grew, their stems bent insect (Khapra beetle) were reared on a basal toward the window. This bending was most likely diet (complete diet without cholesterol) with caused by an [2009] IIT JEE NEET eBooks: www.crackjee.xyz EBD_7100

B-170 Topicwise AIIMS Solved Papers – BIOLOGY (a) unequal distribution of auxin in the stem. 20. Which of the following is a viral disease of poultry (b) unequal distribution of a neurotransmitter birds? [2017] in the stem. (a) Anthrax (b) Ranikhet (c) equal distribution of auxin in the stem. (c) Coccidiosis (d) None of these (d) equal distribution of a neurotransmitter in TYPE B : ASSERTION REASON QUESTIONS the stem. 13. Phytotron is a facility to [2010] Directions for (Qs. 21-25) : These questions consist (a) grow plants under disease-free conditions. of two statements, each printed as Assertion and (b) conserve endangered species of plants. Reason. While answering these questions, you are (c) grow plants under controlled conditions. required to choose any one of the following five (d) induce mutations. responses. 14. Essential oils are those which [2011] (a) If both Assertion and Reason are correct and (a) are essential to the plant itself the Reason is a correct explanation of the (b) are used as lubricants Assertion. (c) produce perfumes (b) If both Assertion and Reason are correct but (d) are essential for human beings Reason is not a correct explanation of the Assertion. 15. Coconut water is rich in [2011] (c) If the Assertion is correct but Reason is (a) auxins (b) gibberellins incorrect. (c) abscisic acid (d) cytokinin (d) If both the Assertion and Reason are incorrect. 16. Which of the following is the pair of (e) If the Assertion is incorrect but the Reason is biofertilizers? [2014] correct. (a) Azolla and BGA 21. Assertion : In plant tissue culture somatic (b) Nostoc and legume embryos can be induced from any plant cell. (c) Rhizobium and grasses Reason : Any viable plant cell can differentiate (d) Salmonella & E. coli into somatic embryos. [2003] 17. Hisardale is a new breed of sheep developed in 22. Assertion : Use of fertilizers greatly enhances Punjab by one of the breeding technique in crop productivity. which superior male of one breed is mated with Reason : Irrigation is very important in increasing superior females of another breed. crop productivity. [2003] Identify the breeding technique from the option 23. Assertion : Fish meal is a rich source of protein given below. [2016] (a) Inbreeding (b) Out crossing for cattle and poultry. (c) Out breeding (d) Cross breeding Reason : Fish meal is produced from non-edible 18. Biodiversity loss occurs due to the reasons parts of fishes like fins, tail etc. [2004] given below. [2017] 24. Assertion : Cattle breeds can be improved by (i) Habitat loss and fragmentation super ovulation and embryo transplantation. (ii) Co- Reason : Superovulation in high milk-yielding (iii) Over-exploitation cows is induced by hormonal injection.[2004] (iv) Alien species invasion 25. Assertion (A) : Vernalization is acceleration of Identify the correct reasons. subsequent flowering by low temperature (a) (i) and (ii) (b) (i), (ii) and (iii) treatment. (c) (ii), (iii) and (iv) (d) (i), (ii), (iii) and (iv) Reason (R) : Site of vernalization is apical 19. Explant is required to be disinfected before meristem. [2007] placing in culture. This is done by [2017] Directions for (Qs. 26-31) : Each of these questions (a) autoclaving contains an Assertion followed by Reason. Read them (b) ultra-violet rays carefully and answer the question on the basis of (c) clorax or hypochlorite following options. You have to select the one that (d) X-rays best describes the two statements. IIT JEE NEET eBooks: www.crackjee.xyz

Strategies for Enhancement in Food Production B-171 (a) If both Assertion and Reason are correct and Reason : Hybrid vigour is mostly used in Reason is the correct explanation of Assertion. vegetatively propagated plants. [2013] (b) If both Assertion and Reason are correct, but 29. Assertion : Yeasts such as Saccharomyces Reason is not the correct explanation of cerevisiae are used in baking industry. Assertion. Reason : Carbon dioxide produced during (c) If Assertion is correct but Reason is incorrect. fermentation causes bread dough to rise by (d) If both the Assertion and Reason are incorrect. thermal expansion. [2015] 26. Assertion : In plant tissue culture, somatic 30. Assertion : Somatic embryos can be induced embryos can be induced from any plant cell. from any cell in plant tissue culture. Reason : Any viable plant cell can differentiate Reason : Any living plant cell is capable of dif- into somatic embryos. [2008] ferentiating into somatic embryos. [2017] 27. Assertion : Cattle breeds can be improved by 31. Assertion : A major advantage of tissue culture superovulaton and embryo transplantaion. is protoplast fusion. Reason : Superovulation in high milk-yielding Reason : A hybrid is formed by the fusion of cows is induced by hormonal injection.[2013] naked protoplasts of two plants. [2017] 28. Assertion : In case of vegetatively propagated crops, pure-line selection is not required. IIT JEE NEET eBooks: www.crackjee.xyz EBD_7100

B-172 Topicwise AIIMS Solved Papers – BIOLOGY

Type A : Multiple Choice Questions that control cell division, shoot meristem 1. (d) Phytoalexins are produced by plants that initiation, leaf and root differentiation, are under attack. Phytoalexins produced in senescence. plants act as toxins to the attacking 7. (c) Somaclonal variations are differences in organism. They may puncture the cell wall, plants that are raised from the callus by tissue culture. If these variations are of delay maturation, disrupt metabolism or economic value, e.g. , induce tolerance of prevent reproduction of the pathogen in pests, diseases, etc, such plants are question. However, phytoalexins are often selected and multiplied. This technique has targeted to specific predators; a plant that been used in wheat, rice, potato and has anti-insect phytoalexins may not have tomato. the ability to repel a fungal attack. 8. (d) Somaclonal variations refer to heritable 2. (a) Auxanometer is an apparatus for measuring changes which accumulate in callus (tissue increase or rate of growth in plants. It automatically detects and measures plant culture) from a somatic explant and express growth and stores information in a data in the progeny of in vitro regeneration logger. obtained from callus. These variations have been used to develop several useful. 9. (a) According to graph, growth of Khapra beetle is directly proportional to cholesterol concentration. 10. (b) The quality of fruits produced by the grafted plant is determined by the genotype of scion. 11. (c) Cocoa is obtained from seeds of cocoa plant 3. (a) Crop rotation increases the soil fertility by Theobroma. It belongs to family sowing different crops, usually legume and sterculiaceae. This plant is a native of non-legume, in successive seasons on the tropical America and its seeds are used to same piece of land. prepare cocoa and chocolate. 4. (a) Pollen grains are used for haploid cultures 12. (a) The plant hormone, auxin, is more since they possess n-number of distributed on the side away from the chromosomes that are required for haploid unilateral illumination causing cells to grow cultures. faster in the darkerside, which in turn, 5. (c) Azolla, has cyanobacteria which is the causes the plant to bend toward the light. If nitrogen fixing bacteria. Due to this Azolla the distribution were equal, the plant would is able to fix the nitrogen as nitrates, thereby grow just upwards. Neuro-transmitters are making it available to plants in the soluble chemicals secreted by multicellular animals form of nitrogen. and are used in transmitting impulses in the 6. (c) Pruning causes the cutting of plant apex nervous system. which lowers the amount of auxin and 13. (c) Phytotron is a chamber in which plants can relatively an increase in cytokinin content. be grown in controlled condition. In the plant, cytokinin promotes growth of axillary buds causing branching. 14. (c) Essential oils-These are volatile oils and Cytokinins are essential plant hormones possess strong aromatic smell. IIT JEE NEET eBooks: www.crackjee.xyz

Strategies for Enhancement in Food Production B-173

15. (d) Coconut water is rich in cytokinin. 23. (a) Fish meal is made from nonedible parts of Cytokinin initiates cell division and is found the fish and is rich source of protein. in dividing tissues. So, coconut water is Fish is a rich source of protein and many of used as a medium in tissue culture of plant the vitamins and minerals which is required tissues where it initiates cell division. for good health. Fish also has many other Coconut water is liquid endosperm. benefits including helping to protect 16. (a) Azolla and BGA are biofertilizer which against heart disease and a range of other increases the fertility of soil. illnesses. Fish is also a source of zinc, which 17. (d) Hisardale is a new breed of sheep is needed for a healthy immune system, developed in Punjab by crossing Bikaneri iodine, needed for a healthy metabolism ewes and Marino rams. Cross breeding is a and vitamins A and B12. Oil-rich fish are an method in which superior male of one breed important source of omega 3 fats. Your is mated with superior females of another body cannot make these special fats so you breed. It allows the desirable qualities of need to eat foods containing omega 3 two different breeds to be combined. everyday. Fish meal, is a commercial 18. (d) Biodiversity refers to the variety found in product made from the waste of fish oil and biota due to the genetic make-up of plants the bones from processed fish. It is a brown and animals to cultural diversity. The main powder or cake obtained by pressing the cause of the loss of biodiversity can be whole fish or fish trimmings to remove the attributed to the influence of human beings fish oil. The major use of fish meal is as a on the world's ecosystem. The important high-protein supplement in aquaculture factors causing loss of biodiversity are - feed. habitat loss, habitat fragmentation, disturbances, over exploitation of 24. (a) Superovulation is done by hormone resources, pollution, exotic species, co- injection. During artificial insemination, extinction, alien species invasion, intensive 4-10 embryos are transplanted into carrier agriculture and forestry. cows. The seven days old embryos can be 19. (c) Before transferring on the culture preserved at a temperature of –196ºC for medium, the explant is first of all disinfected several years. The assertion & reason are by surface sterilization using clorax water, true and the reason is the correct sodium or calcium hypochlorite solution explanation for the assertion. or methiolate. Too much care must be taken 25. (b) The physiological mechanism of flowering in this operation so that the cells do not in plants is controlled by two factors__ light die. period and low temperature. The cold 20. (b) Coccidiosis is a protozoan disease. treatment of plants to induce flowering is Type B : Assertion Reason Questions called vernalization. Term vernalization was first given by T.D.Lysenko(1928). As a 21. (a) Any living plant cell which is viable can be result of vernalization a flowering hormone used to culture somatic embryos. These called vernaline is formed. Site of embryos have the characteristic features vernalization is apical meristem. of the parent plant. The somatic cultures 26. (a) Any living plant cell which is viable can be are viable from the stem, root or leaves and used to culture somatic embryos. These the most viable part is the meristematic cell. embryos have the characteristic features 22. (b) Irrigation relates to the supply of water to of the parent plant. The somatic cultures the crops. Fertilizers which are mainly NPK are viable from the stem root, or leaves & are required to increase the harvest of the most viable part is the meristematic cell. crops. IIT JEE NEET eBooks: www.crackjee.xyz EBD_7100

B-174 Topicwise AIIMS Solved Papers – BIOLOGY

27. (b) Cattle breeds can be improved by super profitably used in vegetatively propagated ovulation and embryo transfer technique. crops because they do not involve sexual It is also known as Multiple Ovulation reproduction and hence no loss of hybrid Embryo Transfer Technology (MOET). In superiority. this method, a cow is administered 29. (a) Yeast is a fermentation agent. It is a known hormones with FSH like activity, to induce fact that yeast raises bread dough to rise super ovulation instead of one egg, which and hence, yeast is also used to increase they normally yield per cycle they produce the volume, making the dough porous and 6-8 eggs. The animal is either mated with the product soft. It is the carbon dioxide an elite bull or artificially inseminated. The that is produced by the yeast that helps fertilised eggs at 8-32 cells stages are the dough to rise. 30. (a) Somatic embryos are non- zygotic embryo removed non surgically and transferred to like structures that develop into from any surrogate mothers. The genetic mother is type of tissue in plant tissue culture. available for another round of super 31. (b) An important technique of tissue culture, ovulation. somatic hybridization results in the 28. (b) In case of vegetatively propagated crops, production of somatic hybrid plants. Two pure line selection is not required. Pure- different plant varieties each with a line selection is useful only for sexually desirable character can be made to undergo reproducing plants. Hybrid vigour is most protoplast fusion, which further can be grown into a new plant. IIT JEE NEET eBooks: www.crackjee.xyz

32 Microbes in Human Welfare

TYPE A : MULTIPLE CHOICE QUESTIONS C. Bacillus thuringiensis III. Production of human insulin 1. Which of the following antibiotic was discovered D. Pseudomonas putida IV. Biological by Alexander Flemming ? [2000] control of (a) Streptomycin (b) Tetracycline fungal (c) Penicillin (d) Terramycin disease 2. Which one of the following pairs is correctly V. Bio- matched? [2003] decomposed (a) Rhizobium - Parasite in the roots of insectiside leguminous plants. [2016] (b) Mycorrhizae - Mineral uptake from soil. (a) A – III; B – I; C – V; D – IV (c) Yeast - Production of biogas. (b) A – I; B – II; C – III; D – IV (d) Myxomycetes - The ringworm diseases. (c) A – II; B – I; C – III; D – IV 3. The bacteria Pseudomonas is useful because of (d) A – III; B – I; C – V; D – II its ability to [2004] 8. What would happen if oxygen availability to (a) transfer genes from one plant to another. activated sludge flocs is reduced? (b) decompose a variety of organic compounds. [2016] (c) fix atmospheric nitrogen in the soil. (a) It will slow down the rate of degradation of (d) produce a wide variety of antibiotics. organic matter. 4. Chloramphenicol and erythromycin (broad (b) The centre of flocs will become anoxic, spectrum antibiotics) are produced by [2014] which would cause death of bacteria and (a) Streptomyces (b) Nitrobacter eventually breakage of flocs. (c) Rhizobium (d) Penicillium (c) Flocs would increase in size as anaerobic 5. A patient brought to a hospital with myocardial bacteria would grow around flocs. infarction is normally immediately given: [2014] (d) Protozoa would grow in large numbers. (a) Penicillin (b) Streptokinase 9. Which one of the following statement regarding BOD is true? [2017] (c) Cyclosporin-A (d) Statins (a) The greater the BOD of waste water, more 6. Microbes are used in [2015] is its polluting potential. 1. primary treatment of sewage (b) The greater the BOD of waste water, less is 2. secondary treatment of sewage its polluting potential. 3. anaerobic sludge digester (c) The lesser the BOD of waste water, more is 4. production of bioactive molecules its polluting potential. (a) 1, 3 and 4 (b) 1, 2, 3 and 4 (d) The lesser the BOD of waste water, less is (c) 2, 3 and 4 (d) 3 and 4 its polluting potential. 7. Choose the right combination 10. The free-living fungus Trichoderma can be used Column-I Column-II for [2017] A. Escherichia coli I. Nif gene (a) killing insects B. Rhizobium melilotae II. Digestive (b) biological control of plant diseases hydrocarbon (c) controlling butterfly caterpillars of crude oil (d) producing antibiotics IIT JEE NEET eBooks: www.crackjee.xyz EBD_7100

B-176 Topicwise AIIMS Solved Papers – BIOLOGY

TYPE B : ASSERTION REASON QUESTIONS Directions for (Qs.13-15) : Each of these questions contains an Assertion followed by Reason. Read them Directions for (Qs. 11-12) : These questions consist carefully and answer the question on the basis of of two statements, each printed as Assertion and following options. You have to select the one that Reason. While answering these questions, you are best describes the two statements. required to choose any one of the following five (a) If both Assertion and Reason are correct and responses. Reason is the correct explanation of Assertion. (a) If both Assertion and Reason are correct and (b) If both Assertion and Reason are correct, but the Reason is a correct explanation of the Reason is not the correct explanation of Assertion. Assertion. (b) If both Assertion and Reason are correct but (c) If Assertion is correct but Reason is incorrect. Reason is not a correct explanation of the (d) If both the Assertion and Reason are incorrect. Assertion. 13. Assertion : Lichen is important for chemical (c) If the Assertion is correct but Reason is industries. incorrect. Reason : Litmus and Orcein are formed from (d) If both the Assertion and Reason are incorrect. lichens. [2009] (e) If the Assertion is incorrect but the Reason is 14. Assertion : Yeasts such as Saccharomyces correct. cerevisiae are used in baking industry. 11. Assertion : Leguminous plants are nitrogen [2003, 2011] fixers. Reason : Carbon dioxide produced during Reason : Leguminous plants have Rhizobium fermentation causes bread dough to rise by in their root nodules. [1997] thermal expansion. 12. Assertion : Nitrogen-fixing enzyme in legume 15. Assertion : Vitamins B2 is found in cereals, green root nodules function at low oxygen vegetables, brewer's yeast, egg white, milk and liver. concentration. Reason : It can be commercially produced by Reason : Low oxygen concentration is provided some yeasts. [2014] by leghaemoglobin. [2004] IIT JEE NEET eBooks: www.crackjee.xyz

Microbes in Human Welfare B-177

Type A : Multiple Choice Questions the quantity of organic wastes in the water supply is high then the number of 1. (c) Alexander Flemming discovered Penicillin decomposing bacteria present in the water from a fungus penicillium accidentally. It is will also be high. As a result, BOD value a natural antibiotic. will increase. 2. (b) Mycorrhiza is a symbiotic relationship 10. (b) Trichoderma is a free-living saprophytic between fungi and roots of higher plants. fungi that most commonly lives on dead Mycorrhizae present in the soil help in the organic matter in the soil and rhizosphere uptake of minerals from soil. These organisms also help in the binding of roots (root ecosystem). It inhibits pathogens through release of gliotoxin, viridin, to the soil. Since it is active transport in the roots, the mycorrhizae help in this active gliovirin and trichodermin like substances. transport of minerals. Type B : Assertion Reason Questions 3. (b) Pseudomonas is a gram negative rod shaped bacteria. It is useful because of its 11. (a) The Rhizobium bacteria converts the ability to decompose a variety of organic atmospheric nitrogen into soluble nitrates compounds. Other bacteria which help in that is absorbed by the plants. purification, decay are Streptococci, Soybean root nodules, contain billions of Clostridium, Micrococus, Proteus, etc. Bradyrhizobium bacteria. 4. (a) 12. (a) Leghaemoglobin is the oxygen scavenger 5. (b) Streptokinase is immediately given to and it protects nitrogen fixing enzyme dissolve the thrombus carring myocardial nitrogenase from oxygen. 13. (a) Litmus is an important and widely used dye infarction. in chemical laboratories as an acid-base 6. (c) Microbes are not used in the primary indicator. It is obtained from Rocella treatment of sewage which basically montaignei. Orcein, a biological stain, is involves settling down of sludge through obtained from Rocella tinctoria. sedimentation. 14. (a) Yeast is a fermentation agent. It is a known 7. (d) fact that yeast raises bread dough to rise 8. (b) In the secondary treatment of sewage, if and hence, Yeast is also used to increase oxygen availability to activated sludge the volume, making the dough porous and flocs (masses of bacteria associated with the product soft. It is the yeast that helps fungal filaments to form mesh like structure) the dough to rise. is reduced; the centre of flocs will becomes 15. (b) Besides cereals, green vegetables, brewer's anoxic, which would cause death of yeast, egg white, vitamin B is also bacteria and eventually breakage of flocs. 2 produced by intestinal bacteria. The 9. (a) BOD is the method of determining the vitamin was first obtained in 1938 using amount of oxygen required by wild strain of mould Ashbhya gossypii. microorganisms to decompose the waste Vitamin B is essential for normal growth present in the water supply. It is a measure 2 and reproduction in a number of laboratory of organic matter present in the water. If animals. IIT JEE NEET eBooks: www.crackjee.xyz EBD_7100

B-178 Topicwise AIIMS Solved Papers – BIOLOGY Biotechnology: Principles and 33 Processes

TYPE A : MULTIPLE CHOICE QUESTIONS (c) making transient pores in the cell membrane to introduce gene constructs. 1. Restriction endonucleases are used as [1998] (d) purification of saline water with the help of (a) molecular build up at nucleotides. a membrane system. (b) molecular degradation to DNA breakup. 7. What is the first step in the Southern blot (c) molecular knives for cutting DNA at technique? [2004, 2008] specific sites. (a) Denaturation of DNA on the gel for (d) molecular cement to combine DNA sites. hybridization with specific probe. 2. In genetic engineering, which of the following (b) Production of a group of genetically is used ? [2001] identical cells. (a) Plasmid (b) Plastid (c) Digestion of DNA by restriction enzyme. (c) Mitochondria (d) E.R. (d) Denaturation of DNA from a nucleated cell 3. Introduction of foreign gene for improving such as the one from the scene of crime. genotype is called [2002] 8. The polymerase chain reaction (PCR) (a) tissue culture technology was discovered by [2009] (b) vernalization (a) Karry Mullis (c) genetic engineering (b) Saiki et al (d) eugenics (c) Craig Venter 4. An example of gene therapy is [2004] (d) Maxam and Gilbert (a) production of injectable hepatitis B vaccine. 9. After 4 PCR cycles how many DNA molecules (b) production of vaccines in food crops like are formed from one DNA template molecule ? potatoes which can be eaten. [2012] (c) introduction of gene for adenosine (a)4 (b) 32 deaminase in persons suffering from Severe (c) 16 (d) 8 Combined Immuno Deficiency (SCID). 10. Human Genome Project (HGP) is closely (d) production of test tube babies by artificial associated with the rapid development of a new insemination and implantation of fertilized area in biology called as [2013] (a) biotechnology (b) bioinformatics eggs. (c) biogeography (d) bioscience 5. c-DNA probes are copied from the messenger 11. Identify the correct match for the given RNA molecules with the help of [2005] apparatus. [2013] (a) restriction enzymes (b) reverse transcriptase (c) DNA polyermase (d) adenosine deaminase 6. Electroporation procedure involves [2005] (a) fast passage of food through sieve pores in phloem elements with the help of electric stimulation. (b) opening of stomatal pores during night by artificial light. IIT JEE NEET eBooks: www.crackjee.xyz

Biotechnology: Principles and Processes B-179 Apparatus Function (a)‘Ori’ is a sequence responsible for (a) Gene gun Vectorless direct gene controlling the copy number of the linked transfer DNA. (b) Column Separation of (b) Selectable marker selectively permitting the chromatograph chlorophyll growth of the non-transformants. pigments (c) In order to link the alien DNA, the vector (c) Stirred tank Carry out fermentation needs to have single recognition site for bioreactor process the commonly used restriction enzymes. (d) Respirometer Finding out rate of (d) The ligation of alien DNA is carried out at respiration a restriction site present in one of the two 12. Genes of interest can be selected from a genomic antibiotic resistance genes. library by using [2014] TYPE B : ASSERTION REASON QUESTIONS (a) Restriction enzymes (b) Cloning vectors Directions for (Qs. 16-20) : These questions consist (c) DNA probes of two statements, each printed as Assertion and (d) Gene targets Reason. While answering these questions, you are 13. Choose the correct option. [2015] required to choose any one of the following five responses. Cla I Hind III A (a) If both Assertion and Reason are correct and Pvu I the Reason is a correct explanation of the Pst I B Assertion. C (b) If both Assertion and Reason are correct but tetR pBR 322 Sal I Reason is not a correct explanation of the Assertion.

D (c) If the Assertion is correct but Reason is rop incorrect. (d) If both the Assertion and Reason are incorrect. Pvu II (e) If the Assertion is incorrect but the Reason is ABCD correct. (a) Hind I EcoR I ampR ori 16. Assertion : Plasmids are extrachromosomal (b) Hind I BamH I kanR ampR DNA. (c) BamH I Pst I ori ampR Reason : Plasmids are found in bacteria and are (d) EcoR I BamH I ampR ori useful in genetic engineering. [2001] 14. Which one of the following palindromic base 17. Assertion: Plasmids are single-stranded extra sequences in DNA can be easily cut at about chromosomal DNA. the middle by some particular restriction Reason: Plasmids are usually present in enzyme? [2016] eukaryotic cells. [2002] (a) 5'...... CGTTCG...... 3' 18. Assertion: Clones are produced by sexual 3'...... ATGGTA...... 5' reproduction. (b) 5'...... GATATG...... 3' Reason: These are prepared by group of cells 3'...... CTACTA...... 5' descended from many cells or by inbreeding of (c) 5'...... GAATTC...... 3' a heterozygous line. [2002] 3'...... CTTAAG...... 5' 19. Assertion : In recombinant DNA technology (d) 5'...... CACGTA...... 3' human genes are often transferred into bacteria 3'...... CTCAGT...... 5' (prokaryotes) or yeast (eukaryote). 15. Which of the following statement is not correct Reason : Both bacteria and yeast multiply very about cloning vector ? [2017] fast to form huge population which expresses the desired gene. [2005] IIT JEE NEET eBooks: www.crackjee.xyz EBD_7100

B-180 Topicwise AIIMS Solved Papers – BIOLOGY 20. Assertion : Agrobacterium tumefaciens is 21. Assertion : Restriction enzymes cut the strand popular in genetic engineering because this of DNA to produce sticky ends. [2009] bacterium is associated with the roots of all Reason : Stickiness of the ends facilitates the cereal and pulse crops. action of the enzyme DNA polymerase. Reason : A gene incorporated in the bacterial 22. Assertion : “DNA finger printing” has become chromosomal genome-gets automatically a powerful tool to establish paternity and identity transferred to the crop with which the bacterium of criminals in rape and assault cases. [2010] is associated. [2005] Reason : Trace evidences such as hairs, saliva Directions for (Qs. 21-24) : Each of these questions and dried semen are adequate for DNA analysis. contains an Assertion followed by Reason. Read them 23. Assertion : In recombinant DNA technology, carefully and answer the question on the basis of human genes are often transferred into bacteria following options. You have to select the one that (prokaryotes) or yeast (eukaryote). . best describes the two statements. Reason : Both bacteria and yeast multiply very fast to form huge population, which express the (a) If both Assertion and Reason are correct and desired gene. [2008, 2015] Reason is the correct explanation of Assertion. 24. Assertion : Insertion of recombinant DNA (b) If both Assertion and Reason are correct, but within the coding sequence of E-galactosidase Reason is not the correct explanation of results in colourless colonies. Assertion. Reason : Presence of insert results in (c) If Assertion is correct but Reason is incorrect. inactivation of enzyme E-galactosidase known (d) If both the Assertion and Reason are incorrect. as insertional inactivation. [2017] IIT JEE NEET eBooks: www.crackjee.xyz

Biotechnology: Principles and Processes B-181

Type A : Multiple Choice Questions 8. (a) Developed in 1984 by Kary Mullis, PCR is 1. (c) Restriction endonuclease cleaves DNA now a common and often indispensable duplex at specific points in such a way that technique used in medical and biological single stranded free ends project from each research labs for a variety of applications. fragment of DNA duplex called sticky ends. These include DNA cloning for These sticky ends can join similar sequencing, DNA-based phylogeny, or complementary ends of DNA fragment functional analysis of genes; the diagnosis from some other source. of hereditary diseases; the identification 2. (a) Plasmid (extrachromosomal part in bacteria) of genetic fingerprints (used in forensic plays important role in recombinant DNA sciences and paternity testing); and the technology. detection and diagnosis of infectious 3. (c) The process of introduction of foreign gene diseases. In 1993, Mullis won the Nobel for obtaining the desirable trait is called Prize in Chemistry for his work on PCR. genetic engineering. 9. (c) From a single DNA template molecule, it is 4. (c) Gene therapy is a new system of medicine. possible to generate 2n DNA molecules Excellent example of gene therapy is SCID. after n number of cycles in polymerase chain They have defective gene for the enzyme reaction. adenosine deaminase (ADA). They lack T- lymphocytes which fail to combat the 10. (b) Human Genome Project (HGP) is closely infecting pathogen. Lymphocytes are associated with the rapid development of extracted from the patients bone marrow a new area in biology called Bioinformatics and normal functioning copy of gene which is used for storage and analysis of coding for ADA is introduced into these enormous amount of data. lymphocytes with the help of retrovirus. 11. (c) The cells so treated are reintroduced into 12. (c) A hybridization probe is a fragment of DNA patients bone marrow which reactivate patients immune system for life. of variable length which is used in DNA 5. (b) c DNA probes are copied from the mRNA samples to detect the presence of nucleotide molecules with the help of reverse sequence (the DNA target) that are transcriptase. complementary to the sequence in the 6. (c) Electroporation is the method of making probe. The probe hybridize to single– cell membrane permeable for the entry of stranded DNA whose base sequence allow recombinant DNA into the bacteria. probe target base-pairing due to 7. (c) The Southern blot is used to detect and complementary between the probe and identify certain DNA sequences in a sample target. of bodily fluid. It uses single-stranded DNA 13. (d) to search out their complementary strands. 14. (c) Palindromic sequences in DNA molecule When a Southern blot is performed on are group of bases that forms the same DNA, the first step is digestion of DNA sequence when read in both forward and with restriction enzymes. Restriction backward direction. In the given question, enzymes cut DNA at known sequences, and produces DNA fragments of a certain only option (c) represent a palindromic length. Once the DNA is cut into pieces, sequence. scientists conduct electrophoresis to 15. (b) Selectable marker selectively permitting the separate them by size. growth of the transformants. IIT JEE NEET eBooks: www.crackjee.xyz EBD_7100

B-182 Topicwise AIIMS Solved Papers – BIOLOGY Type B : Assertion Reason Questions hydrogen bonds with their complementary counter parts i.e., they can join similar 16. (b) Plasmids are the extrachromosomal part in complementary ends of DNA fragment from the bacteria and are useful in recombinant some other source with the help of DNA DNA technology. ligase. This stickness of the ends facilitates 17. (c) Plasmid is a extra chromosomal DNA the action of the enzyme DNA ligase, not present in prokaryotes e.g., bacteria. DNA polymerase. 18. (d) Cloning is shortcut method to amplify the 22. (a) DNA finger printing has become a powerful number of organisms with a desirable tool due to its ability in tracing evidences constitution. No sex is involved in their of crime and to establish the paternity. production. Members of clone are These evidences can be collected from genetically identical as they are derived hairs, semen and saliva. from single parent. 23. (a) Recombinant DNA is a form of synthetic 19. (a) In recombinant DNA technology, DNA that is engineered through the recombinants DNA are usually transferred combination or insertion of one or more to E. coli, yeast because of their rapid DNA strands, thereby combining DNA multiplication. This technology is sequences that would not normally occur employed for combining DNA from two together. In terms of genetic modification, different organisms to produce recombinant DNA is produced through the recombinant DNA. addition of relevant DNA into an existing 20. (d) A soil inhabiting, plant pathogenic bacteria organismal genome, such as the plasmid Agrobacterium tumefaciens infects broad of bacteria, to code for or alter different leaved crops including tomato, soyabean, traits for a specific purpose, such as sunflower and cotton but not the cereals. immunity. It differs from genetic Tumour formation (crown galls) is induced recombination, in that it does not occur by its plasmid (Ti) into the chromosomal through processes within the cell or DNA of its host plant. The T-DNA causes ribosome, but is exclusively engineered. tumors. As gene transfer occurs without Recombinant protein is protein that is human efforts the bacteria is known as derived from recombinant DNA. natural genetic engineer of plants. 24. (a) Alternative markers have been developed 21. (c) Restriction enzyme, a type of that can differentiate recombinants from endonuclease, functions by "inspecting" non-recombinants based upon their ability the length of a DNA sequence. Once it finds to produce colour in presence of a a recognition sequence, it binds and cut chromogenic substrate. The plasmid in the each of the two strands of the double helix bacteria, lacking an insert produces blue at specific point leaving single stranded coloured colonies, while those plasmids portions at the ends. This results in with an insert do not produce any colour overhanging stretches called sticky ends. due to insertional inactivation of the These are named so because they form enzyme, E-galactosidase. IIT JEE NEET eBooks: www.crackjee.xyz

34 Biotechnology & its Applications

TYPE A : MULTIPLE CHOICE QUESTIONS 6. The scientific process by which crop plants are 1. A tumour inducing plasmid widely used in the enriched with certain desirable nutrients is called production of transgenic plant is that of [2005] [2013] (a) Escherichia coli (a) crop protection (b) breeding (b) Bacillus thuringiensis (c) bio-fortification (d) bio-remediation 7. Which of the following is a variety of Brassica (c) Staphylococcus aureus resistance to white rust disease? [2014] (d) Agrobacterium tumefaciens (a) Himgiri 2. Which one of the following is a correct (b) Pusa Kamal statement? [2005] (c) Pusa Swarnim (Karan rai) (a) "Bt" in "Bt-cotton" indicates that it is a (d) Pusa Sadabahar genetically modified organism produced 8. The first clinical gene therapy was given in 1990 through biotechnology. to a 4 years old girl with enzyme deficiency of (b) Somatic hybridization involves fusion of [2014] two complete plant cells carrying desired genes. (a) Adenosine deaminase (ADA) (c) The anticoagulant 'hirudin' is being (b) Tyrosine oxidase produced from transgenic Brassica napus (c) Monamine oxidase seeds. (d) Glutamate dehydrogenase (d) "Flavr Savr" variety of tomato has 9. Select the correct statement(s)- [2015] enhanced the production of ethylene (1) IARI has released a mustard variety rich in which improves its taste. vitamin C. 3. Cultivation of Bt cotton has been much in the news. The prefix Bt means [2008] (2) Pusa Sawani variety of Okra is resistant to (a) barium-treated cotton seeds aphids. (b) bigger thread variety of cotton with better (3) Hairiness of leaves provides resistance to tensile strength insect pests. (c) produced by biotechnology using (4) Agriculture accounts for approximately restriction enzymes and ligases 33% of India's GDP and employs nearly 62% (d) carrying an endotoxin gene from Bacillus of the population. thuringiensis (a) (1) and (2) (b) (2) and (3) 4. Golden rice is a transgenic crop of the future (c) (1), (3) and (4) (d) None of these with the following improved trait [2012] 10. Read the following four statements (1-4) about certain mistakes in two of them (a) High lysine (essential amino acid) content 1. The first transgenic buffalo, Rosie (b) Insect resistance produced milk which was human alpha- (c) High protein content lactal albumin enriched. (d) High vitamin-A content 2. Restriction enzymes are used in isolation 5. How many varieties of rice has been estimated of DNA from other macro-molecules. to be present in India? [2013] 3. Downstream processing is one of the steps (a) 2,000 (b) 20,000 of R-DNA technology. (c) 200,000 (d) 2,000,000 IIT JEE NEET eBooks: www.crackjee.xyz EBD_7100

B-184 Topicwise AIIMS Solved Papers – BIOLOGY

4. Disarmed pathogen vectors are also used TYPE B : ASSERTION REASON QUESTIONS in transfer of R-DNA into the host. Which are the two statements having mistakes? Directions for (Q. 13) : Each of these questions [2015] contains an Assertion followed by Reason. Read them (a) Statement 2 and 3 (b) Statement 3 and 4 carefully and answer the question on the basis of following options. You have to select the one that (c) Statement 1 and 3 (d) Statement 1 and 2 best describes the two statements. 11. A transgenic food crop which may help in solving the problem of night blindness in developing (a) If both Assertion and Reason are correct and Reason is the correct explanation of Assertion. countries is [2016] (b) If both Assertion and Reason are correct, but (a) golden rice (b) Bt soyabean Reason is not the correct explanation of (c) flavr - savr tomato (d) starlink maize Assertion. 12. Which variety of rice was patented by a U.S. (c) If Assertion is correct but Reason is incorrect. company even though the highest number of (d) If both the Assertion and Reason are incorrect. varieties of this rice is found in India ? [2017] 13. Assertion : Insect resistant transgenic cotton (a) Sharbati Sonara (b) Co-667 has been produced by inserting Bt gene. [2010] (c) Basmati (d) Lerma Roja Reason : The Bt gene is derived from a bacterium. IIT JEE NEET eBooks: www.crackjee.xyz

Biotechnology & its Applications B-185

Type A : Multiple Choice Questions deficiency or ADA-SCID is an autosomal 1. (d) A tumour (crown gall) inducing plasmid recessive metabolic disorder that causes widely used in the production of transgenic immunodeficiency. ADA deficiency is due plant is Agrobacterium tumefaciens. to a lack of the enzyme adenosine 2. (c) Hirudin is a protein that stops blood deaminase. clotting. The gene encoding hirudin was 9. (c) chemically synthesized. This gene was then 10. (d) Transgenic Rosie is actually cow. transferred into Brassica napus where Restriction enzymes cut the DNA at hirudin accumulates in seeds. This hirudin specific sites. is purified & used as a medicine. 11. (a) A transgenic food crop which may help in 3. (d) Bacillus thuringiensis, or Bt, is a bacterium solving the problem of night blindness in that occurs naturally in the soil, produces developing countries is golden rice. Golden a protein that is toxic to certain insect pests, rice is genetically modified rice that has and is widely used as a pest control agent. been engineered to have elevated levels of It is also extremely host-specific. beta carotene in it. It is a pre cursor of 4. (d) Golden rice is a variety of rice produced vitamin A, which gives it a characteristic through genetic engineering to golden colour. biosynthesize beta-carotene, a precusor of 12. (c) Basmati rice was patented by a US pro-vitamin A. company even though the highest number 5. (c) 6. (c) 7. (c) of varieties of this rice is found in India. 8. (a) Gene therapy is an experimental technique that uses genes to treat or prevent disease. Type B : Assertion Reason Questions The first clinical gene therapy was given 13. (b) Bt-cotton is a transgenic crop. Transgenic for treating adenosine deaminase plants are those plants, which have foreign deficiency. A four-year old girl became the gene incorporated in their DNA. This insect first gene therapy patient on September 14, resistant gene is derived from a bacterium, 1990 at the NIH Clinical Center. Adenosine Bacillus thuringiensis. deaminase deficiency, also called ADA IIT JEE NEET eBooks: www.crackjee.xyz EBD_7100

B-186 Topicwise AIIMS Solved Papers – BIOLOGY 35 Organisms and Populations

TYPE A : MULTIPLE CHOICE QUESTIONS 8. Which one of the following is a matching pair of certain organism(s) and the kind of association? 1. Territoriality occurs as a result of [1998] [2003] (a) competition (b) parasitism (a) Shark and sucker fish - Commensalism (c) predation (d) co-operation (b) Algae and fungi in lichens - Mutualism 2. Obligate parasites live on [1999] (c) Orchids growing on trees - Parasitism (d) Cuscuta (dodder) growing on other - (a) living host only flowering plants - Epiphytism (b) living host and dead organic matter 9. The great barrier reef along the east coast of (c) dead organic matter only Australia can be categorised as [2004] (d) artificial liquid medium (a) population (b) community 3. Mycorrhiza help in absorption of [1999] (c) ecosystem (d) biome (a) calcium (b) nutrients 10. Which one of the following correctly represents (c) metals (d) none of these an organism and its ecological niche ? [2005] 4. The plants which can withstand narrow range (a) Vallisneria and pond of temperature tolerance are called [2000] (b) Desert locust (Schistocerca) and desert (a) stenothermal (b) eurythermal (c) Plant lice (aphids) and leaf (d) Vultures and dense forest (c) mesothermal (d) monothermal 11. Keystone species deserve protection because 5. Abundance of a species in a population, within these [2006] habitat is called [2001] (a) are capable of surviving in harsh environ- (a) niche density mental conditions. (b) absolute density (b) indicate presence of certain minerals in the (c) relative density soil. (d) geographic density (c) have become rare due to . 6. The maintenance of internal favourable conditions, (d) play an important role in supporting other by a self regulated mechanisms inspite of the fact species. that there are changes in environment, is known 12. A lizard-like member of reptila is sitting on a tree as [2001] with its tail coiled around a twig. This animal (a) entropy (b) enthalpy could be [2006] (c) homoeostasis (d) steady state (a) Hemidactylus showing sexual dimorphism 7. July 11 is observed as [2003] (b) Varanus showing mimicry (a) World Population Day (c) Garden lizard (Calotes) showing (b) No Tobacco Day camouflage (c) World Environment Day (d) Chamaeleon showing protective (d) World Health Day colouration IIT JEE NEET eBooks: www.crackjee.xyz

Organisms and Populations B-187

13. Carrying capacity is [2009] 20. The figure given below is a diagrammatic (a) the capacity of an individual to produce representation of response of organisms to young ones. abiotic factors. What do A, B and C represent (b) availability of resources in a given habitat respectively? [2015] to support a certain no of individuals of population, beyond which no further growth is possible. (c) gene frequency from one generation to next. (d) gene frequency in same generation. 14. Within biological communities, some species are important in determining the ability of a large number of other species to persist in the AB C community. Such species are called [2010] (a) conformer regulator partial regulator (a) keystone species (b) regulator partial conformer (b) allopatric species regulator (c) sympatric species (c) partial regulator conformer (d) threatened species regulator 15. Presence of flagellated protozoans in the gut of (d) regulator conformer partial regulator termites are the example [2012] 21. The salinity in sea water in parts per thousand (a) Symbiosis (b) Parasitism (ppt) ranges between [2016] (c) Antibiosis (d) Commensalism (a) 5-15% (b) 30-35% 16. The formula for exponential population growth (c) 50-75% (d) more than 100% is [2013] 22. In Urn shaped age pyramid of the population (a) dN/rN = dt (b) rN / dN = dt the trend of growth is [2017] (c) dN / dt = rN (d) dt / dN = rN (a) Rapid (b) Stable (c) Declining (d) Stationary 17. The rate of formation of new organic matter by rabbit in a grassland, is called [2014] TYPE B : ASSERTION REASON QUESTIONS (a) Net productivity Directions for (Qs. 23-25) : These questions consist (b) Secondary productivity of two statements, each printed as Assertion and (c) Net primary productivity Reason. While answering these questions, you are (d) Gross primary productivity required to choose any one of the following five 18. If 4 individuals in a laboratory population of 40 responses. fruitflies died during a specified time interval (i.e., (a) If both Assertion and Reason are correct and a week), the death rate in the population during the Reason is a correct explanation of the that period is [2014] Assertion. (a)1 (b) 0.1 (b) If both Assertion and Reason are correct but (c) 0.01 (d) 0.4 Reason is not a correct explanation of the 19. A population of 500 that experiences 55 births Assertion. and 5 deaths during a one-year period. What is (c) If the Assertion is correct but Reason is the reproductive rate for the population during incorrect. the one-year period ? [2015] (d) If both the Assertion and Reason are incorrect. (a) 0.01/year (b) 0.05/year (e) If the Assertion is incorrect but the Reason is (c) 0.1/year (d) 50/year correct. IIT JEE NEET eBooks: www.crackjee.xyz EBD_7100

B-188 Topicwise AIIMS Solved Papers – BIOLOGY

23. Assertion : Leaf butterfly and stick insect show 26. Assertion : Thick cuticle is mostly present in mimicry to dodge their enemies. disease resistant plants. Reason : Mimicry is a method to acquire body Reason : Disease causing agents cannot grow colour blending with the surroundings. [2003] on cuticle and cannot invade the cuticle.[2012] 24. Assertion : Animals adopt different strategies 27. Assertion : In sigmoid growth curve, population to survive in hostile environment. finally stabilizes itself. Reason : Praying mantis is green in colour which Reason : Finally, the death rate increases than merges with plant foliage. [2004] the birth rate. [2013] 25. Assertion : The sex ratio of Kerala is highest in 28. Assertion : Tropical rain forests are India. disappearing fast from developing countries Reason : In countries like India the population such as India. is increasing at a rapid rate. [2005] Reason : No value is attached to these forests Directions for (Qs. 26-30) : Each of these questions because these are poor in biodiversity. contains an Assertion followed by Reason. Read them [2015] carefully and answer the question on the basis of 29. Assertion : Flora contains the actual account following options. You have to select the one that of habitat and distribution of plants of a given best describes the two statements. area. (a) If both Assertion and Reason are correct and Reason : Flora helps in correct identification. Reason is the correct explanation of Assertion. [2016] (b) If both Assertion and Reason are correct, but 30. Assertion : Species are groups of potentially Reason is not the correct explanation of interbreeding natural populations which are Assertion. isolated from other such groups. (c) If Assertion is correct but Reason is incorrect. Reason : Distinctive morphological characters (d) If both the Assertion and Reason are incorrect. are displayed due to reproductive isolation. [2017] IIT JEE NEET eBooks: www.crackjee.xyz

Organisms and Populations B-189

Type A : Multiple Choice Questions 9. (c) Coral reefs occur in clear, shallow, warm water where temperature rarely falls below 1. (a) Competition exists between individuals of 20°C and there is enough light for the same or different species for food, light, photosynthesis. space, shelter and mate. Establishment of 10. (c) Ecological niche is the status of an territories by animals is meant for ensuring organism within its environment and availability of their needs. community (affecting its survival as a 2. (a) Obligate parasites lead only parasitic life species). and are host specific. 11. (d) Keystone species has disproportionate 3. (b) The roots of higher plants possess effects on its environment relative to its symbiotic mycorrhizae. It is a symbiotic, biomass. Such species play a critical role non-pathogenic association between the in maintaining the structure of an ecological roots and soil fungi. Several conifers lack community and help to determine the types root hair and here the mycorrhizae perform and numbers of various other species in this function. The mycorrhizae are of the community. two types : ectomycorrhizae and endo- 12. (d) Chamaeleon (girgit) shows protective mycorrhizae with an intermediate category colouration with its surrounding e.g. twig. as ectondomycorrhizae. The most common 13. (b) Carrying capacity is the maximum number endomycorrhizae are Vesicular Arbuscular of individuals of population that can be Mycorrhizae (VAM), generally present in sustained by available resources in a given herbaceous plants. Mycorrhizae perform habitat beyond which there is no further the function of root hair, i.e. they absorbs growth. When population reaches the essential ions. carrying capacity then mortality < natality. 4. (a) The climatic conditions in the stenothermal 14. (a) Within biological communities, some plants have made them to get adapted to a species may be important in determining small variations in temperature. the ability of large number of other species 5. (a) Density of a population is the number of to persist in the community. These crucial individual species in a given area. Niche is species are known as key stone species. a suitable habitat of a species. Niche Allopatric species are species having density is the number of a particular species exclusive areas of geographic distribution. in a given area or suitable habitat. Sympatric species are species having 6. (c) For metabolic processes to continue, the overlapping area of geographical living things need to remain in a steady distribution. state maintained by self regulatory 15. (a) Presence of flagellated protozoans in the mechanism called homeostasis. gut of termites are the example of 7. (a) July 11 has been observed as World symbiosis. In this type of association both Population Day. partners are mutally benefited from each 8. (b) Algae and fungi in lichens show mutualism. other. Termites are dependent on protozoa Fungi provide fixation, water, minerals and for breaking down their food stuff and the shelter to the algae. The algae manufactures protozoa are dependent on the termites as food for itself and for fungi. host organisms. IIT JEE NEET eBooks: www.crackjee.xyz EBD_7100

B-190 Topicwise AIIMS Solved Papers – BIOLOGY 16. (c) The formula of exponential growth is Type B : Assertion Reason Questions dN dN rN where is the rate of change 23. (a) Leaf butterfly is green in colour and stick dt dt insect also mimics in order to escape from in population size, r is the biotic potential the enemies and also to catch prey. The and N is the population size. camouflaging mechanism helps the 17. (b) At the trophic level of consumers the rate organism to get adapted to its at which food energy is assimilated is called surroundings. The Leaf butterfly is seen secondary productivity. Rabbit is a more near the leaves while the stick insect camouflages with its surroundings by consumer. living on the branches. 18. (b) Mortality or Death rate refers to the death 24. (a) Animals blend with the surroundings or of individuals in a population. back ground to remain unnoticed for protection and aggression. No. of deaths Death rate 25. (b) Kerala has a sin ratio of 1058 Indian Total population population is growing with 1.2% animal change. 4 0.1 individuals per fruitfly 26. (a) Disease resistant plants possess thick 40 per week cuticle. Infectious organisms can not grow 19. (c) The rate of growth, r, equals (55 births – 5 or invade cuticle. deaths)/ 500 per year, or 0.1/year. 27. (c) In sigmoid growth curve, finally, growth 20. (d) In the graph, the line A represents regulator, rate becomes stable because mortality and line B represents conformer and line C natality rates become equal to each other represents partial regulator. Organisms that and finally the population shows zero are able to maintain homeostasis by growth rate as birth rate equals death rate. physiological means that ensures constant 28. (c) Tropical rain forests have disappeared body temperature are called regulators. mainly due to man’s activities. Due to over Organism that are not able to maintain a population in countries like India, rain constant internal temperature are called forests are cut to make place available for man to live and build houses. To build conformers. buildings and factories man has Partial regulators are organisms that have incessantly cut down trees. This has the ability to regulate, but only over a caused the depletion of rain forests. limited range of environmental conditions, 29. (b) Flora contains the actual account of habitat beyond which they simply conform. and distribution of plants of a given area. 21. (b) Sea water typically has a salinity of around It provides the index to the plant species 35 g/kg although lower values are typical found in particular area. near coasts where river enter the ocean. Rivers and lakes can have a wide range of 30. (b) A group of individuals resembling each salinities, from less than 0-0.1 g/kg to a few other in morphological, physiological, g/kg, although there are many places where biochemical and behavioural characters even higher salinities are found. The dead constitute a species. Such individuals can sea has a salinity of more than 200 g/kg. breed among themselves but cannot breed 22. (c) In Urn Shaped pyramid the individuals with members other than their own to below the reproductive age are fewer in produce fertile offsprings. New species are number than the individuals of formed mainly due to reproductive reproductive age. isolation. IIT JEE NEET eBooks: www.crackjee.xyz

36 Ecosystem

TYPE A : MULTIPLE CHOICE QUESTIONS 9. Mr. X is eating curd/yoghurt. For this food intake in a food chain Mr. X should be 1. The food chain in which microbes breakdown considered as occupying [2003] energy rich compounds synthesized by (a) first trophic level producers is called [1999] (b) second trophic level (a) ecosystem (c) third trophic level (b) parasitic food chain (c) detritus level chain (d) fourth trophic level (d) predator food chain 10. Given below is one of the types of ecological 2. 10 % law of energy transfer was given by pyramids. This type represents [2005] (a) Lindemann (b) Tansley [2000] (c) Stanley (d) Darwin 3. Food chain starts with [2000] (a) autotrophs (b) (c) carnivores (d) decomposers 4. Flora and fauna in lake or ponds is [2000] (a) lentic biota (b) lotic biota (a) pyramid of numbers in a grassland (c) abiotic biota (d) field layer (b) pyramid of biomass in a fallow land 5. The enzyme responsible for the reduction of (c) pyramid of biomass in a lake molecular nitrogen to the level of ammonia in (d) energy pyramid in a spring the leguminous root nodule is [2000] 11. The function of leghaemoglobin during (a) nitrogenase (b) nitrate reductase biological nitrogen fixation in root nodules of (c) nitrite reductase (d) ammoneases legumes is to [2006] 6. The role of bacteria in carbon cycle is [2000] (a) photosynthesis (a) convert atmospheric N2 to NH3 (b) chemosynthesis (b) convert ammonia to nitrite (c) decomposition of organic compounds (c) transport oxygen for activity of nitrogenase

(d) evolution of O2 (d) protect nitrogenase from oxygen 7. Trophic levels are formed by : [2001] 12. An ecosystem, such as an aquarium, is self- (a) plants sustaining if it involves the interaction between (b) animals organisms, a flow of energy, and the presence (c) organisms linked in food chain of [2009] (d) carnivores (a) equal numbers of plants and animals 8. Desert can be converted into green land by planting [2001] (b) more animals than plants (a) oxylophytes (b) psammophytes (c) materials cycles (c) halophytes (d) trees (d) pioneer organisms IIT JEE NEET eBooks: www.crackjee.xyz EBD_7100

B-192 Topicwise AIIMS Solved Papers – BIOLOGY

13. The graph below shows the changes in two (c) the desert insects eat away flowers during populations of herbivores in a grassy field. A day time. possible reason for these changes is that [2009] (d) the desert insects are active during night time. 18. Whale is [2012] (a) Primary producer (b) Carnivorous, secondary consumer (c) A decomposer (d) Herbivorous (a) all of the plant populations in this habitat 19. Which one of the following is not a function of decreased. an ecosystem? [2013] (b) population B competed more successfully (a) Energy flow (b) Decomposition for food than population A did. (c) Productivity (d) Stratification (c) population A produced more offspring 20. How much portion of the Photosynthetically than population B did. Active Radiation (PAR) is captured by the (d) population A consumed the members of plants? [2016] population B. (a) 5 – 10% (b) 7 – 10% 14. A scorpion stalks, kills, and then eats a spider. (c) 8 – 10% (d) 2 – 10% Based on its behavior, which ecological terms 21. Arrange the following ecosystems in increasing describe the scorpion? [2009] order of mean NPP (Tonnes / ha / year) (a) producer, , decomposer A. Tropical deciduous forest (b) producer, carnivore, heterotroph B. Temperate coniferous forest (c) predator, carnivore, consumer C. Tropical rain forest (d) predator, autotroph, herbivore D. Temperate deciduous forest [2017] 15. In the vast marine ecosystem, certain sea (a) B < A < D < C develop red colouration. This red colour is due (b) D < B < A < C to the presence of large population of which (c) A < C < D < B one of the following organisms? [2009] (d) B < D < A < C (a) Trichodesmium erythrium TYPE B : ASSERTION REASON QUESTIONS (b) Physarium (c) Dinoflagellates Directions for (Q. 22) : These questions consist of (d) Diatoms and members of red algae two statements, each printed as Assertion and Reason. 16. The xerophytic plants conserve water by storing While answering these questions, you are required to it in [2009] choose any one of the following five responses. (a) intercellular spaces (a) If both Assertion and Reason are correct and (b) normal parenchymatous cells the Reason is a correct explanation of the (c) intercellular spaces and parenchymatous Assertion. cells (b) If both Assertion and Reason are correct but (d) parenchymatous cells specialized for this Reason is not a correct explanation of the purpose Assertion. 17. Most of the desert plants bloom during night (c) If the Assertion is correct but Reason is time because [2010] incorrect. (a) their blooming is controlled by low (d) If both the Assertion and Reason are incorrect. temperature. (e) If the Assertion is incorrect but the Reason is (b) they are sensitive to the phases of moon. correct. IIT JEE NEET eBooks: www.crackjee.xyz

Ecosystem B-193

22. Assertion: Insectivorous habitat of plants is to 24. Assertion : Pyramid of energy may be upright or inverted. [2011] cope up O2 deficiency. Reason: Only 20% of energy goes to next trophic Reason: Insectivorous plants are partly level. autotrophic and partly heterotrophic. [1998] 25. Assertion : Biotic community has higher Directions for (Qs. 23-28) : Each of these questions position than population in ecological hierarchy. contains an Assertion followed by Reason. Read them Reason : Population of similar individuals carefully and answer the question on the basis of remains isolated in the community. [2012] following options. You have to select the one that 26. Assertion : Net primary productivity is gross best describes the two statements. primary productivity minus respiration. (a) If both Assertion and Reason are correct and Reason : Secondary productivity is produced by heterotrophs. [2013] Reason is the correct explanation of Assertion. 27. Assertion : Net primary productivity is gross (b) If both Assertion and Reason are correct, but primary productivity minus respiration. Reason is not the correct explanation of Reason : Secondary productivity is produced Assertion. by heterotrophs. [2016] (c) If Assertion is correct but Reason is incorrect. 28. Assertion : In a food chain, members of (d) If both the Assertion and Reason are incorrect. successive higher levels are fewer in number. 23. Assertion : A network of food chains existing Reason : Number of organisms at any trophic together in an ecosystem is known as food web. level depends upon the availability of organisms Reason : An animal like kite cannot be a part of which serve as food at the lower level. a food web. [2006, 2008, 2011] [2003, 2017] IIT JEE NEET eBooks: www.crackjee.xyz EBD_7100

B-194 Topicwise AIIMS Solved Papers – BIOLOGY

Type A : Multiple Choice Questions second trophic level and Mr. X will occupy third place (curd eater). 1. (c) In nature, detritus food chains are 10. (c) The given figure shows the pyramid of indispensable as the dead organic matter biomass in a lake. An ecological pyramid of grazing food chains is acted upon by of biomass shows the relationship between the detritivores (bacteria, protozoa, biomass and trophic level by quantifying nematodes) to recycle the inorganic the amount of biomass present at each elements into the ecosystem. trophic level of an ecological community 2. (a) 10 % law of energy transfer (pyramid of at a particular moment in time. energy) was given by Lindemann. 11. (d) Leghaemoglobin is an oxygen scavanger. In this, only 10% of total energy received It combines with oxygen and protects by one trophic level is transferred to next nitrogenase which catalyses the fixation of trophic level. nitrogen under anerobic conditions. 3. (a) All trophic levels in an ecosystem are 12. (c) If a ecosystem is to be self-sustaining, connected by transfer of food as energy. materials such as oxygen, carbon dioxide, The transfer of food and its contained water and nitrogen must to recycle energy from one trophic level to the next between the organisms. trophic level is called food chain. 13. (b) If population B increased while population Food chain always starts with producers A decreased, these organisms were (autotrophs) o Herbivores o probably in competition for the same food Carnivorous o Detrivores are placed at (grass) and population B was better adapted. the top of the food chain. Hence, population A is competitively being 4. (a) Lentic relates to still waters such as lakes excluded from the population. and ponds. Hence, the flora and fauna 14. (c) Because the scorpion stalks, kills and eats constitue the lentic biota. its food, it is a predator. Because it eats a 5. (d) The enzyme responsible for the reduction spider it is a carnivore. Because it ingests of nitrogen to ammonia is ammoneases and food it is a consumer. A producer is an the process is ammonification. e.g. autotroph as it is an organism that makes actinomycetes, Bacillus ramosus, B. its own food from inorganic substances. A vulgaris etc. decomposer breaks down dead matter and 6. (c) The excretory wastes of living organisms a herbivore eats only plants. have accumulated carbon compounds and 15. (a) Trichodesmium erythreum is a they are decomposed after their death by cyanobacteria (blue green alga). Although a blue green alga, it possesses a pigment, micro-organisms in the soil to release CO2 back into the environment for its recycling. phycoerythrin, which is red in colour and 7. (c) The producers and consumers in imparts red colour to the water of the sea in ecosystem are arranged into several which it is found, hence named Red Sea. feeding groups/levels called trophic levels. 16. (d) The xerophytic plants conserve water by 8. (b) Psammophytes are those plants that can storing it in parenchymatous cells grow in desert and mainly in sandy soil. specialized for this purpose. Xerophytes Hence, psammophytes can be used to plants are specially adapted to succeed in convert desert into a green land. an arid climate. They are typically able to withstand long periods of drought and the 9. (c) Producers occupy first trophic level, drying effects of desert winds. Some plants primary consumers i.e. herbivores (cow have adapted to arid lands by developing produce milk formating curd) are placed at the ability to store water. IIT JEE NEET eBooks: www.crackjee.xyz

Ecosystem B-195

17. (d) In desert condition, most of the activity of 24. (d) Energy flow in the ecosystem is a the plants and animals happens during night unidirectional manner. There is a decline in because of very high temperature in day the amount of energy passing from one time. As a result the desert insects make trophic level to the next. Thus the pyramid themselves active and pollinate the flowers of energy is always upright. According to at night. To attract the insects, most of the Lindemann, only 10% of energy goes to desert plants bloom during night. next trophic level. 18. (b) Whale is carnivorous and feeds on primary consumer and occupies the third trophic 25. (c) The organisms of all the species that live level of the ecosystem. in a particular area and interact in various 19. (d) Four important functional aspects of the ways with one another form biotic ecosystem are community. Biotic community is a grouping (i) Productivity that is higher than population in ecological (ii) Decomposition, hierarchy. It is an assemblage of all the (iii) Energy flow and populations of different organisms (iv) Nutrient cycling. occurring in an area. The different 20. (d) The main source of energy for an populations of a community do not remain ecosystem is the radiant energy or light isolated. They show interactions and inter- energy derived from the sun. 50% of the dependence. total solar radiation that falls on earth is 26. (b) Net primary productivity is the rate of Photosynthetically Active Radiation organic matter built up or stored by (PAR). producers in their bodies per unit time and The light energy is converted into area. Net productivity is equal to gross chemical energy in the form of sugar by primary productivity minus loss due to photosynthesis. respiration and other reasons. Rate of 6H2O + 6CO2 + Light o 6C6H12O6 + 6O2 increase in energy containing organic Plants utilize 2-10% of PAR in matter or biomass by heterotrophs or photosynthesis. consumers per unit time and area is known 21. (d) Net primary productivity (NPP) is the as secondary productivity. biomass or storage of energy by green plants. It is equal to the gross primary 27. (b) Net primary productivity is the rate of productivity minus loss due to respiration. organic matter build up or stored by The productivity generally increases from producers in their bodies per unit time and polar regions toward the tropics, because area. Net productivity is equal to gross of the increasing sunlight and temperature. primary productivity minus loss due to Type B : Assertion Reason Questions respiration and other reasons. Rate of increase in energy containing organic 22. (e) Insectivorous plants are those plants which matter or biomass by heterotrophs or capture and digest live prey (normally consumers per unit time and area is known insects) to obtain nitrogen compounds that as secondary productivity. are lacking in its usual marshy habitat. 28. (d) When food is made available, automatically These plants are partly autotrophic and the next higher level of organism in the partly heterotrophic. hierarchy should increase. This is because 23. (c) In the food web, different food chains are when the forest cover got depleted it led to interconnected. Each chain is interconnected and consists of different the increase in the number of endangered trophic levels i.e. producers, consumers and species. If the deer population is more, it detrivorous. So, kite can also be a part of automatically leads to an increase in the food web. tiger population. IIT JEE NEET eBooks: www.crackjee.xyz EBD_7100

B-196 Topicwise AIIMS Solved Papers – BIOLOGY 37 Biodiversity and its Conservation

6. Which one of the following is correct matching TYPE A : MULTIPLE CHOICE QUESTIONS of a plant, its habit and forest type where it 1. Heavy rainfall during summer produces [1998] normally occurs? [2005] (a) Prosopis, tree, scrub (a) desert (b) grassland (b) Saccharum officinarum, grass, forest (c) forest (d) wetland (c) Shorea robusta, herb, tropical rain forest 2. The trees occurring in two seasons is the (d) Acacia catechu, tree, coniferous forest characteristic feature of [1998] 7. One of the ex-situ conservation methods for (a) temperate deciduous forest endangered species is [2005] (b) tropical savannah (a) wildlife sanctuaries (c) grassland (b) biosphere reserves (d) coniferous forest (c) cryopreservation 3. The map given below indicates the former and (d) national parks 8. Genetic diversity in agricultural crops is the present distribution of an animal. [2003] threatened by [2005] (a) introduction of high yielding varieties. (b) intensive use of fertilizers. (c) extensive intercropping. (d) intensive use of biopesticides. 9. The Montreal protocol refers to [2006] (a) persistent organic pollutants (b) global warming and climate change (c) substances that deplete the ozone layer (d) biosafety of genetically modified organisms Former distribution 10. Biosphere reserves differ from National parks Present distribution and Wildlife sanctuaries because in the former [2006] Which animal could it be? (a) human beings are not allowed to enter. (a) Wild ass (b) Nilgai (b) people are an integral part of the system. (c) Black buck (d) Lion (c) plants are paid greater attention than the 4. If the high altitude birds become rare or extinct, animals. the plants which may disappear along with them (d) living organisms are brought from all over are [2004] the world and preserved for posterity. (a) pine (b) oak 11. Which part of the world has a high density of (c) orchids (d) Rhododendrons organism? [2007] 5. Which one of the following is a pair of endangered species? [2004] (a) Deciduous forests (a) Garden lizard and Mexican poppy (b) Grasslands (b) Rhesus monkey and sal tree (c) Tropical rain forests (c) Indian peacock and carrot grass (d) Savannahs (d) Hornbill and Indian aconite IIT JEE NEET eBooks: www.crackjee.xyz

Biodiversity and its Conservation B-197

12. Beta diversity is diversity [2007] 200 (a) in a community 100 (b) between communities 50 (c) in a mountain gradient 30 (d) on a plain 20 13. Which one of the following pairs of Slope geographical areas show maximum biodiversity 10 5 0.15 in our country ? [2008] S = 1.89 (a) Sunderbans and Rann of Kutch S : Number of bird species 2 (b) Eastern Ghats and West Bengal 0.001 0.01 0.1110 100 1000 10,000 100,000 (c) Eastern Himalaya and Western Ghats A : Island land area (km2) [2016] (d) Kerala and Punjab. (a) 17 percent of the bird species will be lost. 14. A tree species in Mauritus failed to reproduce because of the extinction of a fruit-eating bird. (b) 20 percent of the bird species will be lost. Which one of the following was that bird? (c) All of bird species will be lost. (a) Dove (b) Dodo [2010] (d) 93 percent of the bird species will be lost. (c) Condor (d) Skua TYPE B : ASSERTION REASON QUESTIONS 15. Tectonic is the study of [2011] Directions for (Q. 20) : These questions consist of (a) volcanos (b) earth’s crust two statements, each printed as Assertion and Reason. (c) sand dunes (d) Sun While answering these questions, you are required to 16. If the Bengal tiger becomes extinct choose any one of the following five responses. [2004, 2012] (a) If both Assertion and Reason are correct and (a) Hyenas and wolves will become scare the Reason is a correct explanation of the (b) The wild area will be safe for man and Assertion. domestic animals (b) If both Assertion and Reason are correct but (c) Its gene pool will be lost for ever Reason is not a correct explanation of the (d) The population of beautiful animals like Assertion. deers will be stabilized. 17. Which of the following is considered a hot-spot (c) If the Assertion is correct but Reason is of biodiversity in India ? [2013] incorrect. (a) Indo-Gangetic Plain (d) If both the Assertion and Reason are incorrect. (b) Eastern Ghats (e) If the Assertion is incorrect but the Reason is (c) Aravalli Hills correct. (d) Western Ghats 20. Assertion : In tropical rain forests. O-horizon 18. The largest Tiger reserve in India is [2014] and A-Horizon of soil profile are shallow and (a) Nagarhole nutrient-poor. (b) Valmiki Reason : Excessive growth of micro-organisms (c) Nagarjunsagar-Srisailam in the soil depletes its organic content.[2006] (d) Periyar Directions for (Qs. 21-24) : Each of these questions 19. Using the figure, determine the percentage of contains an Assertion followed by Reason. Read them bird species that will be lost if the island's carefully and answer the question on the basis of inhabitable land area is reduced from 100,000 following options. You have to select the one that km2 to 1 km2. best describes the two statements. IIT JEE NEET eBooks: www.crackjee.xyz EBD_7100

B-198 Topicwise AIIMS Solved Papers – BIOLOGY

(a) If both Assertion and Reason are correct and 22. Assertion : Diversity observed in the entire Reason is the correct explanation of Assertion. geographical area is called gamma diversity. (b) If both Assertion and Reason are correct, but Reason : Biodiversity decreases from high Reason is not the correct explanation of altitude to low altitude. [2014] Assertion. 23. Assertion : A sanctuary is formed for the (c) If Assertion is correct but Reason is incorrect. conservation of animals only. (d) If both the Assertion and Reason are incorrect. Reason : Restricted human activities are allowed 21. Assertion : Tropical rain forests are in sanctuaries. [2014] disappearing fast from developing countries such as India. 24. Assertion: Communities that comprise of more Reason : No value is attached to these forests species tend to be more stable. because these are poor in biodiversity. Reason: A higher number of species results in [2012, 2013] less animal variation in total biomass. [2017] IIT JEE NEET eBooks: www.crackjee.xyz

Biodiversity and its Conservation B-199

Type A : Multiple Choice Questions 10. (b) Biosphere reserves are a special category of protected areas of land or coastal 1. (b) During summer the land becomes dry and environments where people are an integral rainfall induces formation of grass and component of the system. hence, heavy rainfall during summer 11. (c) Tropical rain forests have a high density produces grasslands. of organisms. Tropical rain forests are 2. (a) In the deciduous forests, the leaves mainly found in America, South America, generally fall off during autumn. These Cargo river basin of Africa, South East Asia. trees are seen in temperate lands. In this biome, rainfall and warmth are 3. (a) This animal which had a wide distribution abundant. Plant growth is luxuriant. This has now considerably decreased in biome possess more than half of the flora number. This is due to the depletion of and fauna of the world. Productivity is forest areas due to man’s intrusion. This very high. Diversity of life is so high that a area has now become a protected area and hectare of the forest may have 200 species hence, this animal is now considered an of trees. 70–80% of all insects and 80 – endangered species. 85% of all birds are known from tropical 4. (b) The oak plant may disappear since they forests. are seen at a high altitude and their 12. (b) D, E, J are the three types of ecosystem dispersal is dependant on these birds. diversities where diversity is the diversity 5. (d) Endangered species are species with low operating between communities. population numbers that are in 13. (c) The Eastern Himalayas Region includes considerable danger of becoming extinct Bhutan, north-eastern India, and southern, Hornbill and Indian Aconite are endagered central, and eastern Nepal. It is home to species. 163 globally threatened species, including 6. (a) Prosopis fuliflora is a shrub or small tree Asia’s three largest herbivores—the Asian native to Mexico, South America and the elephant (Elephas maximus), the greater Caribbean. Its uses include forage, wood one-horned rhinoceros (Rhinoceros and environmental management. unicornis), and the wild water buffalo (Bubalus bubalis)—and its largest 7. (c) Ex situ conservation is the method of carnivore, the tiger (Panthera tigris), as selecting plants or animals in places outside well as several large birds such as vultures, their natural homes, e.g. cryopreservation, adjutant storks, and hornbills. The Western tissue culture etc. They are the sources of Ghats are amongst the world’s biodiversity genetic material for breeders and genetic Hotspots. The various human-induced engineering. threats to the rich biodiversity and the large 8. (d) Genetic diversity is liable to undergo number of endemic species have been degradation and prone to mass scale identified and highlighted by the recently destruction due to fungal and insects completed National Biodiversity Strategy attacks and intensive use of biopesticides. and Action Plan; Western Ghats Ecoregion. 9. (c) Montreal protocol (1987) was a landmark 14. (b) Dodo became extinct in 14th century from international agreement to protect the Mauritius because of large-scale hunting. stratospheric ozone by agreeing to limit the Dodo was helpful in pollinating and production and use the ozone-depleting propagating seed of this tree species. substances. IIT JEE NEET eBooks: www.crackjee.xyz EBD_7100

B-200 Topicwise AIIMS Solved Papers – BIOLOGY

15. (b) Tectonic is the study of the earth's crust. It and has abundant minerals mixed with was considered that earth was made up of humus. tectonic plates. 21. (c) Tropical rain forests are located in the 16. (c) If bengal tiger becomes extinct, its gene equatorial regions where the annual rainfall pool will be lost forever. There are various exceeds 140 cm. They are also called methods for conservation of biodiversity jungles and cover one twelveth of earth's like formation of botanical gardens, surface but contain more than half of the zoological park, sperm bank, gene bank etc. earth's flora and fauna (i.e., rich in 17. (d) Hot spot are those areas which were rich in biodiversity). Now-a-days these forests are biodiversity but now under threat due to disappearing due to excessive cutting of direct or indirect interference of human forests for domestic purposes like fuel, activites. These regions are on the edge to furnitures, accomodations, cloths, resin, get some of their species extinct due to gum, etc. humans. Western Ghats in India are under threat due to continuous developmental 22. (a) activities and Doon valley is under threat 23. (a) A sanctuary is an area which is reserved due to continuous mining activities. for the protection of wild animals only. The 18. (c) Nagarjunsagar-Srisailam Tiger Reserve is activities like harvesting of timber, the largest Tiger reserve in India and the collection of minor forest products and only Tiger Reserve in Andhra Pradesh private ownership rights are allowed, state. however, such activities should not have any adverse effect on animals. 19. (d) 24. (a) Communities with higher number of Type B : Assertion Reason Questions species are more stable as it can resist occasional disturbances. A stable 20. (c) O-horizon occupies the topmost soil and community should show less variation in is rich in mineral and decomposed organic productivity from year to year and matter (humus). A-horizon is dark coloured resistance towards alien species. IIT JEE NEET eBooks: www.crackjee.xyz

38 Environmental Issues

9. Photochemical smog formed in congested TYPE A : MULTIPLE CHOICE QUESTIONS metropolitan cities mainly consists of [2003]

1. Which of the following is biodegradable (a) ozone, peroxyacetyl nitrate and NOx pollutant? [1997] (b) smoke, peroxyacetyl nitrate and SO2 (a) Plastic (b) Asbestos (c) hydrocarbons, SO2 and CO2 (c) Sewage (d) Polythene (d) hydrocarbons, ozone and SOx 2. Positive pollution of soil is due to [1997] 10. Biological Oxygen Demand (BOD) is a measure (a) reduction in soil productivity of [2003] (b) addition of waste to soil (a) industrial wastes poured into water bodies (b) extent to which water is polluted with (c) excessive use of fertilizers organic compound (d) all of the above (c) amount of carbon monoxide inseparably 3. Which of the following metal pollution causes combined with haemoglobin sterility in human beings? [1998] (d) amount of oxygen needed by green plants (a) Mercury (b) Arsenic during night (c) Manganese (d) Chromium 11. Drinking of mineral water with very low level of 4. Which of the following is dissolved in water to pesticides (about 0.02 ppm) for long periods may make Bordeaux mixture ? [1998] (a) produce immunity against mosquito [2003] (a) Calcium chloride (b) Copper sulphate (b) cause leukemia (blood cancer) in most (c) Sodium chloride (d) None of these people 5. Acid rain is due to pollution of [2000] (c) cause cancer of the intestine (a) dust (b) pesticides (d) lead to accumulation of pesticide residues in body fat (c) SO2 and NO2 (d) carbon particle 6. Deforestation causes [2001] 12. Minimata disease is pollution related disease. It results from [2001, 2004] (a) soil erosion (b) soil pollution (a) oil spills in sea (c) noise pollution (d) air pollution (b) DDT pollution 7. Green mufler is useful against [2002] (c) release of industrial waste containing (a) air pollution mercury in fishing water (b) noise pollution (d) accumulation of arsenic (c) soil pollution 13. A lake with an inflow of domestic sewage rich in (d) radioactive pollution organic waste may result in [2004] 8. In almost all Indian metropolitan cities like , (a) drying of the lake very soon due to algal the major atmospheric pollutant(s) is / are[2003] bloom (a) suspended particulate matter (SPM) (b) an increased production of fish due to lot (b) oxides of sulphur of nutrients (c) carbon dioxide and carbon monoxide (c) death of fish due to lack of oxygen (d) oxides of nitrogen (d) increased population of aquatic food web organisms IIT JEE NEET eBooks: www.crackjee.xyz EBD_7100

B-202 Topicwise AIIMS Solved Papers – BIOLOGY

14. Nitrogen oxides, produced from the emission of 20. Which one of the following statement pertaining automobiles and power plants, are the source of to pollutants is correct? [2011] fine air borne particles which lead to [2004] (a) DDT is a non-biodegradable pollutant (a) photochemical smog (b) Excess fluoride in drinking water causes (b) dry acid deposition osteoporosis hardening of bones, stiff (c) industrial smog joints (d) wet acid deposition (c) Excess cadmium in drinking water causes 15. Formation of non-functional methaemoglobin black foot disease causes blue-baby syndrome. This is due (d) Methylmercury in water may cause "Itai to [2005] (a) excess of arsenic concentration in drinking Itai" disease water 21. In the environment, ozone is known for its (b) excess of nitrates in drinking water (a) Harmful effects [2012] (c) deficiency of iron in food (b) Useful effects (d) increased methane content in the (c) Both (a) and (b) atmosphere (d) Inert nature 16. Pollution from animal excreta and organic waste 22. Which one of the following statement is true? from kitchen can be most profitably minimized [2013] by [2006] (a) The greater the BOD of waste water, more (a) storing them in underground storage tanks is its polluting potential. (b) using them for producing biogas (c) vermiculture (d) using them directly as biofertilizers (b) The greater the BOD of waste water, less is 17. Which one of the following is an environment- its polluting potential. related disorder with the correct main cause? (c) The lesser the BOD of waste water, more is [2006] its polluting potential. (a) Black lung disease (pneumoconiosis) (d) The lesser the BOD of waste water, less is found mainly in workers in stone quarries its polluting potential. and crushers. 23. Which one of the following pairs is mismatched? (b) Blue baby disease (methaemoglobinaemia) (a) Fossil fuel burning - release of CO2[2013] due to heavy use of nitrogen rich fertilizers (b) Nuclear power - radioactive wastes in the area. (c) Solar energy - green house effect (c) Non-Hodgkin’s Lymphoma found mainly (d) Biomass burning - release of CO2 in workers involved in manufacture of neem 24. The two gases making the highest relative based pesticides. contribution to the greenhouse gases are [2014]

(d) Skin cancer mainly in people exposed to (a) CO2 and CH4 (b) CH4 and N2O benzene and methane. (c) CFC and N2O (d) CO2 and N2O 18. Ozone in stratosphere extends [2007] 25. A lake near a village suffered heavy mortality of (a) 10-20 km (b) 20-25 km fishes within a few days. Consider the following (c) 15-30 km (d) 25-40 km reasons for this: 19. Which one of the following organisms is likely 1. Lots of urea and phosphate fertilizer were to show the highest concentration of DDT, once used in the crops in the vicinity it has been introduced into the ecosystem? 2. The area was sprayed with DDT by an [2010] aircraft (a) Grasshopper (b) Toad 3. The lake water turned green and stinky (c) Snake (d) Cattle IIT JEE NEET eBooks: www.crackjee.xyz

Environmental Issues B-203

4. Phytoplankton populations in the lake 29. Assertion : Inhabitants close to very busy declined initially thereby greatly reducing airports are likely to experience health hazards. photosynthesis. Reason : Sound level of jet aeroplanes usually Which two of the above were the main causes exceeds 160 dB. [2003] of fish mortality in the lake? [2015] 30. Assertion : Organochlorine pesticides are (a) 2 and 3 (b) 3 and 4 organic compounds that have been chlorinated. (c) 1 and 3 (d) 1 and 2 Reason : Fenitrothion is one of the 26. DDT residues are rapidly passed through food organochlorine pesticides. [2003] chain causing biomagnification because DDT 31. Assertion : Agricultural output increased is [2015] several times after introduction of DDT. (a) moderately toxic Reason : DDT was the first insecticide used on a wide scale. [2004] (b) non-toxic to aquatic animals 32. Assertion : A suspended particulate matter (c) water soluble (SPM) is an important pollutant released by (d) lipo soluble diesel vehicles. 27. Two lakes, A and B are identical in all aspects Reason : Catalytic converters greatly reduce except that lake A has higher temperature. pollution caused by automobiles. [2005] Which of the following is true ? [2016] 33. Assertion : Presently, the global atmosphere is (a) A has higher rate of oxygen dissolution. warming up. (b) B has higher rate of oxygen dissolution. Reason : The depletion of stratospheric ozone (c) Oxygen dissolution of both is the same. layer has resulted in increase in ultraviolet (d) Both the lakes have same BOD. radiations reaching the earth. [2005] 28. Euro II norms stipulate that sulphur be controlled 34. Assertion : Deforestation is one main factor at ______ppm in diesel and ______ppm in contributing to global warming. petrol. [2017] Reason : Besides CO2, two other gases methane (a) 350; 150 (b) 150; 350 and CFCs are also included under green house gases. [2006] (c) 350; 250 (d) 150; 250 35. Assertion : UV radiation causes photo-

TYPE B : ASSERTION REASON QUESTIONS dissociation of ozone into O2 and O, thus causing damage to the stratospheric ozone layer. Directions for (Qs. 29-38) : These questions consist Reason : Ozone hole is resulting in global of two statements, each printed as Assertion and warming and climate change. [2006] Reason. While answering these questions, you are 36. Assertion : The concentration of methane in required to choose any one of the following five the atmosphere has more than doubled in the responses. last 250 years. (a) If both Assertion and Reason are correct and Reason : Wetlands and rice fields are the major the Reason is a correct explanation of the sources of methane. [2006] Assertion. 37. Assertion (A) : Pollution is always caused by (b) If both Assertion and Reason are correct but human activities. Reason is not a correct explanation of the Reason (R) : Pollution is not different from Assertion. contamination. [2007] (c) If the Assertion is correct but Reason is 38. Assertion (A) : Chlorofluorocarbons are incorrect. responsible for ozone depletion. (d) If both the Assertion and Reason are incorrect. Reason (R) : Ozone level decreases by as much (e) If the Assertion is incorrect but the Reason is as 67% every year. [2007] correct. IIT JEE NEET eBooks: www.crackjee.xyz EBD_7100

B-204 Topicwise AIIMS Solved Papers – BIOLOGY

Directions for (Qs. 39-43) : Each of these questions Reason : Nitrates are responsible for blue baby contains an Assertion followed by Reason. Read them syndrome. carefully and answer the question on the basis of 41. Assertion : Water pollutants are measured by following options. You have to select the one that BOD. best describes the two statements. Reason : If BOD is more, the water is polluted. (a) If both Assertion and Reason are correct and [2013] Reason is the correct explanation of Assertion. 42. Assertion : Methane, component of green (b) If both Assertion and Reason are correct, but house gases, contributing to global warming is Reason is not the correct explanation of about 20 percent. Assertion. Reason : Introduction of multi-point fuel (c) If Assertion is correct but Reason is incorrect. injection engines in automobiles has decreased (d) If both the Assertion and Reason are incorrect. methane content in the exhausts. 39. Assertion : Secondary succession takes place [2005, 2015] in recently denuded area. [2009] 43. Assertion : Eutrophication shows increase in Reason : It is caused due to baring of an area. productivity in water. [2013, 2017] 40. Assertion : Excess of nitrates in drinking water Reason : With increasing eutrophication, the are harmful for infants. [2009] diversity of the phytoplankton increases. IIT JEE NEET eBooks: www.crackjee.xyz

Environmental Issues B-205

Type A : Multiple Choice Questions weight of organisms with the rise in trophic level due to their accumulation in fat 1. (c) Sewage is a biodegradable pollutant i.e. (biomagnification). capable of being removed by microbial 12. (c) Mercury compounds in waste water are action. converted by bacterial action into extremely 2. (d) Soil pollution is of two types : positive soil toxic compound, methyl mercury. pollution and negative soil pollution. Biomagnification of mercury into fishes When there is addition of undesirable through the food chain is responsible for substance in the soil then it is called as large number of deaths due to Minamata positive pollution. diseases. It is characterised by numbers of 3. (c) Manganese causes sterility in human limbs, lips and tongue, deafness, blurred beings. It enters the food chain and finally vision, mental retardation etc. reaches human beings & has an effect on 13. (c) A lake rich in (domestic sewage) nutrients the reproductive system. accelerates the growth of algae which use 4. (b) Bodeaux mixture is a combination of copper oxygen at night and deoxygenate the water sulphate and hydrated lime. It is used enough to kill the fish and other animals mainly to control garden, vineyard and farm (eutrophication). infestations of fungus. 14. (a) Nitrogen oxides form peroxyacyl nitrate 5. (c) Acid rain is rainfall and other form of (PAN) by reacting with hydrocarbons. It precipitation with a pH of less than 5. It is leads to photochemical smog formation. due to the excess of SO and NO in the air 2 2 15. (b) Excess nitrate in drinking water, leaves and which along with water form sulphurous fruits, changes into nitrite in alimentary and nitric acid. canal. It reacts with haemoglobin and 6. (a) In India, deforestation is the main cause of produces non-functional met haemoglobin. soil erosion. The quality of top soil loss in This reduces O carrying capacity of blood. India is about 18.5% of the total global loss. 2 The disorder is called methaemoglobinea. 7. (b) Green muffler or green belt vegetation is It causes blue baby syndrome and rows of trees and shrubs grown and breathlessness in adults. maintained to serve as noise absorbers. 16. (c) Vermiculture means artificial seating of 8. (a) Suspended particulate matter (SPM) is worms whose excreta is rich in humus. maximum in metropolitan cities. These worms eat farmyard manure along 9. (c) Photochemical smog or oxidizing type of with other farm wastes and pass out it pollution is characterized by the presence through their bodies and the process of large concentration of ozone, oxides of converts it into vermicompost for the nitrogen and various hydrocarbons. It betterment of human beings. occurs in Los Angeles. 17. (b) Blue baby disease is caused by nitrate 10. (b) BOD is the measure of degree of impurity poisoning (presence of methaemoglobin) of water due to organic matter. resulting in organs & cell tissues that are 11. (d) Pesticides in a drinking water pass into food deprived of oxygen & skin with the chain and they increase in amount per unit characteristic blush tint. IIT JEE NEET eBooks: www.crackjee.xyz EBD_7100

B-206 Topicwise AIIMS Solved Papers – BIOLOGY

18. (b) Stratosphere extends from 20-25 km above 24. (a) The gases that makes highest relative tropospheres. It is also called ozonosphere contribution to the green house gases are

due to presence of ozone. Temperature carbon dioxide (CO2) and methane (CH4). increases in this zone (upto 90°C) due to 25. (c) Lots of urea and phosphate fertilizer were ozone formation. Ozone protects Earth from used in the crops in the vicinity and the harmful ultraviolet radiations of the sun. lake water turned green and stinky. Due to Oxides of Nitrogen + Unburnt Hydrocarbons this, lake near a village suffered heavy mortality of fishes within a few days. UV rays oO3 PAN 26. (d) DDT residues are rapidly passed through (Ozone) (Peroxy acetyl nitrate) food chain causing biomagnification 19. (c) Biomagnification is phenomenon of because DDT is soluble in lipids. Biomagnification, also known as increasing concentration of a compound in bioamplification or biological the tissues of chain, usually as a result of magnification, is the increase in food intake. In this case, the concentration concentration of a substance, such as the of compound increases with increasing pesticide DDT, that occurs in a food chain trophic level. as a consequence of: Pollutant increases in concentration from • Persistence (slow to be broken down Producerso Primary consumers o by environmental processes) Secondary consumers o Tertiary • Food chain energetics consumers o Top consumer. In the given • Low (or non-existent) rate of internal question, only snake represents the tertiary degradation/excretion of the consumer, so the concentration of DDT will substance (often due to water- be highest in snake. insolubility) 20. (a) Excess fluorine in drinking water causes 27. (b) Lake A and B are identical in all aspects hardening of bones and stiffness of joints, except that Lake A has higher temperature. black foot disease is due to arsenic and On the basis of this difference, it is clear itai-itai disease is due to cadmium in that Lake B has higher rate of oxygen contaminated water. dissolution 21. (c) In the environment, ozone is known for its 28. (a) The Govt. of India through a new auto fuel both harmful and useful effects. policy has laid out a roadmap to cut down Ozone present in stratosphere is useful as the vehicular pollution in Indian cities. For it act like shield and protect the earth from example, Euro II norms stipulate that harmful UV-rays. Ozone present in sulphur be controlled at 350 ppm in diesel troposphere act as pollutant and have many and 150 ppm in petrol. deleterious effects. Type B : Assertion Reason Questions 22. (a) 23. (c) Solar energy coming to the earth is not 29. (a) Sound level of jet aeroplanes is about 150 responsible for green house effect. It is the dB beyond the level of human audible increase in green house gases in capacity (80 dB). It affects the hearing and general health of man. atmosphere like CO2 which is released by complete combustion of fossil fuels or 30. (c) Organochlorine are persistent pesticides biomass in industries or transportation (e.g. DDT) which pass into food chain and vehicles that prevent the reradiation of increase in amount per unit weight of infrared radiation from the earth and result organisms with the rise in trophic level. Fenitrothion is organophosphate. in increase in temperature of the earth. IIT JEE NEET eBooks: www.crackjee.xyz

Environmental Issues B-207

31. (a) DDT (dichloro diphenyl trichloroethane) is made. Natural pollution includes volcanic a organochlorine contact insecticide that eruptions, soil erosion, UV- rays etc. kills by acting as a nerve poison. DDT was Pollution is different from contamination. originally used during world war II to Contamination is the presence of harmful control typhus which was spread by body organisms causing disease. louse. Since then it has been used to control 38. (b) Stratosphere zone of Earth's atmosphere mosquito borne malaria and was used contains a layer of ozone which protects extensively as a general agricultural us from harmful ultraviolet radiations of the insecticide. sun.A group of chlorine containing 32. (b) Catalytic converters are involved in compounds called chlorofluoro carbon reducing gaseous pollutions by converting (CFCs) used as coolants in air conditioners  o and refrigerators are primary chemicals CO  o CO2, NO2 N2 etc. Thus, decrease the amount of pollutant. They can responsible for ozone depletion. After their not reduce emission of SPM. release into troposphere, CFCs go to 33. (b) Global warming is due to the increase in stratosphere where they are broken down concentration of green house gases by UV radiations releasing chlorine.In resulted in increase in global temperature. presence of sunlight, chlorine breaks O3 These global gases prevent the escape of into O2. Due to ozone depletion, its levels long wave radiations into space. decreases by 67% every year causing 34. (b) Deforestation results in increase in green higher levels of UV radiations reaching house gases which retains more and more earth which may cause eye cataracts, skin UV radiations and leads to global warming. cancer etc. Global warming is the warming/heating up 39. (a) Biotic or ecological succession is the of the earth's atmosphere due to depletion formation of a series of biotic communities of ozone in the stratosphere. at the same site over a period of time one 35. (c) Ozone in stratosphere is responsible for after the other, till a stable climax community the protection of earth from high energy develops over the area. It occurs generally UV rays i.e. it acts as life saving screen. in bare areas. Primary succession takes place Ozone layer found in troposphere protects on a biological sterile soilless primary barren from warming effect of earth. area. Secondary succession takes place in a Due to human activities, the ozone layer in recently denuded area which still contains a the stratosphere starts thinning, which is lot of organic debris, remains and propagules also called ozone hole. Ozone hole is of previous living organisms. The area has resulting in rain failure, increase in become bared due to destruction of the radiation, cancer (skin) and reduction in community previously present. The baring crop production. of an area can be caused due to forest fire, 36. (a) Wetland and rice fields are the major deforestation for wood, timber and habitat, sources of methane. It is a green house overgrazing, landslides or earthquakes, gas whose concentration is double now excessive and repeated droughts, following than it was 250 years ago. a cropland and repeated floods. 37. (d) Pollution may be defined as an undesirable 40. (a) Excess of nitrates in drinking water are change in physical, chemical or biological harmful for human health and may be fatal characteristics of air, water and land for infants. Excessive use of fertilizers often causing harmful effects on living leads to accumulations of nitrates in water. organisms. Pollution can be natural or man In infants, excess nitrate reacts with IIT JEE NEET eBooks: www.crackjee.xyz EBD_7100

B-208 Topicwise AIIMS Solved Papers – BIOLOGY

haemoglobin to form nonfunctional methane (CH4) and chlorofluoro carbons methaemoglobin that impairs oxygen (CFCs) is 20% and 14% respectively. N2O transport. This condition is termed as also contributes 6% in total global warming. methaemoglobinemia or blue baby Efficient engine such as multi point fuel syndrome. This disease can damage injection engine can reduce the unburnt respiratory and vascular systems and even hydrocarbon (methane) in auto-emissions. cause suffocation. 43. (b) Eutrophication is a natural process which 41. (a) Water pollutants are commonly measured literally means well nourished or enriched. by their main common denominator, called It is a natural state in many lakes and ponds BOD (Biochemical Oxygen Demand), i.e., which have a rich supply of nutrients. the amount of free oxygen absorbed by Eutrophication becomes excessive, when extraneous substances from water. If water abnormally high amount of nutrient from is polluted, it will consume more oxygen, sewage, fertilizers, animal wastage and thereby enhancing the BOD of water. detergent, enter streams and lakes causing 42. (b) Methane is produced by incomplete excessive growth or blooms of biomass combustion, incomplete microorganisms. With increasing decomposition mostly by anaerobic eutrophication, the diversity of the methanogens. phytoplankton community of a lake Carbon dioxide contributes about 60% of increases and the lake finally becomes the total global warming and share of dominated by blue - green algae. IIT JEE NEET eBooks: www.crackjee.xyz

1 G.K. – History

1. Doctors before starting their service take oath 12. In which year was English recommended as the in the name of a scientist to work honestly : medium of instruction for higher education in (a) Hippocrate (b) Darwin [1997] India by Lord Macaulay? [2001] (c) Plato (d) Socrates (a) 1833 (b) 1835 2. Confucianism is famous in : [1997] (c) 1859 (d) 1825 (a) Japan (b) China 13. 'Gayatri Mantra' is related with : [2002] (c) Myanmar (d) Malaysia (a) Athar Veda (b) Rig Veda 3. The tomb of Qutub Shahi is situated in : [1997] (c) Yajur Veda (d) Sam Veda (a) Aligarh (b) Agra 14. The founder of 'Khalsa' was : [2002] (c) Hyderabad (d) Allahabad (a) Guru Gobind Singh 4. “Statue of Liberty” was gifted to USA by : [1997] (b) Guru Nanak Dev (a) Germany (b) Canada (c) Guru Ram Das (c) Greece (d) France (d) Guru Tegh Bahadur 5. The dynasty of Bahadur Shah (II) Jafar was in : 15. Who was the last Viceroy of India : [2002] [1998] (a) Lard David (a) 1658 to 1707 A.D. (b) 1837 to 1857 A.D. (b) Lord Wavell (c) 1857 to 1862 A.D. (d) 1800 to 1829 A.D. (c) Lord Mountbattern 6. Swami Vivekanand was born in which one of the (d) Wellington following year ? [1998] 16. Euthanasia (mercy killing) was first legalized in : (a) 1860 (b) 1882 (a) Switzerland [2003] (c) 1897 (d) 1863 (b) Netherlands (Holland) 7. The Simon Commission had come in which of (c) France the following year ? [1998] (d) Italy (a) 1937 (b) 1938 (c) 1927 (d) 1942 17. Which city was gifted to Charles II by the 8. The Fifth Day commission was headed by justice : Portuguese when he married the sister of the (a) Verma (b) Ahmadi [1999] King of Portugal in 1662? [2003] (c) Anand (d) Vadhwa (a) Bombay (b) Paris 9. Grand Trunk road was built by : [2000] (c) Lisoon (d) Castille (a) Sher Shah Suri (b) Shah Jahan 18. Who is the mother of Bharat in the epic (c) Lord Bentic (d) Lord Mount Battan 'Ramayana'? [2003] 10. At the first time, the song Vande Mataram has (a) Kaushalya (b) Sumitra sung in : [2001] (c) Urmila (d) Kaikayee (a) Indian National Congress session 1986 19. Which of the 'Nawab of Bengal' is supposed to (b) Indian National Congress session 1896 be responsible for 'Back Hole Tragedy' of (c) Quit India Movement 1942 Calcutta (Kolkata)? [2003] (d) Congress session 1911 (a) Mir Jafer (b) Sirajuddaula 11. From where did Mahatma Gandhi start the (c) Alivardi Khan (d) Sarfaraj Khan famous Dandi March? [2001] 20. Who among the following is known as Fuehrer? (a) Surat (b) Mumbai (a) Stalin (b) Lenin [2003] (c) Bardoli (d) Ahmedabad (c) Hitler (d) Bismarck IIT JEE NEET eBooks: www.crackjee.xyz EBD_7100

C-2 Topicwise AIIMS Solved Papers – GENERAL KNOWLEDGE 21. Since the Britishers wanted India to produce and (a) Mahatma Gandhi supply raw materials to feed English factories, (b) Jawaharlal Nehru the pushed a policy that encouraged cultivation (c) Lokmanya Tilak of : [2004] (d) Muhammad Iqbal (a) jute (b) indigo 32. The earliest Iron Age in India has been (c) cotton (d) commercial crop associated with [2010] 22. ‘Brahmo Samaj’ was founded by : [2005] (a) Painted grey ware (a) Devendra Nath Tagor (b) Black and red ware (b) Keshv Chandra Sen (c) Ochre Coloured Pottery (OCP) (c) Raja Ram Mohan Roy (d) Northern black polished ware (d) Annie Besant 33. Who among the following was the ruler of Delhi 23. Who was the first woman ruler of India? [2005] at the time of the invasion of Nadir Shah?[2010] (a) Raziya Sultan (b) Noor Jahan (a) Muhammad Shah (c) Chand Biwi (d) Durga Devi (b) Raj Raj I, the Chola 24. Who were the first to issue gold coins in India? (c) Bahadur Shah [2005] (d) Alamgir II (a) The Kushan's (b) The Gupta's 34. Most of the Ajanta Paintings were done during (c) The Mauryan's (d) The Indo Greeks the period of : 25. Which sea is referred to in our National Anthem? (a) Harshavardhana (b) Guptas (a) Bay of Bengal [2006] (c) Mauryas (d) Kushan (b) Indian ocean 35. Who was the first Indian ruler to face the (c) Arabian sea Mahmud Ghaznavi ? [2011] (d) No sea is mentioned in it (a) Prithviraj Chauhan (b) King Jaipal 26. The national calendar of India is based on : (c) Jaichand (d) Firdausi (a) Gragorian calendar [2006] 36. Which one of the following countries is “Orange (b) Hizrr Era (c) Saka- Era Revolution” associated ? [2012] (d) one of the old Indian Era (a) Brazil (b) Sudan 27. Gaya is associated with Lord buddha, where he (c) Turkey (d) Ukraine (a) was born [2007] 37. Who was the Governor General of India during (b) attained enlightenment the Revolt of 1857 ? [2012] (c) died (a) Lord Canning (b) Lord Dalhousie (d) delivered his first sermon (c) Lord Dufferin (d) Lord Hardinge 28. Which of the following places was known as a 38. Who founded the Brahma Samaj? [2012] centre of learning in ancient India? [2007] (a) Debendranath Tagore (a) Nalanda (b) Ujjain (b) Keshab Chandra Sen (c) Allahabad (d) none of these (c) Raja Rammohan Roy 29. In violation of the Salt Laws, Gandhiji started a (d) Ishwar Chandra Vidyasagar movement called [2008] 39. Consider the following Mughal Emperors: (a) Non-Cooperation Movement i. Farrukhshiyar ii. Jahandar Shah (b) Swadeshi Movement iii. Muhammad Shah iv. Shah Alam II (c) Civil Disobedience Movement What is the correct chronological sequence of (d) None of the above their ascending the throne ? [2012] 30. The Cabinet Mission came to India in [2008] (a) i, ii, iii, iv (b) ii, i, iii, iv (a) 1943 (b) 1944 (c) i, ii, iv, iii (d) ii, i, iv, iii (c) 1945 (d) 1946 40. Match the following : [2013] 31. Who was the founder-editor of the famous A. Sarojini Naidu 1. Muslim League newspaper ‘Kesari’ during the national struggle? B. M.A. Jinnah 2. Indian National [2009] Congress IIT JEE NEET eBooks: www.crackjee.xyz

G.K. – History C-3 C. Sir Tej Bahadur 3. Hindu Mahasabha (c) 1919 - Jallianwala Bagh Massacre Sapru (d) None of the above D. V.D. Savarkar 4. Liberal Party 45. The first meeting of Indian National Congress ABCD ABCD was held in Bombay in1885 A.D. under the (a) 2 1 4 3 (b) 2 134 leadership of __? [2015] (c) 2 4 1 3 (d) 4 132 (a) Dadabhai Naoroji 41. Identify the medical trio of Ancient India from (b) Sir C. Sankaran Nair the following names. [2013] (c) Womesh Chandra Banerjee (a) Charaka, Susruta and Vagbhata (d) Badruddin Tyabji (b) Charaka, Vatsyayana and Vagbhata 46. When did Vasco-da-Gama come to India ? [2016] (c) Charaka, Susruta and Bharata (a) 1492 (b) 1494 (d) Charaka, Susruta and Patanjali (c) 1496 (d) 1498 42. Who among the following first propounded the 47. The Qutub Minar at Delhi was built by ? [2016] idea of Basic Education? [2014] (a) Qutbuddin Aibak (b) Shahjhan (a) Jawahar Lal Nehru (c) Aleu-din-khilji (d) Chandragupta (b) Raja Ram Mohan Roy 48. Who was the first Governor - General of India (c) Mahatma Gandhi (a) Robert Clive [2016] (d) Dayanand Saraswati (b) Sir Charles Metcalfe 43. 'Tebhega' movement is associated with which (c) William Bentinck state ? [2015] (d) Warren Hastings (a) Maharashtra (b) Tamil Nadu 49. ‘Do or Die’ (Karo ya Maro) slogan was given (c) Karnataka (d) Bengal by which freedom fighter in 1942 ? [2017] 44. When was our national song sung for the first (a) Mahatma Gandhi time? [2015] (b) Subhash Chand Bose (a) 1896 session of Indian National Congress (c) Bal Gangadhar Tilak (b) 1857 revolt (d) Bhagat Singh

ANSWER KEY 1 (a) 2 (b) 3 (c) 4 (d) 5 (b) 6 (d) 7 (c) 8 (a) 9 (a) 10 (b) 11 (d) 12 (a) 13 (b) 14 (a) 15 (c) 16 (b) 17 (a) 18 (d) 19 (b) 20 (c) 21 (b) 22 (c) 23 (a) 24 (d) 25 (d) 26 (c) 27 (b) 28 (a) 29 (c) 30 (d) 31 (c) 32 (a) 33 (a) 34 (b) 35 (b) 36 (d) 37 (a) 38 (c) 39 (b) 40 (a) 41 (d) 42 (c) 43 (d) 44 (a) 45 (c) 46 (d) 47 (a) 48 (d) 49 (a) IIT JEE NEET eBooks: www.crackjee.xyz EBD_7100

C-4 Topicwise AIIMS Solved Papers – GENERAL KNOWLEDGE 2 G.K. – Indian Polity

1. Sixty five year plan to promote significant (a) C. Raj Gopalachari (b) Radha Krishana expansion of employment opportunities was (c) Y. C. Grace ( d ) V. V. Gi r i scheduled in the year of : [1998] 10. National Anthem "Jana Gana Mana" was (a) 1980-1985 (b) 1669-1674 adopted on [2001] (c) 1974 -1979 (d) 1961-1966 (a) 26 Jan. 1950 (b) 26 July 1947 2. Sixty first amendment in the Constitution states (c) 15 August 1947 (d) 24 Jan. 1950 about [1998] 11. Who is the chairman of Rajya Sabha? [2001] (a) Extended President rule in Punjab (a) Speaker of Lok Sabha (b) Increasing the ceibling of profession tax (b) Home Minister (c) Reservation of seats for scheduled castes (c) President and scheduled tribes in parliament (d) Vice President (d) Reducing the voting age from 21 years to 12. Which one of the following determines the 18 years salary of attorney General? [2001] 3. Which of the following state, becomes the 22nd (a) Speaker of Lok Sabha state of Indian union ? [1998] (b) Home Minister (a) Tripura (b) Sikkim (c) President of India (c) Meghalaya (d) Assam (d) Prime Minister 4. At present, the total membership of Lok Sabha : 13. The oath of office to a Supreme Court Judge is (a) 552 (b) 525 [1998] administered by : [2001] (c) 527 (d) 545 (a) The Chief Justice 5. Fundamental duties were introduced in the (b) The President of India constitution by : [1999] (c) The Chief Justice of India (a) 42nd amendment (b) 40th amendment (d) The Law minister (c) 48th amendment (d) 53rd amendment 6. The person who served as the President of India 14. Who addressed the U. N. General Assembly for twice, was : [1999] the first time in Hindi? [2001] (a) Radha Krishnan (a) Rajendra Prasad (b) Atal Bihari Vajpai (b) Dr. Rajendra Prasad (c) Jawahar Lal Nehru (d) Swarn Singh (c) Zakir Hussain 15. How many languages are recognised by the (d) V.V Giri constitution of India in the 8th schedule? [2002] 7. President of India gives his resignation to the : (a) 12 (b) 14 (a) Chief Justice (b) Parliament [1999] (c) 16 (d) 18 (c) Vice President (d) Prime Minister 16. Who is known as the 'Iron Man of India'? [2002] 8. The contribution of sarkaria commission was (a) Jawaharlal Nehru related between : [2000] (b) Bal Gangadhar Tilak (a) state and centre (c) Sardar Vallabhbhai Patel (b) centre and union territories (d) Mahatma Gandhi (c) one state to other state 17. The design of the National Flag was adopted by (d) none of these the constituent assembly of India on : [2002] 9. Who was appointed as the first governor general (a) 26 January, 1949 (b) 26 January, 1950 of India ? [2000] (c) 22 July, 1947 (d) 15 August, 1947 IIT JEE NEET eBooks: www.crackjee.xyz

G.K. – Indian Polity C-5 18. When was the golden jubilee of Indian Parliament 29. Who is the constitutional head of the Government celebrated ? [2002] of India ? [2008] (a) 1st January, 1997 (b) 26th January, 2002 (a) President (c) 13th may, 2002 (d) 15th August, 1997 (b) Prime Minister 19. Dravida Munnetra Kargam (DMK) was founded (c) Chief Justice of India by : [2004] (d) Attorney General (a) M.G. Ramachandran 30. Which part of Indian Constitution has been (b) C.N. Annadurai described as the soul of the Constitution ? (c) Kumar Swami Kamraj (a) Fundamental Rights [2008] (d) Lalithambika Antharjanam (b) Directive Principle of State Policy 20. The Planning Commission was set up in : [2005] (c) The Preamble (a) January, 1950 (b) March, 1950 (d) Right to Constitutional Remedies (c) January, 1952 (d) March, 1952 31. The President of India can nominate [2008] 21. How many spokes are in Indian National Flag? (a) 10 members to Rajya Sabha (a) 22 (b) 24 [2005] (b) 2 members to Rajya Sabha (c) 26 (d) 28 (c) 15 members to Rajya Sabha 22. Who was elected as the permanent President of (d) 12 members to Rajya Sabha constituent assembly ? [2005] 32. The Constitution of India was adopted by the (a) Dr. Sachchidanand Sinha Constituent Assembly on [2008] (b) Dr. Rajendra Prasad (a) August 15, 1947 (c) Dr. B. R. Ambedkar (b) June 30, 1948 (d) C. Rajgopalachari (c) November 26, 1949 23. How many articles and schedule are there in (d) January 26, 1950 originally constitution ? [2005] (a) 391 articles and 7 schedules 33. Who is the Chairman of the Planning Commission ? (b) 395 articles and 8 schedules (a) President [2008] (c) 400 articles and 10 schedules (b) Prime Minister (d) 444 articles and 12 schedules (c) Finance Minister 24. The Tenure of first planning holiday was [2005] (d) Governor of Reserve Bank (a) 1964 -1967 (b) 1965 -1968 34. Which Plan give emphasis on removal of poverty (c) 1966 -1969 (d) 1978 -1981 for the first time? [2009] 25. World Trade Organisation established in : [2005] (a) Fourth (b) Fifth (a) 1954 (b) 1988 (c) Sixth (d) Seventh (c) 1994 (d) 1995 35. The Council of Ministers does not include 26. The supreme command of the defence forces is (a) Cabinet Ministers [2009] vested with the : [2006] (b) Ministers of State (a) Field Marshal (c) Cabinate Secretary (b) Commander-in-chief (d) Ministers without portfolio (c) Prime minister 36. Attorney -General of India is appointed by (d) President of India (a) Chief Justice of Supreme Court [2010] 27. India's first battle field missile is : [2006] (b) Parliament (a) Akash (b) Prithvi (c) Law Minister (c) Agni (d) Nag (d) President 28. Who conducts the State assembly elections? 37. Which one of the following Article provide [2007] ‘Right to equality’? [2010] (a) Chief Justice of the High Court concerned (a) Article - 14 (b) Chief Justice of the Supreme Court (b) Article -19 (c) Chief Election Commissioner (c) Article - 20 (d) Governor of the state concerned (d) Article - 18 IIT JEE NEET eBooks: www.crackjee.xyz EBD_7100

C-6 Topicwise AIIMS Solved Papers – GENERAL KNOWLEDGE 38. First speaker of Lok Sabha was : [2011] Choose the correct option from the codes given (a) Sardar Hukum Singh below: (b) G. V. Mavlankar (a) Only 1 (b) Only 2 (c) Neelam Sanjeevan Reddy (c) Only 1 & 2 (d) Only 1 & 3 (d) Bali Ram Bhagat 43. In Lok Sabha of India, the "Leader of the House" 39. What is the intermediate tier of the Panchayati is nominated by ___: [2015] Raj System called? [2012] (a) President (b) Lok Sabha Speaker (a) Zilla Parishad (b) Taluka Panchayat (c) Prime Minister (d) Deputy Speaker (c) Panchayat Samiti (d) Gram Sabha 44. The President of India can nominate how many 40. Which of the following is associated with members to Rajya Sabha and Lok Sabha Panchayati Raj ? [2013] respectively? [2015] (a) Nanavati Commission (a) 10, 3 (b) 12, 2 (b) Balwant Rai Mehta Committee (c) 10, 2 (d) 12, 3 (c) Librahan Commission 45. Supreme Court judge retires upon attaining the (d) Shah Commission age of [2016] 41. How many articles are there in the Indian (a) 65 years (b) 60 years constitution? [2014] (c) 55 years (d) 50 years (a) 395 (b) 396 46. How many times has National Emergency been (c) 398 (d) 399 declared? [2016] 42. The Vice President of India is elected by the (a) Six times (b) Three times electoral collage consisting of the members of (c) Five times (d) Four times ______? [2015] 47. A Municipal Corporation is set up in a city with 1. Rajya Sabha how much population of not less than? [2016] 2. Lok Sabha (a) 2 lakhs (b) 5 lakhs 3. State Legislatures (c) 10 lakhs (d) 15 lakhs

ANSWER KEY 1 (b) 2 (a) 3 (c) 4 (a) 5 (a) 6 (b) 7 (b) 8 (a) 9 (a) 10 (d) 11 (d) 12 (c) 13 (c) 14 (b) 15 (d) 16 (c) 17 (c) 18 (c) 19 (b) 20 (b) 21 (b) 22 (b) 23 (b) 24 (c) 25 (d) 26 (d) 27 (b) 28 (c) 29 (a) 30 (d) 31 (d) 32 (c) 33 (b) 34 (a) 35 (c) 36 (d) 37 (a) 38 (b) 39 (c) 40 (b) 41 (a) 42 (c) 43 (c) 44 (b) 45 (a) 46 (b) 47 (c) IIT JEE NEET eBooks: www.crackjee.xyz

G.K. – Geography C-7 3 G.K. – Geography

1. Dachigan Wild life santuary in Kashmir is (a) The factory where India’s newly designed associated with which of the following animal ? battle tanks are being manufactured (a) Hangul (b) Panther [1997] (b) The samadhi of Indira Gandhi (c) Horned toed Deer (d) Sagui (c) The nuclear reactor at KalpaKam at Chennai 2. ‘Principality of Liechtenstion’ is situated (d) none of these between Switzerland and [1997] 12. Khushi Nagar, the famous Buddhist pilgrimage (a) Austria (b) France centre in the state of : [2000] (c) Italy (d) Germany (a ) U. P. (b) M.P 3. The largest producer of rubber is : [1997] (c) Bihar (d) Orissa (a) Sri Lanka (b) India 13. Nasic is situated on the bank of : [2000] (c) Japan (d) Malaysia (a) Narmada (b) Krishna 4. Kalpakkam in Tamilnadu is known for its : (c) Kauvery (d) Godavari (a) temples [1997] 14. Which city is known as Pink city ? [2000] (b) textiles mills (a) Jaipur (b) Paris (c) handicrafts (c) New York (d) London (d) atomic power plants 15. An indian river, that does not form any delta is : 5. OPEC is : [1997] (a) Cauvery (b) narmada [2002] (a) Organisation of petroleum Exporting (c) Yamuna (d) Singh Companies 16. How many islands are there in lakshadweep? (b) Oil and Petroleum exporting Corporation (a) 47 (b) 36 [2002] (c) Oil and petroleum Exporting Countries (c) 27 (d) 17 (d) Organisation of Petroleum Exporting 17. From which of the following places the Countries international dateline crosses? [2003] 6. Sambalpur is situated on the bank in which of (a) Atlantic ocean the following river ? [1998] (b) Pacific ocean (a) Jammu (b) Saraswati (c) Greenwich (c) Sagar (d) Mahandi (d) Gape of Good Hope 7. Seoul is the capital of : [1999] 18. A man-made tunnel in India transfers water from (a) Japan (b) South Korea which one river to another? [2004] (c) Afganistan (d) Philippines (a) Narmada to Tapti 8. Hirakund dam is constructed on which of the (b) Betwa to Sone following river ? [1999] (c) Beas to Sutlej (a) Mahanadi (b) Ganga (d) Godawari to Krishna (c) Yamuna (d) Kosi 19. The grand Fisher Bank is situated off : [2004] 9. ‘Sun city’ is in : [2000] (a) the coast of New Foundland (a) USA (b) South Africa (b) the Chilean coast (c) France (d Denmark (c) the Spanish coast 10. Which one of the least populated state in India? (d) the coast of Great Britain [2000] 20. Which one of the following organisation's iron (a) Nagaland (b) Himachal Pradesh and steel plant was build to use charcoal as a (c) Orissa (d) Sikkim source of power, to start with, but later switched 11. Shakti sthal is the name given to : [2000] over to hydroelectricity ? [2004] IIT JEE NEET eBooks: www.crackjee.xyz EBD_7100

C-8 Topicwise AIIMS Solved Papers – GENERAL KNOWLEDGE (a) The Tata Iron Steel Company 31. ‘Equinox’ means [2007] (b) The Indian Iron and Steel Company (a) days are longer than nights (c) Mysore Iron and Steel Company (b) days and nights are equal (d) Hindustan Steel Limited (c) days are shorter than nights 21. Rishikesh is famous for the production of (d) none of the above (a) antibiotics [2004] 32. Which country leads in production of aluminium (b) heavy electricals and aluminium goods ? [2007] (c) fertilizers (a) Australia (b) U.S. (d) transistorized radios (c) Russia (d) Japan 22. Nepanagar is famous for : [2004] 33. Which is an ore of aluminium? [2007] (a) paper board industries (a) chromite (b) cuprite (b) craft paper industries (c) bauxite (d) siderite 34. The southern tip of India is [2008] (c) carbon paper industries (a) Cape Comorin (Kanyakumari) (d) newsprint paper industries (b) Point Calimere 23. Where is 'Indira Gandhi Rashtriya Uran (c) Indira Point in Nicobar Islands Akadami's situated? [2004] (d) Kovalam in Thiruvananthapuram (a) Dehradun (b) Raebareli 35. The major coffee producing state in India is (c) Allahabad (d) Mussoorie (a) Kerala (b) Karnataka [2008] 24. Where is 'National Defence Academy' situated? (c) Tamil Nadu (d) West Bengal [2005] 36. Bauxite is an ore of [2008] (a) (b) Khadakvasla (a) Aluminium (b) Boron (c) Dehradun (d) Pune (c) Lead (d) Silver 25. Which river is called 'Bengal's Sorrow'? [2005] 37. Which of the following methods is used to (a) Hughli (b) Ganga determine the age of the earth ? [2008] (c) Damodar (d) Koshi (a) Carbon dating (b) Germanium dating 26. 'Maoris' are : [2005] (c) Uranium dating (d) All of the above (a) inhabitants of New Zealand 38. The coastal tract of Andhra Pradesh and Tamil (b) inhabitants of Hungary Nadu is called [2009] (c) inhabitants of North America (a) Konkan (b) Coromandel (d) inhabitants of North-West Asia (c) East Coast (d) Malabar Coast 27. 'Chittaranjan' is famous for the [2005] 39. Diamond bearing rocks occur in [2010] (a) railway coaches (b) locomotives (a) Panna of Madhya Pradesh (c) iron and steel (d) heavy machinery (b) Mysore of Karnataka 28. Sardar Sarovar Dam is built on the river [2006] (c) Waltair of Andhra Pradesh (a) Jhelam (b) Narmada (d) Ajmer of Rajasthan (c) Tapti (d) Vyas 40. Highest mustard seed producing state is[2010] 29. Lagoon refers to : [2006] (a) Maharashtra (b) Rajasthan (a) A full moon (c) U. P. (d) Gujarat (b) The sea breaking into the land and then 41. The highest city of the world is- [2011] separated by the sand dunes (a) Wan Chan (China) (b) La Paz (Bolivia) (c) A spot in a desert made fertile by presence (a) Tokyo (Japan) (d) New York (USA) 42. Kalahari desert is in [2011] of water (a) North Africa (b) South Africa (d) Horse shoe shaped corel reef (c) South America (d) Australia 30. Ecology deals with : [2006] 43. Which of the following latitudes passes through (a) The earth and planets India ? [2011] (b) The relationship between organisms and (a) Equator environment (b) Arctic Circle (c) The life under the sea (c) Tropic of Capricorn (d) Economical growth of poor people (d) Tropic of Cancer IIT JEE NEET eBooks: www.crackjee.xyz

G.K. – Geography C-9 44. Ghataprabha is a tributary of which one of the (b) magnetic poles following rivers ? [2012] (c) geographic north pole (a) Ganga (b) Indus (d) latitude 45º (c) Godavari (d) Krishna 52. Which place in India is called “The Golden Mine 45. Kiel Canal connects [2012] of Liverworts”? [2014] (a) Baltic Sea and North Sea (a) Eastern Himalayas (b) Red Ocean and Mediterranean Sea (b) Western Himalayas (c) Caribbean Sea and Pacific Sea (c) Western Ghats (d) Atlantic Ocean and Pacific Ocean (d) Eastern Ghats 46. The Victoria Falls in Africa is located on which 53. As per data, which two Indian cities are at river? [2012] highest risk of being devastated by an earthquake? (a) Zaire (b) Orange (a) Guwahati and Nagpur [2015] (c) Zambezi (d) Niger (b) Guwahati and Srinagar 47. The project similar to T.V.A. (Tennessee Valley (c) Jamnagar and Guwahati Authority) of U.S.A. in India is [2013] (d) Nagpur and Srinagar (a) Damodar Valley Project 54. Which is the hottest planet of the solar system. (b) Mahanadi Delta Project (a) Mercury (b) Venus [2016] (c) Ramganga Multipurpose Project (c) Jupiter (d) Saturn (d) Idukki Project 55. Which one of the following zones of the 48. Catch crops are [2013] atmosphere is rich in Ozone gas [2016] (a) crops palnted to attract certain insect pests (a) Mesosphere (b) Troposphere to be destroyed (c) Stratosphere (d) Ionosphere (b) crops planted to attract certain useful insects 56. A sand deposit extending into the mouth of a to be used for biological control of pests bay is a ? [2016] (a) Headland (b) Sea Stack (c) crops to be cut and fed green to the cattle (c) Split (d) Berm (d) substitute crops planted after the regular 57. Which is the Highest milk producer in India? crop has failed. [2016] 49. Which one of the following National Park/ (a) Madhya Pradesh (b) Andhra Pradesh Sanctuary is not in Rajasthan ? [2013] (c) Uttar Pradesh (d) Rajasthan (a) Sariska National Park 58. Which is the longest bridge in India? [2017] (b) Sambar Wildlife Sanctuary (a) Bandra Worli Sea Link (c) Rajaji National Park (b) Mahatma Gandhi Setu (d) Rhanthambore National Park (c) Dhola-Sadia Bridge 50. The black part of the moon is always calm and (d) Arrah-Chhapra Bridge dark which is called [2014] 59. Asia’s longest bi-direction road tunnel is located (a) Sea of tranquility (b) Ocean of storms in—————. [2017] (c) Area of storms (d) None of these (a) Jammu & Kashmir 51. The earth’s magnetic field always has a vertical (b) Sikkim component except at the [2014] (c) Maharashtra (a) magnetic equator (d) Himachal Pradesh

ANSWER KEY 1 (a) 2 (a) 3 (d) 4 (d) 5 (d) 6 (a) 7 (b) 8 (a) 9 (b) 10 (a) 11 (b) 12 (a) 13 (d) 14 (a) 15 (b) 16 (b) 17 (b) 18 (c) 19 (a) 20 (a) 21 (a) 22 (d) 23 (b) 24 (b) 25 (c) 26 (a) 27 (b) 28 (b) 29 (b) 30 (b) 31 (b) 32 (c) 33 (c) 34 (a) 35 (b) 36 (a) 37 (c) 38 (b) 39 (a) 40 (b) 41 (b) 42 (b) 43 (d) 44 (d) 45 (a) 46 (c) 47 (a) 48 (d) 49 (c) 50 (a) 51 (a) 52 (b) 53 (b) 54 (b) 55 (c) 56 (c) 57 (c) 58 (c) 59 (a) IIT JEE NEET eBooks: www.crackjee.xyz EBD_7100

C-10 Topicwise AIIMS Solved Papers – GENERAL KNOWLEDGE 4 G.K. – General Science

1. Halleys comet will be seen in : [1997] 10. Which one of the following function of the (a) 2062 (b) 2060 platelets occurs in our body? [2001] (c) 2068 (d) 2066 (a) It helps in breathing 2. Skylab was launched into space by the US in : (b) It helps in strengthening of gums (a) 1975 (b) 1974 [1997] (c) It helps in circulation of blood (c) 1973 (d) 1979 (d) It helps in clotting of blood 3. Which one of the following is responsible for 11. Which one of the following vitamin can be most the disease ‘dropsy’ ? [1999] easily synthesised inthe human body? [2001] (a) Argemone maxicana (a) Vitamin B (b) Vitamin C (b) Brassica oleracea (c) Vitamin A (d) Vitamin D (c) Oenothera lamarckiana 12. The spinning of the earth on its imaginary axis (d) Brassica campestris is known as : [2002] 4. Which of the following vitamin is required in (a) rotation (b) circulation bone formation ? [1999] (c) orbiting (d) revolution (a)D (b) B 13. Which organ of the body purifies the blood ? (c)C (d) A (a) Heart (b) Lungs [2002] 5. Ecology is the branch of science which deals with : (c) Kidneys (d) Pancreas [1999] 14. Positron Emission Tomography (PET) is one of (a) cell structure (b) soils surface the best methods of functional imaging because (c) balance of nature (d) human anatomy [2003] 6. The disease rheumatism effects : [1999] (a) isotopes of basic body elements are used (a) legs (b) ears for imaging (c) lungs (d) joints (b) isotopes with long half-lives are used 7. Weight of blood in the body is : [2000] (c) isotopes with short half-lives are used (a) about 7 litres in normal body of 7% of the (d) positrons are directly involved used in total body weight imaging (b) about 5 litres in normal body or 5% of the 15. Magnetic Resonance (MR) images are derived total body weight from the proton-bearing species present (c) about 10 litres in normal body or 10% of principally from water and [2003] the body weight (a) long alkane chain protons of the fatty acid (d) none of these moieties 8. The earthquake is measured by : [2001] (b) short alkane chain protons of the fatty acid (a) Lactometer (b) Seismograph moieties (c) Hygrometer (d) Barometer (c) long alkene chain protons of the fatty acid 9. AIDS is caused by : [2001] moieties (a) Helminth (b) Protozoa (d) short alkene chain protons of the fatty acid (c) Virus (d) Bacteria moieties IIT JEE NEET eBooks: www.crackjee.xyz

G.K. – General Science C-11

16. The following separation technique depends on 26. Biometry refers to : [2006] the molecular size of the protein : [2003] (a) Identification of humans by scanning face (a) chromatography on a carboxymethyl (CM) and fingerprints cellulose column (b) Measurement of mechanical displacement (b) iso-electric focusing in humans (c) gel filtration chromatography (c) A method of lie detection (d) chromatography on a diethylaminoethyl (d) Body length relationships across the (DEAE) cellulose column evolutionary scale 17. The approximate number of genes contained in 27. Which one of the following is one of the two the genome of Kalpana Chawala was : [2003] days when the sun rises exactly in the east? (a) 40,000 (b) 30,000 (a) 14th January (b) 21st March [2006] (c) 80,000 (d) 1,00,000 (c) 21st June (d) 23rd December 18. Thanatology is the science that deals with 28. X-rays were discovered by : [2006] (a) death in all its aspects [2003] (a) Wilhelm K. Roentgen (b) solving paternity of child (b) H. Kissinger (c) identification of living (c) Sir C.V. Raman (d) detection of lie (d) Meghnad Saha 19. The disease, Tetanus also known as [2003] 29. One ream of paper equal to : [2006] (a) Gangrene (b) Shingles (a) 100 –110 sheets (c) Lockjaw (d) Whooping cough (b) 256 sheets 20. It lives underwater for up to three years as (c) 180 -500 sheets nymph before emerging as a flying insect, Fossils (d) 1000 sheets of this insect dating back about 300 million years 30. Which of the following gases is most toxic? have been found : [2004] [2006] (a) scorpion fly (b) stone fly (a) Carbon dioxide (b) Carbon monoxide (c) caddis fly (d) May fly (c) Sulpher dioxide (d) None of these 21. Rayon fibre is manufactured from : [2004] 31. The gas used in the manufacture of vanaspati (a) petroleum (b) wood and pulp ghee is : [2006] (c) chemicals (d) naphtha (a) Helium (b) Oxygen 22. Maria Montessori's name is associated with : (c) Nitrogen (d) Hydrogen (a) Christian mission [2004] 32. Which mirror is used as a rear view mirror in (b) Child education vehicles? [2007] (c) Women's rights (a) plain (b) convex (d) Mission hospitals (c) concave (d) spherical 23. The noise produced in office is normally at the 33. Chemical change does not take place in level of : [2004] (a) souring of milk into curd [2007, 2011] (a) 20 db (b) 30 db (b) rusting of iron in atmosphere (c) 40 db (d) 60 db (c) burning of magnesium ribbon in air 24. Ascorbic acid is the chemical name of : [2005] (d) emitting of light by a red hot platinum wire (a) vitamin A (b) vitamin B 34. The process of transfer of heat by matter but (c) vitamin C (d) vitamin D without actual movement of the particles 25. The chemical formulae of Plaster of Paris' is themselves is called [2007] (a) CaSO . 2H O (b) Ca(OH) [2005] (a) conduction (d) convection 4 2 2 (c) radiation (b) none of these (c) CaSO4. 1/2H2O (d) C2H5OH IIT JEE NEET eBooks: www.crackjee.xyz EBD_7100

C-12 Topicwise AIIMS Solved Papers – GENERAL KNOWLEDGE

35. Only zero and one are used for operating 47. The purest form of water is [2009] [2007, 2011] (a) Tap water (b) Rain water (a) Calculator (b) Computer (c) Ground water (d) Distilled water (c) Abacus (d) Type writer 48. Which of the following means ‘change’ in Greek 36. Transistor is [2007] (a) Orangutam (b) Goat [2009] (a) semi conductor (b) inductor (c) Amoeba (d) Hydra (c) modulator (d) demodulator 49. The velocity of light in a medium is maximum for 37. Computer cannot [2007] that colour for which refractive index is [2010] (a) send message (b) read files (a) minimum (b) maximum (c) abstract thought (d) play music (c) optimum (d) very high 38. Which of the following is not a carbohydrate ? 50. Which of the following is/ are the law(s) of (a) wax (b) starch [2007] reflection of light? [2010] (c) sucrose (d) maltose (a) The angle of incidence is equal to the angle 39. Which of the following is an eye disease? of reflection. (a) hepatitis (b) measles [2007] (b) The incident ray, the normal to the mirror at (c) glaucoma (d) bronchitis the point of incidence and the reflected ray, 40. Which of the following is the vaccine for all lie in the same plane. tuberculosis? [2007] (c) Both (a) and (b). (a) OPT (b) BCG (d) The angle of incidence is never equal to (c) salk vaccine (d) rubella vaccine the angle of reflection. 41. Horns, nails and hair are [2007] 51. Which of the following halo-organic compounds (a) soluble fats is used as insecticide, germicide, soil fumigant (b) insoluble carbohydrates and deodorant? [2010] (c) keratin proteins (a) Benzene hexachloride (d) complex lipids (b) Chlorobenzene 42. If the blood group of one parent is AB and that (c) p-chlorobenzene of the other O, the possible blood group of their (d) All of the above child would be [2008] 52. Electronic configuration of deuterium atom is (a) A or B (b) A or B or O (a)1s2 (b) 2s1 [2010] (c) A or AB or O (d) A or B or AB or O (c)1s1 (d) 2s2 43. The vitamin that helps to prevent infections in 53. Which of the material interactions will not the human body is [2008] promote co-evolution? [2010] (a) vitamin A (b) vitamin B (a) Commensalism (b) Parasitism (c) vitamin C (d) vitamin D (c) Heterothallism (d) All of these 44. The gas which turns into liquid at the lowest 54. Bence Jones protein is associated with temperature among the following is [2008] (a) Lymphosarcoma (a) hydrogen (b) oxygen (b) Leukaemia (c) helium (d) nitrogen (c) Multiple myeloma 45. Which of the following disease is caused by a (d) All of these virus ? [2008] 55. A MODEM is connected in between a telephone (a) Diphtheria (b) Malaria line and a [2010] (c) Cholera (d) Hepatitis (a) Serial port 46. An ECG shows the functioning of the [2009] (b) Computer (a) Brain (b) Heart (c) Network (c) Lungs (d) Kidneys (d) Communication adaptor IIT JEE NEET eBooks: www.crackjee.xyz

G.K. – General Science C-13

56. A sequence of precise and unambiguous (b) hypoactivity of thymus instructions for solving a problem in a finite (c) hypoactivity of thyroid number of operations is referred as [2010] (d) hyperactivity of thymus (a) algorithm (b) address 67. Which one among the following is the correct (c) advice (d) all of these order of power consumption for light of equal 57. Polio vaccine was first prepared by [2011] intensity ? [2014] (a) J. Salk (b) L. Pasteur (a) CFL tube < Fluorescent tube < Incan- (c) G. J. Mendel (d) Watson descent bulb < Light emitting diode 58. The Saturn rings were discovered by [2011] (b) Light emitting diode < CFL tube < Fluores- (a) Copernicus (b) Newton cent tube < Incandescent bulb (c) Galileo (d) none of these (c) CFL tube < Fluorescent tube < Light emit- ting diode < Incandescent bulb 4 59. coulomb of charge contains ______(d) Incandescent bulb < Light emitting diode 25 < Fluorescent tube < CFL tube electrons. [2011] 68. When a particle and an antiparticle come in 15 18 (a) 10 (b) 10 contact with each other, they [2014] 20 (c) 10 (d) none of these (a) repell each other 60. Fish can survive inside deep frozen ponds (b) annihilate each other because [2011] (c) go undisturbed (a) Fish are cold blooded (d) spin about a common axis (c) They hibernate 69. Photoelectric effect is [2014] (c) Ice is a good conductor of heat (a) an instantaneous process (d) There is water at 4°C below frozen ice (b) delayed process 61. Which one of the following diseases is not (c) emission of protons caused by virus? [2012] (d) emission of neutrons (a) Chicken pox (b) Measles 70. Candles contains a mixture of [2014] (c) Poliomyelitis (d) Tetanus (a) Bees wax and paraffin wax 62. In the plant body, the water and minerals are (b) Bees wax and stearic acid transported by: [2012] (c) Paraffin wax and stearic acid (a) Bast (b) Collenchyma (d) Higher fatty acid (c) Parenchyma (d) Xylem 71. In chemical terms, what are alums used for 63. What is the main constituent of natural gas ? purifying water for drinking purposes ? [2014] (a) Methane (b) Ethane [2012] (a) Hydrated chlorides (c) Butane (d) Hydrogen (b) Double nitrate 64. Which one among the following has the highest (c) Double sulphates first ionisation energy ? [2012] (d) Nitrates of aluminium (a) Carbon (b) Fluorine 72. Each body segment of Earthworm is called [2014] (c) Nitrogen (d) Oxygen (a) Proglottid (b) Metamere 65. Chemical weathering is at its maximum in : (c) Scolex (d) Rostellum (a) hot and dry regions [2012] 73. The hydraulic brake used in automobiles is a (b) cold and humid regions direct application of [2014] (c) hot and humid regions (a) Archimedes’ principle (d) cold and dry regions (b) Torricellean law 66. Grave's disease is caused due to: [2013] (c) Bernoulli’s Theorem (a) hyperactivity of thyroid (d) Pascal’s law IIT JEE NEET eBooks: www.crackjee.xyz EBD_7100

C-14 Topicwise AIIMS Solved Papers – GENERAL KNOWLEDGE

74. Of the following, which is the fastest? [2014] (a) Tin (b) Mercury (a) CD-ROM (b) RAM (c) Lead (d) Zinc (c) Registers (d) Cache 80. The Bipolar disorder is related to which among 75. ‘Splenic fever’ is another name for [2014] the following? [2015] (a) FMD (b) Anthrax (a) Heart (b) Lungs (c) Cow pox (d) Mastitis (c) Brain (d) Liver 76. Big Bang theory explains ______? [2015] 81. Name the first antibiotic medicine discovered. (a) Origin of Universe (b) Origin of Sun. [2016] (c) Laws of physics. (d) None of above. (a) Penicillin (b) Auromycin 77. Which are the main gases present in Sun? (c) Streptomycin (d) Ampicilin (a) Hydrogen and Helium [2015] 82. The disease scarlet fever is caused by [2016] (b) Hydrogen and Argon (a) culex mosquito (c) Argon and Helium (b) housefly (d) Hydrogen and Carbon Dioxide (c) anopheles mosquito 78. Which of the following is a non metal that (d) haemolytic streptococcal infection remains liquid at room temperature? [2015] 83. In India, person unable to count fingers from a (a) Phosphorous (b) Bromine distance of how many metres is categorised as (c) Chlorine (d) Helium blind? [2017] 79. Which of the following metals forms an amalgam (a)3 (b) 6 with other metals? [2015] (c)7 (d) 8

ANSWER KEY 1 (a) 2 (c) 3 (a) 4 (a) 5 (c) 6 (d) 7 (a) 8 (b) 9 (c) 10 (d) 11 (d) 12 (a) 13 (b) 14 (c) 15 (a) 16 (c) 17 (b) 18 (a) 19 (c) 20 (b) 21 (b) 22 (b) 23 (c) 24 (c) 25 (c) 26 (b) 27 (b) 28 (a) 29 (c) 30 (b) 31 (d) 32 (b) 33 (d) 34 (a) 35 (c) 36 (a) 37 (c) 38 (a) 39 (c) 40 (b) 41 (c) 42 (a) 43 (c) 44 (a) 45 (d) 46 (b) 47 (d) 48 (c) 49 (a) 50 (c) 51 (c) 52 (a) 53 (a) 54 (d) 55 (b) 56 (a) 57 (a) 58 (c) 59 (b) 60 (d) 61 (d) 62 (d) 63 (a) 64 (b) 65 (c) 66 (a) 67 (b) 68 (b) 69 (a) 70 (c) 71 (c) 72 (d) 73 (d) 74 (c) 75 (b) 76 (a) 77 (a) 78 (b) 79 (b) 80 (c) 81 (a) 82 (d) 83 (b) IIT JEE NEET eBooks: www.crackjee.xyz

G.K. – General Awareness C-15 5 G.K. – General Awareness

1. In which of the following year Olympic Games 12. The largest airport in the world is situated in were not played ? [1997] (a) Saudi (b) USSR [1998] (a) 1936 (b) 1925 (c) Russia (d) Denmark (c) 1916 (d) 1932 13. Martyr day is observed on : [1998] 2. The male cricketer who scored maximum runs in (a) 18 April (b) 12 May one day cricket match : [1997] (c) 29 August (d) 30 January (a) Azharuddin (b) Vivian Richards 14. National Housing Bank is subsidary of :[1998] (c) (d) Sayeed Anwar (a) R.B.I. (b) I.D.B.I. 3. Writer of Future Shock is [1997] (c) I.C.I.C.I. (d) S.B.I. (a) Bernad shaw (b) Bertrand 15. What is Nikkie? [1998] (c) Sewitzer (d) Alwyn Toffler (a) Index of shares in Tokyo stock exchange 4. Deep Blue is a : [1997] (b) Rate of interest by Bank of tokyo (a) computer which gives weather report (c) A private firm situated in Japan (b) computer operating system (d) Currency of Korea (c) blue whale 16. ‘Frank Worrel was associated with which of the (d) computer which plays chess following sport ? [1998] 5. The ‘AIDS’ day is observed on : [1997] (a) Hockey (b) Football (a) 10th Dec. (b) 21st Dec. (c) Swimming (d) Cricket (c) 20 Dec. (d) 1st Dec. 17. India’s multi largest surface to air missile is 6. Asian Games were held in India : [1997] known as : [1998] (a) 1961 and 1984 (b) 1956 and 1986 (a) Nag (b) Agni (c) 1962 and 1984 (d) 1957 and 1982 (c) Prithvi (d) Akash 7. Ruble is the currency of : [1997] 18. Beighton cup in India is associated with : [1998] (a) USSR (b) Denmark (a) Cricket (b) Foot ball (c) Volley ball (d) Hockey (c) Germany (d) Japan 19. Which one of the following book is written by 8. The trade name IBM associated with : [1997] Sarojini Naidu ? [1998] (a) scooters (b) refrigerators (a) Gandevata (b) Chitra (c) cars (d) computers (c) Broken wing (d) Great Tragedy 9. Which plant was responsible for Bhopal Gas 20. The common wealth games of 1998 were hosted by Tragedy? [1997] (a) Singapore (b) England [1998] (a) BHEL (c) Malaysia (d) Australia (b) Union Carbide 21. The old name of Thailand is : [1998] (c) Hindustan Insecticides (a) Combodia (b) Siam (d) None of these (c) Zaire (d) Persia 10. Libra is the currency of which of the following 22. Which one of the following country is not the country? [1998] member of SAARC ? [1999] (a) Spain (b) Vietnam (a) Maldeiv (b) Bangladesh (c) Philippines (d) Turkey (c) Nepal (d) Myanmar 11. Which one of the following is the biggest cave 23. Hari Prasad Chaurasia is related to which of the temple in India ? [1998] following instrument ? [1999] (a) Ajanta (b) Tuljapur (a) Tabla (b) Flute (c) Ellora (d) Parli (c) Violin (d) Santoor IIT JEE NEET eBooks: www.crackjee.xyz EBD_7100

C-16 Topicwise AIIMS Solved Papers – GENERAL KNOWLEDGE 24. Which of the following missile of India has the (c) Florence Nightinagale longest range ? [1999] (d) None of these (a) Akash (b) Prithvi 38. Rial is the currency of : [2000] (c) Pinaka (d) Both (b) and (c) (a) Afganistan (b) Iran 25. Which one of the following is the cave temple in (c) Saudi Arabia (d) Jordan India ? [1999] 39. The writer of "Daughter of East" is : [2001] (a) Ellora (b) Ajanta (a) Indira Gandhi (b) Benazier Bhutto (d) Parli (d) Tuljapur (c) Amrita Pritam (d) Marget Tatcher 26. Which one of the following pair is incorrect ? 40. Which one of the following is the largest bridge? (a) Kapil – Cricket [1999] (a) Seawise (b) Petronas [2001] (b) M. F. Husain – Actor (c) Strahov (d) George (c) Abul Fazal – Author 41. Teacher day is celebrated on : [2001] (d) Feroz Gandhi – Politics (a) 5th September (b) 16 August 27. The great poetry ‘Madhushala’ was composed by : (c) 21 September (d) 1st April (a) Mulk Raj Anand [1999] 42. Who discovered the sea route to India?[2001] (b) Harivansh Rai Bachchan (a) Vasco de Gama (b) Columbus (c) Mahadevi Verma (c) Magellan (d) Hopkins (d) Surender Sharma 43. Garba dance is a dance style of : [2001] 28. Grand prix is a term associated with : [1999] (a) Gujrat (b) Uttar Pradesh (a) Chess (b) Table tennis (c) Nagaland (d) Bihar (c) Hockey (d) Badminton 44. A former cricketer after whose name no 29. William’s cup is related to : [2000] (a) basket ball (b) table tennis championship has been started in India : [2001] (c) volley ball (d) foot ball (a) G. K. Naidu (b) Daleep Singh 30. Full form of H.T.T.P. is : [2000] (c) Lala Amarnath (d) (a) Hyper Terminal Transformation 45. Currency note bearts the signature of the : [2001] (b) Hyper Text Transfer Protocol (a) Finance Minister (c) High Technology Test Principles (b) Governor, Reserve Bank of India (d) Hyper Text Training Program (c) Cabinet Secretary 31. Tallest tower in the world is : [2000] (d) President (a) C.N.Tower (b) Kutub Minar 46. When was the first football world cup held? (c) Angel (d) None of these (a) 1930 (b) 1950 [2002] 32. ‘The Satanic Verses’ a controversial book is (c) 1954 (d) 1968 written by : [2000] 47. 'Human Organ Development Centre for (a) Gyani Jail Singh (b) Khuhwant Singh Transplantation' is going to be established in (c) Kuldip Nayyar (d) Salman Rushdie India at : [2002] 33. World Tourism day was declared on : [2000] (a) Vellore (b) Mumbai (a) 1st October (b) 11th February (c) Hyderabad (d) Chennai (c) 27th September (d) none of these 48. July 11 is celebrated as [2002] 34. Which one of first Indian missile (earth to earth) (a) Doctor's Day was tested successfully from Shri Hari Kota ? (b) Van Mahotsava Day (a) Prithvi (b) Nag [2000] (c) AIDS Day (c) Agni (d) none of these (d) World Population day 35. “Divine” comedy was written by : [2000] 49. Which one of the classical dance forms (a) Goethe (b) Milton originated in Andhar Pradesh? [2002] (c) Dante (d) Shakespears (a) Odissi (b) kathakali 36. ‘Abhigyan shakuntalam’ was written by : [2000] (c) Kuchipudi (d) Bharat Natyam (a) Surdas (b) Tulsidas 50. Who is called Nightingale of India [2002] (c) R.N. Tagore (d) Kalidas (a) Indira Gandhi 37. Who was known as the “ Lady of the Lamp”? (b) Lata mangeshker (a) Sarojini Naidu [2000] (c) Asha Bhonsle (b) Joan of Arc (d) Sarojini Naidu IIT JEE NEET eBooks: www.crackjee.xyz

G.K. – General Awareness C-17 51. In Internet what does 'http' mean? [2003] (c) it is interdisciplinary i.e., involves sociologists, (a) High Transfer Text Protocol economist, computer sciences etc. (b) Highest Transfer Text Protocol (d) it plans to produce wearable computer (c) Hyper Text Transfer Protocol 62. Who one of the following is a medical doctor ? (d) Hyper Transfer Text Protocol (a) Samuel Taylor Coleridge [2004] 52. The India-born US physicist who was awarded (b) John Webster the Nobel Prize in Physics for his work on (c) Somerset Maugham astrophysics is : [2003] (d) Thomas Gray (a) H. G. Khorana 63. Who was the first Indian to win Nobel Prize? (b) Subrahmanyam Chandrashekhar (a) Mother Terresa [2005] (c) Sivaramakrishna Chandrashekhar (b) C.V. Raman (d) C.V Raman (c) Ravindra Nath Tagore 53. Which German Physicist invented the electron (d) Amartya Sen microscope which won him the 1986 Nobel Prize 64. Oldest religious text in the world is : [2005] in Physics ? [2003] (a) Rig Veda (b) Sama Veda (a) E. Ruska (b) Van't Hoff (c) Yajur Veda (d) Atharva Veda (c) J. H. D. Jensen (d) Eugene P. Wigner 65. Who is the inventor of 'Insulin'? [2005] 54. Who was the first Indian to be awarded the world (a) Loard Lister (b) Jonos Salk Food Prize in 1987? [2003] (c) Ronald Ross (d) Banting and Best (a) M. S. Swaminathan 66. Who is the author of 'An Area of Darkness'? (b) Sunderlal Bahuguna (a) Nirad C. Choudhari [2005] (c) (b) Vikram Seth (d) B. R. Barwale (c) V.S. Naipaul 55. Israel's Prime Minister Yitzhak Rabin won the (d) B.C. Chatterjee 67. Which one of the following literary titles is Nobel Prize for : [2003] correctly matched with its author? [2006] (a) Peace (b) Literature (a) Ramayan – Tulsidas (c) Chemistry (d) Economics (b) Mahabharat – Vedvyas 56. Who was the world's first space tourist? [2003] (c) Kumarsambhav – Ravidas (a) Desmond Rickett (b) Dennis Tito (d) Shakuntala – Bhushan (c) Igor Kajlnikov (d) Li Wang 68. The jungle in Rudyard Kippling's Jungle book, 57. How many "World Cultural Hertitage Sites" are describes which part of Indian forest ? [2006] in India ? [2003] (a) Central Indian forest near Satpura range (a) 10 (b) 17 (b) Uttranchal thick forest (c) 14 (d) 15 (c) Himalayan Forest in Himachal 58. Birbal Sahni was a : [2004] (d) Nilgiri Jungles (a) zoologist 69. Which of the following honour is given by (b) founder of Central Drug Research Institute UNESCO? [2006] (c) ornithologist (a) The Kalinga Prize (d) paleobotanist (b) Magasay Award 59. Primary sector refers to : [2004] (c) Pulitzer Prize (a) industry (b) agriculture (d) Order of the Golden Ark Award (c) trade (d) banks 70. 'Body line' in the cricket refers to : [2006] 60. All of the following won the title of 'Miss World' (a) Bowling that hits the body except : [2004] (b) The line of body chose to line (a) Lara Dutta (b) Aishwarya Rai (c) The white line on ground within which the (c) Yukta Mukhi (d) Priyanka Chopra player sands 61. All of the following are correct about 'Media (d) The line of moving ball Lab' except : [2004] 71. 'Hindu view of life' is written by : [2006] (a) the third media lab is situated in india (a) S.Radhakrishnan (b) R.K. Narayan (b) it is supported by government funding (c) V.D. Savarkar (d) John Ruskin IIT JEE NEET eBooks: www.crackjee.xyz EBD_7100

C-18 Topicwise AIIMS Solved Papers – GENERAL KNOWLEDGE 72. The compilation “Meri Ekyawan Kavitayen’s is 84. Environment Day is celebrated on [2009] by [2007] (a) 5th June (b) 11th Sep. (a) A.B. Vajpayee (c) 20th Feb. (d) 5th Oct. (b) Harivanshrai Bachchan 85. Among which of the below is a script? [2009] (c) Dharam Vir Bharti (a) Hindi (b) English (d) Shiv Mangal Singh Suman (c) Gurumukhi (d) Sanskrit 73. Who was known as “Nightingale of India”? 86. On which cartoon character’s name a gene is [2007] also named : [2009] (a) Vijaylaxmi Pandit (b) Sarojini Naidu (a) Tintin (b) Sonic (c) Suraiya (d) None of these (c) Asterix (d) Obedix 74. Kalidas was [2007] 87. Which of the following river’s name means (a) A poet during the Gupta period ‘Elephant -river’ [2009] (b) A dramatist during Harshvardhana’s reign (a) Krishna (b) Errabadi (c) An astronomer during Gupta period (c) Godavari (d) Mahanadi (d) None of the above 88. Reserve Bank of India’s emblem carries the 75. India's first human DNA Bank has been setup in sketch of a tiger and a tree. What kind of tree is (a) Patna (b) New Delhi [2008] depicted in this emblem? [2010] (c) Lucknow (d) Kolkata (a) Palm tree (b) Cactus 76. What is the motto incorporated under our (c) Banyan (d) Banana National Emblem ? [2008] 89. The term 'smash' in sports is associated with (a) Satyam Shivam (a) Lawn Tennis (b) Badminton (b) Satyam Shivam Sundaram (c) Satyameva Jayate (c) Volleyball (d) Hockey (d) Jai Hind 90. Match List-1 (personality) with List-II (their field 77. Bhabha Atomic Research Centre is situated in of activity) and select the correct combination/ (a) Delhi (b) Mumbai [2009] option : [2010] (c) Chennai (d) Hyderabad List - I List - II 78. FERA in India has been replaced by [2009] (A) Yamini Krishnamurthy (1) Paintings (a) FEPA (b) FEMA (B) Wasim Jaffar (2) Politics (c) FENA (d) FETA (C) Raja Ravi Verma (3) Bharat Natyam 79. The missile Agni II of India is a [2009] (D) Rahul Gandhi (4) Cricket (a) Nuclear missile Code : (b) Surface-to -air missile (A) (B) (C) (D) (c) Surface-to-surface missile (a)3 41 2 (d) Surface-to-sea missile (b) 3 42 1 80. is a maestro with which (c)2 43 1 instrument? [2009] (d) 4 21 3 (a) Violin (b) Sitar 91. Which of the following sites has been included (c) Sarod (d) Sarangi in UNESCO’s list of World Heritage sites?[2010] 81. With which sport do you associate the name of (a) Akbar’s Tomb at Sikandara Geet Sethi? [2009] (b) Gateway of India (Mumbai) (a) Golf (b) Billiards (c) Agra Fort (c) Lawn Tennis (d) Cricket (d) Bibi ka Maqbara (Aurangabad) 82. In which of the following games, left hand is not 92. With which program, the slogan ‘Do Boond to be used? [2009] Jindgi Ki’ is associated? [2010] (a) Tennis (b) Hockey (a) Blood Donation (b) Save Water (c) Polo (d) cricket (c) Pulse Polio (d) Pollution Control 83. Among countries given below, Albert Einstein 93. On whose birthday is Teacher’s Day celebrated? had citizenship of which country along with (a) S. Radhakrishnan [2011] Germany and US? [2009] (b) Maulana Abul Kalam Azad (a) Sweden (b) Austria (c) Rajendra Prasad (c) Israel (b) Netherlands (d) Jawaharlal Nehru IIT JEE NEET eBooks: www.crackjee.xyz

G.K. – General Awareness C-19 94. Dronacharya Award is given to [2011] 105. Which of the following is not correctly paired? (a) fire fighting operation (a) Jwala Gutta — Tennis [2013] (b) archery (b) — Cricket (c) outstanding coaching in sports and games (c) Harbhajan Singh — Kabaddi (d) outstanding coaching in athletics (d) Saina Nehwal — Badminton 95. Yuan is the currency of [2011] 106. Which one of the following is an example for a (a) China (b) Indonesia non-economic good? [2013] (c) Thailand (d) Japan (a) Doctor’s service (b) Teacher’s service 96. National game of Australia is [2011] (c) Mother’s service (d) Banker’s service (a) base -ball (b) cricket 107. Which one of the following does not match? (c) rugby football (d) hockey (a) Hindu Marriage Act : 1955 [2013] 97. Which one of the following pairs is not correctly (b) Medical Termination of Pregnancy Act : 1971 matched ? [2012] (c) Domestic Violence on women Act : 1990 State/U.T. High Court (d) Cruelty against Women : 1995 (a) Goa – Bombay 108. Ward Cunningham is famous for developing for (b) Andaman and – Calcutta the first time [2013] Nicobar Islands (a) the free encyclopedia on Internet (c) Sikkim – Guwahati (b) a computer language called 'Java' (d) Pondicherry – Madras (c) a digital camera 98. Who is the author of Das Kapital ? [2012] (d) a software which can take Hebrew language (a) Karl Marx (b) Friedrich Engels as input (c) Joseph Stalin (d) Vladimir Lenin 109. Which of the following is/are instance(s) of 99. Which National Highway connects Delhi and violation of human rights? [2013] Mumbai? [2012] 1. A person was detained by the security (a) NH 6 (b) NH 8 forces while going for casting vote in (c) NH 10 (d) NH 12 Parliamentary Election. 2. A civilian was killed by the army while 100. Who among the following advocated scientific undertaking combing operation. socialism? [2012] Select the correct answer using the code given (a) Robert Owen below: (b) Proudhon Pierre Joseph Code: (c) Karl Marx (a) 1 only (b) 2 only (d) Saint Simon Henri Claude (c) Both 1 and 2 (d) Neither 1 nor 2 101. Where are the headquarters of the UNO ? [2012] 110. Navroze is a festival celebrated in India by the (a) Geneva (b) The Hague (a) Hindus (b) Muslims [2013] (c) New York (d) Paris (c) Parsis (d) Christians 102. For controlling inflation, the central bank should 111. Who is the author of the book ‘Conquest of Self’? (a) sell Government securities in the open (a) Aurobindo Ghosh [2014] market [2013] (b) Rabindra Nath Tagore (b) lower the bank rate (c) Mahatma Gandhi (c) purchase Government securities in the open (d) S. Radhakrishnan market 112. Who is known as the ‘Piccaso of India’? [2014] (d) lower the reserve ratio of the banks (a) Amrita Shergil (b) M.F. Hussain 103. United Nations Conference on Environment and (c) Sudhir Vyas (d) Shafqat Hussain Development is called [2013] 113. Which is the capital of Mali ? [2014] (a) Earth Summit (b) Water Summit (a) Mopti (b) Bamako (c) Air Summit (d) Resource Summit (c) Cairo (d) Nairobi 104. The five permanent members of the U.N. 114. Which one of the following though called a Security Council are [2013] garden is infact, not a garden? [2014] (a) China, France, Russia, U.K. and U.S.A. (a) Vrindavan Garden of Mysore (b) China, Canada, France, U.S.A and Germany (b) Hanging Garden of Mumbai (c) China, Germany, Russia, U.K. and U.S.A. (c) Eden Garden of Kolkata (d) China, Germany, U.S.A., U.K and Canada (d) Shalimar Garden of Kashmir IIT JEE NEET eBooks: www.crackjee.xyz EBD_7100

C-20 Topicwise AIIMS Solved Papers – GENERAL KNOWLEDGE 115. Who advocated the adoption of ‘PURA’ model (c) Liquid Ratio to eradicate rural poverty? [2014] (d) Statutory Ratio (a) Dr. A. P. J. Abdul Kalam 122. Which movie has won maximum awards at the (b) Sri Abhijit Sen International Indian Film Awards? [2015] (c) Maulana Abdul Kalam Azad (a) Queen (b) Ek Villain (d) Prof. A. M. Patha (c) Haider (d) PK 116. Who caught the wicket of Sachin Tendulkar in 123. Who has made the first attempt to initiate his last match? [2014] economic planning in India [2016] (a) Chris Gayle (a) M. Visvesvaraya (b) Zakir Hussain (b) Darren Sammy (c) Amarty Sen (d) Narendra MOdi (c) Shiv Narayan Chandrapaul 124. When was decimal coinage introduced in India? (d) Dwane Bravo (a) 1949 (b) 1965 [2016] 117. Gilt-edged market stands for…..? [2015] (c) 1935 (d) 1957 (a) bullion market 125. Who founded the Ayurveda system of medicine? (b) market of government securities [2016] (c) market of guns (a) Susruta (b) Thirumoolar (d) market of pure metals (c) Atreya (d) Agasthiya 118. The best indicator of economic development of 126. What is the full form of GST? [2017] a country is ______? [2015] (a) General Sales Tax (a) Its agriculture (b) Goods and Sales Tax (b) Its transport (c) Goods and Services Tax (c) Its gross production (d) General Services Tax (d) Its per capita income 127. ICMR signs agreement to provide healthcare 119. The data of estimation of India's National income is through solar based solutions.What is the full issued by ? [2015] form of ICMR [2017] (a) Planning Commision (a) Indian Council of Medical Research (b) National Data Center (b) International Council of Medical Research (c) Central Statistical Organisation (c) India Council of Medical Research (d) None of above (d) Indian Counciling of Medical Research 120. First five year plan in India was from? [2015] 128. Which tiger reserve in Madhya Pradesh has (a) 1947 – 1952 (b) 1949 – 1954 become the first tiger reserve in India in officially (c) 1950 – 1955 (d) 1951 – 1956 introduce a mascot which has been named 121. The banks are required to maintain a certain ratio Bhoorsingh the Barasingha? [2017] between their cash in hand and total assets. This (a) Kanha Tiger Reserve is called ? [2015] (b) Kaziranga Tiger Reserve (a) Statutory Liquid Ratio (c) Buxa Tiger Reserve (b) Cash Reserve Ratio (d) Sunderbans Tiger Reserve

ANSWER KEY 1 (c) 2 (d) 3 (d) 4 (d) 5 (d) 6 (d) 7 (a) 8 (d) 9 (b) 10 (b) 11 (b) 12 (a) 13 (a) 14 (a) 15 (b) 16 (b) 17 (b) 18 (a) 19 (b) 20 (c) 21 (b) 22 (d) 23 (b) 24 (b) 25 (a) 26 (b) 27 (b) 28 (a) 29 (a) 30 (b) 31 (a) 32 (d) 33 (c) 34 (a) 35 (c) 36 (d) 37 (c) 38 (b) 39 (b) 40 (d) 41 (a) 42 (a) 43 (a) 44 (c) 45 (b) 46 (a) 47 (c) 48 (d) 49 (a) 50 (d) 51 (c) 52 (b) 53 (a) 54 (a) 55 (a) 56 (b) 57 (b) 58 (d) 59 (b) 60 (a) 61 (d) 62 (c) 63 (c) 64 (a) 65 (b) 66 (c) 67 (b) 68 (a) 69 (a) 70 (a) 71 (a) 72 (a) 73 (b) 74 (a) 75 (c) 76 (c) 77 (b) 78 (b) 79 (b) 80 (c) 81 (b) 82 (c) 83 (b) 84 (a) 85 (c) 86 (b) 87 (b) 88 (a) 89 (b) 90 (a) 91 (c) 92 (c) 93 (a) 94 (c) 95 (a) 96 (b) 97 (c) 98 (a) 99 (b) 100 (c) 101 (c) 102 (a) 103 (a) 104 (a) 105 (c) 106 (c) 107 (c) 108 (a) 109 (c) 110 (c) 111 (c) 112 (b) 113 (b) 114 (c) 115 (a) 116 (b) 117 (b) 118 (d) 119 (c) 120 (d) 121 (a) 122 (c) 123 (a) 124 (d) 125 (a) 126 (c) 127 (a) 128 (a) IIT JEE NEET eBooks: www.crackjee.xyz

G.K. – Current Affairs G-21 6 G.K. – Current Affairs

1. Noble Prize for physiology and medicine for the 10. In the year (2003) the chemistry Nobel Prize was year 1998 was given for the discovery of : awarded to the following work : [2004] (a) Prion (b) Viagra [1999] (a) aquaporins (b) Na++ channels (c) Streptomycin (d) Invading germs (c) Ca++channels (d) methyl chavicol 2. How many countries adopted Euro currency? 11. Who is known as 'Deshbandhu'? [2004] (a) 11 (b) 6 [1999] (a) Chandra Shekhar (b) C.R Das (c)9 (d) 8 (c) A.O. Hume (d) Annie Besant 3. Present speaker in 12th Lok Sabha is : [1999] 12. The Ramakrishna Mission was established by (a) G. M. C. Balayogi (a) Swami Vivekananda [2004] (b) Nazma Haptullah (b) Ramakrishna Paramhansa (c) P. A. Sangama (c) Swami Dayanand Saraswati (d) Murali Manohar Joshi (d) None of these 4. Miss Universe event 2000 was held at [2001] 13. In which space Shuttle Kalpana Chawla killed (a) Peuotro Rico (a) Discovery (b) Columbia [2005] (b) London (c) Sputnik (d) None of the above (c) Paris 14. A very much publicized treatment method (d) Nicosia (Cyprus) "DOTS" is being adopted for the cure of : 5. Who was chosen ‘Time Magazines Person’ for (a) Dementia [2006] the year 2001? [2002] (b) Tetanus (a) Collin Powel (b) Mike Monore (c) Tuberculosis (c) George Bush (d) Rudolf Guilani (d) Sexually transmitted disease 6. Which of the following Hindi Indian movies 15. Which of the following Indian circket player after Language film for Oscar Award 2002? [2002] India-Pakistan ODI (one-day International) at (a) Mansoon Wedding Abudhabi became no. 1 ODI batsman in the ICC (b) Dil Chahata Hai (international Cricket Club) ranking : [2006] (c) Gadar Ek Prem Katha (a) (b) Yuvraj (d) Lagaan (c) Sachin Tendulkar (d) M.S. Dhoni 7. Men's Single US Open, 2001 Championship won 16. Who is the highest wicket taker in Indian Cricket by : [2002] team ? [2007] (a) Leyton Herwitt (b) Pete Sampras (a) Javagal Srinath (b) (c) Safin (d) Stefan Edberg (c) Maninder Singh (d) 8. Who is CEAT International Cricketer of the year 17. India test-fired Agni III on [2008] 2000 -2001? [2002] (a) May 7, 2008 (b) March 20, 2008 (a) Sachin Tendulkar (c) May 20, 2008 (d) March 7, 2008 (b) Muttiah Muralitharan 18. The train which was started on April 14, 2008 (c) Shane Warne (d) Brayan Lara between Kolkata (India) and Dhaka 9. When seen from earth, which of the following (Bangladesh) has been named ? [2008] planet eclipsed (crossed a cross) of the sun on 7 (a) Shanti Express May, 2003? [2003] (b) Maitri Express (a) Mercury (b) Uranus (c) Aman Express (c) Saturn (d) Jupiter (d) Samjhauta Express IIT JEE NEET eBooks: www.crackjee.xyz EBD_7100

G-22 Topicwise AIIMS Solved Papers – GENERAL KNOWLEDGE 19. Which cricketer holds the record for scoring 29. Schemes run under the National Rural highest number of runs in a test match innings? Employment Guarantee Act are sponsored[2013] [2008] (a) by the Central Government alone (a) Gary Sobers (b) Vivian Richards (b) partly by the Central Government and (c) (d) Brian Lara partly by the State Government 20. What is the name given to the common currency (c) by Centre, State and Panchayat bodies of the European Union? [2009] together (a) Rupee (b) Euro (d) on public-private partnership basis (c) Rouble (d) Franc 30. ‘Catch Me If You Can’ is a Hollywood movie 21. Who has written “Two Lives’? [2010] made on the life of Frank Abagnale who was (a) Vikram Seth (b) Kiran Desai (c) Arundhati Roy (d) Khushwant Singh (a) a famous lawyer [2013] 22. Who amongst the following is the ICC Cricketer (b) a famous swimmer of the year? [2011] (c) a famous sprinter (a) Sachin Tendulkar (b) Kevin Pietersen (d) an infamous imposter (c) M. S. Dhoni (d) Sanath Jayasuriya 31. Who among the following was awarded the first (e) None of these Tagore Award for Cultural Harmony for the year 23. ‘Goodbye Shahzadi’ is a book written by- 2012 ? [2013] [2011] (a) (b) Ravi Shankar (a) Shyam Bhatia (b) Ashok Mehra (c) Nikhil Banerjee (d) Bhimsen Joshi (c) Janardhan Thakur (d) Arun Gandhi 32. Which team has won the Men's National Hockey (e) None of these Championship ? [2015] 24. Who is Robert Zoelick? [2011] (a) Karnataka (b) Telangana (a) IMF Chief (c) Indian Railways (d) Madhya Pradesh (b) World Bank President 33. Which bank won the Asian Banker Achievement (c) ADB President Award ? [2015] (d) Chief UNIDO (a) ICICI Bank (e) None of these (b) Bharatiya Mahila Bank 25. Which country’s Parliament has banned ‘Burqa’ by passing a law of 14th September, 2010? (c) State Bank of Mysore (a) USA (b) France [2011] (d) Axis Bank (c) Italy (d) Holland 34. Which among the following state does not 26. Which among the following is not correct with implement the National Food Security Act regard to Sampoorna Garmeena Rozgar Yojana? recently empowers 87% of the priority (1) The cash component of the programme is households? [2016] borne exclusively by the Central (a) Manipur Government. (b) Sikkim (2) Foodgrains are provided free of costs to (c) Nagaland the States/Union Territories. (d) Arunachal Pradesh Select the answer unsing the code given below: 35. Smt. Maneka Sanjay Gandhi launched Beti (a) (1) only (b) (2) only [2012] Bachao Beti Padhao Scheme in additional how (c) Both (1) and (2) (d) Neither (1) nor (2) many districts of the country? [2016] 27. Which of the following countries does not (a) 66 (b) 67 belong to the group of G-8 nations? [2012] (c) 51 (d) 61 (a) Italy (b) Canada 36. Which Government banned digging of bore (c) Germany (d) Australia 28. Who is author of One Night @ the Call Centre ? wells beyond 200ft to prevent the declining of [2013] Ground water level? [2016] (a) Vikram Seth (b) Chetan Bhagat (a) Tamilnadu (b) B.Karnataka (c) Anurag Mathur (d) Robin Sharma (c) Madhya Pradesh (d) Maharashtra IIT JEE NEET eBooks: www.crackjee.xyz

G.K. – Current Affairs G-23 37. India is collaborated with which country on (b) It has been developed by the National tackling offshore tax evasion and increase Co- Payments Corporation of India operation in sharing of cross-border tax (c) The receiver of transferred money needs Information? [2016] to compulsorily be a BHIM user (a) United Kingdom (b) United States (d) A person without a bank account cannot (c) Japan (d) China use BHIM 38. Which country has been declared as drought 42. WannaCry’, recently seen in the news, is which Emergency for first time ever in the History? one of the following types of malware? [2017] [2016] (a) Virus (b) Worm (a) Hungary (b) South Africa (c) Trojan Horse (d) Ransomware (c) Zimbabwe (d) El Salvador 43. According to Urban Jyoti Abhiyaan or Urja app 39. Who is the defence minister of India? [2017] which states top the list in providing (a) Sushma Swaraj (b) George Anthony uninterrupted power? [2017] (c) Prithviraj Chauhan (d) Arun Jaitley (a) Jharkhand, Uttarakhand and Assam 40. Who is the new chief of the Chief of the Army (b) Tamil Nadu, Rajasthan and West Bengal Staff (COAS) of the Indian Army? [2017] (c) Kerala, Maharashtra and Rajasthan (a) Balbir Singh Sandhu (d) Punjab, Assam and Andhra Pradesh (b) Velu Nair 44. Who has been honoured as the Wisden’s leading (c) Bipin Rawat cricketer in the World for 2016? [2017] (d) Amit Sarin (a) Chris Woakes (b) Misbah-ul-Haq 41. The Central government has recently announced (c) Younis Khan (d) Virat Kohli a new digital payments app called BHIM. Which 45. How many ‘priority pathogens’ are listed by of the following statements about BHIM is not the World Health Organization for the first time true? [2017] that pose the greatest threat to human life? (a) The BHIM digital payments app is based (a) 10 (b) 11 [2017] on the Unified Payments Interface (c) 12 (d) 15

ANSWER KEY 1 (b) 2 (a) 3 (c) 4 (a) 5 (d) 6 (d) 7 (c) 8 (b) 9 (a) 10 (a) 11 (b) 12 (a) 13 (b) 14 (c) 15 (d) 16 (b) 17 (a) 18 (b) 19 (d) 20 (b) 21 (a) 22 (a) 23 (a) 24 (b) 25 (b) 26 (a) 27 (d) 28 (b) 29 (a) 30 (d) 31 (b) 32 (c) 33 (b) 34 (b) 35 (d) 36 (d) 37 (b) 38 (d) 39 (d) 40 (c) 41 (d) 42 (d) 43 (c) 44 (d) 45 (c) IIT JEE NEET eBooks: www.crackjee.xyz EBD_7100

7 G.K. – Logical Thinking

1. A duck, a goose, a goat, and a horse all entered 4. Find out the number of students who play only the bar n at different times one day last week. cricket. [2017] [2017] 40 (1) A mammal entered the bar n first. Cricket Tennis (2) The duck entered before the goose. Players 16 Players (3) The goose entered ahead of the horse. 25 22 Who entered the bar n first? (a) A goat (b) A duck (a) 25 (b) 18 (c) A goose (d) A horse (c)9 (d) 41 2. Choose or find odd word [2017] 5. Arrange the following words as per order in the (a) Piano (b) Guitar dictionary [2017] (c) Sitar (d) Violin 1. Eyelid 2. Eyeless 3. Eyesore 4. Eyesight 3. If Ram lives east of a post office. In the north of (a) 2, 1, 4, 3 (b) 2, 1, 3, 4 post office is big bazar. Then what is the position (c) 4, 3, 1, 2 (d) 4, 2, 3, 1 of Ram’s house with respect to the post office? 6. If the day before yesterday was Thursday, when [2017] will Sunday be? [2017] (a) North-west (b) North-east (a) Today (c) South-west (d) South-east (b) Two days after today (c) Tomorrow (d) Day after Tomorrow

ANSWER KEY 1 (d) 2 (a) 3 (d) 4 (a) 5 (a) 6 (c) IIT JEE NEET eBooks: www.crackjee.xyz Mock AIIMS

Time : 3½ Hrs. Questions : 200 Maximum Marks : 200 Negative Marking : –1/3 for each incorrect answer.

6. A charge q is fixed. Another charge Q is brought SECTION I - PHYSICS near it and rotated in a circle of radius r around 1. A block is kept on a inclined plane of inclination it. Work done during rotation is of length . The velocity of particle at the bottom T A q.Q of inclined is (the coefficient of friction is P ) (a) zero (b) SH0r4 2/1 (a) A cos(g2[ sin TTP )] q.Q (b) A(sing2 cos TPT ) (c) (d) None of these H0r2 (c) (sing2 cos TPT ) A 7. Advantage of optical fibre (d) A(cosg2 sin TPT ) (a) high bandwidth and EM interference 2. If earth is supposed to be a sphere of radius R, if (b) low band width and EM interference (c) high band width, low transmission capacity g30 is value of acceleration due to gravity at lattitude of 30° and g at the equator, the value of and no EM interference g – g30 is (d) high bandwidth, high data transmission 1 3 capacity and no EM interference. (a) Z2R (b) Z2R 4 4 8. In an electromagnetic wave, direction of 1 propagation is in the direction of (c) Z2 R (d) Z2R 2 (a) E (b) B 3. An organ pipe open at one end is vibrating in first overtone and is in resonance with another (c) (d) None of these pipe open at both ends and vibrating in third u BE harmonic. The ratio of length of two pipes is 9.F1 and F2 are focal length of objective and eyepiece (a) 1 : 2 (b) 4 : 1 (c) 8 : 3 (d) 3 : 8 respectively of the telescope. The angular 4. A coil takes 15 min to boil a certain amount of water, another coil takes 20 min for the same magnification for the given telescope is equal to process. Time taken to boil the same amount of F F water when both coil are connected in series, (a) 1 (b) 2 (a) 5 min (b) 8.6 min F2 F1 (c) 35 min (d) 30 min 5. Two capillary of length L and 2L and of radius R FF 21  FF 21 and 2R are connected in series. The net rate of (c) (d)  FF FF flow of fluid through them will be (given rate to 21 21 10. Critical velocity of the liquid SPR 4 the flow through single capillary, X = ) (a) decreases when radius decreases KL8 (b) increases when radius increases 8 9 5 7 (c) decreases when density increases (a) X (b) X (c) X (d) X 9 8 7 5 (d) increases when density increases IIT JEE NEET eBooks: www.crackjee.xyz EBD_7100

2 Topicwise AIIMS Solved Papers 11. An organ pipe, open from both end produces 5 18. A period of a planet around Sun is 27 times that beats per second when vibrated with a source of Earth. The ratio of radius of planet’s orbit to of frequency 200 Hz. The second harmonic of the radius of Earth’s orbit is the same pipes produces 10 beats per second (a)4 (b) 9 (c) 64 (d) 27 with a source of frequency 420 Hz. The 19. 3 particles each of mass m are kept at vertices of fundamental frequency of organ pipe is an equilateral triangle of side L. The gravitational (a) 195 Hz (b) 205 Hz field at centre due to these particles is (c) 190 Hz (d) 210 Hz (a) 1.5 V (b) 2.0 V (c) 2.5 V (d) 5 V 12. Two rings of radius R and nR made up of same material have the ratio of moment of inertia about GM3 GM9 12 GM an axis passing through centre as 1 : 8. The value (a) zero (b) (c) (d) 2 2 2 of n is L L 3 L 1 20. A solid sphere of radius R is rolling with velocity (a)2 (b) 22 (c)4 (d) v on a smooth plane. The total kinetic energy of 2 13. One drop of soap bubble of diameter D breaks sphere is into 27 drops having surface tension V . The 7 3 (a) mv 2 (b) mv 2 change in surface energy is 10 4 (a) VS D2 2 (b) VS D4 2 1 1 (c) mv2 (d) mv2 2 2 4 (c) VS D (d) VS D8 2 21. A diode having potential difference 0.5 V across 14. The gas having average speed four times as that its junction which does not depend on current, of SO2 (molecular mass 64) is is connected in series with resistance of 20: (a) He (molecular mass 4) across source. If 0.1 A current passes through

(b) O2 (molecular mass 32) resistance then what is the voltage of the source? (c)H2 (molecular mass 2) (a) 1.5 V (b) 2.0 V (c) 2.5 V (d) 5 V (d) CH4 (molecular mass 16) 22. Potentiometer wire of length 1 m is connected in 15. A container having 1 mole of a gas at a series with 490: resistance and 2 V battery. If temperature 27°C has a movable piston which 0.2 mV/cm is the potential gradient, then maintains at constant pressure in container of 1 resistance of the potentiometer wire is atm. The gas is compressed until temperature (a) 4.9 : (b) 7.9 : becomes 127°C. The work done is (CP for gas is 7.03 cal/mol-K) (c) 5.9 : (d) 6.9 : (a) 703 J (b) 814 J (c) 121 J (d) 2035 J 23. A dipole is placed parallel to the electric field. If 16. An electron having mass (9.1 × 10–31 kg) and W is the work done in rotating the dipole by 60°, charge (1.6 × 10–19 C) moves in a circular path of then work done in rotating it by 180° is radius 0.5 m with a velocity 106 m/s in a magnetic W field. Find the strength of magnetic field. (a) 2 W (b) 3 W (c) 4 W (d) 2 5  6 (a) u1013.1 T (b) u106.5 T 24. An electron of charge e moves in a circular orbit of radius r around the nucleus at a frequency Q. (c) u108.2 6 T (d) None of these The magnetic moment associated with the orbital 17. A cylinder rolls down an inclined plane of motion of the electron is inclination 30°, the acceleration of cylinder is 2 SQr2 SQe Ser g g g2 (a) SQer2 (b) (c) (d) (a) (b) g (c) (d) e r Q 3 2 3 IIT JEE NEET eBooks: www.crackjee.xyz

Mock AIIMS 3 25. A and B are two identically spherical charged 30. The ratio of frequencies of two pendulums are bodies which repel each other with force F, kept 2 : 3, then their length are in ratio at a finite distance. A third uncharged sphere of 2 3 4 9 the same size is brought in contact with sphere (a) (b) (c) (d) B and removed. It is then kept at mid point of A 3 2 9 4 and B. Find the magnitude of force on C. 31. The value of escape velocity on a certain planet F F is 2 km/s. Then the value of orbital speed for a (a) (b) (c)F (d) Zero 2 8 satellite orbiting close to its surface is 26. A composite rod made of copper (a) 12 km/s (b) 1 km/s u D u 108.1(  15 )K and steel (c) 2 km/s (d) 2 2 km/s  15 32. The electrochemical equivalent of a metal is u D u 102.1( )K is heated then it 3.3 × 10–7 kg/C. The mass of metal liberated at (a) bends with steel on convex side cathode by 3 A current in 2 sec will be (b) bends with copper on convex side (c) does not expand (a) u 7 kg108.19 (b) u  7 kg109.9 (d) data is insufficient 7  7 27. A wave has the equation y = 0.1 sin (c) u kg106.6 (d) u kg101.1 S kxt100[ ] and wave velocity 100 m/s, its 33. For a paramagnetic material, the dependence of wave number is equal to the magnetic susceptibility, F on the absolute temperature is given as (a) 1m 1 (b) 2m 1 1 (c) Sm 1 (d) S m2 1 (a) F v T (b) vF T2 28. Volume temperature graph at atmospheric 1 3 (c) vF (d) Independent pressure for a monatomic gas (V in m , T in °C) T is 34. An optically active compound (a) rotates the plane polarised light V V (b) changes the direction of polarised light (c) do not allow plane polarised light to pass (a) (b) through (d) none of the above 35. Three particles A, B and C are thrown from the (T qC) T(qC) top of a tower with the same speed. A is thrown up, B is thrown down and C is horizontally. They

V V hit the ground with speeds VA, VB and VC respectively.

(a)VA = VB = VC (b) VA = VB > VC (c) (d) (c)VB > VC > VA (d) VA > VB = VC 36. The equivalent resistance between A and B is T(qC) T(qC) A R R 29. In X- ray experiment KD , KE denotes R (a) characteristic lines B (b) continuous wavelength R R (c) D , E emissions respectively (d) None of these R8 R5 R3 R7 (a) (b) (c) (d) 5 8 8 8 IIT JEE NEET eBooks: www.crackjee.xyz EBD_7100

4 Topicwise AIIMS Solved Papers 37. The variation of maximum kinetic energy (a) 1 A (b) 1.5 A (c) 0.9 A (d) 0.6 A photoelectrons with applied frequency X )( is 42. Which of the following is false ? (a) (b) (a) convex lens always forms image with m < 1 . . (b) a simple mirror produces virtual, erect and x x a a m m

. same-sized image . E E . . (c) a concave mirror produces virtual, erect K K and magnified image (d) a convex lens can produce real and same- (c) (d) sized image. . . x x a a 43. A star having wavelength O is reaching with m m . . E E . .

K velocity v from earth. The apparent shift in

K s wavelength will be 38. The angle of projection for which range is T Ov Ov equal to maximum height attained by projectile (a) s (b) – s is c c (a) tan 1 4 (b) tan 1 5 Ov 2 Ov 2 1 1 (c) – s (d) s (c) tan 5/4 (d) tan 4/5 2 2 39. The range of projectile will be maximum, when c c angle of projection is 44. The deflection in a galvanometer decreases from S S S (a) (b) (c) (d) None 25 divisions to 5 divisions when a resistor of 3 2 4 20: is connected in series. Find resistance of 40. Forces of 4 N and 5 N are applied at origin along x-axis and y-axis respectively. The resultant force galvanometer. will be (a) 4: (b) 5: (c) 6: (d) 7: (a) 1§ 5 · 41 tan,N ¨ ¸ 45. A current source drives a current in a coil of © 4 ¹ resistance R1 for a time t. The same source drives 1§ 4 · current in another coil of resistance R for same (b) 41 tan,N ¨ ¸ 2 © 5 ¹ time. If heat generated is same, find internal resistance of source § 5 · (c)  41 tan,N 1¨ ¸ © 4 ¹ RR 21 (a) (b)  RR 21  RR § 4 · 21 (d)  41 tan,N 1¨ ¸ © 5 ¹ (c) zero (d) RR 21 41. Current in 2: resistor is (see given figure) 46. The waves used by artificial satellites for 2: communication is (a) microwaves (b) radio-waves, AM 3: (c) radio-waves, FM (d) X-rays 1: 47. The ratio of de-Broglie wavelengths of proton and D -particle having same kinetic energy is (a) 1:2 (b) 1:22 6V 2: (c) 2 : 1 (d) 4 : 1 IIT JEE NEET eBooks: www.crackjee.xyz

Mock AIIMS 5

48. The dimensions of Planck’s constant is Reason : A polaroid is capable of producing plane polarised beams of light. (a) M L T122 (b) M LT 22 54. Assertion : An induced current develop in a (c) TLM 12 (d) TLM  22 conductor moved in direction parallel to the magnetic field. 49. Which of these requires quantum nature of light for their explanation? Reason : An induced current is developed when the number of magnetic lines of force associated (a) diffraction (b) polarisation with conductor is changed. (c) interference (d) black body spectrum 55. Assertion : If the length of the conductor is 50. If blue light is used in place of red light in a doubled, the drift velocity will become half of diffraction experiment the original value (keeping potential difference (a) diffraction pattern remains unchanged unchanged). (b) fringes come closer Reason : At constant potential difference, drift (c) fringes become broader velocity is inversely proportional to the length of the conductor. (d) none of these 56. Assertion : Circuits containing capacitors should be handled cautiously even when there DIRECTION : is no current. NOTE : Instructions for Q. 51 to Q. 60 Reason : The capacitors are very delicate and (a) Both Assertion and Reason are true and so quickly breakdown. ‘Reason’ is the correct explanation of ‘Assertion’ 57. Assertion : The absorbance of a perfect black (b) Both Assertion and R are true and Reason is not body is unity. the correct explanation of ‘Assertion’ Reason : A perfect black body when heated emits (c) ‘Assertion’ is true but ‘Reason’ is false radiations of all possible wavelengths at that temperature. (d) Both ‘Assertion’ and ‘Reason’ are false 58. Assertion : The phase difference between two (e) Assertion is false but ‘Reason’ is true medium particle having a path difference O is 51. Assertion : The dominant mechanism for motion . of charge carriers in forward and reverse biased 2S silicon p-n junction are drift in both forward and Reason : The phase difference is directly reverse bias. proportional to path difference of a particle. Reason : In reverse biased, no current flow 59. Assertion : The impurities always decrease the through the junction surface tension of a liquid. 52. Assertion : The force of repulsion between Reason : The change in surface tension of the atomic nucleus and D -particle varies with liquid depends upon the degree of distance according to inverse square law. contamination of the impurity. Reason : Rutherford did D -particle scattering 60. Assertion : Orbital velocity of a satellite is experiment. greater than its escape velocity. 53. Assertion : The unpolarized light and polarized Reason : Orbit of a satellite is within the light can be distinguished from each other by gravitational field of earth whereas escaping is using polaroid. beyond the gravitational field of earth. IIT JEE NEET eBooks: www.crackjee.xyz EBD_7100

6 Topicwise AIIMS Solved Papers 72. Ozonolysis is relation in which double bond is SECTION II - CHEMISTRY cleaved to give carboxyl group. Which of these will gives formaldehyde as one of its product ? 61. Arene diazonium salt results from reaction of nitrous acid with (a) 3  CHCHCHCH 3 (a) 1° aliphatic amine (b) 2° aromatic amine (b)  CHCHCHCH 3223 (c) 1° aromatic amine CH (d) 1° aromatic amide 3 (c) CHC 2CH3 62. Collidal system constituting the liquid as CH3 dispersed phase and solid as dispersion medium is CH3 CH3 (a) gel (b) emulsion (d) C C CH CH (c) solution (d) suspension 3 3 63. Calculate number of valance electrons in 73. Which of these reacts with Grignard reagent to 2 give carboxylic acid? complex 52 SCN)OH(Cr[ ] (a) 17 (b) 15 (c) 16 (d) 19 (a) CO (b) CO2 64. The ground state valance shell electronic (c) RCOOR (d) HCHO configuration of an element is s4d3 15 . The metal 74. Dehydration of tertiary alcohals is presence of is an acid involves (a) Cr (b) Fe (c) Mn (d) V (a) Formation of carbocation 65. X reacts with acid chloride to give ester X is (b) Formation of carbanion (a) Phenol (b) Benzoic acid (c) Formation of free radical (c) Methnoyl chloride (d) Acid anhydride (d) Formation of Transition state 66. Aldehyde which do not show Cannizarro 75. Reagent of Clemmenson’s reduction is reaction O (a)  NHNH 22 (b) Zn  Hg HCl/ || (c) red HI/P (d) PdH/ 2 , BaSO4 (a)  HCH (b) CH( 33  CHO) 76. The number of structural isomer of (c) 56 CHOHC (d) CH3CHO CrCl 3 .6H2O are 67. Lucas test is used to distinguish between (a)4 (b) 2 (c)3 (d) 1 (a) 1°, 2° and 3° amines 77. How many atoms are present in bcc ? (b) 1°, 2° and 3° alcohol (a)3 (b) 4 (c)2 (d) 1 (c) Aromatic and aliphatic acids 78. Which of following pair present pseudohalide (d) 1°, 2° and 3° amides and polyhalide respectively ? 68. The number of radial nodes in 5d is – – – – (a) OCN and BrI2 (b) RCOO and BrI2 (a)1 (b) 2 (c)3 (d) 4 – – (c) NNN and IF5 (d) OCN and IF5 69. Increasing order of electronegativity of hybrid 79. Units of rate constant of a given chemical orbital is reaction is L mol–1s–1. What is order of reaction? (a) spspsp 32 (b) !! spspsp 32 (a)0 (b) 1 (c)2 (d) 3 23 3 2 80. Which of following is least paramagnetic in nature? (c) !! spspsp (d) !! spspsp (a) Mn (b) Fe (c) Ni (d) Cu 70. Which is longest bond ? 81. Which among the following will have heighest (a) N2 (b) O2 (c) I2 (d) Cl2 B.P at 1 atm. pressure ? 71. Hinsberg’s reagent is used to distinguish (a) 0.1 M NaCl (b) 0.1 M BaCl2 between (c) 0.1 M Sucrose (d) 0.1 M urea (a) acids (b) alcohols 82. Which of the following has smallest ionic radii? (c) amides (d) amine (a) Na+ (b) Mg+ (c)F– (d) Al+3 IIT JEE NEET eBooks: www.crackjee.xyz

Mock AIIMS 7

83. Given compound is subjected to chemical 92. Which of following principle/experiment shows analysis. Results are quantisations of energy in an atom ? I. –ve test to Ninhydrin (a) Heisenberg’s uncertainty principle II. +ve test to Benedict’s solution (b) Auf baus principle Compound is (c) Pauli’s exclusion principle (a) Lipid (b) Monosaccride (d) H-spectrum (c) Protein (d) Amino acid 93. Conjugate acid of CH NH is 84. Iodoform test is show by 23  (a) Glycol (b) Propanaldehyde (a) CH3NH (b) NH2 (c) Ethanol (d) Diethyl ether  85. The correct order of penetrating power is (c) CH3OH (d) CH NH33 (a) J ! D ! E (b) J D E 94. Sublimation energy of I )s(2 is 57.3 kL/ mol and (c) J D ! E (d) J ! E ! D enthalpy of fusion is 15.5 kJ/mol. 86. Given electron would enter which of the The enthalpy of vaporisation of I is following shells first 2 (a) n = 5, = 0 (b) n = 3, = 2 (a) 41.8 kJ/mol (b) 72.8 kJ/ mol A A (c) – 72.8 kJ/ mol (d) – 41.8 kJ/mol (c) n = 6, A = 0 (d) n = 5, A = 1 95. 2-butyne on reaction with Pd/ BaSO gives 3 4 87. SCN(Cr[ )( 52 ])OH and (a) Cis – 2 – butene (b) Trans – 2 – butene 3 Cr[ NCS( )( 52 ])OH show (c) 1 – butene (d) 2 – hydroxy butene (a) Ionization isomerism 96. If 'H is (–) and 'S is (+) 'G will be (b) Hydration isomerism (a) (–) ve (b) (+) ion (c) Link age isomerism (c) Zero (d) ! '  'STH (d) Co-ordination isomerism 97. The given reaction has reagent X as o 88. Buna – S is polymer of (a) Butadiene X 3 { CHCCCH 3 o (b) Butadiene and nitrile 2 Zn/OH (c) Butadiene and styrene 3 CCH C CH3 (d) Butadiene and isoprene |||| 89. Which of following has bond order zero ? O O

(a) CO (b) O2 (c) F2 (d) Be2 (a) O2 (b) HNO3 90. Maximum enol content is in (c) O (d) KMnO O O O 3 4 | | | | | | 98. Which of following is most reactive towards (a) (b) nucleophilic substituting reaction (a) CH2 = CH – Cl O O O (b) C H – Cl | | | | | | 6 5 (c) CH3CH = CHCl (c) H (d) H (d) ClCH2 – CH = CH2 91. Which of following reduces Benedict’s solution? 99. BaCO3 BaO + CO2 is an endothermic O O reaction formation of BaO is favoured by | | | | (a) Decrease in temperature C (b) Decrease in pressure (a) (b) R OH (c) Increase in concentration (c) HCHO (d) R – O – R (d) Increase in pressure IIT JEE NEET eBooks: www.crackjee.xyz EBD_7100

8 Topicwise AIIMS Solved Papers 105. Which is not true about hydrogen peroxide ? 100. A is 0.5M solution of Ca NH( )23 and B is 0.75 m solution of KOH (a) If acts as both oxidising and reducing agent (b) If is pale blue liquid Depression in freezing point is (c) If can be oxidised by O3 (a) Greater for b due to more concentration (d) Two OH bonds lies in same plane (b) equal in both A and B freezing point is less 106. Which is a true peroxide ? then zero (a) CO2 (b) MnO2

(c) equal to 0°C in both as ionic concentration (c) Na2O (d) BaO2 is negligible 107. The conductance of Li salts is lowest of all group (d) Greater for a because number of ions is greater I metals (a) easy difusion of Li+ ion 101. Calculate the volume of H2 gas at NTP (b) lower ability of Li+ ion to polarise water obtained by passing 2 ampere through acidified molecules water for 1 hour. (c) lowest charge to radius ratio (a) 0.0836 L (b) 0.0432 L (d) high degree of hydration of Li+ (c) 0.1672 L (d) 0.836 L 108. Which of following has highest protective power on lyophobic collids ? 102. In an antiflourite structure cation occupies (a) gum arabic (b) sodium oleate (a) Octahedral void (b) Tetra hedral void (c) gelatin (d) starch (c) Centre of cube (d) Edges of cube 109. 0.1 M solution of Ag[ ], Ba[  ] and Ca[  ] is 103. Which is not permissible ? added to solution of sodium sulphate. Ksp 1 (a) n = 3, = 1, m = – 1, s = values for these salts are : 11 A 2 BaSO4 10 ; 6 5 1 CaSO4 10 ; Ag SO42 10 (b) n = 2, = 2, m = – 1, s = – A 2 Which of those will precipitate Ist ? (a) (b) Ag SO 1 BaSO4 42 (c) n = 4, A = 2, m = – 1, s = – 2 (c) CaSO4 (d) All the above 1 110. Which is not a reducing agent ? (d) n = 4, = 2, m = 2, s = – A 2 (a) LiNH4 (b) Na/liq ammonia

Lindlar s' (c) Lindlar’s reagent (d) SeO2 104. A m 3 { CHCCH 3 reagent DIRECTIONS : In the following questions an Assertion (A) is given followed by a Reason (R). Mark Na liq/ NH. 3 o B; A and B are your responses from the following options. (a) Both Assertion and Reason are true and Reason (a) Cis, trans 2 butene is the correct explanation of 'Assertion' (b) Trans, trans 2 butene (b) Both Assertion and Reason are true and Reason (c) Cis, cis 2 butene is not the correct explanation of 'Assertion' (c) Assertion is true but Reason is false (d) Trans, cis 2 butene (d) Assertion is false but Reason is true IIT JEE NEET eBooks: www.crackjee.xyz

Mock AIIMS 9

111. Assertion (A) : A very dilute acidic solution of 119. Assertion (A) : If hydration energy is greater Cd2+ and Ni2+ gives yellow precipitate of CdS than lattice energy the solid dissolves in liquid on passing hydrogen sulphide. Reason (R) : The solubility of a solid in a liquid Reason (R) : Solubility product of CdS is more depends upon lattice energy and hydration than that of NiS. energy 2– 112. Assertion (A) : [Ni(CN)4] has square planar 2– 120. Assertion (A) : Reduction of m-dinitrobenzene and [NiCl4] has tetrahedral shape. 2– with ammonium sulphide gives m-nitroaniline. Reason (R) : [Ni(CN)4] is diamagnetic while 2– Reason (R) : m-Nitroaniline formed gets NiCl4 is paramagnetic. 113. Assertion (A) : Rusting of an iron is an example precipitated and hence further reduction is of corrosion. prevented. Reason (R) : Rusting of iron is decreased by acid and electrolytes. SECTION III - BIOLOGY 114. Assertion (A): Trihydroxyglutaric acid (HO2C– CHOH–CHOH–CHOH–CO H) exists in four 2 121. Sequence of taxonomic categories is stereoisomeric forms; two of which are optically (a) Class – Phylum – Tribe – Order – Family – active while the other two are meso-forms. Genus – Species Reason (R) : It contains two asymmetric and (b) Division – Class – Family – Tribe – Order – pseudo-asymmetric carbon atom. Genus – Species 115. Assertion (A) : The endothermic reactions are (c) Division – Class – Order – Family – Tribe – favoured at lower temperature and the exothermic Genus – Species reactions are favoured at higher temperature. (d) Phylum – Order – Class – Tribe – Family – Reason (R) : When a system in equilibrium is Genus – Species disturbed by changing the temperature, it will 122. Genophore/bacterial genome or nucleoid is made tend to adjust itself so as to overcome the effect of of change. (a) Histones and nonhistones 116. Assertion (A) : For each ten degree rise of (b) RNA and histones temperature the specific rate constant is nearly (c) A single double stranded DNA doubled. Reason (R) : Energy-wise distribution of (d) A single stranded DNA molecules in a gas is an experimental function of 123. Claviceps purpurea is causal organism of temperature. (a) Smut of Barley (b) Rust of Wheat 117. Assertion (A) : HNO3 is a stronger acid than (c) Ergot of Rye HNO2 (d) Powdery Mildew of Pea. Reason (R) : In HNO3 there are two nitrogen-to- 124. Chloroplast of Chlamydomonas is oxygen bonds whereas in HNO2 there is only one. (a) Stellate (b) Cup-shaped 118. Assertion (A) : A metal having negative (c) Collar-shaped (d) Spiral

reduction potential when dipped in the solution 125. Oxysomes of F0 – F1 particles occur on of its own ions has a tendency to pass into the (a) Thylakoids solution. (b) Mitochondrial surface Reason (R) : Metal having negative reduction (c) Inner mitochondrial membrane potential have large hydration energy. (d) Chloroplast surface IIT JEE NEET eBooks: www.crackjee.xyz EBD_7100

10 Topicwise AIIMS Solved Papers

126. Two linked genes a and b show 20% 135. An ovule which becomes curved so that the recombination. The individuals of a dihybrid nucellus and embryo sac lie at right angles to cross between + +/ + + × ab/ab shall show the funicle is gametes (a) Hemitropous (b) Campylotropous (a) + + 80 : ab : 20 (c) Anatropous (d) Orthotropous (b) + + 50 : ab : 50 136. Which of the following movement is not related (c) + + 40 : ab 40 : + a 10 : + b : 10 to auxin level (d) + + 30 : ab 30 : + a 20 : + b : 20 (a) Bending of shoot towards light 127. In Escherichia coli, lac operon is induced by (b) Movement of root towards soil (a) Lactose (b) Promoter gene (c) Nyctinastic leaf movements (c) E-galactosidase (d) I-gene (d) Movement of sunflower head tracking the 128. If a diploid cell is treated with colchicine then it sun becomes 137. An interesting modification of flower shape for (a) triploid (b) tetraploid insect pollination occurs in some orchids in (c) diploid (d) monoploid which a male insect mistakes the pattern on the 129. Syngenesious condition is found in orchid flower for the female of his species and (a) Asteraceae (b) Labiate tries to copulate with it, thereby pollinating the (c) Solanaceae (d) Fabaceae flower. This phenomenon is called 130. Floral formula of Tomato/Tobacco is (a) Mimicry (a) + K4–5 AG 10 (2) (b) Pseudopollination (c) Pseudocopulation (b) + + K2+2 CA 4 2+4G 1 (d) Pseudoparthenocarpy (c) + P A G + 2 31 138. The most common indicator organism that (d) represents polluted water is + + K(5) C(5)AG 5 (2) (a) E. coli (b) P. typhi 131. Ectophloic siphonostele is found in (c) C. vibrio (d) Entamoeba (a) Osmunda and Equisetum 139. In order to obtain virus-free plants through (b) Marsilea and Botrychium tissue culture the best method is (c) Adiantum and Cucurbitaceae (a) Embryo rescue (b) Anther culture (d) Dicksonia and Maidenhair fern (c) Meristem culture (d) Protoplast culture 132. If a cell A with DPD 4 bars is connected to cell B, 140. Which one among the following chemicals is C, D whose OP and TP are respectively 4 and 4, used for causing defoliation of forest trees? 10 and 5 and 7 and 3 bars, the flow of water will be (a) Phosphon-D (a) A and D to B and C (b) Malic hydrazide (b) A to B,C and D (c) 2, 4 Dichlorophenoxy acetic acid (c) B to A, C and D (d) C to A, B and D (d) Amo-1618 133. The size of chlorophyll molecule is 141. Which of the following is not true for a (a) Head 15 × 15 Å, tail 25 Å species? (b) Head 20 × 20 Å, tail 25 Å (a) Members of a species can interbreed. (c) Head 15 × 15 Å, tail 20 Å (b) Gene flow does not occur between the (d) Head 10 × 12 Å, tail 25 Å populations of a species. 134. Terminal cytochrome of respiratory chain which (c) Each species is reproductively isolated donates electrons to oxygen is from every other species. (a) Cyt. b (b) Cyt. c (d) Variations occur among members of a species. (c) Cyt. a1 (d) Cyt. a3 IIT JEE NEET eBooks: www.crackjee.xyz

Mock AIIMS 11

142. The catalytic efficiency of two different 149. If Henle's loop were absent from mammalian enzymes can be compared by the nephron, which of the following is to be (a) formation of the product expected? (b) pH optimum value (a) The urine will be more dilute (c)Km value (b) There will be no urine formation (d) molecular size of the enzyme (c) There will be hardly any change in the 143. Fire bellied toad is quality and quantity of urine formed (a) Amphiuma (b) Banbina (d) The urine will be more concentrated (c) Necturus (d) Salamandra 150. Number of cervical vertebrae in camel is 144. American water plant that has become a (a) More than that of Rabbit troublesome water weed in India is (b) Less than that of Rabbit (a) Cyperus rotundus (c) Same as that of Whale (b) Eichhornia crassipes (d) More than that of Horse (c) Trapa latifolia 151. Which of the following cranial nerves can (d) Trapa bispinosa regulate heart beat? 145. Characteristics of smooth muscle fibres are (a)X (b) IX (a) Spindle-shaped, unbranched, unstriated, (c) VIII (d) VII uninucleate and involuntary 152. Which one of the following pairs correctly (b) Spindle shaped, unbranched, unstriped, matches a hormone with a disease resulting from multinucleate and involuntary its deficiency? (c) Cylindrical, unbranched, unstriped, (a) Luteinizing - Failure of multinucleate and involuntary hormone ovulation (d) Cylindrical, unbranched, striated, (b) Insulin - Diabetes insipidus multinucleate and voluntary (c) Thyroxine - Tetany 146. An adolescent human below 17 years of age normally has dental formula as (d) Parathyroid - Diabetes mellitus hormone 2,3,1,2 2,3,2,2 153. The growth of corpus luteum is initiated by (a) (b) 2,3,1,2 2,3,2,2 (a) Human chorionic gonadotropin (b) Follicle stimulating hormone 0,2,1,2 2,2,1,2 (c) (d) (c) Luteinizing hormone 0,2,1,2 2,2,1,2 (d) Prolactin 147. In alveoli of the lungs, the air at the site of gas 154. Two opposite forces operate in the growth and exchange, is separated from the blood by development of every population. One of them (a) alveolar epithelium only relates to the ability to reproduce at a given rate. (b) alveolar epithelium and capillary The force opposing it is called endothelium (a) environmental resistance (c) alveolar epithelium, capillary endothelium (b) morbidity and tunica adventitia (c) fecundity (d) alveolar epithelium, capillary endothelium, (d) biotic potential a thin layer of tunica media and tunica 155. Red-green colour blindness in humans is adventitia governed by a sex-linked recessive gene. A 148. Splenic artery arises from normal woman whose father was colour-blind (a) Anterior mesenteric artery marries a colour blind man. What proportion of (b) Coeliac artery their daughters is expected to be colour-blind? (a) 3/4 (b) 1/2 (c) Posterior mesenteric artery (c) 1/4 (d) All (d) Intestinal artery IIT JEE NEET eBooks: www.crackjee.xyz EBD_7100

12 Topicwise AIIMS Solved Papers 156. Genetic drift operates only in (d) If both the Assertion and Reason are incorrect. (a) Smaller Populations (e) If the Assertion is incorrect but the Reason is (b) Larger Populations correct. (c) Mendelian Populations (d) Island Populations 161. Assertion : Mango dipped in concentrated 157. Which one of the following statements about sodium chloride solution will contract. fossil human species is correct? (a) Fossils of Homo neanderthalensis have Reason : Water goes out due to exosmosis in been found recently in South America hypertonic solution. (b) Neanderthal man and Cro-Magnon man did 162. Assertion : Nissl’s granules that are basophilic exist for sometime together are present in the cyton. (c) Australopithecus fossils have been found Reason : They are composed of RNA. in Australia 163. Assertion : Phycobilins are destroyed by heat. (d) Homo erectus was preceded by Homo Reason : They are protein linked and proteins habilis are denatured due to heat. 158. In the silk worm, if no juvenile hormone (JH) is 164. Assertion : The inner mucosa coat has present when it moults, it will innumerable finger like projections. (a) die Reason : Absorption increases due to infolds. (b) moult into another larval stage 165. Assertion : Vital capacity is the total volume of (c) moult into pupa air that can be breathed out with minimum effort. (d) moult into an adult Reason : Vital capacity represents the maximum 159. Which one of the following is correct match? capacity of an individual to renew air in the (a) Reserpine — Tranquilizer respiratory system. (b) Cocaine — Opiate narcotic 166. Assertion : Heart wood is non functional. (c) Morphine — Hallucinogenic Reason : Duramen is plugged due to in-growth (d) Bhang — Analgesic of collenchyma. 160. Test tube baby is one who 167. Assertion : Glycogen is called animal starch. (a) is born out of artificial insemination Reason : Glycogen is stored in the liver and (b) has undergone development in a test tube muscles of animals. (c) is born out of the technique of fertilization 168. Assertion : Racemose Inflorescence is an in vitro indeterminate inflorescence. (d) has been developed without fertilization Reason : The Inflorescence shows definite growth. DIRECTIONS : These questions consist of two 169. Assertion : Mouth parts of cockroach, honey statements, each printed as Assertion and Reason. bee & mosquito are analogus organs. While answering these questions, you are required to Reason : These organs follow the same basic choose any one of the following five responses. plan of organization during development. (a) If both Assertion and Reason are correct and 170. Assertion : Crossing over occurs at four strand the Reason is a correct explanation of the or tetrad stage. Assertion. Reason : Parent strand and gene linkages (b) If both Assertion and Reason are correct but disappear at two strand stage. Reason is not a correct explanation of the 171. Assertion : Oncogenes transform normal cell Assertion. into cancer cell. (c) If the Assertion is correct but Reason is Reason : They integrate their DNA with RNA incorrect. of the host cells. IIT JEE NEET eBooks: www.crackjee.xyz

Mock AIIMS 13

172. Assertion : Human immuno deficiency Virus-III 182. On July 18, 2017, the Supreme Court has allowed are retroviruses. the Centre to replace the oversight committee Reason : They exhibit reverse transcription. set up to supervise the functioning of the 173. Assertion : Genes are actual physical units of Medical Coucil of India (MCI) with a fresh panel heredity. of how many eminent doctors? Reason : Genes are in the chromosome in linear (a) 8 eminent doctors order. (b) 7 eminent doctors 174. Assertion : Viruses are obligatory parasites. (c) 6 eminent doctors Reason : They show host specificity and (d) 5 eminent doctors multiply only inside living systems. 183. Who among the following is set to become the 175. Assertion : Allergens are generally weak United Nations' youngest-ever'Messenger of antigens, which are glycogen molecules. Peace'? Reason : Allergy means inappropriate reaction (a) Mark Zuckerberg of a person. (b) Malala Yousafzai 176. Assertion : Interferons are antiviral proteins. (c) Selena Gomez Reason : It is released from the infected and (d) Virat Kohli dying cells. 184. Find out the correct sequence - 177. Assertion : Horticulture is the conscious raise (a) Pain, Doctor, Hospital, Drug of Cereal crop. . (b) Hospital, Doctor, Drug, Pain Reason : Growing of vegetables, fruits and (c) Hospital, Doctor, Pain, Drug ornamental plants is Horticulture. (d) Pain, Hospital, Doctor, Drug 185. Human : Brain :: Computer : ? 178. Assertion : Viruses cannot metabolise outside host cells and use host machinery to produce (a) USB (b) Monitor own nucleic acids and proteins. (c) CPU (d) Internet 186. Which of these is a dwarf planet? Reason : Viruses lack energy yielding and biosynthetic machinery. (a) Neptune (b) Titan (c) Eris (d) Hydra 179. Assertion : All aggregate and multiple fruits 187. M is son of P, Q is the grand-daughter of O, are false. who is the husband of P. How is M related to Reason : They develop from other floral parts O? instead of the ovary (a) Son (b) Daughter 180. Assertion : Casuarina and Betula show (c) Mother (d) Father chalazogamy. 188. How many meaningful English words can be Reason : Pollen tube enters the ovule through formed with the letters URLE using each letter micropyle end. only once in each word? SECTION IV - G. K. (a) None (b) One (c) Two (d) Three 181. On August 12, 2017, Dr Vishwanath Karad MIT 189. Name the country that will host the 2018 Table World Peace University, considered to be the Tennis Team World Cup. first of its kind in India was inaugurated in: (a) South Africa (b) China (a) Pune (b) Bhopal (c) USA (d) England (c) Jaipur (d) Chennai IIT JEE NEET eBooks: www.crackjee.xyz EBD_7100

14 Topicwise AIIMS Solved Papers

190. Select the related word/letters/ number from the 196. On July 21, 2017, Uttar Pradesh Assembly given alternatives. adopted two resolutions on naming the airport Psychology : Human Being :: Ornithology : ? terminals of which two cities? (a) Birds (b) Volcanoes (a) Kanpur and Lucknow (c) Insects (d) Reptiles (b) Lucknow and Bareilly 191. The 2017 International Day for the (c) Kanpur and Meerut Remembrance of the Slave Trade and its (d) Kanpur and Bareilly Abolition is observed on which date? 197. Which country is accused of interfering and (a) August 24 (b) August 25 hacking the US 2016 presidential elections? (c) August 22 (d) August 23 (a) Russia (b) China 192. Who is the head of the 9-judge Constitution (c) Germany (d) Japan bench of the Supreme Court (SC) to determine 198. On July 23, 2017, Arun Jaitley inaugurated 315th whether privacy is a fundamental right or not Rest House of the Kendriya Sainik Board in: under the Constitution? (a) Shimla (b) Dehradun (a) Abhay Manohar Sapre (c) New Delhi (d) Amritsar (b) J S Khehar 199. Arrange the following steps of AIIMS (c) Sanjay Kishan Kaul application form (d) Fali Narima (1) City Choice 193. Which of the following is the correct description (2) Payment of the term 'sex ratio' as used in context of the (3) Password received census? (4) Registration (a) Number of females per 1000 persons (a) 3,4,2,1 (b) 4,3,1,2 (b) Number of females in a sample of 1000 (c) 3,4,1,2 (d) 3,1,4,2 persons 200. Which of the following diagrams indicates the (c) Number of males per 1000 females best relation between Pluto,Planets, Sun and (d) Number of females per 1000 males Earth ? 194. In India, which city is also known as 'City of Palaces'? (a) (b) (a) Jaipur (b) Kolkata (c) Gwalior (d) Udaipur 195. Heena Sidhu, who recently won a World Cup (c) (d) Gold Medal for India, is associated with (a) Shooting (b) Archery (c) Weightlifting (d) Boxing IIT JEE NEET eBooks: www.crackjee.xyz

Mock AIIMS 15 SOLUTIONS

SECTION I - PHYSICS V3 Ÿ n1 4 1 1. (b) From the F.B.D. A N = mg cos T V3 V3 Given nn 21 Ÿ or F = ma = mg sin T – PN 2A 4A12 1 TPT Ÿ (singa cos TPT ) A1 2 N PN A2 V 2 HR 4. (c) H t or t m sing T mgcosT R V 2 xmg The voltage, V remains same, H is also same T HR HR 22 1 2 Now using,  as2uv t1 ; t 2 V2 V2 or, 2 (sing2v cos TPTu ) A  21 )RR(H or t =  tt 21 ( A = length of incline) V2 = 15 + 20 = 35 min or, v = A (sing2 cos TPT ) 2. (b) Acceleration due to gravity at lattitude’ O ’ 8KA 5. (a) Fluid resistance is given by R = 4 is given by 22 Sr O ee cosRgg OZ When two capillary tubes of same size are At equator, O = 90° joined in parallel, then equivalent fluid Ÿ cos O = cos 90° = 0 resistance is   RRR or gO = ge = g (as given in question) 21S 8K uK L28 § KL8 · 9 22 3 2 A  ¨ ¸u At 30°, Z cosRgg 30 g RZ = 4 4 = ¨ 4 ¸ 30 4 SR S )R2( © SR ¹ 8 Rate of flow 3 2 or,  gg RZ 4 30 P SR 8 8 ª SPR 4 º 4 = u = X «as X » R KL8 9 9 3. (a) For 3rd harmonic/2nd over tone of organ S ¬« 8KA ¼» pipe open at ends 6. (a) The charge is moving in an equipotential A 2 line. So no work is done. 7. (d) Optical fibers carry immense no. of signals as compared to other wires. There is no EM interference in these fibres. 8. (c) An EMW is the one constituted by O / 4 O O / 4 oscillating electric and magnetic field which V3 oscillate in two mutually perpendicular Ÿ n 2 2 planes. The wave itself propagates in a A 2 direction perpendicular to both of the For 1st overtone of organ pipe open at one end directions of oscillations of electric )E( A1 and magnetic fields )B( , i.e. u .BE 9. (a) The angular magnification, angle subtended by the image at eye M = angle subtended at eye with object O 2/ O/4 in actual position IIT JEE NEET eBooks: www.crackjee.xyz EBD_7100

16 Topicwise AIIMS Solved Papers 13. (d) Volume of bigger bubble fo F1 For telescope, M = = volume of 27 smaller bubbles fe F2 10. (c) As per Reynold’s formula critical velocity 4 3 4 3 D Su SŸ 27D Su d Ÿ d of a liquid is defined as 3 3 3

KK 1 1 2 v Ÿ vc v & vc v Initial surface energy S D4S V c Ur U r i Where K is coefficient of viscosity of the 2 Final surface energy f 27S D4 VSu liquid, U its density and r is the radius of the tube. K is a dimensionless constant D  '  SSS and using d called the Reynold number. Thus critical if 3 velocity increases when density and radius 2 of the tube decreases. ª D 2 º 11. (b) Let the fundamental frequency of organ ' «274S uSuV D » « 9 » pipe be f ¬ ¼ 2 2 Case I : f = 200 r 5 = 205 Hz or 195 Hz = VuSu SV D84D2 V M 64 14. (a) 1 1 Ÿ 4 V2 M 2 M1

or 1 4M i.e. He 15. (b) At constant pressure

D   if nR)VV(PW D if )TT( Case II : frequency of 2nd harmonic of = 1× 8.14 (127 – 27 ) = 8.14 × 100 = 814 J organ pipe = 2f (as is clear from the second mv2 figure) 16. (a) qvB 2f = 420 r 10 or f = 210 r 5 r or f = 205 or 215 mv  uu 10101.9 631 Hence fundamental frequency of organ B = qr 19 pipe 106.1 uu 5.0 = 205 Hz = 1.13 u105 T 12. (a) The moment of inertia (I) of circular ring 17. (a) Remember that acceleration of a cylinder whose axis of rotation is passing thought down a smooth inclined plane is 2 N its center, 11 RmI PN 2 Also, 22 nR(mI ) m sing T mgcosT Since both rings have same density, xmg T m2 m1 Ÿ 2 nR(2 uS A) uS AR2 sing T mR 2 1 a where I is the Where A is cross-section of ring, § I · 2 ¨1 ¸ 2 AA 21 (Given) ? nmm 12 © mR ¹ moment of Inertia for cylinder I 1 Rm 2 Rm 2 Given 1 = 1 = 1 1 2 2 sing 30q g u I2 8 nR(m ) 1 nR(nm ) a 2 g 2 2 § mR 1 · 1 3 ¨1 u ¸ 1 1 1 ¨ 2 2 ¸ 2 Ÿ or n = 2 © mR ¹ 8 n3 IIT JEE NEET eBooks: www.crackjee.xyz

Mock AIIMS 17 18. (b) According to Kepler’s third law, current flowing due to charge e. Further 2 orbital motion of electron is equivalent to a R § T · 3 § 27 T · current R v T23 ¨ ¸ ¨ e ¸ 9 R ¨ T ¸ ¨ T ¸ e © ee ¹ © e ¹ I eX 19. (a) The gravitational field intensity at the T centre (of an equilateral triangle), 1 equidistant from the three vertices due to 3 (where T is the time period) equal masses will be zero. The vector sum X of the forces due to the 3 masses will be ? M = IA XS re 2 zero. 25. (c) Initial force between the two spheres 20. (a) Kinetic energy = translational kinetic carrying charge (say q) is energy + rotational kinetic energy 1 q2 1 1 22 F K. E = mv IZ 2 2 2 4SH0 r 2 (r is the distance between them) Moment of inertia of sphere (I) = MR 2 5 Further when an uncharged sphere is kept in touch with the sphere of charge q, the 2 1 2 1 2 2 § V · 7 2  0q q ? .E.K mv u MR ¨ ¸ mv net charge on both become . 2 2 5 © R ¹ 10 2 2 21. (c) 'V = V × IR = 0.5 + 0.1 × 20 = 2.5 V Force on the 3rd charge, when placed in 0.5V 20: center of the 1st two r / 2 r / 2 0.1A

1 3 2 q V q / 2 q / 2 2 22. (a) Pot. gradient = 0.2mV/cm § q · § q · q¨ ¸ ¨ ¸ u102.0 3 1 © 2 ¹ 1 © 2 ¹ u102  2 m/v F  = 2 = 3 2 2 10 4SH0 § r · 4SH0 § r ·  2 ¨ ¸ ¨ ¸ Emf of cell = 2×10–2×1m = u102 V © 2 ¹ © 2 ¹ = 0.02 V As per the condition of potentiometer 1 q 2  F]12[ 0.02 (R + 490) = 2 (R) or 1.98 R = 9.8 2 4SH0 r 8.9 Ÿ R = = 9.4 : 26. (b) A bimetallic strip, on uniform heating, bends 98.1 in the form of an arc and the metal with 23. (c) Work done in rotating a dipole by an angle greater ‘ D ’ lies on the convex side. ‘ T ’ is 27. (c) General wave equation pE sin(Ay Z kxt )  T pEW cos1( T pE)  cos1( )60 2 On comparing, we get Z 100S Again, pEW  cos1( 180) Z 100S 180 ? Wave number, k = S m 1  pE  )]1(1[ pE2 W4 v 100 24. (a) Magnetic moment = M = IA, where A is 28. (c) For constant pressure,V v T K the area of the orbit S 2 )r( and I is the 29. (c) K D and E are characteristics line in the X-ray line spectrum. IIT JEE NEET eBooks: www.crackjee.xyz EBD_7100

18 Topicwise AIIMS Solved Papers 36. (b) The equivalent circuit can be redrawn as A 30. (d) 2T S R g R R R 1 1 A x x B { x Ÿ Frequency, n v R R T length R R 2R R R n 2 9 1 A 2 Ÿ A2 Ÿ A1 { { n 3 2R / 3 2 A1 A1 A 2 4 R 5R / 3 31. (c) gR2V and gRV e 0 8/R5 { 2 V2V V0 Ÿ 2 km/s 37. (b) As per Einstein’s photoelectric equation : 0e 2 32. (a) m = Zit E = X wFh  KEmax Ÿ m = 103.3 7 uuu 23 = u108.19 7 i.e. till a certain valve of X , KE remains 0, it only starts increasing once the Work function C 33. (c) F (as per Curie’s law) (WF) of the metal surface is achieved. T Paramagnetic materials obey Curies law. 2 sinu 2T sinu 22 T C = Curies constant 38. (a) R ; Rmax 34. (a) When the plane polarised light passes g g2 through certain substance, the plane of polarisation of the light is rotated about

the direction of propagation of light hmax. through a certain angle. 35. (a) For A: It goes up with velocity u will it T reaches its maximum height (i.e. velocity R becomes zero) and comes back to O and sin2 T Equating we get 2sin T attains velocity u. 2 22 2 2 Using  as2uv Ÿ A  gh2uv or sin4 cos TT sin T 1 u Ÿ tanT 4 or T tan 4 2 sinu 2T 39. (c) R will be maximum for O u Vch g u S S sin 2T = 1 2TŸ or T = 45q h 2 4 40. (a) 22 V V  54R 41N B A u Vch 5 VC The angle T will be given by tan T V VC 4 For B, going down with velocity u 2 Ÿ B  gh2uv For C, horizontal velocity remains same, i.e. ‘u’. 5N R Vertical velocity =  gh20 = gh2 T 4N 2 2 2 The resultant vC = x  vv y =  gh2u . 5 or T tan 4 Hence vvv CBA IIT JEE NEET eBooks: www.crackjee.xyz

Mock AIIMS 19

41. (c) At steady state the capacitor will D be fully V charged and thus there will be no current in R = I = 2 2  RR the 1: resistance. So the effective circuit 2 becomes Further, as heat generated is same, so 2: 2 2 1 21 2tRItRI

I1 2 2 A B § V · § V · ¨ ¸ ¨ ¸ R I2 3: or ¨ ¸ R1 = ¨ ¸ 2 ©  RR 1 ¹ ©  RR 2 ¹ 2 2 I Ÿ  21 )RR(R =  12 )RR(R 2 2 6V 2.8: Ÿ 211  RR2RRRR R 21 Net current from the 6V battery, 2 2 2 = 12 2  RR2RRRR R 21 6 6 3 I A5.1 2 § u32 · 8.2  8.22.1 2 Ÿ   )RR(RR)RR(R ¨ ¸  212121  32 1 © ¹ Ÿ Between A and B, Voltage is same in both R = RR 21 resistances, 46. (a) Microwaves are used for communication I3I2 21 where  21 5.1III in artificial satellites. Ÿ Ÿ A9.0I)I5.1(3I2 h 1 11 47. (c) de Broglie wavelength, = 42. (a) Convex lens can form image with m < 1, m > O mE2 E.K 1 and m = 1 depending upon the positionof the object. Convex lens forms magnified Op m m4 p image (m > 1) when the object is pole and ? D 2f, same size as the object (m = 1) when OD mp mp the object is at 2f and smaller image (m < 1), [ ] when the object is beyond 2f. ' D EE )p(E.K)(E.K 43. (a) When the source (star) is moving towards Op 2 the observer (earth), ? = Apparent wavelength, OD 1

'  VC § V · V  22 O = s O = ¨1 s ¸O = s OO E ML T 12 C C C 48. (c) h = = = ML T © ¹ X T1 Ÿ OO ' = apparent shift in wavelength 49. (d) Black body spectrum V 50. (b) Fringe width v O . Also blue O!O red = s O (C = velocity of sound) C Therefore, fringes come closer when blue light is replaced by red light in diffraction 44. (b) Case - I : When resistor is not connected pattern. Using V = IR Ÿ V = 25G )R( ...... (i) 51. (d) In p–n junction, the diffusion of majority Case - II : When resistor is connected carriers takes place when junction is V = 5(20 + R ) = 100 + 5 R ...... (ii) forward biased and drifting of minority G G carriers takes place across the function, From (i) and (ii), 20 R G = 100 when reverse biased. The reverse bias Ÿ R = 5: opposes the majority carriers but makes the G minority carriers to cross the p–n junction. 45. (d) Let internal resistance of source = R Thus the small current in µA flows during Current in coil of resistance reverse bias. V 52. (b) In Rutherford’s D–particle scattering R1 = I1 =  RR 1 experiments, some of D-particles was found Current in coil of resistance to be scattered at very large angles, inspite IIT JEE NEET eBooks: www.crackjee.xyz EBD_7100

20 Topicwise AIIMS Solved Papers of having very high kinetic energy. This capacitor is touched by someone, he may shows that there are the D–particle which feel shock due to large charge still present will be passing very close to nucleus. on the capacitor. Hence it should handled Rutherford’s confirmed the repulsive force cautiously otherwise this may cause a on D–particle due to nucleus varies with severe shock. distance according to inverse square law 57. (b) A perfect black body is one which absorbs and that the positive charges are concentrated at the center and not heat radiation of all wavelengths, which fall distributed throughout the atom. This is on it. Such a body neither reflects nor the nuclear model of Rutherford. transmits any part of the incident heat 53. (a) When a polaroid is rotated in the path of radiation and hence appears black unpolarised light, the intensity of light irrespective of the colour of the incident transmitted from polaroid remains radiation. Obviously the absorbance of a undiminished (because unpolarised light perfect black body is unity. The radiation contains waves vibrating in all possible given out by a perfect black body are called planes with equal probability). However, black body radiations or full radiation or when the polaroid is rotated in path of plane total radiations. polarised light, its intensity will vary from maximum (when the vibrations of the plane § 2S · polarised light are parallel to the axis of the 58. (b) As we know, I' ¨ ¸ 'u x . This is © J ¹ polaroid) to minimum (when the direction of the vibrations becomes perpendicular phase difference between two particles to the axis of the crystal). Thus using whose path difference is 'x. If 'x = O, then polaroid we can easily verify that whether 'x = 2S. Thus, the phase difference between the light is polarised or not. two medium particles having a path difference O is 2S, i.e., the particles are in 54. (e) An induced current develop in a conductor cannot moved in a direction parallel to the same phase of oscillations. magnetic field. This is because when the 59. (a) The presence of impurities either on the conductor moved in a direction parallel to liquid surface or dissolved in it, considerably affect the force of surface magnetic field, amount of flux linked with tension, depending upon the degree of the conductor does not change. Thus the contamination. A highly soluble induced current develops only when substance like sodium chloride when conductor cuts the lines of magnetic force. dissolved in water, increased the surface The direction of flow of induced current tension of water. But the sparingly soluble can also be found by applying Fleming’s like phenol when dissolved in water right hand rule, when the direction of reduces the surface tension of water. motion of conductor inside the magnetic 60. (c) The orbital velocity, if a satellite close to field and the direction of magnetic field action on if are known. earth is gRV e0 , While the escape 55. (a) Drift velocity of free electrons is given by, velocity for a body thrown from the earth’s eE v W surface is e gR2V e . d m gR Potential difference V V0 e 1 Thus = Where, E = V gR2 2 length A e e eV or V2V ? v d W 0e mA i.e., if the orbital velocity of a satellite 1 eVW revolving close to the earth happens to i.e., v d v where is constant. A m increase to 2 times, the satellite would 56. (c) A charged capacitor, after removing the escape. battery, does not discharge itself. If this IIT JEE NEET eBooks: www.crackjee.xyz

Mock AIIMS 21

SECTION II - CHEMISTRY 67. (b) HCl( conc( .)  ZnCl2 ) is Lucas reagent. 61. (a) Aromatic diazonium salts are generally Different alcohals react at different rate with prepared by adding cold aqueous solution this reagent. of sodium nitrite to solution/suspension ZnCl2 of 1° aromatic amine at 273 - 278 K ROH + HCl o RCl + 2 OH ZnCl/HCl ArNH NaNO  HX o273 278K alc1 . q 2 o 22 HNO 2 do not react at room temp. ArN X NaX OH2 2 2 alc2 . q ZnCl/HCl 2 o turbidity appears in 62. (a) Some sols have a high concentration of dispersed solid and change spontaneously 5 min. into semisolid form on cooling these are alc3 . q ZnCl/HCl 2 o turbidity appears at called gels thus they form liquid as dispersed phase and solid as dispersion once. medium. 68. (b) No. of nodes = n – A – 1 15 For 5d = 5 – 2 – 1 = 2 63. (c) Cr has valance shell s4d3 69. (b) More is s-character more is ? Number of valance electrons = 6 electronegativity of hybrid orbital.. 2OH is monodentate sp has 50 % s-character 2 ? 2 electrons from each 2 OH sp has 33 % s-character ? Total electrons contributed by OH 3 2 sp has 25 % s-character = 2 × 5 = 10 ? Order of electronegativity SCN gives 2 electrons 3 Total valance electrons = 6 + 10 + 2 = 18 sp > sp 2 > sp Over all charge on complex is + 2 70. (c) Bond length is in order as given ? Valance electrons in complex are 16 Single bond > double bond > triple bond 15 64. (a) Valance shell is s4d3 , It means inner N2 has triple bond; O2 has double bond shells are p3s3p2s2s1 62622 andICl22 have single bond ?At no. = 24. Hence element is Cr. Out of these I2 has longer bond length O as, due to big atoms the inter nuclear || distance is large. OH CO  R O 71. (d) SOHC 256 Cl is Hinsberg reagent || 65. (a) +  XCR o 1 q a m i n e q amine2

Ester R  N H 2 2  NHR 66. (d) Aldehyde which do not contain DH 6 HC 5 S O 2 C l 6 HC 5 S O 2 C l

atom on treatment with alkali solution HC 56 S O 2 N H R  H C l SOHC NR 2256  HCl undergo self oxidation - reduction reaction NaOH NaOH (Cannizaro reaction) D has CH3 CHO 6 HC 5 S O 2 N ( N a ) R No reaction D  H S ol u b l e ? Cannot show Cannizaro reaction. e.g. of Cannizaro reaction : 3q amine HCHO + NaOH o CH OH + HCOONa 3 Do not react at all IIT JEE NEET eBooks: www.crackjee.xyz EBD_7100

22 Topicwise AIIMS Solved Papers 76. (a) 4 Isomers are 72. (b) 23  CHCHCHCH O 32 o Cl])OH(Cr[ 362 ; Cl]Cl)OH(Cr[ 2252 OH. O Cl]Cl)OH(Cr[ 2242 OH2. ;

C2H5  CH CH2 2332 OH3.]Cl)OH(Cr[ 77 (c) Contribution by 8 atoms present at corner = 1/8 × 8 = 1 O O Contribution by atom present within the  H2O / H body = 1 Numbers of atoms present per unit cell HC 52 CHO  HCHO = 1 + 1 = 2 78 (a) A few ions, consisting of two or more GG electro negative atoms of which at least 73. (b) MgR  OCOX o one is N and properties similar to halide ions are called pseudohalides of NNN , R  C O OCN  , CN  etc. OH X M g O o2   )HO(OCR MgX Halide ions often react with molecules of halogens or interhalogen to form poly HO  halides like BrI2 , where as IF5 is an inter 74. (a) 3° alcohals undergo reaction with S1 N  mechanism halogen compound. Note - RCOO is not pseudohalide † OHCR H  o  HOCR dx 3 3 2 79. (c) = K Conc[ ]n dt †  o X 3CR oRCX3 dx 1 1 1 Carbocation k = × = u dt Conc[ .]n Time Conc[ .] 1n R For 2nd order reaction,  CR † is most stable carbocation. 1 1 R k = × = sec 1– mol 1– L sec mol[ ]L/ 12 Note :- 1° alcohol show S2N mechanism † 5 2 H 80. (d) 25Mn o d3 , s4 CHR 2OH o CHR  HO 22 Number of unpaired electrons = 5  G G X  o ª º Fe o s4,d3 26 « CHX 2 ˜˜˜˜˜˜˜˜˜˜˜˜˜˜˜˜ HO 2 » 26 « | » Number of unpaired electrons = 4 « R » 28 ¬« .S.T ¼» 28 Ni o s4,d3 75. (b) Clemmenson’s reaction Number of unpaired electrons = 2 Zn Hg/ 10 1 CHOR o CHR 3 28Cu o s4,d3 HCl Number of unpaired electrons = 1 Note :- Wolf Kishner reduction is More is number of unpaired electrons CHOR   NHNH 22 o RCH higher is paramagnetism. 3 81. (b) Elevation in B.P. is colligative property HI/P Also CHOR o  CHR 3 which depends upon number of particles. Rosenmund reaction :- 2  Each BaCl2 ion gives aB and Cl2 . RCOCl  BaSOH/Pd 42 o RCHO Thus number of particles is highest out of Lindlar reagent given compounds. IIT JEE NEET eBooks: www.crackjee.xyz

Mock AIIMS 23 82. (d) Size of ion depends upon nuclear charge. linkage isomerism occurs. More is nuclear charge compared to number In this case SCN has S and N as donor of electrons, Tightly are electrons held and atom which can link with the central atom. thus smaller is size Nuclear charge No. of electrons HC CH2  Na o 11 10 88. (c) 2  CHCHCHnCH 2 + n 2 1, butadiene-3 Mg o 12 10 Styrene  F o 9 10 3 ( CH  CH CH  CH  CH  CH  CH ) Al o 13 10 2 2 2 2 n 3 In Al 13 units of positive charge tend Buna - S to attract 10 electrons more towards nucleus thus reducing sphere of electron 89. (d) CO has triple bond; ? B.O. must be 3 cloud around the nucleus i.e. size. 83. (b) Ninhydrin test is shown by proteins and O2 has double bond, ?B.O. must be 2 amino acid. Negative result show absence F is single bonded, ?B.O. must be 1 of these two. 2 Benedict’s solution test is for aldehyde Be2 does not exist, ?B.O. = 0 group which is present in monosaccrides Alternatively B.O. and not lipids. Number of bonding  No of. antibondinge 84. (c) Iodoform test is shown by compounds of = aldehyde, ketone or alcohol which have 2 D  22 CH gp at D position like CHOHC , Be Ÿ VV *2 s2,s2 ; B.O.= 0 3 3 2 2 D 90. (b) enolic form increases when there is H- 3  CORHC , 3CRCH OH( ) etc. Ethyl bonding. If this H-bonding is further alcohol has 3CHCH OH( ) thus shows stabilised by conjugation enolic form iodoform test further increases. O O H | | | | | ' C C 3 CHC OH  I4 2  NaOH6 o CHI3 ? CH 3 CH3 CH3 is max enolic form. NaI5 2OH5  HCOONa 85. (d). Penetration power is more when particle is Tautomerism small in size and is moving with high speed. ? order of penetrating power is OH O J ! rayX !E!D | | C C 86. (b) Order of filling follows )n( rule, small is A CH 3 CH CH3 H-bonding  A)n( lower is energy so filling is first. Conjugation If  A )n( values are same than lower n is   o given preference for O H  O (a) n = 5, A = 0, n + A = 5 C C (b) n = 3, A = 2, n + A = 5 CH (c) n = 6, A = 0, n + A = 6 CH3 CH3 (d) n = 5, A = 1, n + A = 6 91. (c) Benedict’s sol. test is shown by aldehydes. Reason for easy oxidation of aldehyde to n = 3, A = 2, i.e. 3d is filled Ist 87. (c) When more than one atom in a acid is presence of H- atom on carboxyl gp. monodentate ligand can act as donor Due to which it acts as strong reducing IIT JEE NEET eBooks: www.crackjee.xyz EBD_7100

24 Topicwise AIIMS Solved Papers agent and can thus reduce weak oxidising Carbonyl compounds  reagents like Tollens, Benedict and Fehling 2OH/H solution. OCC 3 {  o   CC  Zn |||| Note :– Benedict’s solution is alkaline OO solution of Cu 2 complexed with citrate di ketones ions. 98. (d) More easily Nucleophilc can replace halide Fehling solution is alkaline solution of ion more is reactivity. In case of vinyl C = Cu+2complexes with Rochelle salt (is Sod. C, C – Cl bond aquires some double bond potassium tartrate). Tollens solution is character and become strong thus is not ammonical silver nitrate (AgNO3 / NH4OH). easily replacable. 92. (d) H – Spectrum confirms quantisation of ? order is Allyl chloride > Vinyl > Chloro energy within an atom. Benzene 93. (d) Conjugate acid and base differ by one hydrogen only xx 4  om ClCC[ om C C Cl] CH3NH2 is a base its conjugate acid is 99. (b) According to Le Chatelier’s principal CH NH 3 3 (Which is +ve, electrons deficient (1) Increase in concentration of any reactant species) being consumed during the reaction favours the reaction. 94. (a) Given I (s) o I(g) , 2 (2) For an endothermic reaction increase in 'Hsub limation  57.3kJ / mol temperature favours reaction in forward direction. I(s) o I() 22A (3) High pressure is favourable for the reaction , 'Hfusion  15.5kJ / mol in which there is decrease in volume. In given case faourable conditions are : I2 (A ) o I(g) (a) Increase in conc. of BaCO3 , ' vap sub '' HHH fusion (b) Increase in temparature = 57.3 – 15.5 = 41. 8 kJ/mol (c) Decrease in pressure BaSO/Pb 4 95. (a) HC { CHCC 3   o 100. (b) Ionic concentration of A i.e. Lindlar reagents' Ca NO( 23 u 5.135.0) CH CH 3 3 Ionic concentration of B i.e. C C H H KOH = 0.75 × 2 = 1.5 Cis-2-butene Ionic concentration is same for A and B 96. (a) 'H = (–) i.e. exothermic reaction ? depression in freezing point will also be 'S = (+) i.e. entropy factor also favours. same. Thus for a spontaneous reaction 'G is 101. (d) Change Q = Current (Ampere) × time (sec) always negative. ? Q = 2 × 60 × 60 'G = (  'H )  'ST = – (ve) 1 o HOH  O Note : '  ve(G ) , reaction is non 2 2 22 spontaneous  ; o He2H2 2 (1 mole is 22400cc ' 0G , reaction is in equlibrium. 97. (c) Ozone changes S bond compound to at NTP) For 1 mol of H 2 liberated 2 faradays carbonyl compounds are required 2F = 2 × 96500  2OH/H 0.2 × 60 × 60 will give H2 CC + O3   o C O Zn 22400 uuu 60602 = 835 64. cc + C O u965002 = 0.836 L IIT JEE NEET eBooks: www.crackjee.xyz

Mock AIIMS 25

102. (b) In fluorite structure, cations form face 2 centered cubic a array and anions fit into Conc. of SO4 is SO[ 4 ] in Ag SO42 tetrahedral void. 105 In antifluorite structure case is reversed,the = 103 oxide ions fill half tetrahedral holes. u 1.01.0 103. (b) Fon any n; 1n , m = – A to 0 to + A 2 A Conc. of SO4 is SO[ 4 ] in CaSO4 1 and s = r ? 106 2 = 105 for n = 2, A z 2 1.0 104. (a) , BaSO poisoned Lindlar catalyst isPd/H 2 4 Ionic product is minimum for BaSO4 with sulphur or quinoline o It gives ? it must precipitate Ist. us product Na / liq. NH (Birch reduction) 3 110. (d) SeO in alkaline/acidic medium gives trans alkenes. 2 Note there are variety of other reducing dehydrogenates ketones to give D,E agents which gives trans or cis products according to mechanism they follow. Some unsturated ketone. (Removal of hydrogen are listed below is oxidation). 1.Wilkinson’s catalyst o 111.(b) Cd2+ is a 2nd group radical and Ni2+ is a 4th group radical. So solubility product of RhCl )(H( PPh 32 o cis) addition 2.Hydroboration NiS has to be more than CdS. Further Cd2+

HB 62 o cis addition gives yellow colour of CdS with H2S, but Ni2+ gives black colour of NiS with H S. So Note : LiAlH4 and NaBH4 normally 2 donot reduce olefinic bonds. They are both assertion and statement are wrong. selective towards carbonyl C = O double (d) is correct choice. bond. 2– – 112.(b) In [NiCl4] the Cl ligands present in the 105. (d) Value of dipole moment is not zero thus – H – O – O – H is not complex ion are less basic than CN . As supposed structure where both OH are in such no pairing of electrons in the 3d- same plane. subshell takes place. This results in sp3 Dipole moment is some what near 2.1 D hybridisation and the complex so formed ? Struture comes out to be is tetrahedral. On the other hand in case of H 2– – [Ni(CN)4] the CN ligands present in the OO complex ion are more basic than Cl–. As H such pairing of electrons can take place in 106. (d) In CO2 O has – 2 valency the 3d subshell. Due to pairing of electrons In MnO O has – 2 valency 2 in 3d subshell, one of the d-orbital becomes In Na O O has – 2 valency 2 vacant. This results in dsp2 hybridisation 2  In BaO2 O is O 2 and the complex so formed in square planar. ? it is true peroxide 113.(c) Rusting involves reduction of absorbed Note[ Ba: O22  o BaO is Barium oxide] oxygen to OH– ions and oxidation of iron 107 (d) Li + due to small size have high degree of to Fe2+ ions. The two ions and oxidation of hydration. iron to Fe2+ ions. The two ions combine to 108 (c) Gelatin has maximum protective power yield Fe(OH) which gets oxidised to give starch has minimum protective power 2 Fe O .nH O (rust). The presence of acid 2 2 3 2 109. (a) Conc. of SO4 is SO[ 4 ] in BaSO4 helps dissolution of pure iron to ferrous 11 ions while electrolytes increase the 10 10 = 10 conductivity and assist cell action. 1.0 IIT JEE NEET eBooks: www.crackjee.xyz EBD_7100

26 Topicwise AIIMS Solved Papers

114.(a) R is the correct explanation of A. equilibrium in the forward direction of those reactions which proceed with absorption COOH of heat (endothermic reactions), and in the backward direction of those reactions H * OH which proceed with the evolution of heat H OH (exothermic reactions).” E is clearly true H * OH again according to Le-chatelier principle. 116.(a) Assertion is correct as for every 10ºC raise COOH in temperature, the specific rate constant, meso-form K nearly doubles. (Although it is not correct for all reactions. For some reactions K even gets tripled for 10ºC raise). The COOH statement is clearly true and it explains the H * OH assertion, as the rate of collision among the molecules doubles for 10ºC rise in HO H temperature. So the answer is (a). H * OH 117..(c) Among oxyacids, the acidic character increases with increase in oxidation state COOH meso-form of the central atom. Hence assertion is correct. Structure of HNO2 : MIRROR O H N O ; Structure of HNO3 : O COOH HON O HO * H The assertion is true but the reason is H OH wrong as can be clearly seen from the above structures. H * OH 118.(b) Metal having negative reduction potential COOH or positive standard oxidation potential has (+)-form a tendency to get itself oxidised and pass into the solution. 119.(a) Lattice energy is the amunt of energy COOH required to dissociate one mole of an ionic crystal into its ions and hydration energy H * OH is the amount of energy released when one mole of ions undergo hydration. So for the HO H solubility of a solid in liquid hydration HO * H energy must be greater than lattice energy 120.(c) The correct reason is : The overall electron COOH deficiency in m-nitroaniline is much less (–)-form (due to –R-effect of NO2 group and +R- effect of NH2 group) than in m- The central carbon atom is pseudochiral dinitrobenzene (–R-effect of the two NO2 carbon atom. groups) and hence does not accept 115.(d) The statement is clearly wrong in context additional electrons from a weak reducing to Le-chateliers principle, which states that agent such as (NH4)2S and thus further “increase in temperature shifts the reduction is prevented. IIT JEE NEET eBooks: www.crackjee.xyz

Mock AIIMS 27

SECTION III - BIOLOGY 128. (b) Certain mitotic and meitotic irregularties such as formation of restitution nucleus, 121. (c) With the discovery of more and more fusions of spindles lead to doubling of organisms, sometimes, it becomes difficult the chromosomes in a cell. Chromosome to adjust an organism to the traditional doubling is introduced by high categories. Therefore to make taxonomic temprature, X-ray treatments, callus position of a species more precise. The formation hybridisation and chemical various obligate categories in hierarchial treatment like chlorofrom, chloral hydrate colchicine, auxins, gibberellins and classification are explained below nicotine sulphate etc. induce polyploidy. Division —o Class —o order —o family 129. (a) Asterceae constitutes the most advanced —o Genus —o Species —o Tribe family among dicots. The syngenesious 122. (c) The bacterial genome/nucleoid is made of condition of the stamen is an adaptation double stranded DNA without histone for entomophily protein. The bacterial genome thus 130. (d) Tomato and tobacco both belong to the represented by a single circular double family Solanaceae. helical DNA. The genome contains some 131. (a) The central pith is surrounded by xylem, 100 chemical sites or loci. Each locus phloem, pericycle and endodermis . The contains many genes. E coli contains about phloem occurs only outside the xylem e.g 4000 genes. Equisetum, Osmunda 123. (c) The fungus that causes the disease ‘Ergot 132. (c) D.P.D. or suction pressure (S.P.) of Rye’ is Claviceps purpurea. It contains = O.P. – T. P. Hence D.P.D. for A = 4 bars, B = 0 bars, C = many poisonous alkaloids. The 5 bars, D = 4 bars hallucinogenic drug LSD is extracted from We know that direction of movement of this fungi. water is from lower D.P.D. (S.P.) to higher • Rust of wheat is used by Puccinia D.P.D. (S.P.). graminis ? flow will be from B to A, C, and D. • Powdery Mildew of Pea is caused by 133. (c) Chlorophyll has a tetrapyrrole porphyrin Erysiphe. head (15 × 15Å) and a long chain alcohol 124. (b) Shape and number of chloroplast in called phytol (20 Å). different member of algae is different 134. (d) Cyt a3 posseses two copper centers. The latter help in transfer of electron to oxygen. Chlamydomonas – cup shaphed, 1/cell 135. (a) In campylotropous ovule, the body is Zygnema – Stellate, 2/cell curved but the embryosac is straight eg. Spiral – Spirogyra 1/ cell Capsella. In Anatropous, the body of the Collar shaped - Ulothrix 1/cell ovule is inverted and gets fused with 125. (c) The inner membrane of mitochondria funiculus along its whole length on one possess small tennis like particles called side (most of the angiosperms). In orthotropous condition the body of ovule elementary particles, F0 – F1 particles or oxysomes. There are 1 × 104 – 1 × 105 lies straight and upright over the funicle. oxysomes in a mitochondria. e.g. Piperaceae, Polygonaceae. 136. (c) Nyctinastic (sleeping) movements : The 126. (c) + + 40 : ab 40 : + a 10 : + b : 10 diurnal (changes in day & night) 127. (a) Lac operon is an inducible operon system movements of leaves cause sleeping which regulates genetic material. The movements during night which are called genetic material remains switched off nyctinastic movements. It may be normally but becomes operational in the photonastic as in oxalis or thermonastic as presence of inducer. in tulip flower. IIT JEE NEET eBooks: www.crackjee.xyz EBD_7100

28 Topicwise AIIMS Solved Papers

137. (a) Mimicry is the resemblance of an organism • Smooth muscle tissue occurs within to its natural surroundings, like initating a almost every organ, forming sheets, non-living object or another organism for bundles or sheaths around other conceding itself from its enemies. Flowers tissues. of Ophrys muscifera resemble the female Smooth muscle differs from both skeletal wasps of Colpa aurea so that the male and cardiac muscle tissues in structure wasp tries to copulate with the flowers and and function. Sarcomeres or myofibrils are pollinate them. not present and are therefore not striated, 138. (a) E. coli are found in the human intestine. If ie. smooth. they are present in water it indicates that 146. (d) The permanent teeth appear completely by the water is polluted. the age of 12 years, except for the last 139. (c) Meristem culture is done for the molars, which , if present, are formed after development of virus free plants. the age of 18 years. The dental formula Meristematic tissue cells can be taken before the wisdom teeth appear would be either from shoot or root tip. 2 1 2 2 140. (c) 2, 4 Dichlorophenoxy acetic acid is used i c, pm; m, 2 1 2 2 for causing defoliation of forest trees. after wisdom teeth appear 141. (b) Species is the basic unit of classification. 2 1 2 3 The term was coined by John Ray. Most i c, pm; m, taxonomists define species as 2 1 2 3 morphologically distinct and reproductively 147. (b) Alveoli are the site of the respiratory isolated natural population or groups of exchange of gases. Oxygen from the populations where individuals resemble one alveolar air diffuses through the alveolar another more closely that with the members epithelium and the capillary endotheliun of other species, interbreed freely and form into the capillary blood and carbon dioxide a genetically closed system. Gene transfer diffuses in the opposite direction. occurs between populations of a species 148. (b) by gene flow i.e. emigration and immigration. Hepatic artery Liver 142. (c) Km (Michealis Menten constant). It is defined as that substrate concentration at Heart Carotico systemic arch Dorsal aorta Coeliac artery which under optimum conditions the rate of an enzyme catalysed reaction reaches Splenic (spleen) Lienogastric

half the maximum rate. Km is inversely Gastric (stomach) proportional to the affinity of enzyme for its substrate. 149. (a) Henle’s loop is associated with 143. (b) Fire bellied toad (Bombina) is a member of concentration of the urine and production order – Anura of the class Amphibia. of hypertonic urine. 144. (b) Eichhornia crassipes is an American origin 150. (c) All mammals have 7 cervical vertebrae in plant and now a troublesome water weed their neck (it is one of the typical mammalian in India. characters). Though they become long in 145. (a) Characteristics of smooth muscle cells camel & giraff but the number is 7 only. As • range from 5 to 10 µm in diameter and whale is also a mammal it must possess the 30 to 200 µm in length same 7 cervical vertebrae. • spindle-shaped 151. (a) Xth pair of cranial nerves (vagus nerves) • single, centrally located nucleus has a motor branch called cardiac nerve which innervate cardiac muscles. IIT JEE NEET eBooks: www.crackjee.xyz

Mock AIIMS 29 152. (a) Luteinizing hormone (LH) stimulates 160. (c) In test tube baby, fertilization of the sperm ovulation. Deficiency of insulin causes and the ova is carried out in lab conditions diabetes mellitus. Deficiency of ADH or (in vitro) and when the embryos have vasopressin causes diabetes insipidus. reached the 32-celled stage, it is implanted Deficiency of parathormone causes tetany. back into the uterus of the surrogate Deficiency of thyroxine causes cretinism mother. in infants and myxoedema in adults. 161. (a) Due to the process of exosmosis, plasmoly- 153. (c) Follicle stimulating hormone (FSH) sis takes place. Hence water goes out of stimulates maturation of ovarian follicles. the mango and the mango shrinks. Hence, the reason is the correct explanation for Luteinizing hormone stimulates testes to the assertion. secrete testosterone. Prolactin stimulates 162. (b) Nissl’s granules are both basophilic and development of mammary glands during also contain RNA. They are basophilic pregnancy. Human chorionic gonadotropin since they stain with basic dyes. The rea- released from the placenta also maintains son is therefore not the correct explana- the corpus luteum during pregnancy. tion of the assertion. 154. (a) Biotic potential is the inherent ability of an 163. (a) Phycobilins are accessory photosynthetic organism to reproduce. Resistance refers pigments present in algae. These pigments are present along with chlorophyll – Į and to the sum total of all the limiting factors help in capturing light for photosynthesis. that inhibit further growth of population. Phycobilins are pigments which contain 155. (b) Normal woman whose father was colour proteins and proteins are easily denatured blind would have received the X by heat. Hence, the reason is the correct explanation for the assertion. chromosome from her father o cXX , 164. (d) The inner mucosa is mainly meant to marries a colour blind man XYc , their increase the surface area and not for progeny would be: absorption. Hence, both assertion and reason are false. c u c YXXX 165. (e) Vital capacity is the total volume of air that can be breathed out with maximum effort. progeny cc c c XYYXXXXX The assertion in this case is false. blindcolour normal colour blind sonnormal daughter daughter son 166. (c) Heart wood is non functional but it is plugged due to the growth of parenchyma 156. (a) Genetic drift can operate only in smaller that thickens later on. The reason in this population where in fluctuation can be case is false, but the assertion is true. observed in the proportion of allele 167. (a) Glucose which is the final product of car- bohydrate digestion is converted to gly- distribution in the presence of external cogen in the liver and stored both in the disturbances. liver and muscles of animals. This process 157. (c) Cro-Magnon succeeded Neanderthal. takes place only in animals and hence it is Fossils of Neanderthal man have been called as animal starch. Hence the reason found from Eroupe, Asia and north Africa. is the correct explanation for the assertion. Fossils of Australopithecus have been Electron micrograph of a section of a liver found in Africa. cell showing glycogen. 158.(b) The presence of juvenile hormone is 168. (c) Racemose inflorescence shows an indefi- necessary for metamorphosis in to adult nite growth, which is why it is said to be an 159. (a) Morphine is an opiate narcotic, Bhang is a indeterminate inflorescence. The reason hallucinogen, Reserpine derived form given is false. Rauwolfia, is used a tranquilizer, cocaine 169. (e) Mouth parts of cockroach, honey bee & is a stimulant. mosquito are homologus organs. Therefore the assertion is false. IIT JEE NEET eBooks: www.crackjee.xyz EBD_7100

30 Topicwise AIIMS Solved Papers 170. (b) The crossing over takes place at the four 176. (b) The interferons are a special defensive strand stage so that recombinations can mechanism produced by the infected cells. take place when the chromosomes sepa- On reaching the nearby uninfected cells rate as chromatids. The gene linkages do they help in the formation of certain pro- disappear if crossing over takes place at teins that prevent the multiplication of the two strand stage. The reason is therefore virus. Hence the reason is not the correct not the correct explanation of the asser- explanation of the assertion. tion. 177. (e) Horticulture is the conscious raise of Veg- etables, fruits & Ornamental plants and not of cereal crops. Hence the assertion is false. 178. (a) Viruses are bodies which have either DNA or RNA as a genetic material. For multipli- cation, it is the genetic material that enters into the host cells which then with the help of the DNA of the host cell prepare their A double crossing over. protein shield. The reason is the correct 171. (c) Oncogenes integrate their DNA with DNA explanation for the assertion. of the host cells and not their RNA. The 179. (a) All aggregate and multiple fruits develop reason in this case is false. from other floral parts like the thalamus, 172. (a) HIV is found in body fluids like blood & calyx, etc. Hence the reason is the correct semen. It is a retrovirus which has RNA as explanation for the assertion. its genetic material. It hence shows reverse 180. (c) In Casuarina & Betula the pollen tube enters transcription. Hence the reason is the cor- through the Chalaza and not the rect explanation for the assertion. microphyle. Hence the reason is false. 173. (b) Genes although show a linear order but are arranged in the DNA as a helical coiled SECTION IV - G.K. structure. Hence the reason is not the cor- 181. (a) 182. (d) 183. (b) rect explanation for the assertion given. 184. (d) 185. (c) 186. (c) 174. (a) The virus cannot multiply outside living systems. They require a host in order to 187. (a) 188. (b) 189. (d) multiply. Hence the reason is true for the 190. (a) 191. (d) 192. (b) assertion. 193. (d) 194. (b) 195. (a) 175. (e) The allergens are actually not glycogen 196. (d) 197. (a) 198. (c) molecules but are protein molecules. Hence 199. (b) 200. (d) the assertion is false.